Westindies High School Maths Olympiad

You might also like

Download as pdf or txt
Download as pdf or txt
You are on page 1of 316

2022 Senior Mathematical Olympiad

Qualifying Round Examination (Grades 9 to 11)

NAME

GRADE

SCHOOL

STUDENT CONTACT NUMBER

• EACH entry MUST be accompanied by a nominal entry fee of J$500


• Be sure to staple ALL pages (including this one) together
• All entries must reach the Mathematics Department, U.W.I by Wednesday 14
December
• You may deliver by (a) Hand (b) Courier (c) Local Mail

• The Courier address is


Mathematics Department, UWI
Mona
Kingston 7
• The Mailing address is
Senior Mathematical Olympiad
P.O. Box 94
Mona Post Office
Kingston 7
For each question, determine the letter corresponding to the correct or best response; along
with the question number, indicate this letter by circling or shading it

1. In a group of 50 persons each person is classified as being tall or short. Given that
there are 14 tall men, 31 women and 18 are short persons, how many of the women
are short?
(A) 5 (B) 7 (C) 9 (D) 13 (E) 18
2. In a bag of marbles, 3/5 of the marbles are blue and the rest are red. If the number of
red marbles is doubled and the number of blue marbles stays the same, what fraction
of the marbles will be red?
2 3 4 3 4
(A) (B) (C) (D) (E)
5 7 7 5 5
3. Four politicians are arguing about who is responsible to pay for work done by an
independent contractor. Using first names only, the politicians are Arlene, Barbara,
Canute and Duncan. Arlene says Barbara is, Barbara says Canute is, while Canute
and Duncan says they have idea who is responsible. The only person who is lying is
the person who is responsible to pay for the work done. Who is responsible to pay
for the work done?
(A) Arlene (B) Barbara (C) Canute (D) Duncan (E) Need more in-
formation
4. A multiple choice examination consists of 20 questions. The scoring is +5 for each
correct answer, −2 for each incorrect answer and 0 for questions unanswered. Pete
scored a total of 48 on the examination. What is the maximum number of correct
answers Pete could have had?
(A) 8 (B) 10 (C) 12 (D) 14 (E) 16
5. Thirty dollars is divided among 8 persons according to the following rules

• Each person gets at least $1


• At least one person gets more than $5
• At least four persons gets more than $1
• Each person gets and exact amount of dollars.

What is the largest amount that a person can receive?


(A) $13 (B) $15 (C) $17 (D) $19 (E) $23
6. A sum of money is to be divided between Alyah, Byron and Cecil. Alyah receives $1
plus one-third of what is left. Byron then receives $6 plus one-third of what remains.
Cecil receives the rest, which amounts to $40. How much did Byron receive?
(A) $26 (B) $28 (C) $30 (D) $32 (E) $34

7. The interior angles of a triangle are x◦ , y ◦ and z ◦ where x ≤ y ≤ z. If the angles are
all multiples of 20, how many triples (x◦ , y ◦, z ◦ ) are there?
(A) 6 (B) 7 (C) 9 (D) 27 (E) 28

8. The numbers 1 to 12 are to be placed on the figure, to the right, in such a way that
the sum of the 4 numbers along the six straight lines are all equal.

3 A B 1

8 5

C 6 D E

Where on the diagram should the 7 be placed?


(A) A (B) B (C) C (D) D (E) E

9. At the Centre for High Achievers, there are 100 students of which 99% are females.
Of the student living on campus, 98% are females. How many of the students live
off campus?
(A) 1 (B) 2 (C) 49 (D) 50 (E) 98

10. The three digit number abc is equivalent to 100a + 10b + c. Which of the following is
a factor of abc when a > 0 and a = b = c?
(A) 7 (B) 11 (C) 13 (D) 19 (E) 37

11. Three car washers can wash 4 cars in 5 hours. The the nearest whole number of
hours, how long would it take 7 car washers to wash 18 cars, if all are working at the
same rate all the time
(A) 3 (B) 4 (C) 7 (D) 10 (E) 18
12. On planet ZeeZee, the number of days in a week is the same as the number of weeks
in a month and the number of months in a year is twice the number of days in a
month. On ZeeZee there are 1, 250 days in a year. How many months are there on
ZeeZee?
(A) 50 (B) 25 (C) 20 (D) 10 (E) 5
13. In the figure shown,

what fraction of the total area of the regular octagon is shaded?


(A) 1/3 (B) 1/4 (C) 1/5 (D) 1/6 (E) 3/8

14. A box contains 10 balls numbered 1 to 10. Two balls are randomly chosen, one at
a time, from the box and without replacement. In how many ways can the balls be
drawn so that the sum of the two balls is even?
(A) 5 (B) 10 (C) 15 (D) 30 (E) 40
15. A common notation is n!, which reads “n factorial”, and is defined as
n! = n (n − 1) (n − 2) · · · · · 2 · 1
For example, 4! = 4 (3) (2) (1) = 24.
The number
m = 1! + 2! + 3! + · · · + 2021! + 2022!
What is the units digit of m?
(A) 1 (B) 2 (C) 3 (D) 4 (E) 5
16. The perimeter of a rectangle is 56 metres. The ratio of the width to the length is
3 : 4. The length of the diagonal of the rectangle is
(A) 17.5 m (B) 20 m (C) 25 m (D) 40 m (E) 50 m
17. Let a = 25555 , b = 33333 and c = 62222 . Which of the following is correct?
(A) a < b < c (B) a < c < b (C) c < b < a (D) b < c < a (E)
b<a<c
18. Converting to seconds, 2000 days, 2000 hours, 2000 minutes and 2000 seconds is
equivalent to N million seconds.
N is closest to
(A) 1 (B) 15 (C) 45 (D) 180 (E) 2000

19. The altitude h, of a triangle is increased by m. How much should be taken off the
base b, so that the area of the new triangle is one-half that of the original tiangle?
bm bh b (2m + h) b (m + h) b (2m + h)
(A) (B) (C) (D) (E)
h+m 2 (h + m) m+h 2m + h 2 (m + h)
20. Store A sells red marbles at 4 per dollar and blue marbles at 3 per dollar. Store B
sells red marbles at 4 per dollar and blue marbles at 6 per dollar. Mary spends $10
buying m red marbles and n blue marbles from store A and m red marbles and n
blue marbles from store B. How many marbles (altogether) did Mary buy?
(A) 20 (B) 30 (C) 40 (D) 50 (E) 60

21. D is the centre of a circle and ABCD is a rectangle, where B is on the cirlce. Given
that AB = 3 units and BC = 4 units.

A B

D C

The area, in square units, of the shaded region is between


(A) 4 and 5 (B) 5 and 6 (C) 6 and 7 (D) 7 and 8 (E) 8 and 9

22. Two numbers are such that the ratios of their difference, their sum and their product
are (respectively) 1 : 7 : 18. What is the product of these two numbers?
(A) 21 (B) 23 (C) 25 (D) 27 (E) 29

23. When the time is 3 : 26 on a conventional clock, what is the angle between the minute
hand and the hour hand?
(A) 45◦ (B) 47◦ (C) 49◦ (D) 51◦ (E) 53◦
24. Two circles of radii 1 unit and 3 units overlap as shown in the figure.

π
If the area of the shaded region is , what is the sum of the areas of the unshaded
3
portions in the two circles?
26π 28π 29π
(A) 8π (B) (C) 9π (D) (E)
3 3 3
25. In the diagram, ABCDE is a regular pentagon and triangle AP E is equilateral.

B P D

A E

What is the size of the obtuse angle BP D?


(A) 168◦ (B) 165◦ (C) 132◦ (D) 150◦ (E) 172◦

Please write your name here


The University of the West Indies, Mona
presents
The 2022 Senior Mathematics Olympiad

Qualifying Round Examination (SOLUTIONS)


1. Soln (D) Let x be the number of short women. The number of tall women is
therefore 31 − x. The number of men is 50 − 31 = 19 and so the number of short men
is 5. Since the number of short persons is 18, we have 5 + x = 18. Solving, x = 13.
3 b
2. Soln (C) Let the numbers of red and blue marbles be r and b. = . This
5 b+r
gives 5b = 3b + 3r or 2b = 3r or b = 32 r. If the number of red marbles is doubled and
the number of blue marbles stays the same, the total is 2r + b and the fraction of red
marbles is
2r 2r 4r 4
= 3 = =
2r + b 2r + 2 r 4r + 3r 7

3. Soln (B) Suppose Arlene is responsible and is lying. This would mean that
Barbara is also lying and so Arlene is not the person. Suppose Barbara is responsible
and is lying. In this case, Arlene, Canute and Daniel’s statements are consistent.
Suppose Canute is responsible and is lying. This would mean that Arlene is also
lying and so Canute is not the person. Similarly, Duncan cannot be the persons.
The correct answer is B.

4. Soln (C) Let x be the number of correct response, y the number of incorrect and
z the number of questions unanswered. His score is 5x−2y = 48. where x+y+z = 20.
So

5x − 2y = 48
2x + 2y + 2z = 40

Adding, 7x + 2z = 88 or 7x = 88 − 2z. Now x is greatest when z is least. The least


such z is 2 corresponding to x = 12

5. Soln (D) Clearly the largest amount is received when only one person gets more
than $5 and when the other 7 persons receive as small as possible. Let m be the
largest amount. We therefore want

m + 2 + 2 + 2 + 2 + 1 + 1 + 1 = 30

This corresponds to m = 19.


6. Soln (A) Suppose the original amount of money is $x. Cecil receives
     
2 1 2
x − 1 + (x − 1) + 6 + x − 1 + (x − 1) − 6 = 40
3 3 3
2 2 1 1 2 2
x−1− x+ −6− x+ + x− +2 = 40
3 3 3 3 9 9
2 38
x− = 40
9 9
2x − 38 = 360
2x = 398
x = 199

Byron’s share is
       
1 2 1 2
6+ x − 1 + (x − 1) − 6 = 6+ 199 − 1 + (198) − 6
3 3 3 3
= 26

Alternatively, After Alyah had received his share, Byron received $6 plus one-third
of the remainder. Since Cecil gets the rest, she received two-thirds of the remainder,
which is $40. Thus, one-third of the remainder is $20. Therefore Byron receives is
$20 + $6 = $26.

7. Soln (B) Let x = 20p, y = 20q and z = 20r so that

20p + 20q + 20r = 180 or p + q + r = 9

and p ≤ q ≤ r ∈ Z+ . The possible triplets are

(1, 1, 7) , (1, 2, 6) , (1, 3, 5) , (1, 4, 4) , (2, 2, 5) , (2, 3, 4) , (3, 3, 3)

There are 7 triplets

8. Soln (E) Since 3 + 8 + 6 + 9 = 26, 9 + D + 5 + 1 = 26 and so D = 11. With this


information,

A + B = 22, A + C = 14, C + E = 9 and B + E = 17

The list of numbers not yet used is {2, 7, 10, 12} . We therefore have two possibilities
for C and E. Case 1: C = 7 and E = 2. In this case B would be 15 which is not in
the list. Case 2: C = 2 and E = 7. In this case, B = 10 and A = 12.
9. Soln (D) It is clear that the number of females is 99 and the number of males is
1. This male lives on campus (otherwise all the students living on campus) would be
male. The 2% males living on campus is accounted for by this 1 male. The 100%
living on campus would therefore be 50 × 1 = 50 (49 females and 1 male)There are
7 triplets

10. Soln (E) When a > 0 and a = b = c, the number takes the form aaa = a (111) =
a · 3 · 37. So 37 is a factor of aaa.

11. Soln (D) Because three car washers can wash 4 cars in 5 hours, 1 car washer will
wash 43 cars in 5 hours or 15
4
cars in 1 hour. The 7 car washers will now work at a
4 28
combined rate of 15 × 7 = 15 cars in 1 hour. If h is the number of hours to wash the
18 cars, then
28
h = 18
15
This gives h = 18×15
28
= 135
14
9
= 9 14 . Rounding up, we get 10.

12. Soln (A) Let d be the number of days in a week and the number of months be
m. So d · d = d2 is the number of days in one month. We therefore have

md2 = 1, 250

Also,
m
m = 2d2 of d2 =
2
It follows that
m
m· = 1, 250
2
This gives m2 = 2500 and therefore m = 50.

13. Soln (B) The octagon may be decomposed into 4 congruent rectangles and 8
congruent triangles. The shaded area consists of one such rectangle and 2 of the
triangles. If x is the area of one of the rectangle and y is the area of one of the
triangle, then the fraction is
x + 2y x + 2y 1
= = .
4x + 8y 4 (x + 2y) 4

14. Soln (E) We will count by cases.


Case 1: The first ball is odd: Odd followed by odd.
Number of ways is 5 · 4 = 20
Case 2: The first ball is even: Even followed by even
Number of ways is 5 · 4 = 20
The total is 20 + 20 = 40
15. Soln (C) In all the terms except the first 4, 5 × 2 = 10 is a factor and therefore
their units digit is 0. The units digit for the the terms of n are therefore
1+2+6+4+0+0+···+0+0
This corresponds to a units digit of 3 for n.
16. Soln (B) Let w be the width of the rectangle. 4w = 3ℓ where ℓ is the length.
4 4 14
w + w + ℓ + ℓ = w + w + w + w = w = 56
3 3 3
3×56
This gives w = 14
= 12 The length of the diagonal is
s  2
√ 4
d = w +ℓ = w +
2 2 2 w
3
r ! r !
16 25 5
= 1+ w= w= w
9 9 3
42
Since w = 5
, the length of the diagonal is d = 53 w = 5
3
· 12 = 20 cm.
17. Soln (E) We observe that
1111
25555 = 25 = 321111
3 1111
33333 = = 271111

3
1111
and 62222 = 62 = 361111
From this, it is evident that b < a < c.
18. Soln (D) In seconds
2000 days : 2000 × 24 × 60 × 60 = 48 × 36 × 105 = 1728 × 105 seconds
2000 hours : 2000 × 60 × 60 = 72 × 105 = seconds
2000 minutes : 2000 × 60 = 12 × 104 = seconds
2000 seconds : 2000 seconds
The sum is equivalent to 172.8 × 106 + 7.2 × 106 + 1.2 × 104 + 2 × 103 . In millions,
this is approximately
N ≈ 172.8 + 7.2 ≈ 180
1
19. Soln (E) The area of the original triangle is A0 = 2
bh. The area of the new
triangle is 14 bh. Let x be the reduction in the base.

1 1
(b − x) (h + m) = bh
2 4
The problem now is to find an expression for x.

2 (b − x) (h + m) = bh

Therefore
bh
b−x=
2 (h + m)
From this,

bh b · 2 (h + m) bh
x = b− = −
2 (h + m) 2 (h + m) 2 (h + m)
2bh + 2bm − bh 2bm + bh b (2m + h)
= = = .
2 (h + m) 2 (h + m) 2 (m + h)

20. Soln (C) The costs for the red marbles from store A is $ 14 . The costs for the blue
marbles from store A is $ 13 . The costs for the red marbles from store B is $ 14 . The
costs for the blue marbles from store B is $ 16 . Buying m red marbles and n blue
marbles from stores A and B, Mary spends, in dollars,
m n m n
+ + + = 10
4 3 4 6
Multiplying through by 12 :

3m + 4n + 3m + 2n = 120
6 (m + n) = 120

From this m + n = 20. The total number of marbles bought is 2 (m + n) = 40.


21. Soln (D) It is evident that the radius of the triangle is r = DB. From Pythagoras
theorem, √ √ √
r= AB 2 + BC 2 = 32 + 42 = 25 = 5 units
The area of the shaded region is the area of the sector (quarter circle) minus the area
of the rectangle. This is
1 25π
× π × 52 − 3 × 4 = − 12
4 4
25π
But 76 ≤ 25π ≤ 80 and so 19 ≤ ≤ 20 and
4
25π
19 − 12 ≤ − 12 ≤ 20 − 12
4
The are is therefore between 7 and 8 square units.
22. Soln (D) Let the numbers be x and y where x < y

y − x : x + y : xy is 1 : 7 : 18

So
y−x 1
=
x+y 7
and so (cross multiplying) 7y − 7x = x + y. This gives 6y = 8x or y = 34 x. Since
x + y : xy is 7 : 18
x + 43 x 7 7
3 7 7 7
4 = or 4 = or =
x · 3x 18 3
x 18 4x 18

This means that 4x = 18 or x = 29 . It follows that the product xy = 92 · 34 · 92 = 27.
23. Soln (E) At 3 : 00 the minute hand is vertical and the hour hand is horizontal..
At 3 : 26, let the angle between the vertical (at 12) and the minute hand be a and
the angle between the horizontal (at 3) and the hour hand be b.
26
a= × 360◦ = 156◦
60
This is 180◦ − 156◦ = 24◦ from the vertical at 6. Also
26 1
b= × × 360◦ = 13◦
60 12
This is the angle below the horizontal at 3. The required angle is 90◦ −24◦ −13◦ = 53◦ .
24. Soln (D) From, πr 2 , the areas of the two circles are π and 9π. If A is the unshaded
area in the small circle and B is the unshaded area of the bigger circle, then
π π
A+ = π and B + = 9π
3 3
2π 28π
Adding, A + B + = 10π, From this, A + B = .
3 3
1
25. Soln (A) The interior angles of a regular pentagon are 5
× 3 × 180◦ = 108◦ . Since
angle P AE = P EA = 60◦ , angle

P AB = P ED = 108◦ − 60◦ = 48◦

Furthermore, because triangles BAP and DEP are isosceles, angle


1
AP B = EP D = (180◦ − 48◦ ) = 66◦
2
The obtuse angle BP D is therefore

360◦ − (66◦ + 60◦ + 66◦ ) = 168◦


2022-2023 Senior Mathematical Olympiad
Round One Examination (Grades 9, 10 and 11) - 11:00am

For each question, determine the letter corresponding to the correct or best response; along
with the question number, indicate this letter by shading it on the answer sheet

1. The statement below is true:

(63) + 4 − (2 − 1) = 5.

The symbol  between the 6 and the 3 is representing


(A) ÷ (B) × (C) + (D) − (E) None of these

2. What is another representation of

(2022) (2022)2022 ?

(A) (2022)2023 (B) (4088 484)2022 (C) (2022)4044 (D) (4044)4044 (E) (2023)2022

3. The product of four of the digits


3, 4, 5, 6, 7
is 360. Which digit was not used?
(A) 3 (B) 4 (C) 5 (D) 6 (E) 7

4. Let J, M and O be distinct (different) positive integers. Given that JMO = 2022. What
is the largest possible value of J + M + O?
(In the above, JMO = J · M · O = J × M × O)
(A) 1014 (B) 342 (C) 343 (D) 2023 (E) 60

5. The following five cards are arranged to form the smallest 9 digit number.

4 8 31 59 107

Which card must be placed furthest on the right? The card marked
(A) 4 (B) 8 (C) 31 (D) 59 (E) 107
2022-2023 SMO (Questions) 2

6. What is the value of x in the diagram below?

x◦

115◦ 125◦

(A) 50 (B) 55 (C) 60 (D) 65 (E) 70

7. The diagram shows an octagon consisting of 10 unit squares. The shapes below P Q is a
unit square and a triangle with base 5.

X
Q
Y

If P Q divides the area of the octagon into two equal parts, what is the value of the ratio
XQ
?
QY
(A) 2/5 (B) 1/2 (C) 3/5 (D) 2/3 (E) 3/4

8. The quadrilateral ADCB is such that AB = 11 cm, BC = 7 cm, CD = 9 cm and


AD = 3 cm.
C

A B
If the angle at points A and C are 90◦ , in square centimetres, what is the area of the
quadrilateral?
(A) 30 (B) 44 (C) 48 (D) 52 (E) 60
2022-2023 SMO (Questions) 3

9. Charmaine pays an on-line service provider a fixed monthly fee plus an hourly charge
for connect time. Her December bill was $1248.00, but in January her bill was $1754.00
because she used twice as much connect time as in December. What is the fixed monthly
fee?
(A) $253.00 (B) $506.00 (C) $624.00 (D) $742.00 (E) $877.00

10. The diagram below consists of three shapes of equal perimeter: a square, a rectangle and
an equilateral triangle. The side length of the square is 9 cm.

9 cm

What is the length of the short side of the rectangle?


(A) 4 cm (B) 5 cm (C) 6 cm (D) 7 cm (E) 8 cm

11. How many integers between 100 and 300 have only odd digits?
(A) 25 (B) 50 (C) 75 (D) 100 (E) 150

12. Under cards with the same colour, the same number is always found. If the three hidden
numbers in one row are added, one obtains the number to the right of the row.

34
32
26

Which number is hidden under the black card?


(A) 6 (B) 8 (C) 10 (D) 12 (E) 14

13. A large watermelon weighs 20 kg, with 98% of its weight being water. It is left to stand
in the sun, and some of the water evaporates so that now only 95% of its weight is water.
What does it now weigh?
(A) 17 kg (B) 19.4 kg (C) 10 kg (D) 19 kg (E) 8 kg
2022-2023 SMO (Questions) 4

14. Two radii of a circular quadrant are OX and OY . A semi-circle is drawn on XY as


shown. T, S and C denote the resulting triangle, segment and crescent.

Y
C
S
T

O X
Area of T
What is the value of the ratio ?.
Area of C
(A) 3/π (B) 1 (C) 13/4π (D) 7/2π (E) 15/4π

15. The triangle P RS is equilateral and its area is half that of the triangle P QR.

What is the size, in degrees, of the angle P RQ?


(A) 75 (B) 80 (C) 90 (D) 100 (E) 120

16. Adam, Barney and Joe carry 999 books out of the library. Adam works for 3 hours,
Barney works for 4 hours and Joe works for 5 hours. They work at different speeds, with
Adam carrying 5 books for every 3 books Barney carries and every 2 books Joe carries.
How many books did Adam carry?
(A) 305 (B) 405 (C) 505 (D) 605 (E) 705

17. Two non-zero real numbers, a and b, satisfy ab = a − b. What is a possible value
a b
+ − ab?
b a

(A) −2 (B) −1/2 (C) 1/3 (D) 1/2 (E) 2


2022-2023 SMO (Questions) 5

18. Karen and Jacqui are in separate classes. Karen’s class and Jacqui’s class are sharing
some apples donated by a local farmer and everyone in each classroom will have 6 apples.
If Karen’s class shared all the apples, each student would have 10 apples. If Jacqui’s class
shared all the apples, how many apples would each student in the class have?
(A) 5 (B) 8 (C) 10 (D) 12 (E) 15

19. Aaa, Bee, Cee, Dee, Eee, Fff and Gee are 1, 2, 3, 4, 5, 6 and 7 years old, in some order
but not this order. Dee is three times as old as Bee. Cee is four years older than Eee.
Fff is older than Aaa and Aaa is older than Gee, but the combined ages of Aaa and Gee
is greater than the age of Fff. What is Aaa’s age?
(A) 2 (B) 3 (C) 4 (D) 5 (E) 6

20. Dried dog food is available in three sizes: 1 kg bags, which cost $60; 3 kg bags, which
cost $150; and 8 kg bags, which cost $250. What is the smallest number of bags you can
buy such that the average price per kilogram is exactly $40?
(A) 4 (B) 5 (C) 6 (D) 7 (E) 8

21. One square is drawn inside each of the two congruent isosceles right-angled triangles.
The area of square P is 45 units.

P R

How many units is the area of square R?


(A) 35 (B) 40 (C) 45 (D) 50 (E) 60

22. Professor T gave an exam to a class of 5 students. The scores, in order, are

71, 76, 80, 82, 91.

In random order, Professor T enters the scores into a spreadsheet, which recalculated
the class average after each score was entered. Professor T noticed that after each score
was entered, the average was always an integer. What was the last score Professor T
entered?
(A) 71 (B) 76 (C) 80 (D) 82 (E) 91
2022-2023 SMO (Questions) 6

23. In rectangle ABCD, AD = 1, P is on AB, and DB and DP trisect ∠ADC.

A P B

D C

What is the perimeter of ∆BDP ?


√ √ √ √
3 4 3 √ 3+3 5 5 3
(A) 3 + (B) 2 + (C) 2 + 2 2 (D) (E) 2 +
3 3 2 3
24. A shape is made up of a triangle and a circle that partially overlap. The grey area is
45% of the entire area of the shape. The white part of the triangle is 40% of the total
area of the shape.

What percent of the area of the circle is the white part, outside the triangle?
(A) 20% (B) 25% (C) 30% (D) 35% (E) 50%

25. Two different prime numbers are chosen between 4 and 18. When their sum is subtracted
from their product, which of the following number could be obtained?
(A) 21 (B) 60 (C) 119 (D) 180 (E) 231
2022-2023 Senior Mathematical Olympiad
Round One Examination (Grades 9, 10 and 11) Solutions

1. Soln: (A) We have (63)+4−1 = 5 or (63) = 2. Of the symbols given,  represents


the ÷ symbol.
2. Soln: (A) (2022) (2022)2022 = (2022)1 (2022)2022 = (2022)1+2022 = (2022)2023 .
3. Soln: (E) 360 = 3 · 120 = 3 · 4 · 30 = 3 · 4 · 5 · 6
4. Soln: (A) 2022 = 2 × 1011 × 1. This result in the largest value of J + M + O =
2 + 1011 + 1 = 1014
5. Soln: (B) The number is smallest when the leading digits/cards are arranged smallest
to largest from left to right. In order, they are
107, 31, 4, 59, 8

6. Soln: (C) Since angles on a straight line adds to 180◦ , the base angles of the triangle
are 65◦ and 55◦ . The sum of the angles in a triangle is 180◦ and so x = 180 − (65 + 55) =
60
7. Soln: (D) Let QY = x. The area below P Q is the area of the triangle plus the area of
1 square. This must be equal to 5 squares. So
1
1+ · 5 (1 + x) = 5
2
8
This gives 2 + 5 (1 + x) = 10 and so 1 + x = 5
or x = 35 . Since QY = x = 35 , XQ =
XQ 2
1 − x = 52 and the ratio = .
QY 3
8. Soln: (C) With the straight line DB, we construct two right angled triangles. The area
of the quadrilateral is now
1 1 1 1
(AB) (AD) + (CD) (CB) = (11) (3) + (9) (7) = 48
2 2 2 2
9. Soln: (D) Let f be the fixed monthly fee, h be the hourly rate and t, the total connect
time in December. We have
f + ht = 1248 and f + h · 2t = 1754
From this, 2f + 2ht = 2496 and f + 2ht = 1754. Subtracting, f = 2496 − 1754 = 742.
2022-2023 SMO (Questions) 2

10. Soln: (C) The perimeter of each shape is 4 × 9 = 36 cm. The length of one side of
the triangle is therefore 36
3
= 12 cm. This is the length of the long side of the rectangle.
Because 12 + 12 = 24, the short sides add to give 12 cm and the length of the short side
is 6 cm.

11. Soln: (A) There is only one choice for the first digit (namely 1). There are 5 choices
for the second digit (namely 1, 3, 5, 7, 9) and there are 5 choices for the third digit (namely
1, 3, 5, 7, 9). The total is 1 × 5 × 5 = 25

12. Soln: (D) Let the number under the white card be w, the number under the grey card
be g and the number under the black card be b. It follows that

g + 2w = 34
g + w + b = 32
w + 2g = 26

Adding the first and third equation, 3g + 3w = 60. Dividing by 3, g + w = 20. Because
g + w = 20, b = 32 − 20 = 12
2 2
13. Soln: (E) The weight of non-water in the watermelon is 100
× 20 = 5
kg. Let the present
weight of the watermelon be x kg.
The 52 kg non-water component now represents 5% = 1
20
of x. The value of x is therefore
2
5
× 20 = 8 kg

14. Soln: (B) Let OX = OY = r and let XY = d. Using Pythagoras’ Theorem, d2 =


r 2 + r 2 = 2r 2 . The areas of T, T + S and S + C are respectively

1 1 1 1 d2 πr 2
r · r = r 2 , πr 2 and · π =
2 2 4 2 4 4
It follows that
1 2
Area of T 2
r
= πr 2
Area of C 1

4
− 4
πr 2 − 12 r 2
Multiplying through by 4 and dividing by r 2 :
Area of T 2
= =1
Area of C π − (π − 2)

15. Soln: (C) Since the area of P RS is half that of P QR it has the same area as SRQ.
Because these triangles have the same area and the same altitude (from R to P Q), the
bases P S and SQ are equal and hence RS = SQ. It now follows that ∠QSR = 120◦
and ∠SRQ = ∠SQR = 30◦ . So ∠P RQ = 60◦ + 30◦ = 90◦ .
2022-2023 SMO (Questions) 3

16. Soln: (B) Suppose Joe works at x books per hour, then Barney works at 32 x per hour
and Adam works at 25 x per hour. For the times specified, the number of books carried is
   
5 3
3 x +4 x + 5x = 999
2 2
Multiplying by 2 :
15x + 12x + 10x = 1998
That is, 37x = 1998 and x = 1998
37
= 54. From this, Joe carries 5 × 54 = 270, Barney
carries 4 × 2 × 54 = 324 and Adam carries 3 × 52 × 54 = 405.
3

17. Soln: (E) We have

a b a2 + b2 − (ab)2 a2 + b2 − (a − b)2
+ − ab = =
b a ab ab
2 2 2 2
a + b − (a + b − 2ab) 2ab
= = =2
ab ab

18. Soln: (E) Let k and j be the number of students in Karen and Jacqui’s class. The total
number of apples shared is 6k + 6j. It follows that
6k + 6j 6j
=6+ = 10
k k
6j k 1 6k 36
From this = 4 and = or = = 9. The average for Jacqui’s class is
k 6j 4 j 4
6k + 6j 6k
=6+ = 6 + 9 = 15.
j j

19. Soln: (D) Since Dee is three times as old as Bea, Dee can either be 3 or 6 corresponding
to Bea being 1 or 2
A B C D E F G A B C D E F G
or
1 3 2 6

Because C=E+4, this means that C= 5, 6 or 7. The possibilities are now


A B C D E F G A B C D E F G A B C D E F G
or or
1 6 3 2 2 5 6 1 2 7 6 3

F>A and A>G means F>A>G


A B C D E F G A B C D E F G A B C D E F G
or or
5 1 6 3 2 7 4 4 2 5 6 1 7 3 4 2 7 6 3 5 1
2022-2023 SMO (Questions) 4

Since A+G>F, the only possibility is

A B C D E F G
5 1 6 3 2 7 4

So Aaa is 5.

20. Soln: (A)For x, y, z bags respectively, the average price per kilogram is
60x + 150y + 250z
= 40
x + 3y + 8z
So

60x + 150y + 250z = 40x + 120y + 320z


20x + 30y = 70z
2x + 3y = 7z

The smallest integer quantities correspond to x = 2, y = 1, z = 1. The smallest number


is 2 + 1 + 1 = 4

21. Soln: (B) Let the legs of the triangle be 2x. The length of the long side is therefore
q √ √
(2x) + (2x)2 = 8x2 = 2 2x. The area of square P is x2 = 45. The area of square R
2

is
√ !2
2 2x 8 8
= x2 = × 45 = 40
3 9 9

22. Soln: (C) The sum of the first 2 must be even, the sum of the first three must be
divisible by 3, the sum of the first 4 must be divisible by 4, the sum of the first 5 must be
divisible by 5. The first two numbers must be both even or both odd. On dividing the
given numbers by 3, the remainders are 2, 1, 2, 1, 1 and so the first three numbers must
be 76, 82 and 91 (91 is the third). The sum is 76 + 82 + 91 = 249. This is one more than
a multiple of 4 and so the fourth number must be 71 and the last 80.
√ √
2 2 3 3
23. Soln: (B) Because angle ADP = 30 and AD = 1, DP = √ =

and AP = .
3 √3 3
Because angle √ BDC√= 30 and BC = 1, DB = 2 and DC = 3. It follows that

√ 3 2 3
PB = 3 − = . The perimeter of ∆BDP is therefore
3 3
√ √ √
2 3 2 3 4 3
DP + P B + DB = + +2=2+ .
3 3 3
2022-2023 SMO (Questions) 5

24. Soln: (B) Let g be the area of the grey region, w the white part of the triangle and
p the area of the white part outside the triangle. We are given that
g 45 w 40
= and =
g+w+p 100 g+w+p 100
p 15 p
So = . We want . Taking g + w + p to be 100, the ratio of g : w : p
g+w+p 100 g+p
is 45 : 40 : 15 and
p 15 1
= = = 25%
g+p 45 + 15 4
25. Soln: (C) The prime numbers between 4 and 18 are 5, 7, 11, 13, 17. Let the chosen
prime numbers be x and y so that the number obtained is

xy − (x + y) = xy − x − y = (x − 1) (y − 1) − 1

Since the primes here are all odd, the required answer must be odd. The least value is
4 · 6 − 1 = 23 and the largest is 12 · 16 − 1 = 191. This eliminates all the answers but
119. Note that (11 − 1) (13 − 1) − 1 = 119.
2022-2023 Senior Mathematical Olympiad
Round Two Examination (Grades 9,10 and 11) - 11:00am

SECTION A

For each question, determine the letter corresponding to the correct or best response; along
with the question number, indicate this letter by shading it on the answer sheet

1. What is the smallest four-digit positive integer which has four different digits?
(A) 1032 (B) 2012 (C) 1021 (D) 1234 (E) 1023

2. In the following expression each  is to be replaced with either + or − in such a way


that the result of the calculation is 100.

123  45  67  89

Let p be the number of + signs used and m the number of − signs used. What is the
value of p − m?
(A) −3 (B) −1 (C) 0 (D) 1 (E) 3

3. How many two-digit numbers have remainder 1 when divided by 3 and remainder 2 when
divided by 4?
(A) 8 (B) 7 (C) 6 (D) 5 (E) 4

4. Which one of the following is equal to



916x2

for all values of x?


2 2 2
(A) 34x (B) 34x (C) 38x (D) 94x (E) 98x

5. After playing 500 games, Sarah’s success rate at Solitaire is 49%. If Sarah wins every
game from now on, how many more games does she need to play in order that her success
rate increases to 50%?
(A) 1 (B) 2 (C) 5 (D) 10 (E) 50
2022-2023 SMO (Questions) 2

6. P QRS is a quadrilateral inscribed in a circle of which P R is a diameter. The lengths of


P Q, QR and RS are 60, 25 and 52 respectively.
Q

60
25

P R

52

What is the length of SP ?


(A) 21 23 (B) 28 11
13
(C) 33 (D) 36 (E) 39

7. Peter wrote a list of all the prime numbers that could be produced by changing one digit
of the number 200. How many of the numbers in Peter’s list are prime?
(A) 0 (B) 1 (C) 2 (D) 3 (E) 4

8. Two externally tangent circles (Circles touching each other) with centers at points A and
B have radii of lengths 5 and 3, respectively. A line externally tangent to both circles
intersects ray AB at point C. What is the length of BC?
(A) 4 (B) 4.8 (C) 10.2 (D) 12 (E) 14.4

9. The parallel sides of a trapezium have lengths 2x and 2y respectively. The diagonals are
equal in length, and one diagonal makes an angle θ with the parallel sides as shown.

2x

θ
2y

What is the length of each diagonal?


x+y x+y
(A) x + y (B) (C) (x + y) cos θ (D) (x + y) tan θ (E)
sin θ cos θ
2022-2023 SMO (Questions) 3

10. The interior angles of a triangle are

(5x + 3y)◦ , (3x + 20)◦ and (10y + 30)◦

where x and y are positive integers. What is the value of x + y?


(A) 15 (B) 14 (C) 13 (D) 12 (E) 11

SECTION B

For each question, provide a complete solution by showing all your workings.

1. The diagram shows triangle ABC, in which ∠ABC = 72◦ and ∠CAB = 84◦ . The point
E lies on AB so that EC bisects ∠BCA. The point F lies on CA extended. The point
D lies on CB extended so that DA bisects ∠BAF .
D

E
C

A F

Prove that AD = CE.

2. Let P (n) and S(n) denote the product and the sum, respectively, of the digits of the
integer n. For example,
P (23) = 6 and S(23) = 5.
Suppose N is a two-digit number such that N = P (N) + S(N). Determine the units
digit of N?

3. The letters a, b, c, d, e and f and represent single digits and each letter represents a
different digit. They satisfy the following equations:

a + b = d, b + c = e and d + e = f.

One solution for the ordered set (a, b, c, d, e, f ) is (2, 1, 4, 3, 5, 8). Find all the other
solutions.
2022-2023 SMO (Questions) 4

4. Two overlapping triangles P OR and QOT are such that points P, Q, R and T lie on the
arc of a semicircle of centre O and diameter P Q, as shown in the diagram.

R
T
S

3x◦ 5x◦
P O Q

Lines QT and P R intersect at the point S. Angle TOP is 3x◦ and angle ROQ is 5x◦ .
In terms of x, what is the measure of ∠RSQ?

5. In a sequence, every term after the second is equal to the sum of the previous two terms.
Also, every term is a positive integer. The eighth term in the sequence is 400. What is
the maximum value of the third term in the sequence.
2022-2023 Senior Mathematical Olympiad
Round Two Examination (Grades 9,10 and 11) - Solutions

1. Soln: (E) To get the smallest possible number we must use the four smallest digits,
0, 1, 2 and 3. A four digit number cannot begin with a 0. So we must put the next smallest
digit, 1, in the thousands place. The hundreds digit must be the smallest remaining digit,
0. Similarly the tens digit must be 2 and the units digit must be 3. The required number
is 1023
2. Soln: (B) If x is the sum of all the positive numbers and y is the sum of all the
negative numbers then x + y = 100 and
x − y = 123 + 45 + 67 + 89 = 324. Adding these two equations 2x = 424 and x = 212
(which means y = −112). A look at the numbers suggest there are 2 negative numbers
−45 and −67. So p = 1 and m = 2. The value of p − m is −1.
3. Soln: (A) Let n be the two digit number. From the information n = 3k + 1 and
4p + 1
n = 4p + 2 where n and p are positive integers. So k = which are integers for
3
p = 2, 5, 8, 11, 14, 17, 20, 23 etc. For p = 2, n = 10 (the first) and for p = 23, n = 94 (the
last). The total is 8.
√ c
4. Soln: (E) Using the fact that a = a1/2 and ab = abc , we have
√  2 1/2
 1 2 2
916x2 = 916x = 9 2 ×16x = 98x

The others are not correct (Put x = 1 to illustrate)


49
5. Soln: (D) Presently Sarah wins 100
× 500 = 245 games. If she plays x more games to
get to 50%, then
245 + x 1
=
500 + x 2
This gives 2x + 490 = 500 + x or x = 10.
6. Soln: (E) Because P R is a diameter, the angles ∠P QR and ∠P SR are both right
angles. By Pythagoras’ Theorem
P R2 = P Q2 + QR2 and P R2 = P S 2 + RS 2 .
Hence P R2 = 602 + 252 = 4225. Also P S 2 = P R2 − RS 2 = 4225 − 522 = 1521 = 392 . So
P S = 39.
2022-2023 SMO (Solutions) 2

7. Soln: (A) If the first or second digit is changed then the number ends in 0 and so
will be divisible by 10 and hence will not be prime. If the last digit is changed to an
even number the resulting number will not be prime because 2 will be a factor. The
only choices left are 1, 3, 5, 7, 9. If the last digit is changed to 1 or 7 the number will
be divisible by 3 (because the sum of the digits is divisible by 3) and so will not be
prime. If the last digit is 5 the number will be divisible by 5 and will not be prime. Now
203 = 7 × 29 = and 209 = 11 × 19 and so they are not prime. So none are prime.
8. Soln: (D) Let D and E be the point of contacts of the tangent line with the larger
and smaller circles respectively. Triangles CBE and CAD are similar. If BC = x, then
AD BE
AC = 5 + 3 + x = 8 + x. From the similarity of the two triangles, = and so
AC BC
5 3
= .
8+x x
This gives 5x = 24 + 3x and so x = 12. The length of BC is x = 12.
9. Soln: (E) We label the vertices of the trapezium P, Q, R and S as shown, and we let
T, U be the points where the perpendiculars from P , Q, respectively meet the line RS.

P 2x Q

θ
S R
T 2y U

Since the diagonals SQ and P R are equal and P T = QU. So the right-angled triangles
QUS and P T R are congruent. So SU = T R and hence ST = UR. Now ST + UR =
SR − T U = SR − P Q = 2y − 2x. Hence ST = y − x and therefore SU = ST + T U =
SU
ST +P Q = (y−x)+2x = x+y. From the right-angled triangle QSU we have = cos θ.
SQ
SU x+y
Hence SQ = = .
cos θ cos θ
10. Soln: (A) The sum of the interior angles of a triangle is 180◦ and so

5x + 3y + 3x + 20 + 10y + 30 = 180
8x + 13y = 130

This gives 8x = 13 (10 − y) . This suggests that x = 13 and 10 − y = 8. So x = 13 and


y = 2 resulting in x + y = 15.
The three angles are 71◦ , 59◦ and 50◦ .
2022-2023 SMO (Solutions) 3

SECTION B

1. Solution
Filling in angles, ∠BCE = ∠ECA = 180−(72+84)
2
= 12◦ . Also, ∠ABD = 180 − 72 = 108◦
180−84
and ∠BED = 2 = 48◦ with ∠BDA = 180 − (48 + 108) = 24◦ . It is now evident
that ∆ACD is isosceles with AD = AC. ∠CEA = 180 − 96 = 84◦ and so ∆ACE is also
isosceles with AC = CE. So AD = CE.

2. Solution
Let N = uv be the two digit number. P (N) + S(N) = u + v + u · v.
N = 10u + v and so v is the units digit of N
Since N = P (N) + S(N),
10u + v = u + v + u · v
and so 10u = u + u · v and 9u = uv. This gives v = 9 (since u 6= 0) which is the units
digit of N.

3. Solution
None of the digits a, b, c, d, e and f may be 0 since this would force two of the others to
be equal. Also d ≥ 3, e ≥ 3 and 7 ≤ f ≤ 9.
Now f = 7 means that the d, e pair is {3, 4} . This is not possible because 4 = 2 + 2 or
4 = 1 + 3. So 8 ≤ f ≤ 9.
Case f = 8. The d, e pairs are{3, 5} .In this case, the (a, b, c) triplets are (2, 1, 4) and
(4, 1, 2) .
Case f = 9. The d, e pairs are{3, 6} and {4, 5} . For the d, e pair {3, 6}, the (a, b, c)
triplets are (2, 1, 5), (5, 1, 2) , (1, 2, 4) , (4, 2, 1) . For the d, e pair {4, 5}, the (a, b, c) triplets
are (1, 3, 2) and (2, 3, 1) .
The solutions are

(2, 1, 4, 3, 5, 8), (4, 1, 2, 5, 3, 8), (2, 1, 5, 3, 6, 9), (5, 1, 2, 6, 3, 9),


(1, 2, 4, 3, 6, 9), (4, 2, 1, 6, 3, 9), (1, 3, 2, 4, 5, 9), (2, 3, 1, 5, 4, 9).

Note that the equations imply a + 2b + c = f.

4. Solution
Since OP = OR are radii, triangle OP R is isosceles with ∠RP O = ∠P RO. From the
exterior angle theorem, they add to 5x◦ and so ∠RP O = ∠P RO = 52 x◦ . By a similar
argument, ∠T QO = 23 x◦ . Finally, by the exterior angle theorem, ∠RSQ = 52 x◦ + 32 x◦ =
4x◦ .
2022-2023 SMO (Solutions) 4

5. Solution
Let the first two terms in the sequence be a, b. The third to eighth terms are

a + b, a + 2b, 2a + 3b, 3a + 5b, 5a + 8b, 8a + 13b

and 8a + 13b = 400. From this 13b = 400 − 8a = 8 (50 − a) .


This suggests that 50 − a is a multiple of 13. This gives a = 50 − 13 = 37 or 50 − 26 =
8 (50 − a)
24 or 50 − 39 = 11. Using b = , we compute the following table:
13

a b a+b
37 8 45
24 16 40
11 24 35

The third term is a + b which has maximum value of 45.


2022-2023 Senior Mathematical Olympiad
Final Round Examination (Grades 9, 10 and 11)
Provide complete solutions to all 8 questions (2 hours)

1. A sum of money is being divided among Altiman, Britannie and Cecil. First, Altiman
receives $100 plus one-third of what is left. Britannie then receives $600 plus one-third
of what remains. Finally Cecil receives $4000 which is the remaining amount. What is
the total amount of money that is being shared?

2. Freddie has a number of square tiles, each measuring 1 cm by 1 cm. He tries to put these
small square tiles together to form a larger square of side length n cm, but finds that he
has 92 tiles left over. If he had increased the side length of the larger square to (n + 2)
cm, he would have been 100 tiles short of completing the larger square. How many tiles
does Freddie have?

3. What is the value or values of the digit k that makes the five-digit number

275k2

divisible by 12?

4. A function f is such that

f (x) − f (x − 1) = 4x − 9 and f (5) = 18.

(i) Show that f (3) = 0.


(ii) If f (x) = 2x2 + px + q, determine the values of p and q.

5. ABC is an equilateral triangle with sides of length 4 cm. Points P, Q and R are chosen
on sides AB, BC and CA, respectively, such that AP = BQ = CR = 1 cm. What is
the area of triangle P QR?

6. If wxyz is a four-digit positive integer with w 6= 0, the layer sum of this integer equals

wxyz + xyz + yz + z.

For example, the layer sum of 4089 is 4089 + 089 + 89 + 9 = 4276.


If the layer sum of wxyz equals 2024, what are the possible numbers for wxyz?

The questions continue on the flip side of this page ......


2022-2023 SMO (Solutions) 2

7. The sum of the squares of 5 consecutive positive integers is 1815. What are the five
consecutive numbers?

8. Determine the smallest positive integer N for which

x4 + 2023x2 + N

can be factored as (x2 + rx + s)(x2 + tx + u) where r, s, t, u are integers and r 6= 0.


2022-2023 Senior Mathematical Olympiad
Final Round Examination (Grades 9, 10 and 11) - Solutions

1. Soln: Let the amount being shared be $x. The three dollar shares are
1 1
Altiman : 100 + (x − 100) = (x + 200)
3 3 
1 1 2 3400
Britannie : 600 + x − (x + 200) − 600 = x +
3 3 9 9
Cecil : 4000
 
1 2 3400
Since Cecil’s amount is the remainder, it represents x − 3 (x + 200) + x + =
9 9
4 4000
x− . So
9 9
4 4000
x− = 4000
9 9
4x − 4000 = 36000. From this, 4x = 40000 giving x = 40000
4
= 10 000.
Alternative Solution: After Britannie receives $600, Cecil’s amount represent 23 of
what is left which is $4000. So Britannie’s amount is $600 + $2000 = $2600. Solving
2 3400
x+ = 2600
9 9
gives x = 10 000.

2. Soln: The area of the bigger square is n2 cm2 and so the number of tiles available is
T = n2 + 92. After the increase, the area of the bigger square is (n + 2)2 = n2 + 4n + 4
and T = n2 + 4n + 4 − 100 = n2 + 4n − 96. So

n2 + 92 = n2 + 4n − 96
188
This gives 4n = 188 or n = 4
= 47. The number of tiles available is T = 472 + 92 =
2301.

3. Soln: If the number is divisible by 12 it must be divisible by both 3 and 4. It is


divisible by 4 if the two digit number k2 is divisible by 4. The choices of k are 1, 3, 5, 7, 9.
If the number is divisible by 3 the sum of the digits must be divisible by 3. The sum of
the digits is 16 + k. The choices of k are 2, 5, 8. For the number to be divisible by 12, the
only possible value of k is 5 corresponding to 27552.
2022-2023 SMO (Solutions) 2

4. Soln:

(i) If we put x = 5 we get f (5) − f (4) = 20 − 9 = 11. So

f (4) = f (5) − 11 = 18 − 11 = 7

Similarly, f (4) − f (3) = 16 − 9 = 7. So

f (3) = f (4) − 7 = 7 − 7 = 0.

(ii) For f (x) = 2x2 + px + q, f (3) = 0 and f (4) = 7 implies

18 + 3p + q = 0
32 + 4p + q = 7

Subtracting, 14 + p = 7. This gives p = −7 (and q = 3).

5. Soln: It is easy to show that ∆RAP , ∆P BQ and ∆QCR are all congruent (side-
angle-side) and so P Q = QR = RP . Therefore ∆P QR is equilateral. Using the cosine
rule in ∆RAP , we can find P R:

P R2 = P A2 + AR2 − 2(P A)(AR)cos(60◦ )


P R2 = 12 + 32 − 2(1)(3) cos(60◦ )
= 12 + 32 − 3 = 7

So P R = 7 and the area of triangle P QR is
√ √
1 1√ √ 3 7 3
(P Q) (QR) sin 60 =

7· 7· = cm2
2 2 2 4

6. Soln: The layer sum of wxyz equals 2024 and so

1000w + 100x + 10y + z + 100x + 10y + z + 10y + z + z = 1000w + 200x + 30y + 4z

So 1000w + 200x + 30y + 4z = 2024


Now w cannot be 3 or greater and so, w = 1 or w = 2. If w = 2 then x = y = 0 and
4z = 24 giving z = 6. So one possible solution is 2006.
If w = 1 then 200x + 30y + 4z = 1024. Clearly z = 1 or z = 6. These give

200x + 30y = 1020 or 200x + 30y = 1000

That is,
20x + 3y = 102 or 20x + 3y = 100
2022-2023 SMO (Solutions) 3

Clearly x = 4 or x = 5. When x = 4, no solution exist and when x = 5, the second


equation gives y = 0. The number is therefore 1506. Here
1506 + 506 + 06 + 6 = 2024
The two solutions are 2006 and 1506.
7. Soln: Let the numbers be m − 2, m − 1, m.m + 1 and m + 2
(m − 2)2 + (m − 1)2 + m2 + (m + 1)2 + (m + 2)2 = 1815
and so
m2 − 4m + 4 + m2 − 2m + 1 + m2 + m2 + 2m + 1 + m2 + 4m + 4 = 1815
5m2 + 10 = 1815

This gives 5m2 = 1805 and so m2 = 361. It follows that m = 361 = 19. The integers
are 17, 18, 19, 20, 21.
8. Soln: Expanding,
(x2 + rx + s)(x2 + tx + u) = x4 + (t + r) x3 + (u + s + tr) x2 + (ur + st) x + su
Now
x4 + (t + r) x3 + (u + s + tr) x2 + (ur + st) x + su = x4 + 2023x2 + N
means that
t+r = 0
u + s + tr = 2023
ur + st = 0
and su = N
So t = −r and so
u + s − r 2 = 2023
ur − sr = r (u − s) = 0
and su = N
Because r 6= 0, u − s = 0 or u = s. Therefore
r 2 = 2u − 2023
and u2 = N
N is least when u is least. This occurs when r 2 = 1 corresponding with u = 1012. The
least N is u2 = 10122 = 1024 144.
Note: Here
(x2 + x + 1012)(x2 − x + 1012) = x4 + 2023x2 + 1024 144
2022-2023 Senior Mathematical Olympiad

Final Round Examination (Grades 7 and 8)


Provide complete solutions to all 7 questions (2 hours)

1. Find all pairs of positive integers (x, y) where the ratios x : 4 and 9 : y are equal?
2. A sum of money is being divided among Altiman, Britannie and Cecil. First, Altiman
receives $100 plus one-third of what is left. Britannie then receives $600 plus one-third
of what remains. Finally Cecil receives $4000 which is the remaining amount. What is
the total amount of money that is being shared?
3. The diagram shows a rectangle ABCD with AD = 10 cm and the area of the shaded
area is 100 cm2 .
D C

A B
What is the shortest distance between the two semicircles?
4. What is the value or values of the digit k that makes the five-digit number

275k2

divisible by 12?
5. An altitude (whose length is called height) of a triangle is the perpendicular drawn
from one vertex of the triangle to the opposite side.
The sides of a triangle have lengths 30 cm, 40 cm and 50 cm. Calculate the length of
the shortest altitude?
It may help to know that 302 + 402 = 502 .

The paper continues on the flip side of this page.....


2022-2023 SMO (Solutions) 2

6. Consider the numbers

w = 2129 × 381 × 5131 ,


x = 2127 × 381 × 5131 ,
y = 2126 × 382 × 5131 ,
and z = 2125 × 382 × 5132

Show the necessary work to demonstrate that x is smallest and w is largest.

7. Freddie has a number of square tiles, each measuring 1 cm by 1 cm. He tries to put these
small square tiles together to form a larger square of side length n cm, but finds that he
has 92 tiles left over. If he had increased the side length of the larger square to (n + 2)
cm, he would have been 100 tiles short of completing the larger square. How many tiles
does Freddie have?
2022-2023 Senior Mathematical Olympiad
Final Round Examination (Grades 7 and 8) - Solutions

1. Soln: If the ratios x : 4 and 9 : y are equal, then


x 9
=
4 y
which gives xy = 36. The pairs are

(1, 36) , (2, 18) , (3, 12) , (4, 9) , (6, 6) , (9, 4) , (12, 3) , (18, 2) , (36, 1)

2. Soln: Let the amount being shared be $x. The three dollar shares are
1 1
Altiman : 100 + (x − 100) = (x + 200)
3 3 
1 1 2 3400
Britannie : 600 + x − (x + 200) − 600 = x +
3 3 9 9
Cecil : 4000
 
1 2 3400
Since Cecil’s amount is the remainder, it represents x − 3 (x + 200) + x + =
9 9
4 4000
x− . So
9 9
4 4000
x− = 4000
9 9
4x − 4000 = 36000. From this, 4x = 40000 giving x = 40000
4
= 10 000.
Alternative Solution: After Britannie receives $600, Cecil’s amount represent 23 of
what is left which is $4000. So Britannie’s amount is $600 + $2000 = $2600. Solving
2 3400
x+ = 2600
9 9
gives x = 10 000.
3. Soln: The area of the complete circle is A = πr 2 = 25π. The area of the rectangle is
therefore 100 + 25π and the length of side AB is 100+25π
10
= 10 + 2.5π. The closest distance
between the two circles is then 10 + 2.5π − 10 = 2.5π.
4. Soln: If the number is divisible by 12 it must be divisible by both 3 and 4. It is
divisible by 4 if the two digit number k2 is divisible by 4. The choices of k are 1, 3, 5, 7, 9.
If the number is divisible by 3 the sum of the digits must be divisible by 3. The sum of
the digits is 16 + k. The choices of k are 2, 5, 8. For the number to be divisible by 12, the
only possible value of k is 5 corresponding to 27552.
2022-2023 SMO (Solutions) 2

5. Soln: Because 302 + 402 = 502 , the triangle is right angled with hypotenuse of length
50 cm. The area of the triangle is therefore 12 (30) (40) = 600 cm2 . If h is the shortest
altitude, then
1 1
Area = (base) (height) = (50) h = 25h
2 2
600
So 600 = 25h and h = 25 = 24 cm.

6. Soln: Let t = 2125 × 381 × 5131

24 2125 × 381 × 5131 = 16t,



w =
22 2125 × 381 × 5131 = 4t

x =
21 · 31 2125 × 381 × 5131 = 6t

y =
31 · 51 2125 × 381 × 5131 = 15t

and z =

This shows that x < y < z < w.

7. Soln: The area of the bigger square is n2 cm2 and so the number of tiles available is
T = n2 + 92. After the increase, the area of the bigger square is (n + 2)2 = n2 + 4n + 4
and T = n2 + 4n + 4 − 100 = n2 + 4n − 96. So

n2 + 92 = n2 + 4n − 96
188
This gives 4n = 188 or n = 4
= 47. The number of tiles available is T = 472 + 92 =
2301.
2022-2023 Senior Mathematical Olympiad
Round Two Examination (Grades 7 and 8) - 11:00am
SECTION A

For each question, determine the letter corresponding to the correct or best response; along
with the question number, indicate this letter by shading it on the answer sheet

1. What is half of 1.01?


(A) 5.5 (B) 0.55 (C) 0.505 (D) 0.5005 (E) 0.055

2. Tallyah plants 60 bulbs of tulip. When they flower, she notes that half are yellow; one
third of those which are not yellow are red; and one quarter of those which are neither
yellow nor red are pink. The remainder are white. What fraction of the tulips are white?
(A) 1/24 (B) 1/12 (C) 1/6 (D) 1/5 (E) 1/4

3. A small ink cartridge has enough ink to print 600 pages. Three small cartridges can print
as many pages as two medium cartridges. Three medium cartridges can print has many
pages as two large cartridges. How many pages can be printed using a large cartridge?
(A) 1200 (B) 1350 (C) 1800 (D) 2400 (E) 5400

4. In the rectangle P QRS, the ratio of ∠P SQ to ∠P QS is 1 : 5.


Q R

P S

What is the size of ∠QSR?


(A) 15◦ (B) 18◦ (C) 45◦ (D) 72◦ (E) 75◦

5. Alfred says his age is

50 years, 50 months, 50 weeks and 50 days old.

What age will Alfred be on his next birthday?


(A) 57 (B) 56 (C) 55 (D) 54 (E) 53
2022-2023 SMO (Questions) 2

6. After playing 500 games, Sarah’s success rate at Solitaire is 49%. If Sarah wins every
game from now on, how many more games does she need to play in order that her success
rate increases to 50%?
(A) 1 (B) 2 (C) 5 (D) 10 (E) 50
7. The diagram, which is not to scale, shows a square with side length 1, divided into four
rectangles whose areas are equal.

What is the length labelled x?


(A) 2/3 (B) 17/24 (C) 4/5 (D) 49/60 (E) 5/6
8. What is the smallest prime number that is equal to the sum of two prime numbers and
is also equal to the sum of three different prime numbers?
(A) 11 (B) 13 (C) 19 (D) 23 (E) 29
9. Shernette makes a nonstandard checkerboard that has 31 squares on each side. The
checkerboard has a black square in every corner and alternate red and black squares
along every row and column. How many black squares are there on such a checkerboard?
(A) 480 (B) 481 (C) 482 (D) 483 (E) 484
10. P QRS is a quadrilateral inscribed in a circle of which P R is a diameter. The lengths of
P Q, QR and RS are 60, 25 and 52 respectively.
Q

60
25

P R

52

What is the length of SP ?


(A) 21 23 (B) 28 11
13
(C) 33 (D) 36 (E) 39
2022-2023 SMO (Questions) 3

SECTION B

For each question, provide a complete solution by showing all your workings.

1. What is the units digit in the answer to the sum 32022 + 32023 ?

2. The Olympiad family went to a restaurant and bought two Pizzas, three Chillies and
four Burgers. They paid $5300 in total. The Mathematics family went to the same
restaurant and bought five of the same Pizzas, six of the same Chillies and seven of the
same Burgers. They paid $10700 in total. How much more does a Pizza cost than a
Burger?

3. The letters A, B and C stand for different, non-zero digits.

A B C
B C A
+ C A B
A B B C

Find all the possible solutions to the word-sum shown.

4. A regular octagon is formed by cutting an isosceles right triangle from each of the corners
of a square with sides of length 10. What is the length of each side of the octagon?

5. The letters a, b, c, d, e and f represent single digits and each letter represents a different
digit. They satisfy the following equations:

a + b = d, b + c = e and d + e = f.

One solution for the ordered set (a, b, c, d, e, f ) is (2, 1, 4, 3, 5, 8). Find all the other
solutions.
2022-2023 Senior Mathematical Olympiad
Round Two Examination (Grades 7 and 8) - Solutions

SECTION A
1 1
 1 1
1. Soln: (C) We want 2
1 + 100 = 2 + 200 = 0.5 + 0.005 = 0.505

2. Soln: (E) From the information given, 30 are not yellow and 13 (30) = 10 are red. So
20 are are neither yellow nor red. 34 of this which is 15 are white. The corresponding
fraction is 15
60
= 1/4.

3. Soln: (B) Let s, m, ℓ be the number of pages a small, median and large cartridge can
print. We have 3s = 2m and 3m = 2ℓ. From this, 9s = 6m and 6m = 4ℓ. Therefore
9s = 4ℓ and ℓ = 49 s = 94 × 600 = 1350.

4. Soln: (E) Let ∠P SQ = x◦ so that ∠P QS = 5x◦ . Therefore x◦ + 5x◦ = 6x◦ = 90◦ and
x = 15. Now ∠QSR = ∠P QS = 5x◦ = 5 × 15◦ = 75◦ .

5. Soln: (B) 50 = 12 × 4 + 2 and so 50 months is approximately 4 years and 2 months


(or 4 years and 8 weeks) 58 = 52 × 1 + 6 and so 58 weeks is approximately 1 year and 6
weeks (or 1 year and 42 days). So Alfred’s present age is approximately 50 + 4 + 1 = 55
years and 50 + 42 = 92 days. So on his next birthday, he will be 56 years old.
49
6. Soln: (D) Presently Sarah wins 100
× 500 = 245 games. If she plays x more games to
get to 50%, then
245 + x 1
=
500 + x 2
This gives 2x + 490 = 500 + x or x = 10.

7. Soln: (A) Let the length of the square be x + y = 1. The area of each of the rectangle
is therefore 41 and one side of the lowest rectangle is therefore 14 . The area of the other
rectangles are therefore 34 y and 38 x. Solving 83 x = 14 . This gives x = 2/3.

8. Soln: (C) If the sum of two primes is prime then one of the prime must be 2 because
odd + odd is even which is not prime. The two prime numbers must take the form
p, p + 2. Similarly, the three primes added must ALL be odd. 3 + 5 + 7 = 15 is not prime.
3 + 5 + 11 = 19 and 19 = 17 + 2.

9. Soln: (B) 16 rows (1, 3, . . . , 31) has 16 black squares and 15 black squares in the other
15 rows (2, 4, . . . , 30). The total number of black squares is therefore 162 + 152 = 256 +
225 = 481.
2022-2023 SMO (Solutions) 2

10. Soln: (E) Because P R is a diameter, the angles ∠P QR and ∠P SR are both right
angles. By Pythagoras’ Theorem

P R2 = P Q2 + QR2 and P R2 = P S 2 + RS 2 .

Hence P R2 = 602 + 252 = 4225. Also P S 2 = P R2 − RS 2 = 4225 − 522 = 1521 = 392 . So


P S = 39.

SECTION B

For each question, provide a complete solution by showing all your workings.

1. Solution
The units digit of powers of 3 are 3, 9, 7, 1, 3, 9, 7, 1, · · · For example, 37 = 2187. The
cycle length is 4. Now 2022 = 505 × 4 + 2 and so the 32020 ends in 1 and 32022 ends in 9
and 32023 ends in 7. Therefore the units digit of 32022 + 32023 is 6 (9 + 7 = 16).
2. Solution
Let the cost of a Pizza, Chilli and a Burger be p, c and b respectively. We have

2p + 3c + 4b = 5300
5p + 6c + 7b = 10700

Multiplying the first equation by 2 and subtracting from the second

4p + 6c + 8b = 10600
5p + 6c + 7b = 10700

gives p − b = 100. So p = 100 + b. The answer is $100.


3. Solution
From the right most column A + B = 10 (because C is the units). 1 is carried and so
1 + A + C = 10 or 1 + A + C = 20 (not possible).
So

A + B = 10
A+C = 9

Finally, the first column says A = 1. This results in B = 9 and C = 8.


The sum is 198 + 981 + 819 = 1998.
2022-2023 SMO (Solutions) 3

4. Solution
Let x represent the length of each side of the octagon, which is also the length of the
hypotenuse of each of the right triangles. If ℓ is the length of the legs of the right triangles
x
then ℓ2 + ℓ2 = x2 and so 2ℓ2 = x2 and ℓ = √ . The length of the square is
2
x √ 
2ℓ + x = 2 √ + x = x 2 − 1 = 10
2
From this, √
10 10 2−1 √ 
x= √ =√ ·√ = 10 2−1 .
2+1 2+1 2−1
5. Solution
None of the digits a, b, c, d, e and f may be 0 since this would force two of the others to
be equal. Also d ≥ 3, e ≥ 3 and 7 ≤ f ≤ 9.
Now f = 7 means that the d, e pair is {3, 4} . This is not possible because 4 = 2 + 2 or
4 = 1 + 3. So 8 ≤ f ≤ 9.
Case f = 8. The d, e pairs are{3, 5} .In this case, the (a, b, c) triplets are (2, 1, 4) and
(4, 1, 2) .
Case f = 9. The d, e pairs are{3, 6} and {4, 5} . For the d, e pair {3, 6}, the (a, b, c)
triplets are (2, 1, 5), (5, 1, 2) , (1, 2, 4) , (4, 2, 1) . For the d, e pair {4, 5}, the (a, b, c) triplets
are (1, 3, 2) and (2, 3, 1) .
The solutions are

(2, 1, 4, 3, 5, 8), (4, 1, 2, 5, 3, 8), (2, 1, 5, 3, 6, 9), (5, 1, 2, 6, 3, 9),


(1, 2, 4, 3, 6, 9), (4, 2, 1, 6, 3, 9), (1, 3, 2, 4, 5, 9), (2, 3, 1, 5, 4, 9).

Note that the equations imply a + 2b + c = f.


2022-2023 Senior Mathematical Olympiad
Round One Examination (Grade 7 and 8) - 11:00am

For each question, determine the letter corresponding to the correct or best response; along
with the question number, indicate this letter by shading it on the answer sheet

1. What is the value of


(20 + 22) ÷ (20 − 22)?
(A) −42 (B) −21 (C) −2 (D) 21 (E) 42
2. The statement below is true:
(63) + 4 − (2 − 1) = 5.
The symbol  between the 6 and the 3 is representing
(A) ÷ (B) × (C) + (D) − (E) None of these
3. The product of four of the digits
3, 4, 5, 6, 7
is 360. Which digit was not used?
(A) 3 (B) 4 (C) 5 (D) 6 (E) 7
4. Alex, Barry and Carl altogether are 15 years old. Alex and Barry together are 11 years
old. Barry and Carl together are 12 years old. How old is the oldest of the three?
(A) 4 (B) 5 (C) 6 (D) 7 (E) 8
5. Let J, M and O be distinct (different) positive integers. Given that JMO = 2022. What
is the largest possible value of J + M + O?
(In the above, JMO = J · M · O = J × M × O)
(A) 1014 (B) 342 (C) 343 (D) 2023 (E) 60
6. What is the value of x in the diagram below?

x◦

115◦ 125◦

(A) 50 (B) 55 (C) 60 (D) 65 (E) 70


2022-2023 SMO (Questions) 2

7. The following five cards are arranged to form the smallest 9 digit number.

4 8 31 59 107

Which card must be placed furthest on the right? The card marked
(A) 4 (B) 8 (C) 31 (D) 59 (E) 107

8. Starting at 0, Fogo the frog jumps on a number line. He starts with two big jumps and
then three small jumps landing on the numbers

3, 6, 7, 8, 9

He keeps repeating his jumps in the same way, over and over again. On which of the
following numbers will he land in the course of his jumps?
(A) 82 (B) 83 (C) 84 (D) 85 (E) 86

9. Marbles are sold in packages of 5, 10 or 25. Tom buys exactly 95 marbles. What is the
minimum number of packages Tom has to buy?
(A) 4 (B) 5 (C) 7 (D) 8 (E) 10

10. The diagram shows an octagon consisting of 10 unit squares. The shapes below P Q is a
unit square and a triangle with base 5.

X
Q
Y

If P Q divides the area of the octagon into two equal parts, what is the value of the ratio
XQ
?
QY
(A) 2/5 (B) 1/2 (C) 3/5 (D) 2/3 (E) 3/4
2022-2023 SMO (Questions) 3

11. ABCD is a big rectangle consisting of 7 congruent smaller rectangles (see diagram).

A B

D C
AB
What is the value of the ratio ?
BC
(A) 1/2 (B) 4/3 (C) 8/5 (D) 12/7 (E) 7/3

12. What is the degree measure of the smaller angle formed by the hands of an analogue
(circular) clock when the time is 2 o’clock?
(A) 30 (B) 45 (C) 60 (D) 75 (E) 90

13. How many integers between 100 and 300 have only odd digits?
(A) 25 (B) 50 (C) 75 (D) 100 (E) 150

14. The diagram shows the midpoints of both longer sides of a rectangle connected as vertices
of the shaded region.

What fraction of the rectangle is shaded?


(A) 1/5 (B) 1/4 (C) 2/7 (D) 1/3 (E) 2/5

15. A rectangular garden 50 m long and 10 m wide is enclosed by a fence. To make the
garden larger, while using the same fence, its shape is changed to a
square. By how many square metres does this enlarge the garden?
(A) 100 (B) 200 (C) 300 (D) 400 (E) 500

16. Ali, Boo, Cloe, Dre and Ezra have different amounts of money. Neither Dre nor Ali has
as much money as Cloe. Both Ali and Boo have more than Ezra. Dre has more than
Ezra, but less than Ali. Who has the least amount of money?
(A) Ali (B) Boo (C) Cloe (D) Dre (E) Ezra
2022-2023 SMO (Questions) 4

17. Charmaine pays an on-line service provider a fixed monthly fee plus an hourly charge
for connect time. Her December bill was $1248.00, but in January her bill was $1754.00
because she used twice as much connect time as in December. What is the fixed monthly
fee?
(A) $253.00 (B) $506.00 (C) $624.00 (D) $742.00 (E) $877.00

18. The quadrilateral ADCB is such that AB = 11 cm, BC = 7 cm, CD = 9 cm and


AD = 3 cm.
C

A B
If the angle at points A and C are 90◦ , in square centimetres, what is the area of the
quadrilateral?
(A) 30 (B) 44 (C) 48 (D) 52 (E) 60

19. Winnie has written some numbers on a piece of paper, the sum of which is 22. Rhianna
has then subtracted each number from 7 and has also written down the results. The sum
of Rhianna’s numbers is 34. How many numbers Winnie wrote down?
(A) 7 (B) 8 (C) 9 (D) 10 (E) 11

20. The square P QRS shown measures 12 m by 12 m

P 4m Q
4m

4m
S R
4m

What is the (total) area of the shaded region?


(A) 36 m2 (B) 40 m2 (C) 44 m2 (D) 46 m2 (E) 48 m2
2022-2023 SMO (Questions) 5

21. Each of the faces of 2 discs has a different whole number on it. The numbers on two of
the faces are shown.

5 6

If the discs are tossed, the possible sums of the numbers showing are 10, 11, 12 and 13.
What is the product of the two numbers that are on the other side of these two discs?
(A) 24 (B) 25 (C) 30 (D) 32 (E) 35

22. Bob has an awesome money changing machine. When he puts in a $100 bill, it returns
five $20 bills; when he puts in a $20 bills, it returns twenty $1 bills; and when he puts
in a $1 bill, it returns two $100 bills. Bob starts with just one $1 bill. Which of the
following amounts could Bob have after using the machine repeatedly?
(A) $363 (B) $513 (C) $630 (D) $745 (E) $996

23. There are two clocks, Clock 1 and Clock 2. Clock 1 is one minute fast every hour and
Clock 2 is two minutes slow every hour. Yesterday both clocks were set to the correct
time. Today, Clock 2 is reading 11 : 00 and Clock 1 is reading 12 : 00. What time
yesterday were the clocks set?
(A) 23 : 00 (B) 19 : 40 (C) 15 : 40 (D) 14 : 00 (E) 11 : 20

24. Tom’s dad is one year older than Tom’s mom and next year the product of Tom’s parents
ages will be over 1000 for the first time. What is the present age of Tom’s father?
(A) 24 (B) 25 (C) 30 (D) 32 (E) 35

25. A large watermelon weighs 20 kg, with 98% of its weight being water. It is left to stand
in the sun, and some of the water evaporates so that now only 95% of its weight is water.
What does it now weigh?
(A) 17 kg (B) 19.4 kg (C) 10 kg (D) 19 kg (E) 8 kg
2022-2023 Senior Mathematical Olympiad
Round One Examination (Grade 7 and 8) Solutions

42
1. Soln: (B) 20 + 22 = 42 and 20 − 22 = −2. The answer is = −21
−2
2. Soln: (A) We have (63)+4−1 = 5 or (63) = 2. Of the symbols given,  represents
the ÷ symbol.

3. Soln: (E) 360 = 3 · 120 = 3 · 4 · 30 = 3 · 4 · 5 · 6

4. Soln: (E) Alex and Barry together are 11 years old. This makes Carl age 15 − 11 = 4.
Barry and Carl together are 12 years old. This makes Alex age 15 − 12 = 3. Barry’s age
is therefore 15 − (4 + 3) = 8 years old.

5. Soln: (A) 2022 = 2 × 1011 × 1. This result in the largest value of J + M + O =


2 + 1011 + 1 = 1014

6. Soln: (C) Since angles on a straight line adds to 180◦ , the base angles of the triangle
are 65◦ and 55◦ . The sum of the angles in a triangle is 180◦ and so x = 180 − (65 + 55) =
60

7. Soln: (B) The number is smallest when the leading digits/cards are arranged smallest
to largest from left to right. In order, they are

107, 31, 4, 59, 8

8. Soln: (C) Fogo lands on the numbers

3, 6, 7, 8, 9, 12, 15, 16, 17, 18, . . .

It is now evident that he lands on all the positive integers that are divisible by 3. Of the
numbers 84 = 3 · 28 and so he lands on 84. Note that he land on

. . . 81, 84, 87, 88, 89, 90, . . .

9. Soln: (B) The minimum number of packages is bought when Tom buys 3 packages of
25 marbles and 2 packages of 10 marbles. This purchase results in exactly 95 marbles.
Any other purchase will result in more packages being bought.
2022-2023 SMO (Questions) 2

10. Soln: (D) Let QY = x. The area below P Q is the area of the triangle plus the area of
1 square. This must be equal to 5 squares. So
1
1+ · 5 (1 + x) = 5
2
8
This gives 2 + 5 (1 + x) = 10 and so 1 + x = 5
or x = 35 . Since QY = x = 35 , XQ =
XQ 2
1 − x = 52 and the ratio = .
QY 3
11. Soln: (D) Let the dimension of the smaller rectangle be x × y where x > y. From
the diagram, 4y = 3x or y = 34 x

AB 4y 3x 12x 12
= = 3 = =
BC x+y x + 4x 4x + 3x 7

12. Soln: (C) At 2 o’clock, the minute hand is at 12 and the hour hand is at 2. Each
1
unit difference on the clock represents 12 × 360 = 30 and so the angle represented is
1
2 × 12 × 360 = 60.

13. Soln: (A) There is only one choice for the first digit (namely 1). There are 5 choices
for the second digit (namely 1, 3, 5, 7, 9) and there are 5 choices for the third digit (namely
1, 3, 5, 7, 9). The total is 1 × 5 × 5 = 25

14. Soln: (B) Dividing the rectangle into 2 × 4 × 2 = 16 congruent triangles. The shaded
4
region represents 4 such triangles. The fraction is 16 = 1/4

15. Soln: (D) The original area is 50 m × 10 m = 500 m2 . The perimeter is 2 (50 + 10) =
120 m. Made into a square, each side will be of length 120
4
= 30 m and the resulting area
is 30 m × 30 m = 900 m2 . The increase is 400 m2 .

16. Soln: (E) From the second sentence, Cloe has more than someone so she can’t have
the least. From the third sentence both Ali and Boo have more than someone so that
eliminates them. And, from the fourth sentence, Dre has more than someone, so that
leaves Ezra with the least.

17. Soln: (D) Let f be the fixed monthly fee, h be the hourly rate and t, the total connect
time in December. We have

f + ht = 1248 and f + h · 2t = 1754

From this, 2f + 2ht = 2496 and f + 2ht = 1754. Subtracting, f = 2496 − 1754 = 742.
2022-2023 SMO (Questions) 3

18. Soln: (C) With the straight line DB, we construct two right angled triangles. The area
of the quadrilateral is now
1 1 1 1
(AB) (AD) + (CD) (CB) = (11) (3) + (9) (7) = 48
2 2 2 2
19. Soln: (B) If she wrote down n numbers then Rhianna’s sum is 34 and so
7 × n − 22 = 34
or 7n = 56. Dividing through by 7 gives n = 8
20. Soln: (E) The square P QRS may be thought of as comprising of 4 congruent kites
(we want the area of two) and 4 congruent triangles. The total area of the triangles is
4 × 21 × 4 × 6 = 48. The area of the 4 congruent kites is therefore 122 − 48 = 96. The
area of two of them is 21 × 96 = 48
21. Soln: (E) Let the outcomes on the first and second coin be (5, x) and (6, y) . The possible
sums are
5 + 6, 5 + y, x + 6 and x + y
x = 7 and y = 5 produces the sum of the outcomes as 11, 10, 13 and 12. The product is
7 × 5 = 35
22. Soln: (E) Bob’s money increases only when he puts in a 1$ and his net increase is
$199. Over time, the amount Bob has must take the form $1 + $199n where n is the
number of times a $1 is inserted. Of the answers, only 996 can be written in this form.
In fact, 996 = 1 + 199 × 5.
23. Soln: (C) In any one hour period the time differential between the two clocks is
1 + 2 = 3 minutes. Today the time differential is 1 hour or 60 minutes. This means that
60
= 20 hours elapsed. It now means that the true time on Clock 1 is 12 : 00 minus 20
3
mins or 11 : 40. Finally 20 hours before 11 : 40 is 35 : 40 − 20 : 00 = 15 : 40
24. Soln: (D) Let Tom’s dad present age be x. His mother’s age is x − 1. Next year, their
ages are x + 1 and x. So
x (x + 1) > 1000
Since 31 × 32 = 992 and 32 × 33 = 1056, x = 32.
2 2
25. Soln: (E) The weight of non-water in the watermelon is 100
× 20 = 5
kg. Let the present
weight of the watermelon be x kg.
The 52 kg non-water component now represents 5% = 1
20
of x. The value of x is therefore
2
5
× 20 = 8 kg
2022-2023 Junior Mathematical Olympiad
Final Round Examination (Grades 5 and 6)

For each question, determine the letter corresponding to the correct or best response; along
with the question number, indicate this letter by shading it on the answer sheet

1. The product
22 × 22020 × 2
is equal to
(A) 24040 (B) 22022 (C) 24041 (D) 22023 (E) 28080
2. The human heart beats an average of 70 times per minute. On average how many times
does it beat during one hour?
(A) 42, 000 (B) 7, 000 (C) 4, 200 (D) 700 (E) 420
3. How many of the integers
123, 234, 345, 456, 567
are multiples of 3?
(A) 1 (B) 2 (C) 3 (D) 4 (E) 5
4. Which of the following fractions below is the greatest (largest)?
7 66 555 4444 33333
(A) (B) (C) (D) (E)
8 77 666 5555 44444
5. Quadrilateral ABCD is a square and its side is 10 cm long. Quadrilateral AT MD is a
rectangle and its shorter side is 3 cm.
D M C

A T B
What is the difference between the sum of the lengths of all the sides of the square and
the sum of the lengths of all the sides of the rectangle?
(A) 14 cm (B) 10 cm (C) 7 cm (D) 6 cm (E) 4 cm
2022-2023 JMO (Questions) 2

6. In any order, Julie, Kassie, Zoe and Helen have their birthdays on March 1st, May 17th,
July 20th and March 20th. Kassie and Zoe were born in the same month. Julie and Zoe
were born on the same day of a month. Which of the girls was born on May 17th?
(A) Julie (B) Kassie (C) Zoe (D) Helen (E) Cannot say

7. All Mother Hubbard have to feed her children is one Giant chocolate bar. She gave each
of her children one-twelfth of the chocolate bar. One third of the bar was left. How many
children does Mother Hubbard have?
(A) 6 (B) 8 (C) 12 (D) 15 (E) 18

8. In the diagram below, 6 angles are marked.

a◦
b

f◦

d◦ c◦

e◦

What is the value of a + b + c + d + e + f ?


(A) 90 (B) 180 (C) 240 (D) 300 (E) 360

9. Peter Peters picked a peck of purple peppers. You are given that

1 peck = 1/4 bucket and 1bucket = 1/9 barrel.

How many more pecks of purple peppers must Peter Peters pick to fill a barrel?
(A) 12 (B) 13 (C) 34 (D) 35 (E) 36

10. The area of a certain rectangle is equal to 1 m2 . What is the area of a triangle that was
cut off from that rectangle along the line connecting the midpoints of the two adjacent
sides?
(A) 33 dm2 (B) 25 dm2 (C) 40 dm2 (D) 3, 750 cm2 (E) 1, 250 cm2

11. Diana is 3 years old and her mother is 28 years older than her. How many years later
will Diana’s mother be three times older than her?
(A) 9 (B) 12 (C) 10 (D) 1 (E) 11
2022-2023 JMO (Questions) 3

12. A square is divided into three congruent rectangles. The middle rectangle is removed
and placed on the side of the original square to form an octagon as shown.

Square Octagon

What is the ratio of the length of the perimeter of the square to the length of the
perimeter of the octagon?
(A) 3 : 5 (B) 2 : 3 (C) 5 : 8 (D) 1 : 2 (E) 1 : 1

13. One medal can be cut out from a golden square plate. If four medals are made from
four plates, the remaining parts of those four plates can be used to make one more plate.
What is the largest number of medals that could be formed when 16 plates are used?
(A) 17 (B) 19 (C) 20 (D) 21 (E) 32

14. The longest five-set tennis match in history lasted 11 hours and 5 minutes. The fifth set
of that match lasted 8 hours and 11 minutes. Approximately what fraction of the whole
match was taken up by the fifth set?
(A) 1/5 (B) 2/5 (C) 3/5 (D) 3/4 (E) 9/10

15. The students in Grade Five are holding an election with 4 candidates. A candidate
receiving more votes than the other 3 candidates wins. The four candidates receive a
total of 83 votes between them. What is the smallest number of votes the winner could
receive?
(A) 21 (B) 22 (C) 23 (D) 41 (E) 42

16. How many different three-digit numbers divisible by 25 can be made with the digits
0, 3, 5, 7 if the digits can be repeated?
(E) 16 (B) 9 (C) 81 (D) 64 (E) 3
2022-2023 JMO (Questions) 4

17. In the diagram below, the area of triangle ABD is equal to 15, the area of triangle ABC
is equal to 12 and the area of triangle ABE is equal to 4.

D C

A B

What is the area of pentagon ABCED?


(A) 19 (B) 31 (C) 23 (D) 27 (E) 35

18. Ida has 14 gray balls, 8 white balls and 6 black balls in a bag. What is the least number
of the balls she has to take out of her bag having her eyes closed to make sure that she
took at least one ball of each color?
(A) 23 (B) 22 (C) 21 (D) 15 (E) 9

19. A cyclist went up a hill with the speed of 12 km/h and went down the hill with the speed
of 20 km/h. The ride up the hill took him 16 minutes longer than the ride down the hill.
How many minutes did the cyclist take to go down the hill?
(A) 24 (B) 40 (C) 32 (D) 16 (E) 28

20. On July 1st in Norwich the sun will rise at 4 : 53 A.M. and set at 9 : 25 P.M. In the
middle of that period is local noon. At what time will the local noon be in Norwich
on July 1st?
(A) 12 : 00 P.M. (B) 12 : 39 P.M. (C) 1 : 09 P.M. (D) 4 : 32 P.M. (E) 11 : 08 A.M.

21. In one month three Sundays were on even dates. What day of the week was the 20th
day of the month?
(A) Monday (B) Tuesday (C) Wednesday (D) Thursday (E) Saturday

22. A computer virus destroys computer memory. On the first day it destroyed 21 of this
memory. On the second day it destroyed 13 of the memory remaining after the first day;
on the third day it destroyed 14 of the memory remaining after two days and on the
fourth day it destroyed 51 of the memory remaining after three days. What part of all
the computer memory was left after those four days?
(A) 1/5 (B) 1/6 (C) 1/10 (D) 1/12 (E) 1/24
2022-2023 JMO (Questions) 5

23. What is the greatest value of the sum of the digits of the number made from the sum of
the digits of a three-digit number?
(A) 9 (B) 10 (C) 11 (D) 12 (E) 18

24. In a chess competition 32 players were competing. The competition was taking place
by steps. In each step all the players were divided into groups of four. In each of these
groups every player played once with every other player. The two best players from the
group went to the next step and the two worst players were out of the competition. After
the step in which four last players played, the two best players played an additional final
game. How many games were played during the whole competition?
(A) 49 (B) 89 (C) 91 (D) 97 (E) 181

25. Symbols P, Q, R, S indicate the total weight of the figures drawn above them.

P Q

R S

It is known that any two figures of the same shape have the same weight. If P < Q < R,
then
(A) P < S < Q (B) Q < S < R (C) S < P (D) R < S (E) R = S
2022-2023 Junior Mathematical Olympiad
SOLUTIONS Final Round Examination (Grade 4)

1. Soln: (A) Since 2 × 0 = 0, the value of 2 × 0 × 2 × 3 is 0.

2. Soln: (C) Since 60 minutes are in an hour, the number of beats in an hour is 70 × 60 =
4, 200 times per hour.

3. Soln: (D) The number of children is the number of girls plus the number of boys. There
are 3 girls and 2 boys in this family. The total is 5.

4. Soln: (E) Of the numbers given, the sum of the hundreds and units digits are 3, 8, 8, 9
and 3. After tripling them, they are respectively 9, 24, 24, 27, 9. From this list 9 is the
only square and corresponds to the number 231.

5. Soln: (D) Because Kassie and Zoe were born in the same month, they were both born
in March. Because Julie and Zoe were born on the same day of a month, they were both
born on the 20th. This means that Helen (the only girl left) was born May 17th.

6. Soln: (D) 22 × 22020 × 2 = 22 × 22020 × 21 = 22+2020+1 = 22023

7. Soln: (A) The sum of the 4 sides of the square is 4 × 10 = 40 cm. The sum of the 4
sides of the rectangle is 3 + 3 + 10 + 10 = 26 cm. The difference in length is 40 cm − 26
cm = 14 cm

8. Soln: (E) After putting a melon on the scale with the oranges, the weights of one melon
cancel, leaving one melon weighing 6 oranges.
66 6 (11) 6 5 4 3
9. Soln: (A) = = . Similarly the other fractions are , and . The greatest
77 7 (11) 7 6 5 4
7
is .
8
10. Soln: (E) Since Diana is 3 years old and her mother is 28 years older than her, the
mother is presently 3 + 28 = 31 years. In x years, Diana will be 3 + x years old and her
mother will be 31 + x years old. If we solve

3 (3 + x) = 31 + x

we get 9 + 3x = 31 + x or 2x = 22 or x = 11.
2022-2023 JMO (Solutions) 2

11. Soln: (E) The number of choices he have is found from the multiplication rule for
counting: 2 × 3 × 4 = 24
12. Soln: (D) Since the remaining parts from using four plates can be used to make one
more plate, exactly 3 plates are used to make 4 medals. That is, one medal is made from
3/4 plate of gold. Now 16 = 3 × 5 + 1. Fifteen plates will make exactly 5 × 4 = 20 medals
and the extra plate (the 16th) will make 1 medal. The total is 20 + 1 = 21.
13. Soln: (E) Let x be the number of persons that gained MORE than Tony. The number
receiving less than Tony is therefore 2x. The total is therefore 2x + 1 + x = 28. This
gives x = 9. This means that x = 9 persons got more than Tony and so Tony finished
10th.
14. Soln: (D) The next number greater than 187569 consisting of different digits is 187590.
The difference is 187590 − 187569 = 21.
15. Soln: (C) The shaded area is the area of a large rectangle of size (5 + 6) × (5 + 3) = 88
MINUS the area of a smaller rectangle of size 5 × 6 = 30. The difference is 88 − 30 = 58.
16. Soln: (A) Let today be the day after Jake’s birthday. Since the day after tomorrow will
be Thursday, tomorrow will be Wednesday which means today is Tuesday and Jake’s
birthday was on a Monday.
17. Soln: (D) From the information given 4 pecks = 1 bucket and 9 buckets = 1 barrel.
Since 4 × 9 = 36 pecks = 9 buckets, 36 pecks = 1 barrel. Peter already picked one peck
an so he must pick an additional 35 pecks of peppers.
18. Soln: (C) In grams, the mass of the largest coin is 100 × 1000 = 100 000. Since a
Jamaican $1 coin has a mass of 10 g. The number of such coins that can be made is
100000
10
= 10 000 which has a value of $10 000.
19. Soln: (B) The greatest three-digit number with all different digits is 987 and the smallest
three-digit number with all different digits is 102. The difference is 987 − 102 = 885.
20. Soln: (C) If the length of the sides of the squares I, II, III and IV are ℓ1 , ℓ2 , ℓ3 and
ℓ4 respectively, then ℓ1 = 16
4
= 4 m, , ℓ2 = 24
4
= 6 m. Also ℓ3 = ℓ1 + ℓ2 = 10 m and
ℓ4 = ℓ2 + ℓ3 = 6 + 10 = 16 m. The perimeter is 4× 16 = 64 m.
21. Soln: (A) Only when 5 balls are in the bag can we guarantee that she took at least one
ball of each color. This would require her to draw at least 14 + 8 + 6 − 5 = 23 balls.
22. Soln: (C) The sum of the areas of the three unwanted triangles is
1 1 1
(4) (5) + (6) (2) + (2) (1) = 17
2 2 2
The area of the required triangle is 24 − 17 = 7.
2022-2023 JMO (Solutions) 3

23. Soln: (E) Let c and d be the locations of C and D. because AB = 2BC and BC = 2CD,

20 − 16 = 2 (c − 20) and c − 20 = 2 (d − c)

The first equation gives c − 20 = 2 and c = 22. The second equation gives d − c = 1
which means d = 23.

24. Soln: (C) If the rays are numbered from 1 (left-most) to 5 (right-most), then the angles
are formed form rays

(1, 2) , (1, 3) , (1, 4) , (1, 5) (2, 3) , (2, 4) , (2, 5) , (3, 4) , (3, 5) and (4, 5)

The total is 10. Of this 10, two pairs are equal (1, 3) with (3, 4) and (2, 4) with (4, 5).
The number with different angle measures is 8.

25. Soln: (D) Let A be the set of all the multiples of 3 and B the set of numbers ending in
3.
A = {3, 6, . . . , 99} and the number in A is 33. Also, B = {3, 13, 23, . . . , 93} and the
number in B is 10. The numbers common to A and B are 3, 33, 63 and 93 (4 in total).
The number of claps is 33 + 10 − 4 = 39.
THE UNIVERSITY OF THE WEST INDIES, MONA
The 2016 Jamaican Mathematical Olympiad

Test for Grades 7 and 8

NAME:

GRADE:

SCHOOL:

YEAR OF BIRTH:

STUDENT PHONE:

EXAMINATION QUESTIONS

1) Which of the following fractions has the smallest value?


1 2 3 4 5
a) b) c) d) e)
2 6 12 20 30

2) The figure on the right shows six equal squares placed side-by- ..........................................................................................................................................................................................................
................. ....
... ............... .... ... ........
......................
.
..... .................. ..... ...........
.. ............................... .. ................
side to form a rectangle. What fraction of the area of the rect- ......................... ..... ... ........................................................... .... ... .................................
............................ .. ... ..................................................... ... ... ..............................
...............................................................................................................................................................................................................................................................................................................................
angle is shaded?
1 1 1 2 2
a) b) c) d) e)
2 3 4 5 3

3) In a class with 16 boys and 10 girls, 75% of the boys and 50% of the girls registered for a trip to the
art museum. How many students registered for the trip?
a) 12 b) 15 c) 16 d) 17 e) 18

4) A pharmacist has to pour 3.2 litres of medicine into bottles that hold 80 millilitres each. How many
bottles will he fill?
a) 40 b) 80 c) 25 d) 400 e) 250

5) What is the value of the expression (1 − 2) − (3 − 4) − (5 − 6) − (7 − 8) − (9 − 10) − (11 − 12)?


a) −6 b) 0 c) 4 d) 6 e) 13

6) In the figure below, some of the angles have the values shown. What is the value of x?
......
......
......
...... ◦
80 ◦............................. .
... ..........
x
..
..... ... .........
... ..
...
... ... ............
... ... ......
......
.... ...
... ......
.... ....
......
......
.... .... ......
.... ....
......
......
..... . ......
...
..... ◦ .
... ◦ ......
......
.... 50 .
. 125 ......
.......................................................................................................................................................................

a) 55◦ b) 32.5◦ c) 5◦ d) 20◦ e) 25◦


7) Mr. Williams asked his five students how many of them were ready for their mathematics test. Al said
none of them, Beth said one of them, Chris said two of them, Don said three of them, and Earl said
four of them. Mr. Williams knows that students always lie if they are not ready for a test and always
tell the truth if they are. How many students were ready for the test?
a) 0 b) 1 c) 2 d) 3 e) 4

8) A palindrome is a positive integer which gives the same number when its digits are reversed. For
example, 6556 is a palindrome. What is the smallest number that can be added to 2002 to give another
palindrome?
a) 11 b) 110 c) 108 d) 18 e) 1001
..............................................
9) The figure on the right shows three squares of side length 1. The square on top is centred ...............................................
... .......................
... ...................
over the two squares below it. What is the area of the shaded region? .. ...............
.............................................................................
... ... ..................
... ... ................
3 7 1 1 ...
...
...
..
.........
.......
a) 1 b) c) d) 1 e) 1 .......................................................
...
4 8 4 2

10) In a set of five numbers, the average of two of them is 12 and the average of the other three is 7. What
is the average of all five numbers?
1 1 3 1
a) 8 b) 8 c) 8 d) 9 e) 9
3 2 4 2

11) The figure below shows a parallelogram that has been divided into 6 equilateral triangles. The sides of
the triangles have been coloured in such a way that each triangle has one red, one green, and one blue
side. The sides marked “b” below are blue, and those marked “g” are green. What colour is the side
marked “x”?
g g
........................................................................................................................................................................
... ... . . . . .
... ..... ... ..... ... ..... ...
... ... ... ... ... ... ...
... ... .. ... .. ... ..
.
... ... ..... ... .
.... ... ....
.
.. ... . .... . ... .
... ... ... ... ... ... ...
... ... .... ... ..... ... ....
... ... ...
. ... .. ... ...
.
..................................................................................................................................................................

b x b
a) red b) blue c) green
d) blue or green, but not red e) red, blue, or green

12) The figure below shows three semicircles with the same centre. Their radii are 1, 2, and 4, respectively.
The two regions marked x have equal area, and the three regions marked y have equal area. What is
the ratio x : y?
.......................................
............. .........
......... .......
....... ........
..
............. ... ..........
..
.
....
... .
... ...
.... y ..
.
.... .....
....
...
... ... ... ...
...
....
. .. ...
. ...
... ... ......... .. .
. .
. .
. .. ...
. .............. ..
. . ................ ...
... .. .
. ...
.
.
.
.
.. y .... .......
.
..
.
.
.
.
......
.
....
.
y ...
...
...
..
.
.... ..
.
.
... x ...
.
............. .........
.
......
. .
x ...
....
.
...
..
... .... .... ...
....
...
..
..
..
.. ... .... . ..
.......................................................................................................................................................................

a) 1 : 3 b) 1 : 2 c) 2 : 3 d) 3 : 8 e) 4 : 9

13) In a ring-toss game at a carnival, a ring on peg A scores 1 point, one on peg B scores 3 points, and one
on peg C scores 5 points. If all three rings land on pegs, how many different point totals are possible?
(Two or more rings can land on the same peg.)
a) 12 b) 7 c) 10 d) 13 e) 6
14) There are positive integers w, x, and y such that
1 97
w+ = .
1 19
x+
y
What is the value of w + x + y?
a) 16 b) 17 c) 18 d) 19 e) 26

........................
15) When the net shown on the right is folded into a cube, three numbered faces will meet ...
...
...
.... 5 ...
..
at each corner. If Janice multiplies all three numbers on the faces that meet at one of ........................
... ...
... ...
these corners, what is the largest product she could get? ... 6 ...
.
.....................................................................
... ... ... ...
... ... ... ...
a) 144 b) 168 c) 240 d) 280 e) 336 .... 7 3
.... 2 ... ...
.................................................................
... ...
...
...
8 ... ...
.......................

16) In the figure below, eight isosceles right triangles have been arranged to form two regular octagons. One
octagon is the outer boundary of the shaded region and the other is its inner boundary. If the outer
octagon has side length 1, what is the side length of the inner octagon?
.............................................................................
...............................................
.....................................................................
...
..........................................................................................................
...........................................................
...................................................................
....................................................................................................................................................................................
............................................... ............................................
........................................
................................ ...........................
................................... . .
........................................... .........................................................
..................................... ..... . . . . . . ...
............................................. ................................................
.. ....................
........................................................................................................................................................................
...................................................................
..................................................................
..................................................................................
...........................................................................
................................................
............................................................
...........................................................

1√ 1 √ √ √
a) 2 b) c) 2 − 2 d) 2−1 e) 3
2 2

17) Which expression below is equal in value to 42015 + 42015 + 42015 + 42015 ?
a) 42016 b) 162015 c) 48064 d) 168064 e) (42016 )4

18) Neither of the positive integers a and b are divisible by 10. If a × b = 10, 000, what is the value of a + b?
a) 1024 b) 641 c) 1258 d) 2401 e) 1000

19) In rectangle ABCD below, AB = 5 and AD = 2. A ball is rolled from point A to side DC on a path
making an angle of 45◦ with side AB. At side DC, and every time it reaches another side, it bounces
off the side at a 45◦ angle. It continues until it reaches D. How many times will the ball bounce as it
moves from A to D?
5
A ......................................................................................................................... B
... ...
... ...
... ...
... ...
2 ...
...
...
...
...
...
... ....
..........................................................................................................................
D C

a) 9 b) 8 c) 7 d) 5 e) 4

20) If m is an even integer and n is odd, which of the following represents an odd integer?
a) 3m + 4n b) 5mn c) 5m + 6n d) m3 n3 e) (m + 3n)2
21) If Al gave Ben 6 of his marbles, they would have the same number of marbles. If Al gave Ben half of his
marbles, Ben would have 8 marbles more than Al. How many marbles do the two boys have together?
a) 20 b) 12 c) 28 d) 24 e) 36

22) Sherika wants to place a positive integer in each of the nine boxes below so that the product of the
integers in each row, column, and diagonal are equal. She has already placed numbers in three of the
boxes. How many values of N can she choose that will allow her to complete the square?
......................................................................
... ... ... ...
...
..N ...
..
...
.. 24 ...
.
.......................................................................
.... .... .... ...
...
...
... 12 ...
..
...
.. ...
.
........................................................................
... ... ... ...
6 ...
...
...
...
...
...
...
..
..............................................................

a) 7 b) 15 c) 9 d) 6 e) 12

23) Let √ √ √ √ √ √ √ √
2− 1 3− 2 4− 3 9− 8
S= √ + √ + √ + ···+ √ .
2 6 12 72

Which of the expressions below is equal in value to S?


1 1 3 1 2
a) √ b) c) d) e)
2 2 4 3 3

24) In the figure below, points A, B, O, and C are collinear, BC is the diameter of a semicircle with
centre O, and line AD intersects the semicircle at D and E. If AE = OC and 6 DOC = 45◦ , what is
the measure of 6 DAO?
................................................
........ .......
....... ......
..
....... .....
...
.
....
.
....
.... D
........................
.
. . .
. ....
.... ........
. .
.......... .
.. .. ...
...
... .................... ..... ...
.
.. ............. ... ...
E ..........................
.
. ...... ...
.............. ... ..
.
........ ..
. ...
.............. .
. .. ◦ ..
........................ .
.
.
.
.
... ..
. 45 .
. .
....................................................................................................................................................................................................
A B O C

a) 10◦ b) 15◦ c) 20◦ d) 22.5◦ e) 25◦

25) The number 2004 is divisible by 12, and the sum of its digits is 6. All together, how many four-digit
numbers are divisible by 12 and have 6 as the sum of their digits?
a) 10 b) 12 c) 13 d) 15 e) 18

END OF QUESTIONS

You may mail your completed question paper to:

Mathematical Olympiad
P.O. Box 94
Mona Post Office
Kingston 7

You may also deliver your entry by hand or by courier directly to the Department of Mathematics at the
UWI, Mona Campus. In all cases, an entry must be received by December 7, 2015, in order to be considered.
THE UNIVERSITY OF THE WEST INDIES, MONA
The 2016 Jamaican Mathematical Olympiad
Solutions for Grades 7 and 8
1. The fraction 1/2 is in lowest terms. Regarding the other fractions, 2/6 = 1/3, 3/12 = 1/4, 4/20 = 1/5,
and 5/30 = 1/6. The fraction with the smallest value is 5/30 = 1/6.

2. A diagonal of a square divides it into 2 equal triangles. If each of the 6 squares were divided by a diagonal
line, there would be 12 equal triangles in all. Since 4 of them are shaded, 4/12 = 1/3 of the rectangle’s area
is shaded.
................................................................................................................................................................................................................
........... .. ..... ... ....... .. ... .. ........
................ ... ..... .. ................
................. ... .... ... ...............
...................... .. .... . ........................ . .... ... .............
................................ .... ... .... ........................................................... .... ...... . ................................
.
.... .. .................................... .. ... .... ...............................
................................ ... ... .. ........................................ . ... .. .. . . . . .
......................................................................................................................................................................................................................................................................................

3. Since 75% of the boys registered for the trip, 12 boys registered. Since 50% of the girls registered for the
trip, 5 girls registered. Then 12 + 5 = 17 students registered for the trip.

4. The pharmacist has 3,200 millilitres of medicine. If he pours it into bottles holding 80 millilitres each, he
will fill 40 bottles.

5. We have

(1 − 2) − (3 − 4) − (5 − 6) − (7 − 8) − (9 − 10) − (11 − 12)


= −1 − (−1) − (−1) − (−1) − (−1) − (−1) = −1 + 1 + 1 + 1 + 1 + 1 = 4

6. Label the vertices of one of the triangles A, B, and C, as shown below. The measure of 6 ABC is 55◦ .
This is because 6 ABC is the supplement of an angle measuring 125◦ . The measure of 6 CAB
......
......
......
...... A ◦
◦............................ .
80 x
..... ... ..........
... .........
.... .. ....
......
.... ...
... ......
... ... ......
... ......
.... ...
... ......
......
.... .... ......
.... ....
......
......
.... .... ......
....
. . ......
......
.. ◦ ... ◦ ......
...... 50 ....
.
.
.... 125
...................................................................................................................................................................
C
B

is 75◦ . This is because the sum of the angles in triangle ABC is 180◦ . Finally, the angle marked 80◦ , angle
CAB, and the angle marked x◦ together make a straight angle. Thus 80◦ + 75◦ + x◦ = 180◦ . Solving for x
gives x = 25.

7. Since any 2 students gave different answers, they could not both be telling the truth. So only 1 student
told the truth, or else nobody did. If nobody told the truth then nobody was ready for the exam. In this
case, Al would have told the truth. Since this is a contradiction, it is impossible that nobody told the truth.
Exactly 1 student told the truth (and this student was Beth). Then 1 student was ready for the test.

8. The next palindrome after 2002 is 2112. So the smallest number that can be added to 2002 to give another
palindrome is 110.
9. The combined area of the shaded and unshaded regions is 3 The unshaded region ...............................................
... .................................
consists of a large right triangle and a small rectangle immediately to its left. The base ... ........................
... ....................
............
.. . ..
of the triangle is 3/2 and its height is 2. Its area is (1/2)(3/2)(2) = 3/2. The rectangle ............................................................................
... ... ... .....................
... ... ... .................
..........
has base 1/2 and height 1. Its area is 1/2. The total area of the unshaded region is ...
...
...
..
...
..
.........
........
..........................................................
(1/2) + (3/2) = 2. The area of the shaded region is 3 − 2 = 1.

10. Let x1 and x2 be the two numbers whose average is 12. Then (x1 + x2 )/2 = 12. Multiplying both sides
by 2 gives x1 + x2 = 24. Let x3 , x4 , and x5 be the numbers whose average is 7. Then (x3 + x4 + x5 )/3 = 7.
Multiplying both sides by 3 gives x3 + x4 + x5 = 21. The average of all five numbers is

x1 + x2 + x3 + x4 + x5 (x1 + x2 ) + (x3 + x4 + x5 ) 24 + 21 45
= = = = 9.
5 5 5 5

11. Let the vertices of the triangles be labeled A, B, . . . , H, as shown in the figure below. If BC is blue
then triangle ABC would have two blue sides. If BC is green then triangle BCD would have two

B..........................g D F g
.................................................................................................................................... H
.... .... .... ..
... ... ... ... ... ... ...
... ..... ... ..... ... ..... ...
... ... ... ... ... ... ...
.
..
..
r
...
... b .
...
...
r
...
... g .
...
...
r
...
... .
...
...
... ...
...
...
...
...
...
... ...
... .....
...
... .....
...
... .....
...
... ... .. ...... ... ..
..............................................................................................................................................................
A
b C x E b G

green sides. So BC is red. Since BD is green, CD must be blue. Also, F G must be red. (If it were green
then F HG would have two green sides, and if it were blue then F GE would have two blue sides.) Since EG
is blue, EF must be green. If side DE is green then triangle DF E would have two green sides. If DE is
blue then triangle DEC would have two blue sides. So DE is red. Finally, triangle DEC has a blue side, a
red side, and a side marked x. The side marked x is green.

12. The concentric semicircles have radii 1, 2, and 4, respectively. The full circles that contain them have
areas π(12 ) = π, π(22 ) = 4π, and π(42 ) = 16π, respectively. The area of the middle band, containing
.....................................................
.......... ........
........ .......
...
............. ... ...........
..
..
.....
....
...
...
.
...
y .....
.. .....
....
....
... ... ..
....
. .
. .... ...
...
... ... ... ...
. . ...............
............... .. ............ .. . ...
... . . . ... ...
.... ......
.
.
.
.
.. y ........
.. .
.
.
.
.
.
......
....
y ...
...
..
. . ... x ......
.
. .. x . ...
.... ..
. ..
......... ............ ...
...
..
.. ... ... ..
... . .. .... . ...
... ... .... .
... ..
..........................................................................................................................................................................

the two regions marked with x and two other regions equal in size and shape, is 4π − π = 3π. Each region
marked with x has one-fourth of this area. So x = 3π/4. The area of the outer band, containing the three
regions marked with y and three regions equal in size and shape, is 16π − 4π = 12π. Each region marked
with y has one-sixth of this area. So y = 2π. Then x : y = 3π/4 : 2π = 3π : 8π = 3 : 8.

13. Since all scores for the pegs are odd numbers, the total score for three rings will be an odd number. The
least possible score will be 3 points, occuring when all three rings land on Peg A. The greatest possible score
is 15, occuring when all three rings land on Peg C. Any odd number between 3 and 15 is also possible. For
example, to generate scores of 5, 7, 9, 11, and 13, the rings could land on pegs A, A, B, A, A, C, A, B, C,
A, C, C, and B, C, C, respectively. The possible point totals are 3, 5, 7, 9, 11, 13, and 15. There are 7 such
totals in all.
14. First,
1 1 y 97 2
w+ =w+ =w+ = =5
1 xy + 1 xy + 1 19 19
x+
y y
Also, xy + 1 > y because x is a positive integer and hence at least 1. Thus y/(xy + 1) is a fraction between 0
and 1. Since w is a positive integer, we must have w = 5 and y/(xy + 1) = 2/19. One can prove that y = 2
and xy + 1 = 19. Substituting for y gives x = 9. Thus w = 5, x = 9, and y = 2, and w + x + y = 16.

15. When the net is folded into a cube, the numbers 6 and 8 will be on opposite sides. ....
...
.....................
...
...
So it is not possible for Janice to obtain the product 8 × 7 × 6 = 336. However, the sides ... 5
.........................
..
... ...
...
...
with 8, 7, and 5 will meet at a corner. The largest product Janice can get is ... 6 ...
.
.................................................................
.
.
.... .. ... ...
8 × 7 × 5 = 280. ...
... 7 ...
3
... 2 ...
..
...
..
......................................................................
... ...
... ....
8 ...
........................
...

16. Figure 16(b) below shows an enlarged view of two adjacent triangles from the octagon. The side length
of the interior octagon is equal to DC. In turn, this is equal to AC − AD. By the Pythagorean theorem,

........................................................ B
........................................... ....
................................................................... ..........
..................................................... ....................
................................................................................................................ ....................
.
............................................................. ................................................
...................................................................... .
.
..............................
.......................................................................................................................................................................... .....................................................
............................................ ............................... ..........................................
..................................
....................................... .............................................
..................................... ....................................................................................................................................
...................................
..........................................
.
...............................................
. A .....................................
...............................................
C
........................................ .
......................
..........................................
. . ........................................................ D
............................................... . . .
.........................
. .........................................
................................................................................................................................................................................ .................................
.... ............................................................. ....................................
..............................................................
................................................................
. . . . . .. . . . . . . .. . . . .............................
...................................................... .................
................................................................. .............
..................................... ...
............................................................ ........

Figure 16(a) Figure 16(b)

(AC)2 √
= (AB)2 + (BC)2 = 12 + 12 = 2. (Note that AB is the measure of a side√of the outer octagon.) Then
AC = 2. Length AD is also 1 because AD = AB. Then DC = AC − AD = 2 − 1.


17. We have 42015 + 42015 + 42015 + 42015 = 4 42015 = 41 42015 = 42016 .

18. We have a × b = 10, 000 = 104 = 24 × 54 . Since a is a factor of 24 × 54 , its prime factorization is of
the form 2p × 5q , where 0 ≤ p, q ≤ 4. It is not possible that p = 0 and q = 0. Otherwise, a = 1 and so
b = 10, 000. But then b is divisible by 10. So either p ≥ 1 or q ≥ 1. If both p ≥ 1 and q ≥ 1, then a would
be a multiple of 10. Since this is not so, either p ≥ 1 and q = 0, or q ≥ 1 and p = 0. In other words, a is
either a power of 2 or a power of 5. Reasoning in the same way, b is either a power of 2 or a power of 5.
Since a × b = 24 × 54 , either a = 24 and b = 54 , or vice-versa. In either case, a + b = 16 + 625 = 641.

19. As shown below, AB = DC = 5 and AD = BC = 2. Starting from A, the ball will bounce 5 times
before it reaches D.
A .....................................................................................................................................................
... ..... . B
.... ...... .... ....... ... ....
... ..... .... ..... .... ................. ....
... .... .... ............ .............
... .... ...... .
. .... ........ .... ..
........ . .... ... . ......
........ . .. ......
... . ............ ... .........
. .....
.
............. .............. .... .... .. ...... ....
... .. ..... .. . . .... . .
............. ....
... ....... .... .... ..... .
.
... .... ... ...... ... ...... ...
.
.....................................................................................................................................................
D C
20. Since m is even, 3m is even. Also, 4n is always even. So, 3m + 4n is even. Since m is even, 5mn is even.
Since m is even, 5m is even and 6n will always be even. So, 5m + 6n is even. Since m is even, m3 is even
and m3 n3 is even. Finally, since n is odd, 3n is odd. Since m is even, m + 3n is odd. Since an odd times an
odd is odd, (m + 3n)2 is odd. Then (m + 3n)2 is the only odd expression.

21. Let a and b be the number of marbles that Al and Ben have, respectively. If Al gave Ben 6 of his marbles,
Al would have a − 6 marbles and Ben would have b + 6 marbles. Since they would have the same number
of marbles, a − 6 = b + 6. Adding 6 to both sides gives a = b + 12. If Al gave Ben half of his marbles, Al
would have a/2 marbles and Ben would have b + a/2 marbles. Since Ben would have 8 marbles more than
Al, b + a/2 = a/2 + 8. Subtracting a/2 from both sides gives b = 8. Then a = 8 + 12 = 20. Together, Al
and Ben have 20 + 8 = 28 marbles.

......................................................................
22. Let M be the number in the corner opposite to N . In the diagonal containing ... ... ... ...
...
. N ...
.
...
. 24 ...
.
6, 12, and 24, the product of its entries is 1728. So the product of the entries in ........................................................................
...
.... .... .... ....
any row, column, or diagonal is 1728. In particular, 12M N = 1728. Dividing both ... 12 ... ...
......................................................................
.. ... ... ...
... ... ... ...
sides by 12 gives M N = 144. Thus 6 ... .. M ..
........................................................................
...

N ∈ {1, 2, 3, 4, 6, 8, 9, 12, 16, 18, 24, 36, 48, 72, 144}.


However, if N = 144 or N = 1 then the box cannot be completed. If N = 144 then the product of the
numbers in the top row would be at least 3456. If N = 1 then M = 144, and the product of the numbers in
the third column would be least 3456. Also, if N = 48 or N = 3 the box cannot be completed. If N = 48
then the number in the middle of the top row would be 1.5. But all numbers in the small boxes must be
integers. Similarly, if N = 3 then M = 48, and the number in the middle of the third column would be 1.5.
Finally, if N = 16 or N = 9 the box cannot be completed. If N = 16 then the number in the middle of the
top row would be 4.5. Similarly, if N = 9 then M = 16, and the number in the middle of the third column
would be 4.5. So we have
N ∈ {2, 4, 6, 8, 12, 18, 24, 36, 72}.
Furthermore, if Sherika uses any of these numbers then the box can be successfully completed. The cases
with N = 2, 4, 6, 8, and 12 are shown in Figures 21(a)-21(e) below. The cases for N = 18, 24, 36, and 72
may also be obtained from the first four boxes. For example, the case with N = 72 may be obtained by
transposing the box in Figure 21(a) about the diagonal with entries 24, 12, and 6. In summary, Sherika has
9 choices for N that allow her to complete the box.

...................................................................... ...................................................................... ...................................................................... ...................................................................... ......................................................................


... ... ... ... ... ... ... ... ... ... ... ... ... ... ... ... ... ... ... ...
...
. 2 36 24 ...
.
...
.
...
.
...
. 4 18 24 ...
.
...
.
...
.
...
. 6 12 24 ...
.
...
.
...
.
...
. 8 9 24
...
.
...
.
...
. 12 6 24
...
.
...
.
...
.
...
.
....................................................................... ....................................................................... ....................................................................... ....................................................................... .......................................................................
... ... ... ... ...
.... .... .... .... .... .... .... .... .... .... .... .... .... .... .... .... .... .... .... ....
144 12 1... ... ...
......................................................................
72 12 2
... ... ...
......................................................................
48 12 3
... ... ...
......................................................................
36 12 4
... ... ...
......................................................................
24 12 6
... ... ...
......................................................................
.. ... ... ... .. ... ... ... .. ... ... ... .. ... ... ... .. ... ... ...
... ... ... ... ... ... ... ... ... ... ... ... ... ... ... ... ... ... ... ...
6 ... 4 72 .. ..
........................................................................
... 6 ... 8 36 .. ..
........................................................................
... 6 12 24
... .. ..
........................................................................
... 6 16 18
... .. ..
........................................................................
... 6 24 12
... .. ..
........................................................................
...

Figure 21(a) Figure 21(b) Figure 21(c Figure 21(d) Figure 21(e)
23. We have
√ √ √ √ √ √ √ √
2− 1 3− 2 4− 3 9− 8
S= √ + √ + √ +···+ √
2 6 12 72
√ √  √ √  √ √  √ √ 
2 1 3 2 4 3 9 8
= √ −√ + √ −√ + √ −√ +··· + √ − √
2 2 6 6 12 12 72 72
       
1 1 1 1 1 1 1
= 1− √ + √ −√ + √ −√ +···+ √ − √
2 2 3 3 4 8 9
       
1 1 1 1 1 1 1 1 1
=1+ − √ + √ + −√ +√ + −√ +√ +···+ − √ + √ −
2 2 3 3 4 4 8 8 3
1 2
=1− = .
3 3

24. Join E and O as in the figure below. Since AE = OC and EO = OC (both lines are radii of the
semicircle), AE = EO. Then the triangle AEO is isosceles with 6 EAO = 6 EOA. Let x be the mea-
..........................................
......... .......
....... ......
...... .....
...... .....
....
.
.
................D
.....
............................ .....
.... ................... ..... ...
...
....... ..
..
.. ............. ... ...
E ............ .... ...
.........................
. ...... ...
.. ..
. .. .
........ ... .................... . . ...
.......... ... ◦
...........
. ........ .
.
.
............
....... ..... 45 ..
......... . ............ .. .
..
.........
.. ................................................................................................................... ..................................................................
. .....
. . ...
A B O C

sure of these angles. Since the sum of the angles in any triangle is 180◦ , 6 AEO = 180◦ − 2x. Since
6 OED is the supplement of 6 AEO, 6 OED = 2x. Since OE = OD (both lines are radii of the semicircle),
triangle EOD is isosceles with 6 ODE = 6 OED = 2x. Finally, since 6 AOD is the supplement of 6 DOC,
6 AOD = 135◦ . Summing the angles in triangle AOD, we have x + 2x + 135◦ = 180◦ . Simplifying gives
3x = 45◦ . Dividing both sides by 3 gives x = 15◦ . The measure of 6 DAO is 15◦ .

25. Let N be a four-digit number which is divisible by 12 and whose digits sum to 6. We know that N is
divisible by 12 if and only if it is divisible by 3 and by 4. (This is because 3 and 4 have no common factors.)
Since the digits of N sum to 6, N is certainly divisible by 3. To be divisible by 4 as well, the last two digits
of N , regarded as a two-digit number by itself, must be divisible by 4. So, the last two digits of N must be
00, 04, 12, 20, 32, or 40. We consider each possibility in turn.
If the last two digits of N are 00 then N could be 6000, 5100, 4200, 3300, 2400, or 1500. There are
6 such numbers in all. If the last two digits of N are 04 then N could be 2004 or 1104. There are 2 such
numbers in all. If the last two digits of N are 12 then N could be 3012, 2112, or 1212. There are 3 such
numbers in all. If the last two digits of N are 20 then N could be 4020, 3120, 2220, or 1320. There are 4
such numbers in all. If the last two digits of N are 32 then N must be 1032. There is only 1 number in this
case. If the last two digits of N are 40 then N could be 2040 or 1140 There are 2 such numbers in all. In
summary, there are 6 + 2 + 3 + 4 + 1 + 2 = 18 numbers which are divisible by 12 and whose digits sum to 6.
THE UNIVERSITY OF THE WEST INDIES, MONA
The 2015 Jamaican Mathematical Olympiad

Test for Grades 9, 10, and 11

NAME:

GRADE:

SCHOOL:

YEAR OF BIRTH:

STUDENT PHONE:

EXAMINATION QUESTIONS

1) The median of the list

n, n + 3, n + 4, n + 5, n + 6, n + 8, n + 10, n + 12, n + 15

is 10. What is the mean?


a) 4 b) 6 c) 7 d) 10 e) 11

102013 + 102015
2) The ratio is closest to which of the following numbers?
102014 + 102014
a) 0.1 b) 5 c) 1 d) 0.2 e) 10

3) Let E be the sum of the first 2014 even counting numbers and D the sum of the first 2014 odd counting
numbers. What is the value of E − D?
a) 0 b) 1 c) 2 d) 2014 e) 4028

..............
4) In the polygon shown on the right, all sides are equal in length and each side is .. ...
............. .............
... ...
............
perpendicular to its adjacent sides. The perimeter of the polygon is 56. What is its .............
.. ...
............
............. ...
area? ...
........... ...............
.... ..
............ ..............
... ..
a) 84 b) 96 c) 100 d) 112 e) 196 ............ ..............
... ...
...........

5) Cindy was asked by her teacher to subtract 3 from a certain number and then divide the result by 9.
Instead, she subtracted 9 and divided the result by 3, giving an answer of 43. If she had worked the
problem correctly, what answer would she have obtained?
a) 15 b) 34 c) 43 d) 51 e) 138

6) A three-digit number abc is called a supersquare if the two-digit numbers ab and bc are both perfect
squares. What is the sum of all the three-digit supersquares?
a) 1013 b) 1177 c) 1465 d) 1993 e) 2016
7) In triangle ABC below, M and N are the midpoints of the sides they lie on. Segments BN and CM
divide the interior of ABC into four regions: three triangles and one quadrilateral. The areas of the
triangles are 3, 6, and 3, as shown below. What is the area of the quadrilateral?
A
..
......
... .....
... ...
..... ...
... ...
... ...
.. ...
.... ...
M .
......
.. ........... .
.
...
............
N
. ....... ... .
...
. ...
. .
...
....... ...
...
.. ..........
.
.
...
. 3 .......
....
... ...
.........
.......
3...
...
..... .
... ......... .......
....... .....
..
.... ........... 6 .
........ ...
.............. ... .
..........................................................................................................
.
B C

a) 4 b) 5 c) 3 d) 7 e) 6

8) Adam had $400 and needed to buy 100 chocolates which cost $4 each. There was a sale at the store:
buy six chocolates and get one free. How much money did Adam have left after he bought 100 chocolate
bars, if he took advantage of the sale?
a) $52 b) $56 c) $60 d) $64 e) $68

9) When the decimal point of a certain positive decimal number is moved four places to the right, the new
number is 4 times the reciprocal of the original number. What is the original number?
a) 0.0002 b) 0.002 c) 0.02 d) 0.2 e) 2

10) In the figure below, 4ABC is isosceles with AB = AC. If AP = P C = CB, what is the measure of
6 A?

.... B
P .......... ....
..........
........... ....
.
. ..
. .............. ..... ....
..... ...
.......... ... ...
..................
A ..........
..........
....
... ....
..
.......... ... ..
..........
.......... ..... ...
.......... .. ...
............
C

a) 36◦ b) 30◦ c) 48◦ d) 72◦ e) 60◦

11) What angle do the hour hand and the minute hand of a clock form at 9:20?
a) 140◦ b) 160◦ c) 150◦ d) 170◦ e) 165◦

12) In the figure below, ABCD is a parallelogram with DC = 16 and CB = 10. Side BA is extended to E,
and EC intersects AD at F . If EA = 4, what is the length of AF ?
E ...............................................A
...................................................................................................... B
........ . .
........ .... ...
........
.. ...
... ............... ...
F ... ........
........ ...
... ........
........ ... 10
... ........ ...
...
. ........ ...
.
........
... ... ....
..........................................................................................................
D C
16

a) 1 b) 2 c) 3 d) 4 e) 5

13) How many ordered pairs of positive integers (x, y) are there such that x + y = 60 and the highest
common factor (HCF) of x and y is 5?
a) 4 b) 5 c) 6 d) 7 e) some other answer
14) In the figure below, each of the nine small squares needs to be painted one of three colours. In how
many ways can this be done so that each of the three colours is present in each row and in each column?
.................................................................
... .. .. ...
.. ... ... ...
.. .. .. ..
.....................................................................
... ... ... ...
... ... ... ..
......................................................................
.. .. .. ...
... ... ... ...
... .. .. ...
...................................................................

a) 4 b) 6 c) 8 d) 10 e) 12

15) Suppose x, y, and z are positive real numbers such that xy = 24, xz = 48, and yz = 72. What is the
value of x + y + z?
a) 18 b) 19 c) 20 d) 22 e) 24

16) The figure below shows six circles arranged as an “equilateral triangle” inside a rectangle with base
6 cm. What is the shortest distance between the two shaded circles?
......................................................................................................................................... ...........
....... ... .................................. ....
............................................
......
...
...
..... .................................................. .. ...
...
... . . .
...... .................................................................... .. ...
... ........ . ............................................................. .......... ...
.... .................................. ...................... .. ...
... .................. ... .. .... ... .
... .. ...... ... ...
... ..... .. . ...
... ... ...
....... ...
.. ... h
... ...
...... .. . .. ... ...
... ............................................................................... .... ..
... .. . ...................... ... ...
..................................................................
.. . .. .. . .. ...
... ...
...
... ........................................ ...
..................................... ... ...
... .. . . . . . . . . . ..
..................................................................................................................................... ..........

.. .
.............................. 6 cm ..............................
.

√ √ √ π
a) 2 b) 3 c) 2 3 − 2 d) e) 2
2

17) Suppose A, B, and C are three numbers such that

1001C − 2002A = 4004 and 1001B + 3003A = 5005

What is the average of the three numbers A, B, and C?


a) 1 b) 3 c) 6 d) 9
e) not uniquely determined

18) A set of tiles numbered from 1 to 100 is modified repeatedly by the following operation: remove all tiles
numbered with a perfect square and renumber the remaining tiles consecutively starting with 1. How
many times must the operation be performed to reduce the number of tiles in the set to one?
a) 20 b) 11 c) 10 d) 19 e) 18

19) For how many positive integers n does 1 + 2 + 3 + · · · + n evenly divide 6n?
a) 5 b) 3 c) 9 d) 11 e) 7

20) How many distinct four-digit numbers are divisible by 3 and have 23 as their last two digits?
a) 27 b) 30 c) 33 d) 81 e) 90
21) In 4ABC below, the segments AL, BM , and CN determine four tri- A .
angles and three quadrilaterals in the interior of 4ABC. The sum .........
... ......
... .. ..
.... .... .....
of the perimeters of the three quadrilaterals is 25 cm. The sum of the ...
...
.. .. ...
... ..
.. ............N
...
.... ....... ....
perimeters of the four triangles is 20 cm. If the perimeter of 4ABC is .
...
....
. ....
..........
.... ...
...
...
M .
..
........... ......... ....
.. ...
19 cm, what is value of AL + BM + CN ? .
.
.
..
... ..........................
.. ........ ......... ...
.... ...
...
...
.
. ....... ............ ...
... . . .
...
.
... ......... ...
................ .
... ....
.......... ....
.....
a) 15 cm b) 16 cm c) 12 cm d) 13 cm C
. .
.................................................................................................................
. ...
B
L
e) 11 cm

22) Suppose that a + 1 = b + 2 = c + 3 = d + 4 = a + b + c + d + 5. What is the value of a + b + c + d?

7 10 5
a) −5 b) − c) − d) e) 5
3 3 3

23) Mr. Earl E. Bird leaves his house and drives to work at exactly 8:00 am every morning. When his
average driving speed is 40 k/h, he arrives at his workplace three minutes late. When his average speed
is 60 k/h, he arrives three minutes early. At what average speed, in k/h, should Mr. Bird drive to arrive
at his workplace precisely on time?
a) 45 b) 48 c) 50 d) 55 e) 58

24) In the figure below, AD = AE = 10, DE = 12, and BD = EC. The perimeter of 4ABC is twice the
perimeter of 4ADE. What is the length of BD?
A
...
...................
....... ... ...................
....... ... .... .........
....... ....
.
............ ...... ....
....
.......
........
.. ...
....... ...
.... .......
............
. ... .... .......
.......
....... ..
. .... .......
.
..... .
........ 10 .... .. .....
.
10 ........
.......
........... .
.... ....
.....
.......
.......
...... .. ..
. ........................................................................................................................................................................................................
.
.... ... .
B D 12 E C

15 17 19
a) b) 8 c) d) 9 e)
2 2 2

25) Let n be the smallest positive integer that is divisible by both 4 and 9, uses only the digits 4 and 9
when written as a numeral, and uses both digits at least once. What are the last four digits of n?
a) 4444 b) 4494 c) 9444 d) 9944 e) 4944

END OF QUESTIONS

You may mail your completed question paper to:

Mathematical Olympiad
P.O. Box 94
Mona Post Office
Kingston 7

You may also deliver your entry by hand or by courier directly to the Department of Mathematics at the
UWI, Mona Campus. In all cases, an entry must be received by December 1, 2014 in order to be considered.
THE UNIVERSITY OF THE WEST INDIES, MONA
The 2015 Jamaican Mathematical Olympiad
Solutions for Grades 9, 10, and 11

1. The numbers are listed from least to greatest. So, their median is the number in the middle
(i.e., the fifth number on the list). Then n + 6 = 10, and hence n = 4. The numbers on the list
are 4, 7, 8, 9, 10, 12, 14, 16, and 19. Their average is

4 + 7 + 8 + 9 + 10 + 12 + 14 + 16 + 19 99
= = 11.
9 9

2. We have
102013 + 102015 102013 (1 + 102 ) 1 + 100 101
= = = = 5.05
102014 + 102014 102013 (10 + 10) 10 + 10 20
This ratio is closer to 5 than to any other number given as a choice.

3. We have E = 2 + 4 + 6 + · · · + 4028 and D = 1 + 3 + 5 + · · · + 4027. Then E − D =


(2 + 4 + 6 + · · · + 4028) − (1 + 3 + 5 + · · · + 4027). Rearranging terms gives

E − D = (2 − 1) + (4 − 3) + (6 − 5) + · · · + (4028 − 4027)
= 1 + 1 + 1 + · · · + 1 (2014 times)
= 2014.

4. The original polygon has 28 equal sides and a perimeter of 56. So, each side has length 2. The
interior of the polygon contains 25 equal squares (see the figure below). Each square has a side
length of 2 and an area of 4. The area of the original polygon is 25 × 4 = 100.
.............
... ...
........................................
.. .. .. ..
............................................................
.. .. .. .. .. ..
....................................................................................
.. .. ... ... ... ... .. ..
.......................................................................
....................................................................
... ... ... ...
...................................
... ...
.. ...
.........

5. Let x be the original number. Cindy subtracted 9 from x and divided by 3, obtaining 43. So,
(x − 9)/3 = 43. Multiplying both sides by 3 gives x − 9 = 129. Adding 9 to both sides gives
x = 138. The original number was 138. Cindy should have subtracted 3, obtaining 135, and then
divided by 9. She should have obtained 15.

6. The two-digit square numbers are 16, 25, 36, 49, 64, and 81. The numbers 16, 36, 64, and 81
are the first two digits of the supersquares 164, 364, 649, and 816. The numbers 25 and 49 are
not the first two digits of any supersquare. So there are 4 supersquares in all, and their sum is
164 + 364 + 649 + 816 = 1993.

7. We may re-orient the diagram so that AC is considered the base of 4ACB. Then the triangles
AN B and BN C have equal bases and equal (perpendicular) heights. Thus they have equal areas.
A
.
.......
... .....
.. ...
....
. ...
.. ...
.
... ...
...
.... ...
M .........
.. ........... ...
.
............
N
. ...
... .................. ...
...
.. ..........
...
. 3 .......
.
.... ...
. .......
.......
3 ...
.
.... .............. ....... .....
.. ....... 6 ....... ...
.
......... ....... ..
................................................................................................
..
.
B C

In particular, the area of 4AN B is 9. Thus the area of the quadrilateral inside 4ABC is 6.

8. Taking advantage of the sale, Adam bought 7 chocolates for the price of 6, which is $24. At
the same rate, he bought 14 chocolates for $48, 21 chocolates for $72, and so on. Eventually, he
bought 98 chocolates for $336. The next 2 chocolates cost $8. So, he bought 100 chocolates for
$344. He had $56 left after that.

9. Let x be the original number. When the decimal point in the representation of x is moved four
places to the right, the new number has the value 10, 000x. Since this is four times the reciprocal
of x, 10, 000x = 4/x. Multiplying both sides by x gives 10, 000x2 = 4. Taking the positive square
root of both sides gives 100x = 2. Dividing throughout by 100 gives x = 0.02.

10. Let x = 6 BAC and y = 6 ABC. Since 4AP C is isosceles with AP = P C, 6 P CA = x as well.
Since 4P CB is isosceles with P C = CB, 6 CP B = y as well. The sum of the angles in 4AP C is
180◦ . Thus 6 AP C = 180◦ − 2x. Also, 6 AP B is a straight angle. Thus (180◦ − 2x) + y = 180◦ .
.... B
P .......... ....
..........
..
.......
............
.......... ..... yy .....
...
........ ...
................. x ... ...
A ..........
..........
...
... ....
..........
.......... x ... ...
.......... ..... ....
...............
..
C
Solving for y gives y = 2x. Thus 6 ABC = 2x. Since 4ABC is isosceles with AB = AC,
6 ACB = 2x as well. Since the sum of the angles in 4ABC is 180◦ , x + 2x + 2x = 180◦ . That is,
5x = 180◦ . Dividing both sides by 5 gives x = 36◦ .

11. In sixty minutes the minute hand of a clock sweeps out one full circle, which is 360◦ . In one
minute it sweeps out one-sixtieth of this, which is 6◦ . In sixty minutes, the hour hand sweeps out
one-twelfth of a circle, which is 30◦ . In one minute it sweeps out one-sixtieth of this, which is 0.5◦ .
At 9:00, the minute hand is at 0◦ and the hour hand is at 270◦ . (We may liken a clock face to a
compass, with 12:00 representing due north.) Twenty minutes later, at 9:20, the minute hand has
advanced by 120◦ and the hour hand has advanced by 10◦ . Their new positions are 120◦ and 280◦ ,
respectively. The difference between these angles is 280◦ − 120◦ = 160◦ .
12. Triangles EAF and CDF are similar. This is because 6 AEF = 6 AEC = 6 ECD = 6 DCF
and 6 EAF = 6 EAD = 6 ADC = 6 F DC. (In both cases these are alternate angles between the

E ...........................4.................A
.............................................................................................. B
..
........ ... ...
..........
..... ...
... ............... ..
..
F ... ............ ..
..
..
.. ........
........
..
..
..
10
........
... ........ ...
... ...... ..
.
...................................................................................................
D C
16
parallel lines EB and DC). Also, 6 EF A = 6 CF D by vertical angles. Since corresponding sides
of similar triangles are in the same proportion,

AF EA 4 1
= = =
FD CD 16 4

Cross multiplying gives F D = 4(AF ). Since opposite sides of a parallelogram are equal, AD = 10.
This means that AF + F D = 10 and hence AF + 4(AF ) = 10. Thus 5(AF ) = 10 and hence
AF = 2.

13. If x and y are positive integers with a highest common factor (HCF) of 5, both of them are
positive multiples of 5. It follows that x, y ∈ {5, 10, 15, · · · , 55}. The ordered pairs (x, y) such that
x + y = 60 are

(5, 55), (10, 50), (15, 45), (20, 40), (25, 35), (30, 30), (35, 25), (40, 20), (45, 15), (50, 10), (55, 5)

The HCF of x and y is 5 in exactly four of these pairs. These are (5, 55), (25, 35), (35, 25),
and (55, 5).

14. Let the colour for the upper left corner be denoted by a. The square next to it (in the top
row) must have a different colour, call it b. The square below it must also have a colour different
................................................................ ................................................................
.. .. .. ... .. .. .. ...
.... a b .. ..
..................................................................
. ...a b ... ...
....................................................................
..
... ... ... ...
..... ..... ..... ....
...
b ...
...
.
...
.
..................................................................
...
. c ... . .
..................................................................
..
... ... ... ... ... ... ... ...
... ... ... .. ... ... ... ..
.. . . .. .. . . ..
....................................................... .......................................................

Figure 14(a) Figure 14(b)

from a. It may be either b or c (see the figures above). In either case, once these three squares
are coloured there is only one way to colour the remaining squares. (Try it.) In the first case,
as in Figure 14(a), there are 3 choices for a and two choices for b. There are 3 × 2 = 6 different
colourings in this case. In the second case, as in Figure 14(b), there are again 3 choices for a and
two choices for b. There are 3 × 2 = 6 different colourings in this case. Then there are 6 + 6 = 12
different colourings in all.

15. Since xy = 24 and xz = 48, we may divide equals into equals to see that

xy 24 y 1
= and hence =
xz 48 x 2
Cross multiplying gives z = 2y. Since yz = 72 we may substitute for z to see that 2y 2 = 72. Then
y 2 = 36 and hence y = 6. (It is given that y is a positive real number.) It follows that x = 4 and
z = 12. Thus x + y + z = 4 + 6 + 12 = 22.

16. In the top row, the diameters of the three circles span 6 cm. So, the diameter of each circle is
2 cm and the radius of each one is 1 cm. The centres of the top shaded circle and the two unshaded
.......................................................................................................................... ........
. ... .... .................................... .... ....... ...
...... ....................................................
...............................
...
.. ...
... ...
...... .......................................................................
. .. ...
. ..................................... ... ...
......... ..
. .. . .. .. ...... . . ..
.. ...
... ...................................................................................................... ..... ...
... .. . . ..... ... ... ...
. . .
... .... ....................................... .. ...
...
..
...
.... ... ..... ..
. . ....
. ...
...
h
.... ....... ......................................... ......... ... ...
................................................. ... ...
... .. . ... .. . .. .. .
... ................................................................................. ... ...
...
... ........................................ ... ...
... .............................................. ... ..
.. ................................ .
............................................................................................ .........
.
............................
. 6 cm ..............................

circles below it are vertices√of an equilateral triangle. Each side of this triangle is 2 cm long. The
altitude of this triangle is 3 cm. (This may be deduced from the Pythagorean theorem.) In a
similar way, the centres of the bottom
√ shaded circle and the two unshaded
√ √ circles above √it form an
equilateral triangle with altitude 3 cm. It follows that h = 1 + 3 + 3 + 1 = 2 + 2 3. From
the top of the rectangle to the bottom of the upper shaded circle is 2 cm. From the bottom of the
rectangle to the top of the√lower shaded √ circle is also 2 cm. Then the shortest distance between
the shaded circles is (2 + 2 3) − 4 = 2 3 − 2.

17. Dividing the equation 1001C − 2002A = 4004 throughout by 1001 gives C − 2A = 4. Adding
2A to both sides gives C = 2A + 4. Dividing the equation 1001B + 3003A = 5005 throughout by
1001 gives B + 3A = 5. Subtracting 3A from both sides gives B = −3A + 5. Let m be the average
of A, B, and C. Then

A+B+C A + (−3A + 5) + (2A + 4) 9


m= = = = 3.
3 3 3

18. On the first step, 10 tiles are removed (numbered 1, 4, 9, . . ., 100) and so 90 tiles remain. On
the second step, 9 tiles are removed (numbered 1, 4, 9, . . ., 81) and so 81 tiles remain. On the next
steps 9, 8, 8, 7, 7, 6, 6, 5, 5, 4, 4, 3, 3, 2, 2, and 1 tiles are removed. At this stage, there is one tile
remaining. The operation must be preformed 18 times to leave 1 tile remaining.

19. The sum 1 + 2 + 3 + · · · + n, which is the sum of the first n positive integers, is equal to
n(n + 1)/2 = (n2 + n)/2. This will evenly divide 6n when 6n is a multiple of (n2 + n)/2. That is,
1 + 2 + 3 + · · · + n evenly divides 6n when 6n = k(n2 + n)/2 for some positive integer k. This holds
if and only if 12n = k(n2 + n) for some positive integer k. In turn, dividing both sides by n, this
holds if and only if 12 = k(n+1) for some k. Since k is a positive divisor of 12, k ∈ {1, 2, 3, 4, 6, 12}.
If k = 1 then n = 11; if k = 2 then n = 5; if k = 3 then n = 3; if k = 4 then n = 2, and if k = 6
then n = 1. The choice k = 12 is not possible because n = −1 in this case. Then there are five
numbers n such that 1 + 2 + · · · + n evenly divides 6n. These are n = 1, 2, 3, 5, and 11.
20. Let n = ab23 be a four-digit number that is divisible by 3. If a = 1 then n has the form 1b23.
This is divisible by 3 when b = 0, 3, 6, or 9. There are four such numbers in this case. If a = 2
then n has the form 2b23. This is divisible by 3 when b = 2, 5, or 8. There are three such numbers
in this case. If a = 3 then n has the form 3b23. This is divisible by 3 when b = 1, 4, or 7. There are
three such numbers in this case. Continuing in this way, if a = 4 there are four possible numbers,
if a = 5 there are three possible numbers, and so on. There are 4 + 3 + 3 + 4 + 3 + 3 + 4 + 3 + 3 = 30
possible numbers in all.

21. Let x = AL + BM + CN and let P , Q, and R be the points of intersection shown below. The

A
.........
.... ......
... .. ..
.... .... .....
.
. ... ...
... .. ......... N
.... ...... ...
... P .........
..... ...
.. ...
...
... ..... .. ...
M ..
.
........... .........
. .............
... ...
...
...
...
.
.. . ........... ..
.... ..........
. ............. Q . ...
..
.. ...... R
.
.
.......... ... ............ .....
. .
..
. .
......... ... .......... ...
........................................................................................................
..
.
C B
L

sum of the perimeters the three quadrilaterals is 25. (All dimensions in this solution are centime-
tres.) Thus

(AN + M R + RP + P A) + (CL + LQ + QR + RC) + (BN + N P + P Q + QB) = 25

Rearranging terms gives

AM + CL + BN + (AP + P Q + QL) + (BQ + QR + RM ) + (CR + RP + P N ) = 25.

Thus AM + CL + BN + AL + BM + CN = 25. That is, AM + CL + BN + x = 25. Also, the


sum of the perimeters of the four interior triangles is 20. Thus

(M C + CR + RM ) + (LB + BQ + QL) + (N A + AP + P N ) + (P R + RQ + QP ) = 20.

Rearranging terms gives

M C + LB + N A + (AP + P Q + QL) + (BQ + QR + RM ) + (CR + RP + P N ) = 20.

Thus M C + LB + N A + AL + BM + CN = 20. That is, M C + LB + N A + x = 20. We therefore


have AM + CL + BN + x = 25 and M C + LB + N A + x = 20. Adding these equations gives
(AM + M C) + (CL + LB) + (BN + N A) + 2x = 45. That is, AC + CB + BA + 2x = 45. But
AC + CB + BA is the perimeter of 4ABC, and this is 19. Thus 19 + 2x = 45. Subtracting 19
from both sides gives 2x = 26. Dividing both sides by 2 gives x = 13. Thus AL + BM + CN = 13.

22. The original expression may be written as the four equations

a + 1 = b + 2, a + 1 = c + 3, a + 1 = d + 4, and a + 1 = a + b + c + d + 5.

From the first three, b = a − 1, c = a − 2, and d = a − 3, respectively. Substituting these


expressions into the fourth equation gives a + 1 = a + (a − 1) + (a − 2) + (a − 3) + 5. Thus
a + 1 = 4a − 1. Subtracting a from both sides gives 1 = 3a − 1. Adding 1 to both sides gives 2 = 3a
and hence a = 2/3. Then b = −1/3, c = −4/3, and d = −7/3. Thus
2 1 4 7 10
a+b+c+d = − − − = −
3 3 3 3 3

23. Let t be the time, measured in hours, after 8:00 am that Mr. Bird’s work begins. Let d be the
distance measured in kilometres from his house to his workplace. When Mr. Bird drives 40 k/h,
he is 3 minutes late. Since rate times time equals distance, and 3 minutes is one-twentieth of an
hour,  
1
40 t + = d and so 40t + 2 = d.
20
When Mr. Bird drives 60 k/h, he is 3 minutes early. Thus
 
1
60 t − = d and so 60t − 3 = d.
20
Eliminating d from these equations gives 40t + 2 = 60t − 3. Subtracting 40t and adding 3 to both
sides gives 5 = 20t. Thus t = 1/4 (measured in hours), and his work begins at 8:15 am. We may
solve either of the first two equations to see that d = 12 km. Then to arrive on time Mr. Bird must
have r(1/4) = 12. Multiplying both sides by 4 gives r = 48. Mr. Bird should drive 48 k/h to arrive
at work on time.

24. Let x = BD = EC and let AF be an altitude for 4ADE. Since 4ADE is isosceles,
DF = F E = 6. By the Pythagorean theorem, (AF )2 + 62 = 102 . Solving for AF gives AF = 8.
Let y = AB. By the Pythagorean theorem again,
y 2 = (x + 6)2 + 82 = (x2 + 12x + 36) + 64 = x2 + 12x + 100.
In the same way, (AC)2 = x2 + 12x + 100. Thus AC = y as well. The perimeter of 4ABC
is twice the perimeter of 4ADE. This means that 2x + 2y + 12 = 2(32) = 64. Dividing by 2
throughout gives x + y + 6 = 32. Subtracting x + 6 from both sides gives y = 26 − x. Finally,
squaring both sides gives y 2 = 676 − 52x + x2 = x2 − 52x + 676. Thus we have y 2 = x2 + 12x + 100
and y 2 = x2 − 52x + 676. Eliminating y 2 gives x2 + 12x + 100 = x2 − 52x + 676. Subtracting x2
from both sides gives 12x + 100 = −52x + 676. Adding 52x to both sides gives 64x + 100 = 676.
Subtracting 100 from both sides gives 64x = 576. Finally, dividing both sides by 64 gives x = 9.
Thus BD = 9.
A
.........
.........................
....... .... .. .... ........
y ....... ... .. .... ....... y
.
...
......... ....... .... ...... .............
...... .
.... .....
.... ........
....... ....
.... .......
....... .... ... .... .......
....... 10 8 ....
10 ........
.
............ .
...... ...
. .... .......
........
...
....... ..
... .......... ....
. .......
..... .. . .. .... ....
. .....................................................................................................................................................................................
.
.... ... .
. .
B x D 6 F 6 E x C

25. Since n is a multiple of 36, it is a multiple of both 4 and 9. Also, the digits of n must include
at least one 4 and one 9. In order for n to be divisible by 9, the sum of its digits must be divisible
by 9. This means that the number of 4’s among its digits must be a multiple of 9. So, the digits
of n must include one 9 and nine 4’s. Its last two digits must be 44, 49, 94, or 99. Since n is
a multiple of 4, the only possibility is 44. The smallest number that meets the stated criteria is
n = 4, 444, 444, 944. The last four digits of n (written as a four-digit number) are 4944. [Note: a
number abcd · · · ef g is divisible by 4 if and only if the two-digit number f g is divisible by 4.]
THE UNIVERSITY OF THE WEST INDIES, MONA
The 2015 Jamaican Mathematical Olympiad

Test for Grades 7 and 8

NAME:

GRADE:

SCHOOL:

YEAR OF BIRTH:

STUDENT PHONE:

EXAMINATION QUESTIONS

1) What is the value of 2014 − (2 × 0 × 1 × 4)?


a) 2014 b) 2000 c) 2006 d) 1986 e) 8048

2) Rose bushes are planted in a line on one side of a path, with the bushes spaced 2 m apart. Then bushes
are planted in a line on the other side of the path, also 2 m apart. If the path is 20 m long, what is the
largest number of bushes that can be planted in this way?
a) 11 b) 20 c) 12 d) 22 e) 10

3) How many rectangles may be found in the figure below?


..................................................................................................................................
.. ... ...
.. .. ..
........................................................................................................................................
... ... ...
... ... ..
......................................................................................................................................
... ... ...
... ... ..
..................................................................................................................................

a) 6 b) 9 c) 12 d) 15 e) 18

4) In how many ways can you place 2 identical $20 dollar coins in three boxes, one red, one blue, and one
green?
a) 8 b) 6 c) 4 d) 3 e) 2

5) The six-sided polygon in the figure below has a right angle at each vertex. What is the area, in square
units, of the shaded region?
...........................................................................................................................................................................................................
............................................................................................................................
..............................................................................................................................
............................................................................................................................
............................................................................................................................. 3
............................................................................................................................................................................................................
............................................................................................................................................
.........................................................
........................................................................
........................................................ 7
.........................................................
........................................................................ 4
........................................................
........................................................
.....................................................................

a) 56 b) 48 c) 42 d) 41 e) 19
6) There were twice as many girls as boys in Miss Smith’s class. After Eric left the class, there were 7
more boys than girls. After Joan left, how many girls were in the class?
a) 6 b) 7 c) 8 d) 9 e) 10

7) Grandmother baked dumplings for her grandchildren. If she gives each of them 2 dumplings, she will
have 3 dumplings left over. If she gives each of them 3 dumplings, she will be 2 dumplings short. How
many grandchildren does Grandmother have?
a) 2 b) 4 c) 3 d) 6 e) 5

8) Three identical equilateral triangles are located in the corners of a larger equilateral triangle. Each side
of the larger triangle measures 6 cm. When the perimeters of the smaller triangles are added,their sum
is equal to the perimeter of the shaded hexagon. What is the length of one side of a smaller triangle?
....
... ...
... .....
..
..............................
.
.
........................
.......
.....................
...........................
..........................................
.
.................................................................
...................................
.........................................
............................................................
... .................................. ..
.... ............................................................ .....
. .. . . . . . . . . . . .. .
...................................................................................................................

a) 1 cm b) 1.5 cm c) 1.25 cm d) 2 cm e) 1.2 cm

9) A digital clock displays the time in 24-hour format using two digits for each hour and two for each
minute. Between one minute after midnight (00:01) and one minute before the next midnight (23:59),
how many times will the clock show a time that reads the same from the left as from the right (for
example, 15:51)?
a) 8 b) 12 c) 14 d) 15 e) 24

10) Stephen and Andrew were running around the track at constant speeds. Stephen ran 5 laps every 12
minutes, and Andrew ran 3 laps every 10 minutes. Both boys started together, and they finished running
the first time they were together again at the starting line. How many laps did the two boys run all
together?
a) 3 b) 86 c) 43 d) 135 e) 90

11) The arrow-shaped polygon in the figure below has right angles at A, C, D, E, and F . The segments
BA and AG have equal length, BC = F G = 5, CD = EF = 20, and DE = 10. What is the area of
this polygon?
B
....
... ..
.... ...
.... .... C
...... ............................................................................. D
..
.... ....
.... ...
.
..... ..
A ..... ...
.... ...
.... ...
.... ..
....
... ..............................................................................
.... .
.... ....
.... .. F E
......
.
G

√ √
a) 300 b) 250 + 20 2 c) 350 d) 250 + 10 2 e) 250

12) When 999 is divided by a certain two-digit number n, the remainder is equal to 3. What will the
remainder be when 2001 is divided by n?
a) 1 b) 3 c) 6 d) 7 e) 9
13) When a carton of juice is three-fourths full, it has enough juice to fill one and one-half glasses. How
many glasses will the juice from 5 new cartons fill?
1 3 1
a) 7 b) 3 c) 10 d) 8 e) 8
2 4 4

14) When Carl the Camel is thirsty, water makes up 84% of his body weight. After he drinks, Carl weighs
800 kg and water makes up 85% of his weight. How much does Carl the Camel weigh when he is thirsty?
a) 672 kg b) 680 kg c) 715 kg d) 720 kg e) 750 kg

15) In the polygon below, each side is perpendicular to the sides adjacent to it and all sides in the figure
are congruent. The perimeter of the polygon is 56 cm. What is its area?
..............
.
... ...
.............. ..............
.
. ............
..........
..... ...
. .............
.............. ....
.............. .
.. ..............
............. .
.
.... ..............
............ .............
... ....
..............

a) 84 cm2 b) 96 cm2 c) 100 cm2 d) 112 cm2 e) 196 cm2

16) A certain broken calculator does not display the digit 7. For example, if we type in the number 3747,
only the number 34 is displayed, with no spaces. Mike typed a six-digit number on this calculator, but
only 2014 appeared on the display. How many different numbers could Mike have typed?
a) 12 b) 13 c) 14 d) 15 e) 16

17) Carlos adds five numbers together. None of them is greater than 20. The numbers may be repeated.
From the sum he subtracts the smallest number (or one of them if more than one number has the
smallest value), and writes the difference as the final result. If the sum of the five numbers is 72, what
is the smallest final result he could obtain?
a) 58 b) 72 c) 57 d) 52 e) 54

18) The area of the biggest square shown on the right is 16 cm2 and the area of the .....................................................................................................
... ..........
.. ..... ...
... .......... .. .... ...
. ..............
smallest square is 4 cm2 . What is the area of the square which is between them? .................... ................................................ ....
. ... .. .....
.. ...
........ .... .... .... .... .
... ... ... ... ... ...
a) 8 cm2 b) 8 21 cm2 c) 10 cm2 d) 10 21 cm2 ... ....
... ...
... .... ....
...
.. .....
...
... ..
... ...
... ...
... ... ... ... ......
e) 12 cm2 ... .... ................................................. ... ... ......
... ... ..
... . .. . .. ...
...... . ....
... ... ...
.
... ..... .......... ...
. .......... .
...........................................................................................................

19) The ones digit of a certain three-digit number is 2. If we move this digit to the beginning of the number,
we get a three digit number that is smaller than the original by 36. What is the sum of the digits of
this number?
a) 1 b) 5 c) 9 d) 7 e) 10

20) Mr. Williams said, “The product of my children’s ages is equal to 1664. My oldest child is twice as old
as my youngest.” How many children does Mr. Williams have?
a) 2 b) 3 c) 4 d) 5 e) 6
E
21) In the figure on the right, ABCD and EF GH are identical squares with B .......
... ......
A .... ....
side length 10. The vertex H of square EF GH is the centre of ABCD. ................................................................. ....
... ... . ...
... ... .... ....
....
. ... ....
Sides HE and HG pass through B and C, respectively. What is the area ...
... .
......
....
.
..
....
...
....
...
..
... H .... ... F
of polygon ABEF GCD? ... .... .
.
. ....
... .... .... .
....
... ... .. ....
.
.... . ...
.... .. ...
a) 75 b) 100 c) 125 d) 150 ............................................................. ....
D ... .....
........
C .
e) 175 G

22) A certain boy always tells the truth on Thursdays and Fridays, always lies on Tuesdays, and randomly
tells the truth or lies on the other days. On seven consecutive days he was asked what his name was.
On the first six days he said, in this order, John, Bob, John, Bob, Peter, and Bob. What answer did he
give on the seventh day?
a) John b) Bob c) Peter d) Kate e) some other name

23) A 3 × 3 table contains positive integers (see the figure on the right). Nick and Pete ..................................................................................
.... .. .. ...
... ... ... ...
crossed out four numbers each. They noticed that the sum of the four numbers Pete ...
...
4 12 ...
...
...
...
8 ...
..
......................................................................................
crossed out was exactly 3 times the value of the sum of the four numbers Nick ...
...
...
...
...
...
...
...

crossed out. Which number was the only one that neither Pete nor Nick had crossed 13 ...
...
24 14 ...
...
...
...
...
...
.....................................................................................
... ... ... ...
....
out? 7
...
... 5
...
...
...
23 ... ...
... ... ... ...
................................................................................
a) 4 b) 7 c) 14 d) 23
e) 24

24) Emily arranged five cardboard pieces into the shape of a bird (see the figure below). One piece was a
rectangle measuring 5 cm by 10 cm, two pieces were quarters of a large circle, and two were quarters of
a small circle. Benjamin moved the pieces around and made the shape of an egg. How much larger is
the perimeter of the bird than the perimeter of the egg?
.............
... ......
...
... ...................................
.. ... ........ .... ......
............................................................... ..... ....
..........
. .
.......
.. ..... .. ...
..
.. .
... .... ... .... ... ...
.
..... . ... ... ... ..
. ...
. ... .. ...
... .. ... ... .... ... ..
... .
. .
. ... ... ... ..
..
... .... ..... .... .................................................................................
..... ... ... . .
.. .. . ...... .
.. ..
............................................................................... ...
...
... . ..
... ... ...... ... ... ....
... . ..............................................................
..... ....
............

Emily’s Bird Benjamin’s Egg

a) 2.5 cm b) 5 cm c) 10 cm d) 20 cm e) 30 cm

25) Peter rides a bicycle from town P to town Q at a constant speed. If he increases his speed by 3 k/h he
will arrive at town Q 3 times faster. How may times faster will he arrive if he increases his speed by
6 k/h?
a) 4 b) 5 c) 6 d) 4.5 e) 8

END OF QUESTIONS

You may mail your completed question paper to:

Mathematical Olympiad
P.O. Box 94
Mona Post Office
Kingston 7

You may also deliver your entry by hand or by courier directly to the Department of Mathematics at the
UWI, Mona Campus. In all cases, an entry must be received by December 1, 2014 in order to be considered.
THE UNIVERSITY OF THE WEST INDIES, MONA
The 2015 Jamaican Mathematical Olympiad
Solutions for Grades 7 and 8

1. The value of 2 × 0 × 1 × 4 is 0. So the value of 2014 − (2 × 0 × 1 × 4) is 2014 − 0 = 2014.

2. Since the path is 20 m long, 11 bushes can be planted on each side. (See the figure below.) A
total of 22 bushes can be planted in this way.
•.................•..................•..................•...................•..................•..................•..................•...................•..................•..................•..
•.................•..................•..................•...................•..................•..................•..................•...................•..................•..................•..

3. On the left side of the figure, there are 6 rectangles: three with a height of one small rectangle,
two with a height of two small rectangles, and one with a height of three small rectangle. On the
right side, there are 6 more rectangles. Also, there are 6 rectangles which span both sides of the
figure. There are 6 + 6 + 6 = 18 rectangles in all.
.........................................................................................................................
... .... ...
..
. .
..........................................................................................................................
..... ..... ...
..........................................................................................................................
... ... ..
... ... ..
..........................................................................................................................

4. The coins can be placed in the same box in three ways: both in the red box, both in the blue
box, or both in the green box. They may be placed in different boxes in three ways: one in the
red box and one in the blue, one in the red box and one in the green, or one in the blue box and
one in the green. There are 6 ways of placing the coins in all.

5. The six-sided polygon may be enclosed by a rectangle as shown below. The outer rectangle has
width 12 and height 7. Its area is 84. The unshaded rectangle in the lower right corner has width 7
and height 4. Its area is 28. The area of the shaded polygon is 84 − 28 = 56.
.................................................................................................................................................................................................
.............................................................................................................................
......................................................................................................................................................................
.............................................................................................................................
....................................................................................................................................................................
..............................................................................................................................
3
..............................................................................................................................
...................................................... ....
.........................................................................
........................................................
.........................................................................
7 ...
...
.......................................................
........................................................
4 ...
...
...
4
...................................................... .
......................................................................................................................................

5 7

6. After Eric left, there were 7 more boys than girls in Miss Smith’s class. So, before he left there
were 8 more boys than girls in the class. Since there are twice as many boys as girls, there are 16
boys and 8 girls in the class. After Eric and Joan left, there were 7 girls in the class.
7. Let c be the number of grandchildren that Grandmother has. If she gives each of them 2
dumplings, she will have 3 dumplings left over. Thus grandmother made 2c + 3 dumplings. If she
gives each of them 3 dumplings she will be 2 dumplings short. Thus Grandmother made 3c − 2
dumplings as well. Then 2c + 3 = 3c − 2. Adding 2 to both sides gives 2c + 5 = 3c. Subtracting 2c
from both sides gives 5 = c. Grandmother has 5 children (and she made 13 dumplings for them).

8. Let x be the side of a smaller triangle and y a longer side of the hexagon. Then 2x + y = 6. (All
dimensions in this solution are in cm.) Since the sum of the perimeters of the smaller triangles
...
.. ...
x......... ..........
......................................
................................................
y......... .
.........................................
...........
............................
. . . . . . . . . . . . . . ..
....................................................
.............................................................................
... ................................... ..
x ... ......................................................................... .....
.. ... . . . . . . . . . . ... ..
...................................................................................................

equals the perimeter of the shaded hexagon, 9x = 3x + 3y. Subtracting 3x from this equation
gives 6x = 3y. Dividing both sides by 3 gives 2x = y. Substituting for y in the first equation gives
2x + 2x = 6. Thus 4x = 6 and hence x = 6/4 = 3/2 = 1.5.

9. Suppose the time on a watch reads the same from the left as from the right. The times with
the first digit 0 are 01:10, 02:20, 03:30, 04:40, and 05:50. There are five such times in this case.
The times with the first digit 1 are 10:01, 11:11, 12:21, 13:31, 14:41, and 15:51. There are six such
times in this case. The times with the first digit 2 are 20:02, 21:12, 22:22, and 23:32. There are
four such times in this case. The total number of times reading the same from the left and the
right is 5 + 6 + 4 = 15.

10. The boys were together again at the starting line when they each had run a whole number
of laps in the same amount of time. Suppose Stephen ran s laps and Andrew ran a laps in this
time. Stephen runs 1 lap in 12/5 minutes and so n laps in 12n/5 minutes. Andrew runs 1 lap in
10/3 minutes and so m laps in 10m/3 minutes. Since these times are equal, 12n/5 = 10a/3. Cross
multiplying gives 36n = 50a and hence 18n = 25a. The smallest value of n such that 18n is a
multiple of 25 is n = 25. In this case, a = 18 as well. Then the number of laps they ran together
was n + a = 25 + 18 = 43.

11. From the information given, CDEF is a rectangle. Then CF = DE = 10 and so BG = 20.
The 4ABG has a right angle at A and AB = AG. It follows that 6 ABG = 6 AGB = 45◦ . By the
Pythagorean theorem,
B .
......
..... ..
.... ..
.
........ .... C
......................................................................... D
..... . .
.... .... ....
....
..... ... ...
......... ... ...
A .....
.....
... ...
..... .
... ...
..... .
..... ..........................................................................
..... ..
..... ..
..... .. F E
.....
G

(AB)2 + (AG)2 = (BG)2 = 202 = 400. Since AG = AB, 2(AB)2 = 400. Dividing both sides by 2
gives (AB)2 = 200. Triangle BAG has base AG and perpendicular height AB. Its area is
1 1 1
(AB)(AG) = (AB)2 = (200) = 100.
2 2 2
The area of CDEF is (20)(10) = 200. The area of ABCDEF G is 100 + 200 = 300.

12. When 999 is divided by n, the remainder is 3. This means that 996 is divisible by n evenly.
Thus 996 = kn for some positive integer k. Multiplying both sides by 2 gives 1992 = (2k)n. Adding
9 to both sides gives 2001 = (2k)n + 9. Then when 2001 is divided by n the remainder is 9.

13. Let j be the capacity of a carton of juice and g the capacity of a glass. Then 34 j = 32 g.
Multiplying both sides by 4/3 gives j = 2g. That is, one carton of juice fills 2 glasses evenly. Then
5 cartons fills 10 glasses evenly.

14. Let w be the weight of Carl the Camel when he is thirsty. After he drinks, he weighs 800 kg
and water makes up 85% of his weight. After he drinks, Carl consists of 120 kg of solid matter and
680 kg of water. Before he drinks, he consists of 120 kg of solid matter and the rest of him, 84%,
is water. So, before he drinks he consists of 120 kg of solid matter and this is 16% of his weight.
Then
120 16 4
= = .
w 100 25
Cross multiplying gives 3000 = 4w. Dividing both sides by 4 gives w = 750.

15. The original polygon has 28 equal sides and a perimeter of 56. So, each side has length 2. The
interior of the polygon contains 25 equal squares (see the figure below). Each square has a side
length of 2 and an area of 4. The area of the original polygon is 25 × 4 = 100.
.............
.. ..
..........................................
.. .. .. ..
.........................................................
.. .. .. .. .. ..
..................................................................................
..................................................................................
... .. .. .. .. ...
..............................................................
... ... ... ...
...................................
... ...
..............

16. Mike typed two 7’s, and the digits 2, 0, 1, and 4 (in that order), The first 7 he typed could
have been in 5 different places. He could have typed 7xxxxx, 27xxxx, 207xxx, 2017xx, or 20147x.
In the first case, there are 5 places where the second 7 could have been. In the second case, there
are 4 places it could have been. In the third case there are 3 places, in the fourth case there are 2
places, and in the last case there is one place the second 7 could be. There are 5 + 4 + 3 + 2 + 1 = 15
different numbers Mike could have typed.

17. The smallest of the five numbers Carlos added was 14 or less. (If it were 15 or more the total
would be at least 75.) Furthermore, the smallest number could be 14. For example, the numbers
he added could be 14, 14, 14, 15, and 15. In this case, the sum of the five numbers minus the
smallest one is 72 − 14 = 58. If the smallest number is less than 14 then 72 minus the smallest
number will be more than 58. So, the smallest final result Carlos can obtain is 58.

18. The square in the middle consists of the inner square plus four right-angled triangles. The area
of the inner square is 4 cm2 and the area of the outer square is 16 cm2 . So, the area of the section
.........................................................................................
... ....
... .......... .. ...
... .......... .. ....
..
. ............. ..
................ ... ........................................ .... ....
........ .... ... ... ..
... ... ...
... ... ... ... ... ...
... ... .. ... ... ...
... ... ... ... ... ...
... .... ... ... ...
... ... ... ... ......
... ... ... .
.. ... .......................................... ... .................
.... ... .. ....
...
. ...
... .... .............
. ....
.. ... ................ .
....................................................................................

in between, consisting of 8 equal triangles, is 16 cm2 − 4 cm2 = 12 cm2 . The area of each triangle
is 12 cm2 /8 = 1.5 cm2 . The area of four triangles is 6 cm2 . So, the area of the middle square is
4 cm2 + 6 cm2 = 10 cm2 .

19. Let the original number be ab2, where a, b ∈ {0, 1, 2, . . . , 9}. Then the new number is 2ab. The
value of the first number is a × 100 + b × 10 + 2 = 100a + 10b + 2. The value of the second one is
2 × 100 + a × 10 + b = 200 + 10a + b. Since the first number is equal to the second one plus 36,

(100a + 10b + 2) = (200 + 10a + b) + 36 = 236 + 10a + b.

Subtracting 10a + b + 2 from both sides gives 90a + 9b = 234. Dividing throughout by 9 gives
10a + b = 26. Then a = 2 and b = 6. The original number is 262. The sum of its digits is
2 + 6 + 2 = 10.

20. The prime factorization of 1664 is 27 × 13. So, the age of one of Mr. Williams’s children is
13 or a multiple of 13, and the ages of his other children are powers of 2. It is not possible that
the youngest child is 13 or a multiple of 13. Otherwise, the oldest child would also be a multiple
of 13 and the product of their ages would have a factors of 132 . Similarly, the age of the oldest
child could not be a multiple of 13. (Otherwise, the younger one would also have to be a multiple
of 13.) Then Mr. Williams has at least three children and one of them (neither the oldest nor the
youngest) is 13 or a multiple of 13. Since the age of the oldest child is a power of 2, he or she is
at least 16. The youngest child is at least 8, and one child in between is at least 13. The product
8 × 13 × 16 is equal to 1664. If Mr. Williams had more than three children, or the oldest one is
more than 16, then the product of their ages would be more than 1664. Since this is impossible,
Mr. Williams has three children (and their ages are 8, 13, and 16).

21. The polygon ABEF GCD is the union of the two overlapping squares. Its area is the sum of
the areas of the two squares minus the area of the overlapping part (shaded in the diagram below).
Each square has an area of 100. The overlapping region is exactly one-fourth of the square ABCD.
Its area is 25. Then the area of the polygon ABEF GCD is 100 + 100 − 25 = 175 square units.
E.
........
A ..........................................B ..
. .. .
.... .........
................. .....
..
..... ... ..
............
.....
.....
... .................. .....
.. ................. .....
.... . ............................................. .....
.
............................... ....
..
..
H ....................
........................ .....
F
.... .................. .......
... ............ .
....
.
. . .. .
.............................................................. .....
..... .....
D ..... ........
C .......
G

22. The boy did not give the same answer on two consecutive days. So, he did not give an answer
on both Thursday and Friday. If he missed Thursday then he started on Friday and went until
Wednesday. If he missed Friday then he started on Saturday and went until Thursday. In the first
case, he would have said “John” on Friday and then “Peter” on Tuesday. This is possible provided
his name is John. In the second case, he would have said “John” on Saturday, “Bob” on Tuesday,
and “Bob” on Thursday. This is impossible because he lies on Tuesdays and tells the truth on
Thursdays. So, He missed Thursday. He started giving his name on a Friday and said “John”. On
the seventh day, a Thursday, he told the truth and said “John”.

23. Let s be the sum of the four numbers Nick crossed out. The sum of the numbers Pete crossed
out is 3s. The sum of all eight numbers they crossed out is s + 3s = 4s. This is clearly a multiple
..................................................................................
... ... ... ...
...
... 4 12 ...
.
...
.
..................................................................................
8 ..
..
.. .. .. ...
.... .... .... ....
13 24 14
...
...
...
...
...
...
..............................................................................
...
.
... ... ... ...
... ... ... ..
....
7 ...
... 5 ...
... 23 ...
...
............................................................................
.

of 4. The sum of the original nine numbers is 4 + 12 + 8 + 13 + 24 + 14 + 7 + 5 + 23 = 110. This has


a remainder of 2 when divided by 4. Since the eight numbers crossed out add up to a 4-multiple,
the number not crossed out has a remainder of 2 when it is divided by 4. Among the nine numbers
in the square, 14 is the only one with a remainder of 2 when divided by 4. So, 14 is be the number
that neither Nick nor Pete crossed out. (One can also find by trial and error that Nick crossed out
4, 5, 7, and 8, and Pete crossed out 12, 13, 23, and 24.)

24. The perimeter of Emily’s bird consists of two quarters of a large circle, two quarters of a small
circle, two segments equal to a long side of the rectangle, and two segments equal to a short side.
.............
... ......
... ... ......................................
. ..
.. ........................................................ ....... .....
.
..
........... ... ..... .... ....
...... ... . .... ... ...
...
. ... ... .. ...
. ... ...
... ... ... .. ...
..... ..
.
...
. .
... ....
. .
..... ...
.
..
...
.. ..... ... ..........................................................................
.... .....
.. .. ..... ... ... ... .
...
........................................................................... ... ... ...
... ... ...
...... .... ... ......
... . ................................................ . . .
..
.... ...
....... ...
....

Emily’s Bird Benjamin’s Egg


The perimeter of Benjamin’s egg consists of two quarters of a large circle, two quarters of a small
circle, and one long side of the rectangle. The perimeter of the bird is larger by an amount equal
to one long and two short sides of the rectangle. This is 10 cm + 2(5 cm) = 10 cm + 10 cm = 20 cm.

25. Let r be Peter’s original speed, in k/h, let t be the number of hours it takes to travel from
P to Q, and let d the number of kilometres from P to Q. Since rate times time equals distance,
rt = d. If Peter increases his speed by 3 k/h, he needs only one-third of the time to get from P
to Q. So,
t rt
(r + 3) = d, and hence + t = d.
3 3
Multiplying the second equation by 3 gives rt + 3t = 3d. Multiplying the original equation, rt = d,
by 3 gives 3rt = 3d. Eliminating 3d from these equations gives 3rt = rt + 3t. Dividing throughout
by t gives 3r = r + 3. Subtracting r from both sides gives 2r = 3. Thus r = 3/2. (This is measured
in k/h.) If Peter increases his speed by 6 k/h. his new speed will be 6 + 32 = 15/2. This is five
times faster then his original speed. So, if he increases his speed by 6 k/h he will arrive 5 times
faster.
The University of the West Indies, Mona Campus
The 2015 Jamaican Mathematical Olympiad
First Round Examination, Grades 9, 10, and 11
January 17, 2015, at 9:30 am

Part A

This part consists of four multiple-choice


 questions. For each one, mark the letter for the correct
answer (a), (b), (c), (d), or (e) on Page 3 of the answer book provided. Each question in this part is
worth 5 marks.

1) When a positive integer n is multiplied by 7, the product is less than 2015. What is the largest
possible value of n?
a) 287 b) 267 c) 277 d) 257 e) 297

2) There were five people in a certain room. Each of them was either a liar who always lies or a
knight who always tells the truth. Each of them was asked, “How many liars are among you?”
Their answers were “one”, “two”, “three”, “four”, and “five”, respectively. How many liars were
in that room?
a) 1 b) 2 c) 3 d) 4 e) 5

3) In the five-pointed star below, three angles have the values shown (measured in degrees). What is
the measure of 6 C?
A
....
......
... ..
... .....
... ...
... ...
... ...
...
... ...
..
E ....... .
.
...................................................
.... . .
. . .
. .. ....
.. ...
.
..............................................
.......
....... .. ..... .. .
...
.
........ B
....... .. ..
... . . . .. ......... ..............
.
.. .. . .
.
... ........ .... . . ......................
100 .
...
.
.. ..
....... .
. .
..
....... ............... ..............
..
.......
....
....... ............ ....
. 93
58 ................. .................. ....... ..
. .... .........
....... ..
D C

a) 35◦ b) 51◦ c) 42◦ d) 109◦ e) 65◦

4) Which of the numbers below divides 102015 + 2 evenly?


a) 4 b) 5 c) 6 d) 9 e) 101008
Part B

This part consists of six written-answer questions. For each one, give your solution in the answer
book provided. Each question in this part is worth 10 marks. To score full marks, you must provide
an answer which is both correct and completely justified.

5) Suppose we toss three dice and add up the number of dots showing. How many different values
for the sum can we obtain? (We count the dots showing on the tops of the dice only.)

6) The sum of seven consecutive odd numbers equals 119. What is the smallest of these numbers?

7) In the figure below, ABC is an equilateral triangle and AKLB and BM N C are squares. What is
the measure of 6 ALM ?
K
....
...... ...
....... .....
......
...... ...
A .
...
.....
...
.. ...
...
.. .....
.
...
...
... .... .
.. ... ..... L
...
. ... ............
..
.
... .
....
....
. ... ......
....................................................
C ... .... B
...
... ....
... ..
... ....
..
.... ..
.............................................
N M

8) Peter took a quiz with 40 questions. He earned 2 points for each right answer and lost 1 point
for each wrong answer. Peter answered all the questions and received a score of 44. How many
questions did he answer correctly?

9) In the square ABCD below, DE is perpendicular to F C, DE = 4 and EC = 3. What is the


length of F B?
A F B
.............................................................................
..... ...
... ...
.. ... ...
.... ... ...
.. ... ...
.... ... ...
...
.. ... ...
.... ...... ...
..
.
...... ......
......... ....
E ...
... . .... ... ...
...
.... .
... ....
.. ........ .. ...
. .. ...
.... .
..... .... .....
.. ........... ... ..
... ..
.... ....... .
...............................................................................
D C

10) How many pairs of numbers (a, b) satisfy the conditions that a > b, a × b = 300, and a and b have
a greatest common factor of 1?
The University of the West Indies, Mona Campus
The 2015 Jamaican Mathematical Olympiad
First Round Solutions, Grades 9, 10, and 11
January 17, 2015, at 9:30 am
1. When 7 is divided into 2015 the quotient is 287 and the remainder is 6. The largest possible value
for n is 287.

2. It is not possible that two or more persons told the truth. This is because no two people gave the
same answer. So, either nobody told the truth or only one person told the truth. Thus there were
either four or five liars in the room. If all five persons were lying then the one who said “five” would
have told the truth. Since this is impossible, not all five people were lying. The only possibility is that
four persons were lying. (One person, the one who said “four”, told the truth.)

3. Let F , G, and H be the points of intersection shown in the diagram below. It is given that
6 AHG = 100◦ , 6 HDG = 58◦ , and 6 AF G = 93◦ . Since 6 DHG is the supplement of 6 GHA, 6 DHG =
180◦ − 100◦ = 80◦ . Since the sum of the angles in 4HDG = 180◦ , 80◦ + 58◦ + 6 DGH = 180◦. It
A
..
......
... ..
.. .....
.
... ....
... ...
.. ...
.
... ...
..
. ...
E ....................................................
.......
..
. ................................................................
.
.......
....... ...
....... ...
. ...
... .......
.......
B
.......
.. ... ..............
H .. ............
. ..... ..........
.
...
.
....... ....... ...
...
.................. ....
F
... ........ ....... ...
... .......... .........
........... G ...
D C

follows that 6 DGH = 42◦ . By vertical angles, 6 F GC = 42◦ as well. Also, 6 GF C is the supplement
of 6 GF A. Thus 6 GF C = 180◦ − 93◦ = 87◦ . Finally, since the sum of the angles in 4CF G is 180◦ , we
have 42◦ + 87◦ + 6 C = 180◦ . Thus 6 C = 51◦ .

4. Let n = 102015 + 2. The numeral for n begins with the digit 1, ends with the digit 2, and has 2014
zeroes between them. It is clear that n is a multiple of 2 and of 3. So, n is a multiple of 6. Also, n is
not a multiple of 4. (The last two digits, regarded as the “two-digit” number 02, is not divisible by 4.)
Clearly, n is not divisible by 5 or 9, either. Finally, it is obvious that n is not a multiple of 101008 . So,
the only number on the list that divides n evenly is 6.

5. The smallest possible sum is 3. (This happens if each die shows a 1). The largest possible sum is 18.
(This happens if each die shows a 6.) Also, any number in between is a possible value for the sum. So,
the values for the sum that may be obtained are 3, 4, 5, . . ., 18. There are 16 such values in all.

6. Let n be the smallest of the seven odd numbers. The next six odd numbers are n + 2, n + 4, n + 6,
n + 8, n + 10, and n + 12. The sum of these numbers is

n + (n + 2) + (n + 4) + (n + 6) + (n + 8) + (n + 10) + (n + 12) = 7n + 42.


Equating this to 119 gives 7n + 42 = 119. Dividing throughout by 7 gives n + 6 = 17. Subtracting 6
from both sides gives n = 11. The smallest of the numbers is 11.

7. Join AL and LM as shown in the figure below. Note that 6 M BC+6 CBA+6 ABL = 90◦ +60◦ +90◦ =
240◦ . Since the total angle around B is 360◦ , 6 LBM = 120◦ . Also, LB = BA

K
......
....... .....
......
.
...
....... ...
...
A ...
........
. ...
.............................. ..
.
.
.. ... ............ .....
..............
.. ... ....
...
. ...
... ..
.......... L
.. ...... ..
.
... ...
.. ............. ....
..
.................................................... ...
C ... ... B ...
... .... ...
... .. ...
...
... .... ....
... .. ...
... ... ...
..
..............................................
N M

(both are sides of the same square), BA = BC (both are sides of the same equilateral triangle), and
BC = BM (both are sides of the same square). Thus LB = BM . It follows that the triangle BLM
is isosceles with 6 BLM = 6 BM L = 30◦ . In particular, 6 BLM = 30◦ . Since the diagonal AL bisects
the angle KLB, we also have 6 ALB = 45◦ . Then 6 ALM = 6 ALB + 6 BLM = 45◦ + 30◦ = 75◦ .

8. Let x be the number of answers Peter got right, and y the number he got wrong. Since Peter
answered every question, x + y = 40. Since he received a score of 44, 2x − y = 44. Adding these
equations gives 3x = 84. Dividing both sides by 3 gives x = 28. Peter got 28 answers right (and 12 of
them wrong).

9. The triangle DEC has a right angle at E, and DE = 4 and EC = 3. By the Pythagorean theorem,
(DC)2 = (DE)2 + (EC)2 = 42 + 32 = 16 + 9 = 25. Therefore, DC = 5. That is, the side

A F B
..................................................................................
... ...
...
...
.... ... ....
.. ... ...
.... ... ...
... ...
.. ...
.... .
...
. ...
.. ....... ......
......... ....
E ...
...
.... ... ... ...
.. ... .. . ...
.... ..
. .
. . ...
... ...
.. .
.....
. .... .....
.... .......... ... ..
... ..
.. ....... .. .
.....................................................................................
D C

length of the square ABCD is 5. Also, the triangles DEC and CBF are similar. To see this, note
that 6 DEC = 6 CBF = 90◦ , and 6 DCE = 6 CF B (both are complements to 6 F CB). It follows that
6 EDC = 6 BCF as well (the sum of the angles in any triangle is 180◦ ). Since corresponding sides of
similar triangles are in the same proportion,
FB CE FB 3
= ; = .
BC ED 5 4
Multiplying the last equation by 5 on both sides gives F B = 15/4.

10. The factors of 300 are 1, 2, 3, 4, 5, 6, 10, 12, 15, 20, 25, 30, 50, 60, 75, 100, 150, and 300. The pairs
(a, b) such that a > b and a × b = 300 are:
(300, 1), (150, 2), (100, 3), (75, 4), (60, 5), (50, 6), (30, 10), (25, 12), (20, 15)
The pairs, (150, 2), (60, 5), (50, 6), (30, 10), and (20, 15) have a common factor greater than 1. (Their
greatest common factors are 2, 5, 2, 10, and 5, respectively.) The other pairs, namely (300, 1), (100, 3),
(75, 4), and (25, 12), have a greatest common factor of 1. There are 4 such pairs in all.
The University of the West Indies, Mona Campus
The 2015 Jamaican Mathematical Olympiad
First Round Examination, Grades 9, 10, and 11
January 17, 2015, at 12:30 pm

Part A

This part consists of four multiple-choice


 questions. For each one, mark the letter for the correct
answer (a), (b), (c), (d), or (e) on Page 3 of the answer book provided. Each question in this part is
worth 5 marks.

1) The result of dividing 20152015 by 2015 is:


a) 11 b) 101 c) 1001 d) 10001 e) 2015

2) Let n be equal to the product of all the positive integers from 11 to 29, inclusive. What is the ones
digit of n?
a) 0 b) 2 c) 4 d) 6 e) 8

3) How many four-digit numbers with all different digits are divisible by 2014?
a) 1 b) 2 c) 3 d) 4 e) 5

4) Each figure below shows a square with a side length of 24 which is filled with one or more shaded
circles. In each square, all circles are equal in size and tangent to their neighbors. Circles not
surrounded by other circles are tangent to the square they are in. Which figure has the largest
amount of shaded area?
.............................................................................................................................. .................................................................................................................................................... ........................................................................................................................................................ ................................................................................................................................................................................
.. ..... . . . . . . . . . . . ..... ... .. .......................................... .......................................... ... ............................... ............................. ................................ ..............................................................................
... ............................................ ............................................................ ............................................................ ................................................................... .............................................................................................................................................
................................................................... ..
....................................................................................................................................
.... ....................................................................................................... .... .................................................................................................... ...................................................................................................................................
.. .................................................................................. ... ............... ...............
.............................................................................................................................. ............................................................................................................................... .. ......................... .......................... .......................... .......................... ...
.... ................................................................................................................ ... . . . . . . . . . . . . . . .. . . . . . . . . . . . . . ....................................... .................................. .................................. .... ................................... ............................ ............................ ...............................
............................................................................................. ............................................................................................................................... .. .................................. .................................. .................................. ... . . . . . . . ... . . . . . ... . . . . . ... . . . . . .
..............................................................................................................................................
................................................................................................ ... ........................................................ ........................................................ .... .................................. ............................... .................................. . . . . . . . .. . . . . . . .. . . . . . . .. . . . . . ..
. . . . . . . . . . . . . . . . . . . . . . . . . . . . . ..
.............................................................................................................................. .... ..................................... .......................... ..
.......................... ..
...........................................................
.................................................................................................................................... .................................................................................................................................................
... .......................................... .... . . . .... . ... ............................... ............................... ............................... ............................... ...
............................................................................................................................. .. .......................................... .......................................................... ....
..............................
.................................................................................................................................. ..............................................................................
........................................................................................................................... ............................................................ ............................................................ ...................................... .................................. .................................. .... .............................................................................................................................................
............................................................................................... ..................................................................................................... .... ................................... ................................... ................................... ... ...................................................................................................................................
.. ................................................................................... ... .............................................................................................. ............................... ............................. ................................ .. ......................... .......................... .......................... .......................... ...
.... .......................................................................................................... ... .............................................................................................................................. .................................................................. ................................... ............................ ............................ ...............................
.. ......................................................................... ... .............................................................................................. ..................................................................................................................................... . . . . . . ... . . . . . ... . . . . . ... . . . . . ..
.... ................................................ ... .... ........................................................ ........................................................ ... ................................................................................................................................. ..............................................................................................................................................
. . . . . . . .. . . . . . . .. . . . . . . .. . . . . . ..
... ........................................................
.. ... .. ... .................................... ..... ............. ...... .. ... .................................. ................................. ................................... ... .........................................................................................................................................
...................................................................................................................... ......................................................................................................... ................................................................................................ .............................................................................................

Figure 1 Figure 2 Figure 3 Figure 4

a) Figure 1 b) Figure 2 c) Figure 3 d) Figure 4


e) There is an equal amount of shaded area in each figure.
Part B

This part consists of six written-answer questions. For each one, give your solution in the answer
book provided. Each question in this part is worth 10 marks. To score full marks, you must provide
an answer which is both correct and completely justified.

5) Carlos opened his dictionary and said, “If I add the number of the page I am looking for to the
number of the next page, I will get 341”. What page is Carlos looking for?

6) Mario saved $1,200. One day he spent 1/3 of his money on a present for his brother. The next
day he spent 1/4 of the remaining money on a book for himself. How much money did he have left
then?

7) The figure below shows a rectangle divided into squares of size 1 × 1. What is the area of the
triangle shown in bold?
......
........................................................................................................................................................
..
......... .....
. . . . . .
.................................................................................................................................................
...
.
. ..

....... ...
.. .. .. .. .. .. .. .. ...
.... .... .... .... .... .... .... ....
.......
...

... ...
..............................................................................................................................................
.
.......
..... ..... ... ..... ..... ..... ..... ..... ..... ....
..
.....
...............................................................................................................................................
...
.
.... ..... .... ....
... .... .... .... .... .... .... ...
.................
..........................................................................................................................................
... ...
..................
..
....
..
....
..
....
..
....
..
....
..
....
..
....
..
.... ...
.................
.................. .....
.
...............................................................................................................................................
.... ... ... ... ... ... ... ... ...
.
......... . . . . .
.......................................................................................................................................
.

8) The sum of the volumes of three pitchers and two bottles equals 16 litres. The volume of each
pitcher is twice as much as the volume of each bottle. What is the total volume of two pitchers
and three bottles?

9) In triangle ABC below, P and Q divide AB into three equal parts, and S and R divide CB into
three equal parts. The area of 4ABC is 1. What is the area of the shaded region?

.
A
.......
...... .....
..... ...
. ... ...... ...
..... ...
. . .... .. .. ...... ...
...
...............
.
P
. . .
.............................
.
. . .. .... . . . .
..
....
... .. ...
..................................
.. ... . . . . . . ...........
. . . . . . .
...... ....................................
..
...
.....
..... ... ..
........................................
....................................................................................................
Q
. . . . .
...... .............................................................. ...
..... ........................................... ...
..... ........................................... ...
...... ............................................ ..
....................................................................................................................................................................................................
C B
S R

10 There were a certain number of people in a room. The average age of these people was equal to
the number of people in the room. Then a 29-year-old person came in. It turned out that the
average age of the people in the room was once again equal to the number of people in the room.
How many people were in the room at the beginning?
The University of the West Indies, Mona Campus
The 2015 Jamaican Mathematical Olympiad
First Round Solutions, Grades 9, 10, and 11

January 17, 2015, at 12:30 pm

1. When 20152015 is divided by 2015 the quotient is 10001.

2. Note that 20 is one of the numbers between 11 and 29, inclusive. So, the product of the positive
integers between 11 and 29, inclusive, is a multiple of 20. Since 20 is a multiple of 10, this product is
also a multiple of 10. Its last digit is 0.

3. Suppose n is a four-digit number divisible by 2014. Then n is a four-digit number and is a multiple
of 2014. The four-digit multiples of 2014 are 2014, 4028, 6042, and 8056. Each of these numbers has
all of its digits different. So, there are 4 such numbers in all.

4. In Figure 1, the radius of the shaded circle is 12. The area of this circle is π(12)2 = 144π. In
Figure 2, the radius of each circle is 6. The area of each circle is π(6)2 = 36π. The total shaded area
................................................................................................................................ .................................................................................................................................................... ..................................................................................................................................................... ...............................................................................................................................................................................
... .... . . . . . . . . . . . ....
............................................
... ... ...................................................... ...................................................... ... ........................................... ........................................ ............................................ .............................................................................
.. ................................................................... .. .............................................. ............................................... .. . . . . . . . . ... . . . . . . . . ... . . . . . . . . .. ................................................................................................................................................
... ....................................................................................................... ..... .................................................................................................... .............................................................................................................................. ................................................................................................................................
. . .
.... ............................................................................................................. ... .. . . . . . . . . . . . . . . . . . . . . . . . . . . . . .
.............................................................................................................................. ........................................................................................................................................................ .... .......................... .......................... .......................... .......................... ...
.. ..................................................................................... ... . . . . . . . . . . . . . . .. . . . . . . . . . . . . . .. ........................... ........................... ........................... ... ............................. .......................... .......................... .............................
................................................................................................ ................................................................................................................................. .... ................................... ................................... ................................... ... .. . . . . . . .. . . . . . . .. . . . . . . .. . . . . . . .
..............................................................................................................................................
............................................................................................. .. ...................................................... ....................................................... ... ............................... ............................. ................................ . . . . . . . .. . . . . . . .. . . . . . . .. . . . . . .
. . . . . . . . . . . . . . . . . . . . . . . . . . . . . ..
.............................................................................................................................. ... ...................................... ............................ ...
.... . . . ... .
..........................................................
..................................................................................................................................... ................................................................................................................................................
. . . . . . . . . . . . . . . . . . . . . . . . . . . . . .. .... ........................................... .............................. .. .... ................................. ................................. ................................. .....................................
.............................................................................................................................. ... ...................................................... ................................................................... .... ........................................................................................................................................................... ......................................................................
......................................................................................................................... .............................................. ............................................... .. ........................... ........................... ........................... ... ................................................................................................................................................
............................................................................................. ................................................................................................... ... ................................... ................................... ................................... ... .................................................................................................................................
.... ............................................................................................................. .... ............................................................................................................................................................. .......................................... ...................................... ......................................... .... .......................... .......................... .......................... .......................... ..
.. ................................................................................ ... . . . . . . . . . . . . . . ... . . . . . . . . . . . . . . .. . . . . . . . . .. . . . . . . . . .. . . . . . . . . .. ............................. .......................... .......................... .............................
.... ........................................................................... ... ................................................................................................................................. .................................................................................................................................... . . . . . . . .. . . . . . . .. . . . . . . .. . . . . . . ..
.. ................................................................ ... .. ...................................................... ....................................................... ... ..................................................................................................................................... ................................................................................................................................................
. . . . . . . .. . . . . . . .. . . . . . . .. . . . . . .
... .........................................
. .. .. ... ....................................... ..... ............. ...... .. ... ..................................... .................................. .................................... ... ................................................................................................................................................
...................................................................................................................... ...................................................................................................... ............................................................................................ .....................................................................................

Figure 1 Figure 2 Figure 3 Figure 4

is 4(36π) = 144π. In Figure 3, the radius of each circle is 4. The area of each circle is π(4)2 = 16π.
The total shaded area is 9(16π) = 144π. In Figure 4, the radius of each circle is 3. The area of each
circle is π(3)2 = 9π. The total shaded area is 16(9π) = 144π. So, there is an equal amount of shaded
area in each figure.

5. Let p be the number of the page Carlos is looking for. The number of the next page is p+1. The sum
of these page numbers is p + (p + 1) = 2p + 1. This is equal to 341. That is, 2p + 1 = 341. Subtracting 1
from both sides gives 2p = 340. Dividing both sides by 2 gives p = 170. Carlos is looking for page 170.

6. On the first day, Mario spent 13 × $1200 = $400. After that, he had $1200 − $400 = $800 remaining.
On the second day, Mario spent 14 × $800 = $200 on a book. After that, he had $800 − $200 = $600
remaining.

7. Let the vertices of the rectangle be A, B, C, and D, and the vertices of the triangle be P , C, and Q
A ........................................................................................................P ..
................................ B
....
..............................
...
..
.... ..
....................
.... .... .... .. .... ....
.................................................................
.. .. .. .. ...
..... .. .. .. . .. ..
.. ... .. ...........
........................................... .................................................
.... ... ... .... .. ... ... ... ..... ....
.
..................
..
.... .........................................................................
.. ..
..... ..... ..... ..... ..... ..... ....... ....
................
.... .....
.............

Q ..........................................................................................................................................................................
. .. . . . . . .
.................................................................................
.......................................................
.. .. .. .. .... ............................. .. ........
........................................................................................ ...
..
... .......
...
D C
(see the figure above). The area of ABCD is 8 × 6 = 48. The area of 4AP Q is
1 1
(AP )(AQ) = (6)(4) = 12.
2 2
Similarly, the area of 4P BC is 12 (2)(6) = 6 and the area of 4CDQ is 12 (2)(8) = 8. Then the area of
4P CQ is the area of rectangle ABCD minus the combined area of triangles AP Q, P BC, and CDQ.
This is 48 − (12 + 6 + 8) = 48 − 26 = 22.

8. Let p be the volume of one pitcher and b the volume of one bottle. (All dimensions in this solution
are litres.) It is given that 3p + 2b = 16, and p = 2b. Substituting for p in the first equation gives
3(2b) + 2b = 16. Thus 6b + 2b = 16 and hence 8b = 16. Dividing both sides by 8 gives b = 2. One
bottle holds 2 litres. It follows that one pitcher holds 4 litres. The total volume, in litres, of 2 pitchers
and 3 bottles is given by 2(4) + 3(2) = 8 + 6 = 14.

9. We have BQ/BA = BR/BC = 1/3. Then the triangles BQR and BAC are similar and QR is
parallel to AC. Since the scaling factor between 4BQR and 4BAC is 1/3, the ratio between

A
...
...... ...
...... .....
...... ...
.. .. ....... ...
.. .......
..... ...
.......P
... .. ... ..
.....
...........................
..
.. . ... . .. .. ....................................
... . .
...........................................
.
...... ...................................
...... .......................................................... Q
...... ........................................... ...
.
.......... . ...
.. ........................................................... ...
.. ........................................ ...
...... ......................................... ...
...... .............................................. ...
...... ............................................ .
..................................................................................................................................................................................................
C B
S R

their areas is 1/9. Since the area of 4BAC is 1, the area of 4BQR is 1/9. Similarly, BP/BA =
BS/BC = 2/3. Then the triangles BP S and BAC are similar and P S is parallel to AC. Since the
scaling factor between 4BP S and 4BAC is 2/3, the ratio between their areas is 4/9. Thus the area
of 4BP S is 4/9. The area of the shaded region is 4/9 − 1/9 = 3/9 = 1/3.

10. Let n be the number of people in the room at the beginning, and let a1 , a2 , . . ., an be their ages.
Since the average age of the people was n,
a1 + a2 + · · · + an
= n.
n
Multiplying both sides by n gives
a1 + a2 + · · · + an = n2 . (1)
After the 29-year-old person entered, there were n + 1 people in the room and their average age was
n + 1. Thus
a1 + a2 + · · · + an + 29
= n + 1.
n+1
Multiplying both sides by n + 1 gives a1 + a2 + · · · + an + 29 = (n + 1)2 . Expanding the right sides
gives a1 + a2 + · · · + an + 29 = n2 + 2n + 1. Subtracting equation (1) from both sides gives 29 = 2n + 1.
Solving for n gives n = 14. There were 14 people in the room at the beginning
The University of the West Indies, Mona Campus

The 2015 Jamaican Mathematical Olympiad

First Round Examination, Grades 7 and 8


January 17, 2015, at 9:30 am

Part A

This part consists of four multiple-choice


 questions. For each one, mark the letter for the correct
answer (a), (b), (c), (d), or (e) on Page 3 of the answer book provided. Each question in this part
is worth 5 marks.

1) What is the value of 2003 × 3002?


a) 736 b) 61306 c) 6013006 d) 600130006 e) 60001300006

2) Suppose 29 is subtracted from the largest two-digit number and the difference is divided by 10.
What will the result be?
a) 6 b) 7 c) 9 d) 10 e) 11

3) There were five people in a certain room. Each of them was either a liar who always lies or a
knight who always tells the truth. Each of them was asked, “How many liars are among you?”
Their answers were “one”, “two”, “three”, “four”, and “five”, respectively. How many liars were
in that room?
a) 1 b) 2 c) 3 d) 4 e) 5

4) In the figure below, 6 BAE and 6 AED measure 90◦ and all sides of the pentagon ABCDE have
length 1. What is the measure of 6 BCA?

A ...............................................................B
.......
.
... .......
............... ......................... .
.. ............. .............
.... ............. .......
....................
... .................
.. ..... C
... .......
.... ...........
............ .
........
....
.. ... ......
.......................................................
E D

a) 15◦ b) 12◦ c) 30◦ d) 20◦ e) 18◦


Part B

This part consists of six written-answer questions. For each one, give your solution in the answer
book provided. Each question in this part is worth 10 marks. To score full marks, you must provide
an answer which is both correct and completely justified.

5) There are 800 students at our school. According to a survey, 50% of them have bicycles. Of the
students who have bicycles, 30% have a skateboard. What percent of the students at our school
have a bicycle and a skateboard?

6) Tom has $147 and Stan has $57. How much money should Tom give Stan so that Tom will have
twice as much money as Stan?

7) A shaded square is placed on the base of a rectangle with a width of 12 cm (see the figure below).
The square is 3 cm from one side of the rectangle and twice as far from the other side. The area
of the unshaded portion of the rectangle is 39 cm2 . What is the height of the rectangle?
.............................................................................................................................................................................
.... ...
... ...
... ...
.. ...
.... ................................................................................................................. ...
... .................................................. ...
... .................................................. ...
.. ................................................................. ...
.... ................................................... ...
... ................................................................ ...
... ................................................. ...
... ................................................... ..
.......................................................................................................................................................................................................................

......................................................................... .
............................................................................
.. 12

x − 3y x
8) Suppose = 12. What is the value of ?
y y

9) In the figure below, lines AD, BE, and CF intersect at O, 6 EOA = 108◦ , and 6 AOC = 124◦ .
What is the measure of 6 EOF ?

A.
...
...
..
◦ ....
F ......
...... 108 ..
..
..
. ... ◦
...... .......... ... .............
. 124
.........
............. ....
.
... ...
... ................
. . ...
B
. ...... .. ..................
..... ................
. ..
. ............................ ...
.....
... ...........
. .
..... ..............
........... O .. ......
E ....
...
......
......
....
...
..
...
C
...
.
D

10) A circus trainer needs 40 minutes to wash an elephant. His son needs 2 hours to do the same
job. How much time will the trainer and his son need to wash three elephants if they work
together?
The University of the West Indies, Mona Campus
The 2015 Jamaican Mathematical Olympiad
First Round Solutions, Grades 7 and 8
January 17, 2015, at 9:30 am

1. We have 2003 × 3002 = 6013006.

2. The largest two-digit number is 99. If 29 is subtracted from this number the difference will be 70.
If 70 is divided by 10 the result will be 7.

3. It is not possible that two or more persons told the truth. This is because no two people gave the
same answer. So, either nobody told the truth or only one person told the truth. Thus there were
either 4 or 5 liars in the room. If all five persons were lying then the one who said “five” would have
told the truth. Since this is impossible, not all five people were lying. The only possibility is that
four persons were lying. (One person, the one who said “four”, told the truth.)

4. Join B and D, as shown in the figure below. The quadrilateral ABDE is a parallelogram because
sides AB and ED are parallel and equal in length. Since it contains two right angles in the interior,

A ...............................................................B
..........
... . ..
............... ......................... .... ............
............. .
..
.... ... ............. .............
... ....................
... ... .................
.. ..... C
... ......
...
....
... .
............
............
... .
.......
.
... ........
.. ... ..
.........................................................
E D

ABDE is a square. Thus BD = AE = 1. It follows that BCD is an equilateral triangle. Then


6 ABD = 90◦ , 6 DBC = 60◦ , and so 6 ABC = 90◦ + 60◦ = 150◦ . The triangle ABC is isosceles with
AB = BC = 1. It follows that 6 BCA = 6 BAC = 15◦ .

5. According to the survey, 50% of the 800 students at our school have a bicycle. This is 400
students. Furthermore, 30% of these students have a skateboard. The number of students who have
a bicycle and a skateboard is

30 3 1200
× 400 = × 400 = = 120.
100 10 10
The ratio of students who have a bicycle and a skateboard to all students in our school is

120 12 3 15
= = = .
800 80 20 100

Then 15% of the students in our school have a bicycle and a skateboard.
6. Tom has $147 and Stan has $57. Between them, they have $147 + $57 = $204. When Tom has
twice as much money as Stan, Tom will have 2/3 of the total and Stan will have 1/3 of the total.
Then Tom will have $136 and Stan will have $68. Tom should give Stan $11.

7. The shaded square is 3 cm from one side and 6 cm from the other side of the rectangle. So, the
width of the square is 3 cm. Its area is square 9 cm2 . Te area of the unshaded region is 39 cm1 2. The
total area of the rectangle is 9 cm2 + 39 cm2 = 48 cm2 . Since the width of the rectangle is 12 cm, its
height is 4 cm.
..............................................................................................................................................................................
... ...
.. ..
.... ...
... .............................................. ....
... ................................................................... ...
.. ................................................... ...
.... ................................................................. ...
... .................................................. ...
... ................................................................. ...
.. ................................................. ...
.... .................................................................. ...
... ................................................. ..
......................................................................................................................................................................................................

......................................................................... ...........................................................................
. 12 .

x − 3y
8. Given that = 12, we may multiply both sides by y to see that x − 3y = 12y. Adding 3y
y
to both sides gives x = 15y. Dividing both sides by y gives x/y = 15.

9. Note that 6 F OA is the supplement of 6 AOC. Thus 6 F OA = 180◦ − 124◦ = 56◦ . Also,
6 EOF + 6 F OA = 6 EOA. Then 6 EOF + 56◦ = 108◦ . Subtracting 56◦ from both sides gives
6 EOF = 52◦ .

A.
....
...
.
F ◦ ......
......
...... 108
...... ...... .
........ ... ..........
..... 124 ◦
........ ..... ....
B
....... .. .. ..............
.... ............ ... .........................
.... ................... .
. .................................. ...
............. .. ...... .....
....... O .
. .
...
..... .....
......
......
E ... ......
......
..
.... C
...
..
...

10. If a circus trainer can wash 1 elephant in 40 minutes, he can wash 2 elephants in 80 minutes and
3 elephants in 120 minutes. His son can wash 1 elephant in two hours. This means he can wash 1
elephant in 120 minutes. Together, they can wash 4 elephants in 120 minutes. At this rate, working
together they can wash 1 elephant in 30 minutes. They can wash 3 elephants in 90 minutes, or 1 21
hours.
The University of the West Indies, Mona Campus
The 2015 Jamaican Mathematical Olympiad

First Round Examination, Grades 7 and 8


January 17, 2015, at 12:30 pm

Part A

This part consists of four multiple-choice


 questions. For each one, mark the letter for the correct
answer (a), (b), (c), (d), or (e) on Page 3 of the answer book provided. Each question in this part
is worth 5 marks.

1) How many hours are there in one-half of one-third of a quarter of a day?


1 1
a) b) c) 1 d) 2 e) 3
3 2

2) Alicia is packing 178 red and 121 blue blocks into boxes. Each box can hold no more than 10
blocks, and all blocks in the same box must have the same colour. What is the fewest number
of boxes Alicia will need?
a) 13 b) 18 c) 24 d) 30 e) 31

3) At the beginning of the game, John, Peter, and Karl had marbles in the ratio of 1 : 2 : 3. At
the end of the game, the marbles were divided between them in the ratio of 4 : 5 : 6. What was
the result of the game?
a) John and Peter lost, and Karl won.
b) John and Karl won, and Peter lost.
c) John lost, Karl won, and Peter kept the same number of marbles.
d) John won, Karl lost, and Peter kept the same number of marbles.
e) None of the situations listed above took place.

4) In the figure below, what is the measure of angle α?

... .....
... .....
... ..... ......
.... ◦ ..
...
..
.....................
. ..
.
..... ..
...
30 ............ .....
.... ............ ........
.... ................ ...
......................
. α ........
...
.... .....
... .....
.... .....
...... ..
......
... ◦ .... . ◦
... 50 ..... 40
....................................................................................................

a) 20◦ b) 25◦ c) 30◦ d) 35◦ e) 40◦


Part B

This part consists of six written-answer questions. For each one, give your solution in the answer
book provided. Each question in this part is worth 10 marks. To score full marks, you must provide
an answer which is both correct and completely justified.

5) Carlos opened his dictionary and said, “If I add the number of the page I am looking for to the
number of the next page, I will get 341”. What page is Carlos looking for?

6) Let s = 2012 + 2013 + 2014 + 2015 + 2016. If s is divided by 2015, what will the remainder be?

7) The star in the figure below is constructed from 12 identical equilateral triangles and has a
perimeter of 36 cm. What is the perimeter of the shaded hexagon?
....
... ...
.. .....
. . .. ...
. .
.................................................................................................................
...
... .. ............................................................................. .
...
... ................................................................................................ ...
... ................................................................................... ...
..................................................................................................................................
... .................................................................................. ...
... .............................................................................................. .....
... ............................................... ..
............................................................................................................................
... ..
... ...
... ...
... ...
....

8) An empty truck weighs 2,000 kg. After the truck was loaded, the freight (that is, the load)
made up 80% of the weight of the loaded truck. At the first stop, one-fourth of the freight was
unloaded. What percent of the loaded truck’s weight was the freight after that?

9) The rectangle below has been divided into 7 squares. Each shaded square has a side length of
8 cm. What is the side length of the largest unshaded square?
.......................................................................................................................................
... ....................................
.. ............................................
.... ......................................
... ....................................
... ..........
.. ............................................................................
.
.... .................................................
... ..............................................
... ....................................
.. ...................................
.... ..................................................................
... ......................
..................................................................................................................
.. .... .... .............................................
.... ... ... ......................................
... . . ......................
..........................................................................................................................................................

10) Let n be the smallest natural number such that the digits of n add up to 2015. What is the
first digit of n? (The first digit is the one furthest to the left.)
The University of the West Indies, Mona Campus
The 2015 Jamaican Mathematical Olympiad
First Round Solutions, Grades 7 and 8
January 17, 2015, at 12:30 pm
1. One day consists of 24 hours. A quarter of a day consists of 6 hours. One-third of a quarter of
a day consists of one-third of 6 hours, which is 2 hours. One-half of one-third of a quarter of a day
consists of one-half of 2 hours. This is 1 hour.

2. To pack the red blocks, Alicia needs at least 18 boxes. This is because 17 boxes can hold only
170 blocks, but 18 or more boxes can hold 180 or more blocks. Similarly, to pack the blue blocks she
needs 13 or more boxes. She needs at least 18 + 13 = 31 boxes in all.

3. At the beginning of the game, John had 1/6 of the marbles, Peter had 2/6 = 1/3 of the marbles,
and Karl had 3/6 = 1/2 of the marbles. At the end of the game, John had 4/15 of the marbles,
Peter had 5/15 = 1/3 of the marbles, and Karl had 6/15 = 2/5 of the marbles. John won because
he went from 1/6 to 4/15 of the marbles. (Using a common denominator of 30, he went from 5/30
to 8/30 of the marbles.) Peter kept the same number of marbles because he started and finished
with 1/3 of them. Karl lost because he went from 1/2 to 6/15 of the marbles. (Using a common
denominator of 30, he went from 15/30 to 12/30 of the marbles.) So, John won, Karl lost, and Peter
kept the same number of marbles.

4. Let x and y be the angles shown below. Since x is the supplement of a 30◦ angle, x = 180◦ −30◦ =
150◦ . Similarly, y = 180◦ − 40◦ = 140◦ . The sum of the angles in any quadrilateral is 360◦ . Thus
150◦ + 50◦ + 140◦ + α = 360◦ . That is, 340◦ + α = 360◦ . Then α = 20◦
..
... .....
.... .....
... ..... ...
. . .... ...
.
...........................
.
.. ◦ ...........
...... 30 .
..............................
.
... ......... ....
............ .....
.... ............ .....
..
... .............
........
α .
..
.....
.....
.... ..
x .....
...
... .....
... .....
... ........
.
.. ◦ .... ◦
....
.... 50 y .....
..... 40
......................................................................................................................

5. Let p be the number of the page Carlos is looking for. The number of the next page is p + 1.
The sum of these page numbers is p + (p + 1) = 2p + 1. Equating this to 341 gives 2p + 1 = 341.
Subtracting 1 from both sides gives 2p = 340. Dividing both sides by 2 gives p = 170. Carlos is
looking for page 170.

6. First, 2014 + 2015 + 2016 = (2015 − 1) + 2015 + (2015 + 1) = 2015 + 2015 + 2015 = 3(2015).
Second, 2012 + 2013 = (2010 + 2) + 2013 = 2010 + (2 + 2013) = 2010 + 2015. then
s = (2012 + 2013) + (2014 + 2015 + 2016) = (2010 + 2015) + 3(2015) = 4(2015) + 2010.
If s is divided by 2015 the quotient will be 4 and the remainder will be 2010.
7. The perimeter of the star consists of 12 equal segments and their total length is 36 cm. So,
each segment has a length of 3 cm. This means that each equilateral triangle has a side length
of 3 cm. The perimeter of the hexagon consists of 6 of these sides. The perimeter of the hexagon is
6 × 3 cm = 18 cm.
.....
... ...
.. .....
. ... ...
..................................................................................................................
... ............................. .
... ................................................................................... ...
... ............................................................. ....
.
... .................................................................................... ...
. . . . . . . . . . .. . . . . . . . . . ... .
.............................................................................................................................
... ............................................................ ..
.. ........................................................................................... .....
.... .................................................
. ... . . . . . . . .
...
.................................................................................................................
... .
... ....
... ...
... ...
..

8. After the truck was loaded, the freight made up 80% of the total weight. So, the truck itself was
20% of the total weight. Let w be the total weight (measured in kg) of the truck and the freight.
Then
2000 20 1
= =
w 100 5
Cross-multiplying gives w = 10, 000 kg. Then the truck weights 2,000 kg and the freight weighs
8,000 kg. At the first stop, one-fourth of the freight was unloaded. This means that 2,000 kg was
unloaded and 6,000 kg remained on the truck. The combined weight of the truck and freight was
then 8,000 kg. The weight of the freight compared to the total weight was
6000 3 75
= =
8000 4 100
The freight made up 75% of the total weight.

9. Let x be the side length of the largest unshaded square. It is clear from the diagram that the side
length of the smaller unshaded squares is x/3. Also, the side length of each shaded square is 8. The
..........................................................................................................................
.. ...................................
.... ...............................................
... ....................................
.. ...................................
.... .......................
... ............................................................................
... .....................................
.. .............................................
.... .....................................
... ....................................
... ..............................................
.. ..........................................
................................................................................................................................
... ... ... ....................................
.. ... ... ...................................
.... ... ... ...............................................
.. .. .. .. . . . . . . . . ..
.......................................................................................................................

height of the outer rectangle is equal to the combined height of the three shaded squares. This is
24 cm. This height is also equal to the sum of the heights of the large unshaded square and one
small unshaded square. This is x + x/3 = 4x/3. Then 4x/3 = 24. Multiplying both sides by 3 gives
4x = 72. dividing both sides by 4 gives x = 18. The side length of the largest unshaded square is
18 cm.

10. If the numeral for a certain positive integer has more digits than the numeral for another one,
the number with more digits is the larger number. Also, the largest digit is 9. When 9 is divided
into 2015 the quotient is 223 and the remainder is 8. So, any integer whose digits add up to 2015
must have at least 224 digits. Furthermore, if the number has 224 digits then 1 of them must be
an 8 and the other 223 must be a 9. The smallest such number is 89,999,. . .,999. The first digit of
this number is 8.
The 2015 Jamaican Mathematical Olympiad
Practice Problems Set 1

1) Nickiesha ate one sweetie one day, and each day afterward she ate one sweetie more than he did the day
before. How many sweeties did she eat during her first week?

2) How many rectangles may be found in the figure below?


................................................................................................................................................................
.... .. .. .. ...
... ... ... ... ...
... ... ... ... ...
... ... ... ... ..
....................................................................................................................................................................
... ... ... ... ...
... ... ... ... ...
... ... ... ... ...
... ... ... ... ...
..............................................................................................................................................................

3 Anna bought a pizza. She gave two-thirds of it to her sister and one-half of the remaining part to her
mother. She ate the rest. What portion of the pizza did Anna eat?

4) The pattern below repeats every six symbols. What are the 100th and 101st symbols (in this order) in
the pattern?
♥ ♣ ♥ ♦ ♥ ♠ ♥ ♣ ♥ ♦ ♥ ♠ ···

5) Which number is half way between 2006 and 6002?

6) In the figure below, each small square has area 1 cm2 . What is the area, in square centimetres, of the
shaded region?

......................................................................................................................................
... . .. . .. .. ...
................... ...
... ..................................................... .... ... ...
... ................................................... ... .. ..
..............................................................................................................................................................................
............................................................................................... ..
...
... ............................................................................................
... ................................................................................................................... .....
... ................................................................ ..
.........................................................................................................................................................................................................................
... ...............................................................................................................
... .......................................................................................
... ......................................................................................
... ..................................................................................
.............................................................................................................................................................................................................
............................................................... ... ...
..
........................................................... ... ...
...................................... ... ... .....
........................... ... ... ... ..
.........................................................................................................................

7) A certain lock uses three numbers, each with one digit, for a combination. For example, 9-1-0 and 4-5-5
could be combinations. How many combinations are possible that use three different odd digits?

8) Suppose you count from 1 to 100 and clap your hands when you count either a multiple of 3 or a number
which is not a multiple of 3 but ends in 3. How many times will you clap your hands?

9) Shania subtracted the smallest three-digit number with all different digits from the largest three-digit
number with all different digits. What was the result?

10) All together, 6 chicks eat 8 cups of grain in 3 days. How many cups of grain will 3 chicks eat in 9 days?
11) Olympiette is a small country with exactly seven towns. No three towns lie on the same straight line.
Each pair of towns is connected by one and only one straight road. Determine the number of straight
roads that connect the seven towns.

12) Akeem’s uncle needs 12 minutes to walk around a square plaza. How many minutes will he need to
walk at the same pace around a plaza that has an area four times greater?
................................................. ....... ....... ....
... ... .
... ... ...
... ... ..
... ...
...
... ...
... ... ..
.......................................... ...
..
... ...
.
..
...
.. ...
..
...
. ....... ....... ....... ....... ....... ......

13) How many four-digit numbers abcd are multiples of 3, 4, and 5 and satisfy the conditions that a is twice
as much as c, and b is equal to 6?

14) An odd whole number between 600 and 800 is divisible by 7 and by 9. What is the sum of its digits?

15) In the figure below AB is parallel to CD, and EF meets AB at G. What is the value of y?

. F
...
...
...

G ..
.. x
......................................................................................................................................................................................
A . ...
.. ........... ◦
B
.
... ◦ ...........
. 40
...
..
.
y ......
......
......
... ......
......
... ......
... ......

.... ......
...... 2x
.....................................................................................................................................................................................
..
C .
..
D
. H
...
...
...
E

16) The sum of the ages of Abigail and Brianna is 25. The sum of the ages of Abigail and Christina is 20.
The sum of the ages of Brianna and Christina is 31. Which of these three people is the oldest and what
is her age?

17) In the figure below, a triangle is inscribed in a regular hexagon. What proportion of the area of the
hexagon is shaded?
........................................................
.. ......... ...
... ...
... ........ ...
.... .........
....................... ...
.
.. .. ...
.... .......................... ...
...
..
. ................
. ...
.... ................................. ...
.
.. .........................
.
. ...
.... .
............................... ...
... . ....................
. . . . . . . . . . . ..
...
... .
.
...............................................
. . . . . . . . . .
.....
... .......................................
. .
...
...
............................................. .
............
... ...
... .............................................................. ....
... .................................. ...
... .................................................. ...
..........................................................
....................................................

18) If 4x = 9 and 9y = 256, what is the value of xy?

19) What is the sum of the digits of the square of 111, 111, 111?
20) Suppose M is 30% of Q, Q is 20% of P , and N is 50% of P . What is the value of M/N ?
3 6 3 4
a) b) 1 c) d) e)
250 5 25 3

21) Emily plants 60 tulip bulbs. When they flower, she notices that half are yellow, one third of those that
are not yellow are red, one quarter of those that are neither yellow nor red are pink. The remainder are
white. What fraction of the tulips are white?

22) In the figure below, ABCD is a rectangle with AB = 5 and AD = 3. The points E and F divide AC
into three equal parts. What is the area of triangle BEF ?

A ..........................................................................................................................
... .......... ...
B
... ......
...... ...
... ...... ...
•E
... ......
......
...
...
... ...... ...
... ......
...... ...
... ......
... ...... .....
•F
... ...... ...
......
... ......
......
...
... ...... ....
... ......
...... ..
... ...... ....
... ...... ...
....................................................................................................................
D C

23) The positive real numbers a, b, c, d, and e satisfy the conditions ab = 2, bc = 3, cd = 4, and de = 5.
What is e/a?

24) Suppose a and b are positive real numbers greater than 1. Which of the following fractions has the
greatest value?
a a 2a 2a 3a
(a) (b) (c) (d) (e)
b−1 b+1 2b + 1 2b − 1 3b + 1

25) In the figure below, ABC is a right triangle with a right angle at B, and BD is an altitude. (So, 6 BDA
is also a right angle.) If AD = 64 and AC = 100, what is AB?

B
...........
......... .........
.........
......... .... ....
.
.....
........... ... ....
..
......... ... ....
....
............ .
. ...
...
......... .... ...
........ ...
........ ..
.....................................................................................................................................
A C
D

26) What is the maximum number of digits a number can have if every pair of consecutive digits is a perfect
square?

27) The average age of a group of doctors and lawyers is 40. If the doctors average 35 and the lawyers 50
years old, what is the ratio of the number of doctors to the number of lawyers?

28) In the figure below, the quadrilateral ABCD is inscribed in a circle with center O. Suppose 6 OAB =
35◦ , 6 OBC = 40◦ , and 6 OCD = 55◦ . What is 6 ODA?
................................
A...................................... ......
.....
....
... .. ............ ....
.. ... ............
.. . . ............... ....
... ... ............ ...
.. ...
.
.. .
.. ...
......
..
... ...
B
..... .... ... ..
. • .... ...
.
... ..
... ... ...
. ...
... ..
......
O ..
... ...
..
...... ..... ...
.
........... . .
D .... ........
..... .........
...... ......... ...
...
....
...
...
........ . ... .......
..................................................

C
29) The ages of Juan and his sister add up to half their father’s age. If Juan is 3 years older than his sister
and 27 years younger than his father, how many years old is Juan?

30) In how many ways can the numbers 1, 2, 3, 4, 5 and 6 be written in the squares in the figure below so
that no numbers in adjacent squares differ by 3? (Squares that share only a corner are not considered
adjacent.)
.................................................................................................
... ... ... ...
... ... ... ...
... ... ... ...
... ... ...
...
...
...
.... ....
..................................................................................................
... ... ..
... .... ....
... ... ...
... ... ...
... .... ...
... ..................................
... ...
...
...
...
... ... ...
... ... ...
...
.... ..... ....
............................................................................................
The 2015 Jamaican Mathematical Olympiad
Solutions for Practice Problems Set 1

1. On the first day, Nikeshia ate 1 sweetie. On the second day she ate 2 sweeties. On the third day she ate
3 sweeties, and so on. During the first week, she ate 1 + 2 + 3 + 4 + 5 + 6 + 7 = 28 sweeties.

2. In the top row, there are 4 small rectangles as shown here:


................................................................................................................................ ...................
.................................................................................................... ...................
.................................................................................................... ...................
....................................................................................................
......................................................................................................................................... ...
....................
... ... ...
.................... ....................
... ... ... ... ... ... ... ... ... ... ...
... .. .. .. . ... .. .. .. .. ... .. .. .. ..
.......................................................................................................... ........................................................................................................ ........................................................................................................
... ... ... ... ... ... ... ... ... ... ... .... ... ... ... ... .... ... ... ... ...
.... ... ... ... . ... ... ... ... .. .... ... ... ... .. .... ... ... ... ..
............................................................................................... ............................................................................................. ........................................................................................... ...........................................................................................

There are 3 more rectangles consisting of two small ones side-by-side:

.............................................................................................................................................................. ... ....................................


.................................................................................................
... ... ... ... ... ... ...
..................................
.................................................................................................
...
... ...
..................................................................................................................................................... ...
................................
... ... ... ... ...
................................
... ...
... ...
...................................................................................................... ......................................................................................................
. . . . ... ... ... ... ... ... ... ... ... ...
.... ... ... ... ...
. ... ... ... ... .. ... ... ... ... ..
................................................................................................... ................................................................................................. ................................................................................................

There are 2 more rectangles consisting of three small ones side-by-side, and 1 more which is the entire top
row of the diagram.

......................................................................................................................................................................... ...
....................................................
.................................................................................................
.. ..
...
..
...
..
...
.
....
............................................................................................................................................................................
...................................................................................................................
.................................................................................................................................................................. ...
...............................................
....................................................................................................
... ... ... ... .................. ........................................................
....
... . .. ... .. . ... . . ... ... ... ... .. ... .... .... .... ....
...
.................................................................................................. ............................................................................................... .............................................................................................

Then there are 4 + 3 + 2 + 1 = 10 rectangles in the top row. In the same way, there are 10 rectangles in the
bottom row. Finally, there are 10 more rectangles which span both rows. Four of there are shown here:

.......................................................................................................................................... ...................
...........................................................................................................
... .. ...
...
...................
.....................................................................................................
... ...
... ... ...
...
... ...
...................
.....................................................................................................
... ...
... ...
.......................................................................................................................... ...
...
..
...
..
...
..
........................................................................................................
.. .... ...
..
...
.
.........................................................................................................
... ... ... ..
... ... ...
.
.........................................................................................................
... ... ... ... ..
... ...
........ .... ... ...
...
........................................................................................................................
... ...
...
..................
... ... ... .................. ..................
. ... .
. .
. .
. .. .
. .
. .
. ..
..................................................................................................... ......................................................................................................... .........................................................................................................

A total of 30 rectangles may be found in the diagram.

3. After Anna gave two-thirds of the pizza to her sister, she had one-third of it left. After she gave half of
that to her mother, the other half was the portion that she ate. Then Anna ate one-half of one-third of the
1 1 1
pizza. This is × = of the total pizza.
2 3 6

4. The sixth symbol in the sequence is the spade suit (♠). Since the pattern repeats every six symbols, the
12th, 18th, 24th, and so on, symbols are all spade suits. This will continue through the 96th symbol, which
is a spade suit again. The 97th symbol is the heart suit (♥), the 98th symbol is the club suit (♣), and the
99th symbol is the heart suit. The 100th symbol is the diamond suit (♦) and the 101st symbol is the heart
suit. The answer is ♦ ♥.

5. If m is half way between 2006 and 6002 (as shown in the figure on the • •
.............................................................................................................................. •
right) then m − 2006 = 6002 − m. Thus 2m = 8008 and so m = 4004. 2006 m 6002

6. The shaded region consists of 5 small squares completely shaded, four small shaded triangles as in
Figure 6(b), and one larger shaded triangle as in Figure 6(c). (See the figure on the next page.) The area of
each small shaded square is 1 cm2 . Each small shaded triangle has area 1/2 cm2 . The larger shaded triangle
............................................................................................ ..........................
.... ............. . .. ... .... .........
... ...................................... .... ... ... ... ......................
... ...................................................... .. .. ... ..............................
..................................................................................................................................................... ................................
... ................................................................................ ...
... ............................................................................................... .... ........................
... ..................................................... .. ................ ..
.................................................................................................................................................................. ................... ....
... .......................................................................................... ............................ ...
... ..................................................................... ..............................
... ..................................................................
............................................................................................................................................................ ...................... ..........................................
................................. ............................................................ .....
....... .............
.............................................. ... ... ...
.................................. ... ... ...
... ... .................. ................................ ...
.......................... ... .. .. ... ........... .......................... ... ..
............................................................................................. .......................... ................................................

Fig. 6(a) Fig. 6(b) Fig. 6(c)

is half of the rectangle that contains it. The area of the rectangle is 2 cm2 , and the area of the shaded triangle
is 1 cm2 . The total shaded area, in square centimetres, is 5(1) + 4(1/2) + 1 = 5 + 2 + 1 = 8.

7. There are 5 odd digits that could be the first number in the combination. (They are 1, 3, 5, 7, and 9.)
For each one, there are 4 others that could be the second number in the combination. For each pair chosen to
that point, there are 3 other digits that could be the last number in the combination. There are 5×4×3 = 60
possible combinations in all. To check this, note that there are 12 combinations whose first number is 1:

1-3-5 1-5-3 1-7-3 1-9-3


1-3-7 1-5-7 1-7-5 1-9-5
1-3-9 1-5-9 1-7-9 1-9-7

Similarly, there are 12 combinations whose first digit is 3, 12 whose first digit is 5, 12 whose first digit is 7,
and 12 whose first digit is 9. This gives 60 combinations in all.

8. The multiples of three from 1 to 100 are 3, 6, 9, 12, . . ., 99. These are the numbers 3 × 1, 3 × 2, 3 × 3,
3 × 4, . . ., 3 × 33. So when you clap your hands on the multiples of 3, you clap 33 times in all. The numbers
ending in 3 are
3, 13, 23, 33, 43, 53, 63, 73, 83, and 93
The numbers 3, 33, 63, and 93 are multiples of 3 and have already been counted. The other numbers, 13,
23, 43, 53, 73, and 83, are not multiples of 3. These are 6 more numbers that you clap your hands on. So
there are 33 + 6 = 39 numbers that you clap your hands on.

9. The smallest three-digit number is 100. However, not all of its digits are different. The next smallest
number is 101, but not all of its digits are different either. The next smallest number is 102, and all of its
digits are different. The largest three-digit number is 999, but not all of its digits are different. None of the
next largest numbers, 998, 997, 996, . . ., down to 988, have all different digits either. The largest number
with all different digits is 987. Then Shania subtracted 102 from 987 and obtained 885.

10. We know that 6 chicks eat 8 cups of grain in 3 days. During the same time, half as many chicks would
eat half as much grain. So, 3 chicks would eat 4 cups of grain in 3 days. These chicks would eat three times
as much grain during three times as many days. So 3 chicks would eat 12 cups of grain in 9 days. (Note:
One can determine that 1 chick eats 4/9 cups of grain in 1 day.)

11. Suppose the towns are called A, B, C, D, E, F , and G. Since A is connected to each of the other
towns, there are 6 roads from A to another town. (There is one from A to B, one from A to C, and so on.)
There are 5 more roads from B to another town. (These do not include the one from B to A, which has
already been counted). There are four more roads from C to another town, three from D to another town,
two from E to another town, and one from F to G. There are a total of 6 + 5 + 4 + 3 + 2 + 1 = 21 roads in
Olympiette.
12. Akeem’s uncle needs 3 minutes to walk along each side of the smaller plaza. The 3 3
................................................... ....... ....... .....
sides of the larger plaza are twice as long as the sides of the smaller one. His uncle ... ...
.... ... ...
.. ... ..
will need 6 minutes to walk along each of side of the larger plaza. He will need 3 ... 3 ...
... ...
.... ... ..
.. .
6 + 6 + 6 + 6 = 24 minutes to walk around the larger plaza. .......................................
...
...
..
.3 ...
..
..
.. ...
..
...
.. ....... ....... ....... ....... ....... ......

13. Since b = 6 the number to be found has the form a6cd. Since this number is a multiple of both 4 and 5,
it is a multiple of 20. Thus it has the form a600, a620, a640, a660, or a680. We consider each possibility
in turn. Since a is twice as much as c, the number in the first case would be 0600. However, this is not
a four-digit number and must be discarded. The number in the second case would be 4620. This is also a
3-multiple and so it satisfies all stated criteria. The number in the third case would be 8640. This is also
3-multiple and it also satisfies all stated criteria. The numbers in the last two cases are impossible because
no single digit a can be twice as much as 6 or 8. There are two numbers that satisfy all stated criteria: 4620
and 8640.

14. For a whole number to be divisible by both 7 and 9, it must be a multiple of both 7 and 9. But a
multiple of both 7 and 9 is a multiple of 63. The multiples of 63 between 600 and 800 are 630, 693, and 756.
The only odd multiple is 693. The sum of its digits is 6 + 9 + 3 = 18.

15. The lines AB and CD are parallel and the line GH crosses them ...
F
..
..
both. Then the angles BGH and GHD are supplementary. That is, G .... x◦
..............................................................................................................................................
A .........
... ...........
B
40◦ + 2x◦ = 180◦ . Then 2x◦ = 180◦ − 40◦ = 140◦ and hence x = 70◦ . ...
.. ◦ ...........
..... 40 ◦
.
.. y ......
Since F GE is a straight line, the angles F GB, BGH, and HGE sum .
...
.......
......

... ......
...... 2x
to 180◦ . That is, x◦ + 40◦ + y ◦ = 180◦ . Then 70◦ + 40◦ + y ◦ = 180◦ C
.. .
...................................................................................................................................................
..
D
.
and so 110◦ + y ◦ = 180◦ . Thus y ◦ = 180◦ − 110◦ = 70◦ . ..
... H
..
E

16. Let the ages of Abigail, Brianna, and Christina be denoted by A, B, and C, respectively. From the
information given, A + B = 25, A + C = 20, and B + C = 31. From the first two equations, A = 25 − B and
A = 20 − C. Then 25 − B = 20 − C and so B − C = 5. Adding this to the third equation gives 2B = 36
and so B = 18. Substituting this into the original equations gives A = 7 and C = 13. Then Abigail is 7,
Brianna is 18, and Christina is 13. In particular, Brianna is the oldest and she is 18.

17. Consider a triangle with the same base as the given triangle and with one vertex being the centre of
the circle. (For example, consider the shorter, darker triangle in the figure below.) The area of the shorter
triangle is 1/6 the area of the hexagon. The taller triangle
............................................................
... ..... ...
... .......... ...
... .......... ...
.... .. .................. ...
... .
.................. ...
. . ...
.... .................................. ...
... ......................... ...
.. .................... ...
. . ...
..... ..............................................
.......
...
... ..
. •
..............................
.
................................................ . ..
...
.
... .. . . ..
..........................................................................................
. . . . . . ..
... . .
.
... ... ............. ............ .. ..
... ........................................................................... ...
... ....................................................................................................................... .....
... ...................................................................... ..
... ................................................................................................. ..
................................................................................................................
...........................................................

has twice as much area because it has the same base as the shorter one and twice its height. Then the
proportion of the area of the hexagon which is contained in the taller triangle is
 
1 2 1
2 = =
6 6 3
18. From the information given, 4xy = (4x )y = 9y = 256. On the other hand, we also have 44 = 4×4×4×4 =
256. Then 4xy = 44 and it follows that xy = 4.

19. Using the usual multiplication algorithm,

111, 111, 111 × 111, 111, 111 = 12, 345, 678, 987, 654, 321

The sum of the digits in this product is 1 + 2 + 3 + · · · + 8 + 9 + 8 + 7 + · · · + 1 = 81.

20. From the information given, M = 0.30Q, Q = 0.20P , and N = 0.50P . Then M = 0.3Q = 0.3(0.2P ) =
0.06P . Since N = 0.5P ,
M 0.06P 0.06
= =
N 0.5P 0.5
 
M 0.06 100 6 3
Multiplying top and bottom by 100 gives = = = .
N 0.5 100 50 25

21. Half of the tulips are yellow and half are not. So, 30 tulips are yellow and 30 are not. Among the 30
tulips not yellow, one-third are red. So, 10 tulips are red. Thus 30 + 10 = 40 tulips are either yellow or
red. This leaves 20 tulips of neither colour. Among them, one-fourth are pink. So, 5 tulips are pink. Thus
30 + 10 + 5 = 45 tulips are yellow, red, or pink. The remaining 15 tulips are white. The fraction of the tulips
that are white is 15/60 = 1/4.

22. From the information given, the rectangle ABCD has area 15. It follows that the triangle ABC has
area 15/2. Furthermore, the triangles BAE, BEF , and BF C all have

A •........................................................................................................................•
...... B
.... .......... ........ .... ....
...... .......... ... ....
... ...... ...................
.. ...
...
...
• ..........
......
...... ....
...
...
...
.... E ......
......
...... ... ...
.
. ...
...
.. ...
... •
.......
......
......
...
..
....
..
F ......
...... .....
.
......................................................................................................
D• •C

equal bases and equal heights. Thus they have equal areas. Then the area of BEF is 1/3 the area of ABC,
and this is
1 15 15 5
× = = .
3 2 6 2

2 3 2 3 3a
and b = . Then = and it follows that c =
23. From the first two equations, b = . From the next
a c a c 2
4 5 4 5 4e 4e 3a
two equations, d = and d = . Then = and it follows that c = . Since c = c we have = .
c e c e 5 5 2
e 15
Then 8e = 15a and so = .
a 8

24. Suppose first that a, b, c, and d are any positive numbers. If a/b > c/d then, multiplying both sides
by bd (which is positive), ad > bc. On the other hand if ad > bc then, multiplying both sides by 1/bd, we
have a/b > c/d. Thus we may say that
a c
> if and only if ad > bc
b d
We now show that a/(b − 1) is larger than any of the other four quantities. First, a/(b − 1) > a/(b + 1).
This is because b − 1 < b + 1 and when a is divided by the smaller quantity its quotient is larger. Second,
a/(b−1) > 2a/(2b+1). To see this, cross multiply to get a(2b+1) > (b−1)(2a) and hence 2ab+a > 2ab−2a.
Subtracting 2ab from both sides gives a > −2a. Since a is positive, this is true and validates the inequality
we started with. Third, a/(b − 1) > (2a)/(2b − 1). To see this, cross multiply to get a(2b − 1) > (b − 1)(2a)
and hence 2ab − a > 2ab − 2a. Subtracting 2ab from both sides gives −a > −2a and hence a < 2a. Since a
is positive, this is true and validates the inequality we started with. Finally, a/(b − 1) > 3a/(3b + 1). To see
this, cross multiply to get a(3b + 1) > (b − 1)(3a) and hence 3ab + a > 3ab − 3a. Subtracting 3ab from both
sides, a > −3a. Since a is positive, this is true and validates the inequality we started with. In summary,
the largest of the five quantities is a/(b − 1).

25. The triangles ADB and ABC are similar. This is because both are right triangles with a common angle
at A. Since the sum of the angles in 4ADB and in 4ABC are the same (both sums are 180◦ ),

90◦ + 6 A + 6 ABD = 90◦ + 6 A + 6 ACB.

It follows that 6 ABD = 6 ACB as well. Since corresponding sides in similar triangles are in the same
proportion, AD/AB = AB/AC. Cross-multiplying gives (AD)(AC) = (AB)2 . Substituting 64 for AD and
100 for AC gives (AB)2 = 6400. Thus AB = 80.

B
.........
........ .........
.........
......... .... .....
................ ... ...
......... ... ....
..
............... ... ....
..
. .
.......... ... ...
. .
........... .. ...
....
......................................................................................................................................
A C
D

26. The two-digit perfect squares are 16, 25, 36, 49, 64, and 81. So suppose every pair of consecutive digits
of a number is a perfect square. If the digit 1 is present the next digit must be 6, when 2 is present the
next digit must be 5, and so on. Note that whenever 5, 7, or 9 occurs there cannot be a next digit. Among
other things, this means that the first digit of the number cannot be 5, 7, or 9. We consider each of the
possibilities 1, 2, 3, 4, 6, and 8 in turn. If the first digit of the number is 1 it has to be 1649. If its first digit
is 2 it has to be 25. If its first digit is 3 it has to be 3649. If its first digit is 4 it has to be 49. If its first
digit is 6 it has to be 649. And if its first digit is 8 it has to be 81649. The longest such number is 81649
and it has five digits.

27. Suppose there are d doctors and ` lawyers in the group. Let a1 , a2 , . . ., ad be the ages of the doctors
and b1 , b2 , . . ., b` the ages of the lawyers. Since the average age of the doctors is 35,
a1 + a2 + · · · + ad
= 35.
d
Multiplying both sides by d gives a1 + a2 + · · · + ad = 35d. Since the average age of the lawyers is 50,
b1 + b2 + · · · + b`
= 50.
`
Multiplying both sides by ` gives b1 + b2 + · · · + b` = 50`. Since the average age of the whole group is 40,
(a1 + a2 + · · · + ad ) + (b1 + b2 + · · · + b` )
= 40.
d+`
Multiplying both sides by d + ` gives (a1 + a2 + · · · + ad ) + (b1 + b2 + · · · + b` ) = 40(d + `). The left side is
equal to 35d + 50`. The right side is equal to 40d + 40`. So, 35d + 50` = 40d + 40`. Subtracting 35d + 40`
from both sides gives 10` = 5d. Then d/` = 10/5 = 2/1. Thus d : ` = 2 : 1.
.................................
............ ........
........
28. Let 6 ODA = x◦ . Note that OA = OB = OC = OD because A...• ......
.......
........................
.......
......
.....
....
... ... ..... ................... ....
OA, OB, OC, and OD are all radial segments. Then 4OAB, 4OBC, . .
... ....
. .
.. ... .....
.... ◦
............
.. ...............
.
...
...
... .
4OCD, and 4ODA are isosceles. Thus 6 OAB = 6 OBA = 35◦ , ..
.
. ◦ 35
... ..............
............ .....
..
.
.
x .
..
.
...
...
....
..............
◦ ......................•B
.... .... .... 35 ....... ... ...
6 OBC = 6 OCB = 40◦ , 6 OCD = 6 ODC = 55◦ , and 6 ODA = ..
. .. ....
.........
.......... . .
.. ... ....
... .................. ◦ ...... ....
.... ... 40
6 OAD = x◦ . Since the sum of the angles in any quadrilateral is 360◦ , .. .... • ..
.........
.
.... ... . ...
.. ...
...
...
... ...
.
.
.......
.......
......
O ..
.. ..
..
...
..
..
2(35) + 2(40) + 2(55) + 2x = 360. Dividing throughout by 2 gives ... ... ◦ ... . . .
x ... ..
... ... ............
......
.. .
..
.. ...
... .
...
..
..... ........ ◦ ..
... ... ...
..
35 + 40 + 55 + x = 180 and hence 130 + x = 180. Subtracting 130 D • 55 ...........
...........
... .........
..
. ◦ ..... ......
... ....... 40
.
◦ ..... ......
.
...
...
from both sides gives x = 50. Then 6 ODA = 50◦ . ....
......
.
..... ...............
.......
55
........ .. ..
........ .... ..... ...........
.....
.....
........ .
............. ......................................

.....................
C

29. Let Juan’s age be J. Juan’s sister is three years younger than him. Her age is J − 3. Juan’s father
is 27 years older than him. His age is J + 27. It is stated that Juan’s age added to his sister’s is half their
father’s age. Then
1
J + (J − 3) = (J + 27)
2
Multiplying throughout by 2 gives 2J + 2(J − 3) = J + 27, hence 2J + 2J − 6 = J + 27, and hence
4J − 6 = J + 27. Subtracting J and adding 6 to both sides gives 3J = 33. Dividing both sides by 3 gives
J = 11. Then Juan is 11 years old. (And his sister is 8 and his father is 38.)

... ... ... ... ...........................................................................................


30. The pairs of numbers which differ by 3 are {1, 4}, {2, 5}, and {3, 6}. This ...
...
...
...
...
...
...
...
... c ...
...
b ...
means that 1 must not be adjacent to 4, 2 must not be adjacent to 5, and 3 ... ...
...
.
......................................................................................................
...
.
... ... ...
must not be adjacent to 6. Suppose now a number, say a, is placed in the large ...
...
...
...
...
...
... .. ...
square (see the figure to the right). There is exactly one number, call it b, that ...
...
...
...
...................................
...
.

cannot be adjacent to a. It must be placed in the upper-right square as shown. ... a ...
...
...
...
... ... ...
... ... ...
...
Now let c be any other number. This may be placed in the upper left corner as ...
...
.
...
.
...................................................................................................
shown. There is also exactly one number, call it d, which may not be placed ad-
jacent to c. This number must be placed in one of the two boxes below b. The fifth number, call it e, may
be placed between c and b. The last number must go in the last box. In summary, there are:
i) 6 possibilities for a, the number in the largest box;
ii) 1 possibility for b, the number in the opposite corner;
iii) 4 possibilities for c, the number in the upper left corner;
iv) 2 possibilities for the box which may contain d (the complement of c);
v) 2 possibilities for e, the number placed between c and b; and
vi) 1 possibility for placing the last number in the last box.
By the multiplication principle, there are 6 × 1 × 4 × 2 × 2 × 1 = 96 ways of filling the squares.
The 2015 Jamaican Mathematical Olympiad
Practice Problem Set 2

1) In which of the following numbers is the product of its digits greater than the sum of its digits?
(a) 112 (b) 209 (c) 312 (d) 222 (e) 211

2) What is the value of 1 − 2 + 3 − 4 + 5 − 6 + · · · + 53 − 54 + 55?

3) Mother gave each of her children one-twelfth of a pizza. Then she ate the remaining one-third of the
pizza. How many children does Mother have?

4) In the figure below, the sides of a triangle have been extended to form three angles as shown. What is
the value of x?
..
...
....
....
..... ◦
..
.......
.... .....
106
.... ...
...... ...
... ...
... ...
.... ...
.... ...
...... ...
...
...... ...
..... ...
◦ ..... ...
142 .
.... ...
..
..........................................................................................................................................
..
◦ .......
... x

5) Which four digits need to be removed from the number 4921508 to get the smallest possible three-digit
number? (When the digits are removed, the ones remaining stay in their original order.)

6) In a certain mathematics class, 17 students are in the Spanish class and 13 are in the French class. If
four of them study both languages, how many study at least one foreign language?

7) The positive integer n is the product of all the prime numbers less than 2014. What is the ones digit
of n?

8) In the figure below, a circle of radius 2 is inscribed in a semicircle. What proportion of the area of the
semicircle is shaded?
..........................................
..................... ....................
................... ...................
..................
..
.................................... ...................
.... .........................................
...
.........................
.
. .... ...................
.
. .
......................... .. ... ...........................
.. . . . . . . .
............................. .... 2 .....................
....................
......................................... . .... .........................................
.
.
.................................
................................................
• . .
. . . . . . . . . ..
....................................
.
. .
..................................................
..........................................
. .
.
............................................................... .
............................................
..............................
................................... ..................................
.......................................................................... .....................................
.................................................................... . ...........................................................
. ...............................................
...................................................................................................................................................................................................................

9) How many times between 00:00 and 23:59 does an electronic watch show all of the following four digits,
2, 0, 1, 2, in any order?

10) Three lights are flashing at regular intervals. One flashes every 2 minutes, one every 2.5 minutes, and
one every 3 minutes. If all three lights flash at exactly 9:00 am, when is the next time they will all flash
together?
11) The first four figures in a patterned tile arrangement are shown below. Figure 1 consists of 1 tile and
Figure 3 consists of 7 tiles (including squares of various sizes). How many tiles will the sixth figure
have?
................................................................................................ .................................................................................................. .................................................................................................... ......................................................................................................
... ... ... ... ... ... ... ... ... ... ... ... ... ...
... ... ... ... ... ... ... ... ... ... ... ............................
... ... ... ... ... ... ... .. .. ... ... .. .. ..
... ...
... ... ... ...
... ... ...................................................... ... .......................................................
... ... ... ... ... ... ... ... ... ... ... ... ...
...
... ... ... ... ... ... ... ... ... ... ... ... ...
... ... ... ... .. .. ... ... .. ..
... ..... ....................................................................................................
.
......................................................................................................... ..........................................................................................................
... ... .. .. .. .. .. .. .. .. ...
... ... ... ... ... ... ... ... ... ... ...
... .... ... ... .... ... ... .... ... ... ....
... ... ... ... ... ... ... ... ... ... ...
... ...
... ... ... ...
... ... ... ...
... ... ... ...
... ... ... ... ... ... ... ... ... ... ...
...
... ... ... ... ... ... ... ... ... ... ...
... ... ... ... ... ... ...
... ... ... ... ... ... ... ... ... ... ...
............................................................................................ ............................................................................................ ............................................................................................ ............................................................................................

Figure 1 Figure 2 Figure 3 Figure 4

1
12) Find the value of 1 + 1 .
1 + 1+1

13) In the figure below, Regions I, II, III, and IV are squares. The perimeter of Square I is 16 cm and the
perimeter of Square II is 24 cm. What is the perimeter, in centimetres, of Square IV?
....................................................................................................................
.. ... ...
.......................... .... ....
... ...
...
I .
. II ..
.
.
..
...
... ..
. ..
. ....
.......................................................... ..
... ... ...
.... .... IV ....
... .. ..
... ... ...
... III .... ....
... .. ..
... ... ...
... .... ....
.. .. .
....................................................................................................................................

14) The value of the two-digit number AB is equal to 5(A + B). What is the number AB? [Note: In the
expression AB, A is the tens digit and B is the ones digit.]

15) A multiplication magic square has the property that the product of the three numbers in each row,
column, and diagonal is equal to 1. For the multiplication magic square below, what is the value of
r + s?
....................................................................................................
.... .... .... ...
.. .. .. ...
...
....
p ...
....
q ...
....
r ...
...
.. .. .. ..
.................................................................................................
..... ..... ..... ..
.. .. .. .....
... s
....
1 ...
....
...
....
t ...
...
.. .. .. ..
.................................................................................................
..... ..... ..... ...
...
.. .. ..
u ...
....
4 ...
....
1/8 ...
....
...
...
.. .. .. ...
............................................................................................

16) What is the last digit of the number 22014 + 32014 ?

17) Determine the number of three-digit numbers in which the third digit (the ones digit) is larger than the
first digit (the hundreds digit).

18) When 1001 is divided by a certain one-digit number the remainder is 5. What is the remainder when
the same one-digit number divides 2014?
19) In the figure below, four congruent rectangles are arranged to form a square as shown. If the area of
the outer square is four times the area of the inner square, what is the ratio of the length of a long side
of a rectangle to the length of a shorter side?
.........................................................................................................
.... ... ...
... ... ...
... ... ...
... ... ...
.................................................................................. ...
...
... ... ... ...
... ... ... ...
... ... ...
... .... .... .....
... ... ... ...
... ... ... ...
... .... .... ....
... ... ... ...
... ... ...
...................................................................................
.
... ... ...
... ... ...
...
... ..... .....
... ... ..
.........................................................................................................

20) A vendor sells 360 chocolates each week. She purchases then at a supermarket at a cost of $300 for a
box of 8. However, she discovered that she can get a case of 60 at a wholesale shop for $2,000. If she
buys her chocolates from the wholesale shop, how much will she save each week?

21) Three married couples are attending a party. In how many ways can they form a three-person group in
which there will not be a married couple?

a2 − b2 ab − b2
22) The difference − is equal to:
ab ab − a2
a a2 − 2b a2
(a) (b) (c) a2 (d) a − 2b (e)
b ab b2

23) Anna started throwing darts at the dart board shown below. She scored A points for each dart in the
innermost circle, B for each one in the middle ring, and C for each one in the outer ring. Each turn,
.................................
.......... .......
....... ......
..... .....
..... ....
..... .. ....................... ...
...
. . .......... ..
. ...
.... ..... ...
... .. ..... ...
.... .... . ...
. .. ........... ... ..
. . .. ...... ...
....
. ..... ..... ... ...
..
..
.. .. .... .. . .
... ... ... .. .. ...
..
...
...
...
....
.................... . A ...
.
.
.
..
.
... ... .. ..
... .... ... ..
..... .... ..
... ....... ..... B ...
...
... ............................... ...
.... ....
..... .
...... .....
........ C ......
.............. ......................
........

she threw three darts. On her first turn two darts landed in ring B and one in ring C, and she scored
10 points. In the second turn two darts landed in circle A and one in ring C, and she scored 22 points.
On her next turn one dart landed in each of the three regions. How many points did she score?

24) In the figure below, a square with side length 12 cm has been divided into three rectangles with equal
perimeter. What is the area of the shaded rectangle?
..............................................................................................
... ...
... ...
... ...
... ...
..
...
................................................................................................................................
................................................................. ...
................................................... ...
.................................................. ...
................................................................. ....
................................................... ....
.................................................. ...
................................................................. ...
...
................................................... ...
.................................................. ...
.................................................................
................................................... ...
.............................................................................................
25) Mr. and Mrs. Dobson have several children. The average age of the Dobson family is 18 years. Without
the 38-year-old father, the average age of the family decreases to only 14 years. How many children do
the Dobsons have?

26) How many positive integers less than 1,000 are neither multiples of 5 nor of 7?

27) In the figure below, the circle P QRS has centre O and radius 4, and P R and QS are perpendicular
diameters of the circle. The points T , U , V , and W are the midpoints of P O, QO, RO, and SO,
respectively. What proportion of the area of the circle is shaded?
...
... Q
................
...
.. .........................................................
.. . . ...... .................................
..
.......
......
... .
.....................................
..... .....
.... ............................................ ....
. .... ..
.......................................................................... ...
...
. ... . . . ....................................U ...................
. . . . . . . . . . ...
... .. . ................................................................................. ...
...
... .. ................................................ ... ................................................... ..
.. . . . . . . . . .. .. ...........................
.... .................................................... .. ........................... ....
... ........................................ .
. ...................................... ...
.. ...........................
. ..
. ...................................... ..
P . ................T ........ .O .......V ................... . R
............................................................................................................................................................................................................................................
... ....................................... .. .... . . . . . . . . . .. .
... ....................................... ... ........................... ...
... ............................. .
. .................................................... ...
.................................... . ......................... ...
... ............................ .... ....................................................
...
... ............................ .. ............................ ...
............................................................................. ..
...
...
...............................................
. ....
...................................W ....... ..
... ..................................................... ...
.... ............................................ ....
.....
...... ........................... .....
...... .......................
.. ...... . ........ . ..........
........ . ..
........... .......................... .................
...........................
....
... S


28) How many right triangles have one side of length 60 and integer values for the lengths of the hypotenuse
and the other side?

29) The numbers 1, 2, and 3 are written on the circumference of a circle (see Figure 1). Then the sum of
each pair of neighboring numbers is written between them (see Figure 2). In this way, 6 numbers are

1 1
.......................... ..........
.......................
......... ....... ......
.......
...... ..... ..... .....
..... ....
... ... ...
....
.. ...
...
...
4 ....
.
. ...
...
...
3
.... .. ... ...
... ... ..... ..
..
.... ... ... .
... .. ... ...
... ... ... ...
... ... ...
... .
..
... ... ..
.
...
... ... ... ...
..
3 ...
.....
......
...
.....
......
2 3 .....
......
........ ......
..... 2
.........
........................... ...............................

5
Fig. 1 Fig. 2

arranged around the circle: 1, 3, 2, 5, 3, and 4. The process of adding each pair of neighboring numbers and
placing their sum between them is repeated 3 more times, resulting in 48 numbers on the circle. What is
the sum of these numbers?

30) In the figure below, two circles of radius 6 overlap in such a way that each passes through the centre of
the other. What is the area of the shaded region?
.................................. ...........................................
......... ........ .......
....... .............. ......
...... ................. .....
.
. ..... .................. ....
...
. ....................................................... ...
...
.... .............................................. ...
..
. ...
............................................. ...
.
.. .
................................................... ..
. .. . . . . . . . . . . . . . . . . ..
.... ......................................................................... ..
... ......................................................................... ...
...
...

......................................................
......................................................
• ...
..
... ....................................................... ...
... ..................................... ..
.
... ................................................
... ........................................... ...
... ................................... ...
... ....................... ...
.... ................... .
. ...
..... .................. ..
...... ............ ....
........ ......
................. ....................... ....................... .......................
... ....
The 2015 Jamaican Mathematical Olympiad
Solutions for Practice Problem Set 2

1. For the number 112, the product of its digits is 2 and the sum of its digits is 4. For 209, the product of
its digits is 0 and the sum of its digits is 11. For 312, the product of its digits is 6 and the sum of its digits
is also 6. For 222, the product of its digits is 8 and the sum of its digits is 6. Finally, for 211 the product of
its digits is 2 and the sum of its digits is 4. Among the given numbers, only 222 has the product of its digits
greater than the sum of its digits.

2. The numbers may be added and subtracted in reverse order to obtain (55 − 54) + (53 − 52) + (51 − 50) +
· · · + (5 − 4) + (3 − 2) + 1. There are 27 expressions of the form 55 − 54, 53 − 52, and so on, up to 3 − 2.
Each of them is equal to 1. There is also one more term, namely 1, at the end. Then the original expression
is equal to 1 + 1 + 1 + · · · + 1, where 1 occurs a total of 28 times. This is equal to 28.

3. Since mother ate one-third of the pizza, she shared two-thirds of it between her children. But the
fraction 2/3 is equivalent to 8/12. Mother shared 8/12 of the pizza among her children. She has 8 children.

4. Let a, b, and c be the measures of the interior angles, as shown in the ....
....
figure to the right. Since the three angles are formed by extending each ....
.
....
.
..... ◦
.... .. 106
side of the triangle, 142 + a = 180, 106 + b = 180, and c + x = 180. From ....
... .....
...
..... ...
.. ◦ ...
the first two equations, a = 38 and b = 74. Furthermore, a + b + c = 180. ....
.... ...
...
b
...
.... ...
Substituting for a and b, 38 + 74 + c = 180. It follows that c = 68. Since ....
. ...
...
....
. ...
...
x + c = 180 we have x = 112. ◦ ...... .. ◦ ◦ .......
142 .... a ... c
...........................................................................................................................................
..
◦ .......
... x

5. To end up with the smallest possible three-digit number, one first looks for the smallest possible digit for
the hundreds place. In this case, it is 1. One then looks for the smallest digit to its right for the tens place.
In this case it is 0. One finally looks for the smallest remaining digit on its right for the ones digit. In this
case, it is 8. The smallest possible three-digit number is 108. It is formed by removing 4, 9, 2, and 5.

6. Since 4 students study both languages, they should not be counted twice. The number who study at least
one language is 17 + 13 − 4 = 26.

7. Since n is the product of the prime numbers less than 2014, n = 2 × 3 × 5 × 7 × 11 × · · ·× 2011. (One may
verify that 2011 is, indeed, prime.) Then n is a multiple of 2 × 3 × 5 = 30, and hence is a multiple of 10.
The last digit of n is 0.

8. Since the radius of the inscribed circle is 2, its diameter is 4. Thus .......................
............................
........................ ... .. ....................................
.. .......................
................... ... ..... . . ......
........................... .... .......................
the radius of the outer semicircle is 4 as well. The area of this semicircle ..
.
.
...
.
. .
............................. ...
.
.
. ..... ...... ....................................
..
........................... .
.
.............................
.................. .. ............................
1 ..................... .......... 2 .....................
............................... ..... .................................
is π(42 ) = 8π. On the other hand, the area of the inscribed circle is .
..............................................
.
......................................
.
.
..
. .
.........................
..................................................
2 ........................................................
...............................
. ..
.
....
.......................................................
... . . . . . . . . . . . ..
................................................. ................................
... .................................................
........................................................ ......................................
π(22 ) = 4π. Thus the area of the shaded region is 8π − 4π = 4π. The ........................................................................................... ...
.
. ...
.
....
..................................................................................
.
.............................................................................................................................................................................................................

proportion of the semicircle that is shaded is 4π/8π = 1/2. 4


9. First, consider the times of the form ab: cd, where a, b, c, and d are the numbers 2, 0, 1, and 2 in some
order. There are 6 ways of replacing two of the letters with 2:

22: cd, 2b: 2d, 2b: c2, a2: 2d, a2: c2, ab: 22.

For each of these ways, we can choose the remaining letters to be 0 and 1 in two different ways. This gives
6 × 2 = 12 possibilities in all. They are:

22: 01, 20: 21, 20: 12, 02: 21, 02: 12, 01: 22,
22: 10, 21: 20, 21: 02, 12: 20, 12: 02, 10: 22.

Each of these displays occurs on an electronic watch exactly once between 00:00 and 23:59. The answer
is 12.

10. The first and third lights flash together every 6 minutes. Thus they flash together after 6, 12, 18, 24, 30,
36, . . . minutes. The second light flashes after 2.5, 5.0, 7.5, 10.0, 12.5, . . . minutes. The first time all three
lights flash together is after 30 minutes. So, after 9:00, the next time all three lights will flash together is
9:30 am.

11. Each new pattern is formed by taking one tile (in the upper right corner) in the preceding pattern and
replacing it with four smaller tiles. So, each new pattern has three tiles more than the old one. The first
............................................................................................. ................................................................................................. ................................................................................................. .....................................................................................................
.... ... .. .. .. .... .. .. ... .. .. .. .. ..
... ... ... ... ... ... ... ... ... ... ... .............................
.... ... .... ... .... ... ... .. .. ... ... .. .. ..
.. ... .. ... ... ... ....................................................... ... ............................................................
... ...
... ... ... ... ... ... .. ...
... ... .. .. ...
.... ... .... ... ...
... ... ... ... ... ... ... ... ...
.. ... .. ... ... ... ... ... ... ... ... ... ...
... ... ... ... ... ... ... ... ... ...
.... ..... ....................................................................................................
. ..................................................................................................
.
..................................................................................................
.. ... .. .. .. ... ... ... ... ... ...
...
... ... ... ... ... ... ... ... ... ... ...
.... .... .... ... .... ... ... .... ... ... ...
.. ... .. ... ... ... ... ... ... ...
... ... ... ... ... ... ... ... ... ... .....
.... ... .... ... ... ... ... ... ... ... ...
.. ... .. ... ... ... ... ... ... ... ...
... ... ... ... ... ... ... ... ... ... ...
. .
............................................................................................. ............................................................................................... .............................................................................................. ...............................................................................................

Figure 1 Figure 2 Figure 3 Figure 4

pattern has 1 tile, the second one has 4 tiles, the third has 7 tiles, the fourth has 10 tiles, the fifth has 13
tiles, and the sixth one has 16 tiles.

12. We have
1 1 1 1 2 3 2 5
1+ 1 =1+ 1 =1+ 2 1 =1+   =1+ = + = .
1 + 1+1 1+ 2 2 + 2
3 3 3 3 3
2

13. Since the perimeter of Square I is 16, each of its sides has length 4. Since the perimeter of Square II is 24,
each of its sides has length 6. Then each side of Square III has length 10. Similarly, each side of Square IV
has length 16. The perimeter of Square IV is 16 + 16 + 16 + 16 = 64.

14. The value of AB is 10A + B. This is equal to 5(A + B) = 5A + 5B. So, 10A + B = 5A + 5B. Subtracting
5A + B from both sides gives 5A = 4B. We know that A is a digit from 1 to 9, and B is a digit from 0 to 9.
The possibilities for 5A and 4B are given by

5A: 5, 10, 15, 20, 25, 30, 35, 40, 45


4B: 0, 4, 8, 12, 16, 20, 24, 28, 32, 36
The only way 5A can be equal to 4B is if both are equal to 20. Then A = 4 and B = 5. The number AB
is 45.

15. From the middle column of Figure 15(a), we have q × 1 × 4 = 1. This means that q = 1/4. From the
bottom row, u × 4 × (1/8) = 1. This means that u = 2. From the diagonal starting with p, p × 1 × (1/8) = 1.
............................................................................................. .............................................................................................
.... .. .. ... .... .. .. ...
... ... ... ... ... ... ... ...
....
.. p ....
..q ....
.. r ...
...
....
.. 8 1/4....
..
....
.. r ...
...
... ... ... ... ... ... ... ...
....................................................................................................... .......................................................................................................
.. .. .. ... .. .. .. ...
... ... ... ... ... ... ... ...
.... .... .... .... .... .... .... ....
.. s 1 .. .. t ... .. s 1 .. .. t ...
... ... ... ... ... ... ... ...
....................................................................................................... .......................................................................................................
.. .. .. ... .. .. .. ...
... ... ... ...
... ... ... ... ...
...
u .... .... ....
1/8 .... .... ....
1/8
.. 4 .. .. ...
...
2 .. 4 .. .. ...
...
... ... ... . ... ... ... .
................................................................................................ ................................................................................................

Fig. 15(a) Fig. 15(b)

Then p = 8. These values are shown in the table in Figure 15(b). From the top row there, 8 × (1/4) × r = 1.
Then r = 1/2. From the first column there, 8 × s × 2 = 1. Then s = 1/16. Thus

1 1 8 1 9
r+s= + = + = .
2 16 16 16 16

16. The powers of 2 are 2, 4, 8, 16, 32, 64, 128, 256, 512, . . . Their last digits are 2, 4, 8, 6, 2, 4, 8, 6, 2, . . .
This repeats every 4 terms and continues forever. The 4th, 8th, 12th, and so on, terms are all 6. Eventually
the 2012th term is 6, the 2013th term is 2, and the 2014th term is 4. The powers of 3 are 3, 9, 27, 81, 243,
. . . Their last digits are 3, 9, 7, 1, 3, . . . This sequence also repeats every 4 terms and continues forever.
The 4th, 8th, 12th, and so on, terms are all 1. Eventually the 2012th term is 1, the 2013th term is 3, and
the 2014th term is 9. The last digit of the number 22012 + 32012 is the last digit of 4 + 9 = 13. This is 3.

17. Suppose abc is a three-digit number with c greater than a. If a = 1 then b could be any digit and c
could be 2, 3, 4, 5, 6, 7, 8, or 9. There are 10 ways to choose b and, for each one, 8 ways to choose c. There
are 10 × 8 = 80 numbers 1bc where c is greater than 1. If a = 2 then b could be any digit and c could be 3,
4, 5, 6, 7, 8, or 9. There are 10 ways to choose b and, for each one, 7 ways to choose c. There are 10 × 7 = 70
numbers 2bc where c is greater than 2. In the same way, there are 60 numbers 3bc where c is greater than 3,
50 numbers 4bc where c is greater than 4, and so on. This continues until there are 10 numbers 8bc where c
is greater than 8. (In other words, c = 9). There are a total of 80 + 70 + 60 + · · · + 10 = 360 numbers abc
with c greater than a.

18. If a one-digit number divided into 1001 leaves a remainder of 5, the number divides 996 evenly. Then
the number must be 1, 2, 3, 4, or 6. However, if 1001 is divided by 1, 2, 3, or 4, any remainder we obtain
would have to be 3 or less. So, the one-digit number dividing 1001 is 6. When 6 divides 2014, the remainder
is 4.
19. Let a be the length of a shorter side and b the length of a longer side of a b
rectangle. The side length of the outer square is a + b and the side length of the .........................................................................................................
.... ...
...
...
....
a ... ... ...
inner square is b − a. Since the area of the outer square is 4 times that of the ...
...
................................................................................
..... .....
...
inner one, .... ... .... ...
... .... .... ....
... ... ... ...
... ..
...
..
...
..
...
... .... .... ....
(a + b)2 = 4(a − b)2 ...
...
...
...
.....
...
.....
...
.....
... ... .. .
..................................................................................
a2 + 2ab + b2 = 4(a2 − 2ab + b2 ) ...
...
...
....
...
....
...
... ... ...
a2 + 2ab + b2 = 4a2 − 8ab + 4b2 ... ...
......................................................................................................
..

0 = 3a2 − 10ab + 3b2

Factoring the right side gives (3a − b)(a − 3b) = 0. Then either 3a − b = 0 or a − 3b = 0. In the first case
a = b/3, and in the second case a = 3b. Since a is the shorter side, a = b/3. Then the ratio of the length of
the longer side to that of the shorter side is
 
b b b 3 3
=  = = = 3.
a b 1 b 1
3

That is, the ratio is 3 : 1.

20. For the vendor to buy 360 chocolates at the supermarket, she must buy 45 boxes with 8 chocolates each.
Since each box costs $300, her total cost will be $13,500. At the wholesale shop, she must buy 6 cases with
60 chocolates each. Since each case costs $2,000, her total cost will be $12,000. If she buys at the wholesale
shop she will save $13, 500 − $12, 000 = $1, 500.

21. Suppose one married couple is denoted by A1 and A2 , another by B1 and B2 , and the third by C1 and
C2 . The only way to form a three-person group with no married couple present is to choose one of the As,
one of the Bs, and one of the Cs for the group. There are two ways to choose one of the As, two ways to
choose one of the Bs, and two ways to choose one of the Cs. There are a total of 2 × 2 × 2 = 8 ways of
choosing a group like this. So, there are eight ways of forming a three-person group in which there will not
be a married couple.

ab − b2 b(a − b) −b(b − a) b
22. First, = = = − . Then
ab − a2 a(b − a) a(b − a) a
 
a2 − b2 ab − b2 a2 − b2 b a2 − b2 b a2 − b2 b2 a2 a
− = − − = + = + = = .
ab ab − a2 ab a ab a ab ab ab b

23. We know from the first turn that 2B + C = 10 and from the second one that 2A + C = 22. Adding these
equations, 2A + 2B + 2C = 32. Dividing throughout by 2 gives A + B + C = 16. Anna scored 16 points on
her third turn.
24. The upper rectangle has width 12 and its height may be de- 12
...........................................................................................
noted by x, as shown in the figure to the right. (All dimensions .... ...
... ...
x .... ...
x
are in centimetres.) The lower rectangles obviously have equal ..
...
...
...
.................................................................................................................
height and it is given that they have equal perimeter. This implies ................................................
..................................................................
...
...
.................................................... ....
that they have equal width as well, which is 6 in both cases. Since .................................................
..................................................................
...
...
.................................................... ....
the upper and lower rectangles have the same perimeter, 12 − x ................................................. ... 12 − x
.................................................................. ...
...
................................................... ...
................................................ ...
................................................................. .
..............................................................................................................
12 + x + 12 + x = (12 − x) + 6 + (12 − x) + 6
6 6
24 + 2x = 36 − 2x
Simplifying further gives 4x = 12 and so x = 3. This means that each lower rectangle has height 9. Then
the area of the shaded rectangle is 9 × 6 = 54.

25. Let k be the number of children the Dobsons have, let a1 , a2 , . . ., ak be their ages, and let m be the age
of Mrs. Dobson. Since Mr. Dobson is 38 years old and the average age of the Dobson family is 18,
a1 + a2 + · · · + ak + m + 38
= 18.
k+2
Multiplying both sides by k + 2 gives a1 + a2 + · · · + ak + m + 38 = 18k + 36. Subtracting 38 from both sides
gives a1 + a2 + · · · + ak + m = 18k − 2. Since the average age of the family without Mr. Dobson is 14,
a1 + a2 + · · · + ak + m
= 14
k+1
Multiplying both sides by k + 1 gives a1 + a2 + · · · + ak + m = 14k + 14. It follows that 18k − 2 = 14k + 14.
Then 4k = 16 and hence k = 4. The Dobsons have 4 children.

26. We first determine the number of positive integers less than 1000 which are in fact multiples of 5 or 7 (or
both). The 5-multiples less than 1000 are 5, 10, 15, 20, . . ., 995, and there are 199 such numbers in all. The
7-multiples less than 1000 are 7, 14, 21, 28, . . ., 994, and there are 142 such numbers in all. However, some
numbers are multiples both of 5 and 7, and these are the multiples of 35. The 35-multiples less than 1000
are 35, 70, 105, 140, . . ., 980, and there are 28 such numbers in all. Then the number of positive integers less
than 1000 which are multiples of 5 or 7 (or both) is 199 + 142 − 28 = 313. The number of positive integers
that are not multiples of 5 or 7 is 999 − 313 = 696.

27. Let a be the radius of the circle. Then T O and OU are each a/2. The area
..
of the triangle T OU is ... Q
................................
.. . .. ......... ............................. ...............
.... .............................. ......
   ....
... . .. . ... .
.. . . . . . . ...
..............................
................................................
.....
...
1 1 a a a2 ....
...
... .....................................................U ...............
............................... ... .................................
. . . . . . . . . .....
...
...
..
(T O)(OU ) = = . .
... .. ..
.. ............................................ ... ............................................. ....
..
2 2 2 2 8 .... .............................................
.
. ....
....................... ..
........................ ...
. . . . . . . .
..P
.....................................................T
.....................................O ...............................V
...............................................R
. ... . . . . . . . . . . . . . . .. .. .
. .. . . . . . . . . ..
......
... .............................. ... ...................
... .................................. ... ................................ ..
... ................................ .... ............................................. ....
It follows that the area of square T U V W , which is four times the area of 4T OU , ... .................................. ... .................................
... ...................................................
............................................................ ...
..
... ...
1 a2 ...
...
............................W
...........................................
. . . . . ..
...
...
is a2 /2. Similarly, the area of 4P OQ is (a)(a) = , and it the area of square ....
...... . . . . .
. . . . . . ..
. . . . .
....... ................................ ............
.
. ... .. .. .
... . .. .

2 2 .......... ............. .........


...........................
..
P QRS is 2a2 . Then the shaded area is the area of the square P QRS minus that .. S
.

of T U V W . This is
a2 4a2 a2 3a2
2a2 − = − = .
2 2 2 2
On the other hand, the area of the outer circle is πa2 . Then the proportion of the area of this circle that is
shaded is
3a2 /2 3a2 1 3a2 3
2
= × 2 = = .
πa 2 πa 2πa2 2π
28. Let h be the hypotenuse of the triangle and a the length of the other side. By the Pythagorean theorem,
√ 2
a2 + 60 = h2 and hence a2 + 60 = h2 . By the laws of algebra, h2 − a2 = 60 and so (h + a)(h − a) = 60.
Since h and a are positive integers with h > a, h + a and h − a are also positive integers and h + a > h − a.
Since h + a and h − a are factors of 60, the possibilities are:
h + a = 60, h − a = 1; h + a = 20, h − a = 3; h + a = 12, h − a = 5;
h + a = 30, h − a = 2; h + a = 15, h − a = 4; h + a = 10, h − a = 6.

When h + a = 30 and h − a = 2 then h = 16 and a = 14. This gives a right triangle with sides 14 and 60
and hypotenuse
√ 16. When h + a = 10 and h − a = 6 then h = 8 and a = 2. This gives a right triangle with
sides 2 and 60 and hypotenuse 8. In the other pairs of equations,
√ the solutions for
√ h and a are not integers.
Thus there are two right triangles meeting our criteria: 14, 60, and 16, and 2, 60 and 8.

29. The numbers added in the first step are 1 + 2, 2 + 3, and 3 + 1. The sum of these numbers is
1 + 1 + 2 + 2 + 3 + 3 = 2(1 + 2 + 3) = 12. Thus the sum of all the numbers on the circle after the first step is
6 + 12 = 18. In general, suppose the numbers on the circle at some stage are a1 , a2 , . . ., ak . Let sk be the
sum of these numbers, so that sk = a1 + a2 + a3 + · · · + ak−1 + ak . Moving to the next stage, the sums of
the neighboring numbers are a1 + a2 , a2 + a3 , . . ., ak−1 + ak , and ak + a1 . The sum of these new numbers is
(a1 + a2 ) + (a2 + a3 ) + · · · + (ak−1 + ak ) + (ak + a1 ) = 2(a1 + a2 + · · · + ak−1 + ak ) = 2sk
When this is added to the numbers already on the circle, the total is sk + 2sk = 3sk . We know that the sum
of the numbers on the circle after the first step is 6, and the sum after the second step is 18. Taking three
more steps, the sums are 54, 162, and 486, respectively. Thus, at the end, the sum of the 48 numbers on the
circle is 486.

30. Let O and P be the centres of the respective circles and let A and B be the points where the circles
intersect. (See Figure 30(a) below.) Note that OA and OP are radii of the circle with centre O, and P O

.............................
A .............................. A .............................. ......................
........... ...................... ........... ....... ..........................................
....... .................
.......
...... ........ ...... ..............................................................
...... ......................................... ..... ............. ..... ...................................................
.....
. . ... .................. ..... .... ......................................................................................................
.
....
. .
.. .
.................................................. ...
... ..
..
.................. ... ...
...
. .
.. . . . . . . . . . . . . . . . . . . . . . . . . . . C
..............................................................................................................................................................................
... ............................................................ ... ...................... .... ... B ........
.......................................................................................................................... A
. .
....................................................... ... . .
................................. ... .....
... . ... . ... ...
... ... ................................................................................................................................................................................................................... .....
... .......................................................................
. .. ............................. .... ... .... ................................................................................... ...
.... ................................................ ...................................................................................
................................................................................. ..
..
... .. .... ................................................ . ..
...
.... ............................................................. .............................. ... ... .... .................................
...
..
... O • ...........................................
........................................
• P
.........................................................................................
...
.....................
..............................
.............................
• P .....
...
..
..
. ...
...
• ........ ..
...
..
... ........................................ ..
. .
. .. ... .
...
...
.......................................................
....................................
...
..
. ...................
....................
.......................... . .... .
...
.. ...
... P .
.
...
..
... ..................................
............................................... .. ....................... ..... ... ...
... ............................... ... ..................... .. ... ... ...
... ............ ... ... ...
...
.....
................................
..... ................... .
..... .......... ... ..
. . ...
.... ... ...
.... .......... . . ...... .
...... ....................
...... ............ ..... ......
....... .......
...................... ....... ........ ...... ...... ......
...........
............................. ............................. .............. .................... ........
.........................................
..........
B B

Fig. 30(a) Fig. 30(b) Fig. 30(c)

and P A are radii of the circle with centre P . Then 4AOP is equilateral. Similarly, 4BOP is equilateral.
Thus 6 AP B = 60◦ + 60◦ = 120◦ . The shaded region in Figure 30(a) consists of two sectors equal to the
one shaded in Figure 30(b). By rotating one sector 90◦ , this may be viewed as the upper portion of an
“ice-cream cone” as in Figure 30(c). The area of the entire ice-cream cone is 1/3 of the area of the circle
with centre P . (This is because 6 AP B is 1/3 of an entire circle.) Its area is
1 1
π(62 ) = (36π) = 12π
3 3
To find the area of 4BP A, let C be the midpoint of BA. Then
√ BP C and AP C are each 30-60-90 triangles.
It follows that BP = P A = 6, CP = 3, and BC = AC = 3 3. Then the area of BP A is
1 1 √  √
(BA)(CP ) = 6 3 (3) = 9 3.
2 2
 √ 
Thus the area of one sector as in Figure 30(b) is 12π − 9 3. The total shaded area as in Figure 30(a)

is 24π − 18 3.
The 2015 Jamaican Mathematical Olympiad
Practice Problem Set 3

1) Each of the boys Alan, Bob, Carl, and Doug has one and only one of the following animals: a cat, a
dog, a goldfish, and a canary. Bob has an animal with fur. Doug has a pet with four legs. Carl has a
bird, and Alan and Bob don’t like cats. Which of the following sentences is not true?
(a) Doug has a dog. (b) Carl has a canary. (c) Alan has a goldfish.
(d) Doug has a cat. (e) Bob has a dog.

2) In the figure below, the angles at A, B, C, E, and F are all 90◦ . If BC = 3, CD = 6, DE = 5, and
EF = 5, what is the area of the shaded region?

A .................................................................................................................................................................................................................................
B
....................................................................................................................
.........................................................................................................................................................
..................................................................................................................
.....................................................................................................................
.................................................................................................................................................................................................
......................................................
........................................................................ C
.........................................................
......................................................
D
.........................................................................
.........................................................
......................................................
.........................................................................
.........................................................
......................................................
...................................................
F E

3) A rosebush in a garden has three branches. On each branch there is a cluster of three roses, and in each
rose there are two bees. How many bees are in the rosebush?

4) A square of area 64 cm2 is cut into four equal rectangles, as indicated below. What is the perimeter of
one of the rectangles?

........................................................................................
... ... ... ... ...
... ... ... ... ...
... ... ... ... ...
... ... ... ... ...
...
... ... ... ... ...
... ... ... ... ..
... ... ... ... ...
... ... ... ... ...
... ... ... ... ...
... ... ... ... ...
... ... ... ... ....
... ... ... ... ...
... ... ... ... ...
........................................................................................

5) An aeroplane has 24 rows numbered from 1 to 25 (excluding 13). Four passengers are seated in Row 15
and six passengers are seated in all the other rows. How many passengers are on the plane?

6) In the figure below, ABCD is a rectangle and AD : AB = 2 : 3. If the area of ABCD is 150 cm2 , what
is its perimeter?

A ..............................................................................................
... ... B
... ...
... ...
... ...
...
... ...
... ...
...
... .....
... ...
... ...
... ..
.............................................................................................
D C

7) Two litres of fruit juice containing 10% sugar are mixed with 3 litres of another juice containing 15%
sugar. What percentage of the mixture consists of sugar?
8) A square is divided into three equal rectangles. The middle rectangle is removed and placed on the side
of the original square to form the octsgon shown. If the perimeter of the square is A and the perimeter
of the octagon is B, what is the value of the fraction A/B?
........................................................................................... ......................................................................................................
.... ....
..........
... ...................................
... ... ... ....................................
... ... ... ..............................................
... ... ... ....................................
... ... ... ...................................
.................................................................................................................................. ............................................................................................................................
................................................................................................ ....................................
................................................................................................. .....................................
.............................................................................................................................. ..............................................
................................................................................................ ....................................
............................................................................................................................... ....................................
............................................................................................................................. .....................................................................................................................................
... ... .... ........................
... ...
... ... ...............................................
... ... ... ....................................
... ... ... ....................................
... . ... ...................................
.............................................................................................. ..............................................................................................................................

9) Martha sold eggs at the market every day from Monday to Friday. On Wednesday she sold 60 eggs and
on Thursday she sold 96 eggs. Friday evening, she noticed something interesting. After Monday and
Tuesday, the number of eggs she sold each day was equal to the sum of the number of eggs she sold the
previous two days. How many eggs did Martha sell on Monday?

10) A rectangular picture of size 80 cm×160 cm is scaled down to fit onto a sheet of paper of size 30 cm×40 cm.
Given that the longer side of the picture fits exactly onto the longer
.........................................................................................................................................................................................................
................................................................................................................................................................................................................
............................................................................................................................................................
............................................................................................................................................................
............................................................................................................................................................................................................... ..............................................................................
............................................................................................................................................................. ..... .
............................................................................................................................................................................................................... ..................................................................................
.............................................................................................................................................................. ................................................................................
.............................................................................................................................................................. .................................................................................
................................................................................................................................................................................................................ .................................................................................
.............................................................................................................................................................. ...........................................................................................................
.............................................................................................................................................................. .................................................................................
................................................................................................................................................................................................................ .................................................................................
.............................................................................................................................................................. ...........................................................................................................
.............................................................................................................................................................. .................................................................................
................................................................................................................................................................................................................................................................................................................. ..........................................................................................................................................................

Picture: 80 × 160 Paper: 30 × 40

side of the sheet of paper, determine the area of the sheet of paper not covered by the picture.

11) Adam is in jail and needs to have some money sent to his lawyer. The amount of money he needs is a
dollar amount with seven digits. Eve, his wife, is very smart and Adam did not want to say openly how
much he needed. Instead, he sent a numerical code to Eve. He wrote:
“The sum of any four adjacent digits is 16 and the sum of any 5 adjacent digits is 19”
Eve decoded the message and sent the money to his lawyer. What amount of money did Eve send the
lawyer?

12) In the figure below, the two overlapping squares each have side length 1. What is the area of the shaded
region?
.......
.... ......
... ...
.. . .... ....
... ....
..
. .... ....
...
. .. .. ....
. .. ....
.. ....
.
. .... ...
....
..... ....
..... ....
..... ...
....
.
........................................................................................................................................................ .
... ........................................................................................... ....
... ..................................................................................... .....
.
... ................................................................................ .
....
.................................................... .
... .................................................. ....
... ................................................ .....
.............................................. ....
... ............................................................... ...
... .............................................
... ..........................................................
................................... ..
... .......................................... ..
... .......................... ...
... .......................... ....
...............
... ........... ...
...
... ....
... ... ...
....
... .... ....
... ... ...
.
...................................................................................................
13) A number less than 3568 is odd, has remainder 2 when divided by 3, and has remainder 4 when divided
by 5. What is the sum of the digits of the largest number which meets these conditions?

14) The diagram below


√ shows a cuboid with four of its vertices marked X, Y , Z, and A, respectively. If
XY = 8, XZ = 55, and Y Z = 9, what is the length of XA?
X

..........
.......... ..
..........................................................................................
.
.......... ....
.................................................................................................... • ..
... . ... ...
... .... Y... ...
... .
. ...
.... ... .... ...
... . ... ...
... . .
. ...
... .... .... ...
... ... ...
... ... ... ...
. .
...
... ....... ....... ....... . ....... ....
... .
...... . .... .
... .
...
.. .... ....... .... ....................
Z • .................................................................................. •
A

15) The numbers 257 and 338 have the property that when their digits are put in reverse order the new
numbers, 752 and 833 respectively, are larger. How many 3-digit numbers have this property?

16) In the figure below, triangles P QR and LM N are equilateral. If 6 QSM is 55◦ , what is the value of x?

L......
.
... .......
... ...
..
...
.
....
... S ..
..............
.... ........................ ....
Q
.... ........
............ ....... ◦ ....
............
..
......... . ..
... .
...
.
...
....
55 ..
.
............ .... ... ..
................ .... ....
P .....
.....
..... ..
.
.. ... ..
.....
.
..... .... ........
..... ..
...... ... ......
.. .. ....
............ .. ......
.
...
..... .
.
◦ ...................................... M
..
.
....
.....
..... x
..... ...............
.....
......
..
.
.
.
......
.. .................. ......... ..
..
.............. .....
..... ....
N ..... ..
.......
.

R
The 2015 Jamaican Mathematical Olympiad
Solutions for Practice Problem Set 3

1. Bob has an animal with fur but doesn’t like cats. Then Bob has a dog. Also, Doug has a pet
with four legs. Then Doug has a cat. Since Carl has a bird, he has a canary. This means that
Alan has a goldfish. The only false sentence is “Doug has a dog”.

2. The figure may be divided into three rectangles as shown below. In the upper right corner, the
area of the smaller rectangle is 6 × 3 = 18. In the upper left, the area of the rectangle is 5 × 3 = 15.
In the lower left, the area of the rectangle is 5 × 5 = 25. The total shaded area is 18 + 15 + 25 = 58.
5 6
A ................................................................................................................................................................................................................................................................. B
..........................................................................................................................................................
......................................................................................................................
3 ....................................................................................................................... 3
.........................................................................................................................................................
..............................................................................................................................................................................
........................................................
.......................................................................
........................................................ D 6 C
........................................................................
........................................................
5 .......................................................
........................................................ 5
........................................................................
.......................................................
......................................................................................
F E
5

3. There are 3 branches in the rosebush, 3 × 3 = 9 roses in the rosebush, and 2 × 3 × 3 = 18 bees
in the rosebush.

4. Since the area of the square is 64 cm2 , each side has length 8 cm. Then each smaller rectangle
has width 2 cm and height 8 cm. Thus each smaller rectangle has perimeter (in centimetres)
2 + 8 + 2 + 8 = 20.
2 2 2 2
....................................................................................
... ... ... ... ...
... ... ... ... ..
... ... ... ... ...
... ... ... ... ...
.. .. .. .. ...
.... .... .... .... ...
.. .. .. .. ...
8 .... .... .... .... ...
...
... ... ... ...
... ... ... ... ...
... ... ... ... ...
... ... ... ... ...
... ... ... ... ..
................................................................................

5. The plane has one row with four passengers and 23 rows with 6 passengers each. A total of
(23 × 6) + (1 × 4) = 138 + 4 = 142 passengers are seated on the plane.

6. Since AD : AB = 2 : 3, there is some x such that AD = 2x and AB = 3x. Since ABCD has

3x
A ............................................................................................. B
.... ...
... ...
.. ...
.... ...
...
2x ..
.. ...
... ...
.. ...
... ...
........................................................................................
D C
area 150 cm2 ,
(2x)(3x) = 150; 6x2 = 150; x2 = 25; x = 5.
Then the rectangle has height 10 cm and width 15 cm. Its perimeter is 10 cm + 15 cm + 10 cm +
15 cm = 50 cm.

7. The total amount of sugar in the mixture, measured in litres, is 2(0.10)+3(0.15) = 0.20+0.45 =
0.65. The mixture itself contains 5 litres of juice all together. The percentage of sugar in the juice
is
0.65
× 100 = (0.13) × 100 = 13.
5
The mixture contains 13% sugar.

8. Let u be the unit of length which is one-third of the side of the square. Then each side of
the square has length 3u. The perimeter of the square, denoted by A, is given by A = 12u. The
A 12u 12 3
perimeter of the octagon, denoted by B is given by B = 16u. Then = = = .
B 16u 16 4
. . .
............................................................................. ................................................................................................................
.. ... .... ...........................
.... ... .. .....................................
... . .... .................
........................................................................................................ ........................................................................................
..................................................................................................... ......................................
............................................................................. .............................
.............................................................................. ............................
.................................................................................................... ...........................
..................................................................... ..........................................................................................
... ... .. ...........................
.... ... .... .....................................
... .. .. ..................
.......................................................................... ................................................................................................................
. . .

9. Suppose Martha sold m eggs on Monday and t eggs on Tuesday. On Wednesday she sold m + t
eggs. So, m + t = 60. On Thursday she sold t + 60 eggs. So, t + 60 = 96. Subtracting 60 from
both sides gives t = 36. Martha sold 36 eggs on Tuesday.
........................................................................................................................................................................................................................................................................................................................................................................................................................
.... .... .... .... .... ...
.... .... .... .... .... ....
...
...
Monday ...
...
Tuesday ...
...
...
...
Wednesday ...
...
Thursday ...
...
Friday
... ... .. ... .. ..
..............................................................................................................................................................................................................................................................................................................................................................................................................................
... ... ... ... ... ..
...
...
...
...
m ...
...
..
t ...
...
..
...
...
..
...
...
..
60 ..
..
96 156
.................................................................................................................................................................................................................................................................................................................................................................................................................

Since m + t = 60 we have m + 36 = 60. Subtracting 36 from both sides gives m = 24. Martha sold
24 eggs on Monday.

10. The side of the picture 160 cm long is scaled to fit exactly on a sheet of paper 40 cm long. Then
the scaling factor is 1/4. The height of the picture, 80 cm, is also scaled by 1/4. Then the height
of the picture’s image is 20 cm. Since the sheet of paper is 30 cm high, the region not covered by
the image is 10 cm high. It is also 40 cm long. Its area is 400 cm2 .
.......................................................................................................................................................................................................................................................................................................................
............................................................................................................................................................
.............................................................................................................................................................................................................
.............................................................................................................................................................
............................................................................................................................................................ .............................................................................
............................................................................................................................................................................................................... .. .
........................................................................................................................................................... ..........................................................................................................................
........................................................................................................................................................... ..........................................................................................................
............................................................................................................................................................................................................... ...........................................................................................................
........................................................................................................................................................... ................................................................................
................................................................................................................................................................................................................ ............................................................................................................
.......................................................................................................................................................... ...............................................................................
........................................................................................................................................................... ................................................................................
................................................................................................................................................................................................................ ............................................................................................................
.................................................................................................................................................................................................................................................................. ................................................................................................................................

Picture: 80 × 160 Paper: 30 × 40


11. Suppose the seven-digit number is abcdef g. Since the sum of any four adjacent digits is 16,
a + b + c + d = 16. Since the sum of any 5 consecutive digits is 19, a + b + c + d + e = 19. This
means that e = 3. Similarly, b + c + d + e = 16 and b + c + d + e + f = 19. Then f = 3. Continuing,
c + d + e + f = 16 and c + d + e + f + g = 19. Thus g = 3. So far, the number abcdef g has the
form abcd333. On the other hand, a + b + c + d + e = 19 and b + c + d + e = 16. Then a = 3.
Similarly, b + c + d + e + f = 19 and c + d + e + f = 16. Then b = 3. Finally, c + d + e + f + g = 19
and d + e + f + g = 16. Then c = 3. Then the number abcdef g has the form 333d333. Finally,
the sum of the first four digits is 16. Then 3 + 3 + 3 + d = 16. This means that 9 + d = 16 and so
d = 7. The number of dollars Eve sent the lawyer is 3, 337, 333.

12. Let A, B, C, D, E, and F be the points shown on the figure below. The area of the shaded
region is the area of triangle ABC minus the area of triangle EF C. The area of the square ABCD
is 1. Since the
.
.........
..... ........
.... .....
.... .... .....
..... .....
.. ..... .....
.....
.. .... .....
...... .....
.....
.. .... .....
.. . ... .....
..
.... .....
...
... .....
A .. B
.................................................................................................................................................................................
.....
.
... ........................................................................................ .....
... ...................................................................................... ......
.
... ............................................................................... ......
..................................................... ...
... ........................................................................ ........
... ............................................... .....
... ................................................................ ....
... .................................................................
.. ...........................................
.............................................. ..
.... .............................. ..
.................................. ..
F
... ........................
... ................... ....
... ....... ..... ..
... ......... ....
...
... E .....
..... ....
..... ...
... .. .
.........................................................................................
D C

triangles ABC and ADC are congruent by the side-side-side theorem, each of them has area 1/2.
To find the area of triangle EF C, note that 6 F EC = 90◦ and 6 ECF = 45◦
√ . Then EF C = 45
6 ◦

as well. Thus EF C is an isosceles triangle


√ with EF = EC.
√ Also, AC = 2 by the Pythagorean
theorem and AE = 1. Then EC = 2 − 1. Thus EF = 2 − 1 as well. The area of 4EF C is
1 √  √  1 √  3 √
2−1 2 − 1 = 2 − 2 2 + 1 = − 2.
2 2 2
Finally, the area of the shaded region is
 
1 3 √ 1 3 √ √
− − 2 = − + 2 = 2 − 1.
2 2 2 2

13. If a number has remainder 4 when divided by 5 then its last digit is either 4 or 9. If the
number is odd then its last digit must be 9. If the number is also less than 3568 then it is one of
these numbers (counting backwards):

3559, 3549, 3539, 3529, 3519, 3509, . . .

The largest number on this list which has remainder 2 when divided by 3 is 3539. The sum of its
digits is 20.
14. Let a, b, and c be the dimensions of the cuboid as shown. Since XZ is perpendicular to ZA,
we have (XZ)2 + (ZA)2 = (XA)2 by the Pythagorean theorem. Furthermore,
X b
a ..........
• .....................................................................................................................................
.......... ........
..........
. ...
.......... .. .. b .......... ...
......................................................................................................................................... • a..
... ... ... .
.
. ...
... . . .. .
. ....
... ... . Y ..
. ...
.. .
. ... .
.
. ...
... .. . .. ...
. . ...
..
...
. ...
.
...
..
..
.
.
.
...
...
c
. .
.. ... ... ..... ...
... . .. . ..
c .. ... ..
. ...
.
.
. c ....
.... . .. ..... ...
... . .
. . ...
... ... .. ... .
. ...
.. .
.
.
.. . . ...
... ... ..
. ... ... .
.. . ...
..... ....... ....... ....... ....... ........ .......... ....... ....... ...................
..... .... ... .... .. .
. .......
..
....... ....... ..........
• .
........................................................................................................................... . .
. ..
• a
Z
b A

(XZ)2 = a2 + c2 and (ZA)2 = b2 . Thus (XA)2 = a2 + b2 + c2 . Now note that a2 + b2 = (XY )2 =


64, a2 + c2 = (XZ)2 = 55, and b2 + c2 = (Y Z)2 = 81. Adding all three of these√equations,
2a2 + 2b2 + 2c2 = 64 + 55 + 81 = 200. Thus a2 + b2 + c2 = 100. It follows that XA = 100 = 10.

15. When the digits of a 3-digit number abc are put in reverse order the new number, cba, is
larger if and only if c > a. The numbers abc for which a < c (and b is unspecified) are as follows:

1b2, 1b3, 1b4, 1b5, 1b6, 1b7, 1b8, 1b9,


2b3, 2b4, 2b5, 2b6, 2b7, 2b8, 2b9,
3b4, 3b5, 3b6, 3b7, 3b8, 3b9,
4b5, 4b6, 4b7, 4b8, 4b9,
5b6, 5b7, 5b8, 5b9,
6b7, 6b8, 6b9,
7b8, 7b9,
8b9,
There are 36 possibilities listed. For each one there are 10 choices for b. Then there are 360 such
numbers in all.

16. Let T , U , and V be the points of intersection as shown in the figure below. From the
information given, the angles at Q, M , and R are each 60◦ . Since the sum of the angles in
L....
.
... ......
... ........ Q
... ..... .
..............
....
.
..... S
.... ........................ ....
..
.... ................................ .
...
. ....
............
.....
..... ...
............. .... ..... ...
............... ..... ....
P .....
.....
..... ....
.
.
.. ...
. ..
......
.. T
..... .. .. .....
....... .. ........
......
..... ... ...
.
.
... ..........
.. ....... M
... ............
... ..... ◦ ..........................
x
.... ..... .............
......... ..
U
.. ............ ......... ...
.................. ..
N V .....
..... ....
..... ...
....

any triangle is 180◦ , 6 QT S = 65◦ . By vertical angles, 6 M T U = 65◦ as well. It follows that
6 M U T = 55◦ . By vertical angles again, 6 RU V = 55◦ . Thus 6 RV U = 65◦ as well. Finally, since
6 P V M and 6 RV M are supplementary, x + 65 = 180. Then x = 115.
The 2015 Jamaican Mathematical Olympiad
Practice Problem Set 4
x/4 4
1) If = , what is x?
2 x/2

2) A square of area 81 cm2 is cut into three equal rectangles and they are placed end-to-end, as shown
below. What is the perimeter of the new rectangle?
.........................................
... ..
...... .... .... .... .... ....
... ... ..............................................................................................................................
... .. ... ..
...... .... .... .... .... ..... .......................................................................................................................
... ..
...........................................

3) Let a = 70 and b = 28. If a is reduced by 20% and b by 25%, what is the product of the reduced
numbers?

4) Five chairs are arranged around a circular table and Ashley, Brian, Carl, Dean, and Elton are sitting in
them. Ashley and Brian are not sitting together, Brian and Carl are not sitting together, and Carl and
Dean are not sitting together. Which two people are sitting next to Elton?

5) In the figure below, some of the angles have the measures shown (where x is measured in degrees).
What is x?
..
........
... ......
... ....
..... ... 5x
. ....
... ....
...
. ...
.
.. ...
.....................................................................................................................................................................................................................
... . .... ...
... .... ... ......
... 3x .. ... ...... 2x
... ... .... ......
... ..... ...
. ....
......
...
...
.... .... 6x
... .... .....
... ... .... ..........
...
... .....
4x .. .......
... .. .
......
.

6) The first three terms in a sequence of numbers are 1, 2, and 3. After that, each term is the sum of the
last three numbers that come before it. So, the fourth number in the sequence is 6, and so on. What is
the eighth number in this sequence?

7) A small box of chocolates costs $100. There is a coupon inside each of the boxes of chocolate. With
three coupons, you can get an additional box of chocolates free. What is the greatest number of boxes
you can get for $1,500?

8) In the figure below, five equilateral triangles have been arranged to form a trapezoid. What fraction of
this trapezoid is shaded?

................................................................................................................................................
... ... .. ... ......... ............. ..
... ..... ... ...................................................................... .....
...
. ... ... ... . . . . . . . . . . ... . .. .
... ... .. .................................................................. .....
.... ...
. .. . ... .............................................................................................................
. ...
.. .................................................................
. . ...
... ................................................................................ ...
... .................................................................................
....................................................................................................................................................................................................
.
...
.
............................................................................................................................................................................................................
.
2x − y 2 x
9) If = , what is ?
x+y 3 y

10) Eighteen years ago, Marco was three times older than his niece Shawana. Now he is twice as old as she
is. How old is Shawana?

11) In the figure below, ABC and ABD are right triangles with right angles at C and D, respectively. If
6 CAB = 60◦ , 6 DBA = 60◦ , and AB = 12 cm, what is the area of the shaded region?

C D
...... .....
... ............ ....... .....
... ....... ............. ...
... .......... ...
.... ... .................. ...
... . ... .............................................. ...
. . . .
.... ...................................................................................... ....
... ............................................................................. ...
. . ........................................................................................................
... . . . . . . . . . . . . . . . . . . . . . . . . . . . ... .
..........................................................................................................................................................................
A B

12) We say that a number is lowly to mean that all of its digits are either 1 or 2. Suppose all of the lowly
4-digit numbers are listed from least to greatest. What is the sum of the 8th and 9th numbers on the
list?

13) In a certain market, bananas cost $10, apples cost $20, and mangos cost $30. In how many ways can
you spend $80 on fruit in this market? (You are not required to purchase at least one of each type. For
example, you may buy 5 bananas and 1 mango.)

14) In the figure below, each side of triangle ABC has been extended by a segment equal in length to that
side. (So, AB = BL, BC = CM , and CA = AK. If the area of triangle ABC is 1, what is the area of
triangle KLM ?

M
.........
.. .......
... ..........
... ... ....
... ... ....
... ... ....
... ....
... ... ...
... . ....
.. ...C ...
...
.
..
........ ..... ....
...
... .
..
..... ...
... ....
..
. .
. ...... . ....
..
. ........ ...
.
...
....
.. . .
.. .......... ... ...
. . .
.............. .
.. .......... ............. .... ....
.
.
.. ........ A ...............
..
..............
...
.... B
.. ........ ............. ..
..
.
....... ............. ......
.................................................................................................................................................
K L

15) The weight of each possible pair of boys from a group of five was recorded. The following results were
obtained: 90 kg, 92 kg, 93 kg, 94 kg, 95 kg, 96 kg, 97 kg, 98 kg, 100 kg, and 101 kg. What is the total
weight of all five boys.

16) In the figure below, ABC is a right triangle with a right angle at B, D is the foot of the altitude from B,
AD = 3, and DC = 4. What is the area of ABC?

A ............
... .....
... ....... 3
... .....
... .....
.....
... .....
... .....
... D
.....
.....
... .........
... .... ......
... .................... .........
... ... . .....
... .
. .....
... ..... 4
...
... . .
..... .....
.....
... ..... .....
.....
... ..... .....
.. ..... .....
..... ...
...........................................................................................................
B C
The 2015 Jamaican Mathematical Olympiad
Solutions for Practice Problem Set 4
x/4 4 x/4 4/1
1. If = then = . Since dividing by a fraction is the same as multiplying by its
2 x/2 2/1 x/2
reciprocal,
x 1 4 2 x 8
× = × and so = .
4 2 1 x 8 x
Cross multiplying gives x2 = 64. then x = ±8.

2. Since the square has area 81 cm2 , each side has length 9 cm. When the square is cut into three
equal rectangles, each one is 3 cm × 9 cm. When the strips are placed end-to-end, the resulting
rectangle is 27 cm × 3 cm. Its perimeter is 27 cm + 3 cm + 27 cm + 3 cm = 60 cm.
........................................................
..... . . ...
... ... ... ... 9 9 9
... ..
... ... ... ... ..................................................................................................................................................................
.. ... ... .. .. ...
.... ... 3 .... .. .. .. 3
... ... ... ... ................................................................................................................................................................
... ... ... ...
... ..
... ... ... ... 9 9 9
...........................................................

3. Let a = 70 and b = 28. If a is reduced by 20% the new number is 80% of 70. This is
(0.8)(70) = 56. If b is reduced by 25% the new number is 75% of 28. This is (0.75)(28) = 21. The
product of the reduced numbers is 56 × 21 = 1176.

4. Let the seats be numbered 1, 2, 3, 4, and 5, as shown in the figures below, where 1 is the seat
number for Ashley. Since Brian is not sitting next to her, he is not in Seats 2 or 5.
......... .
................
.... ...... ....
.... ...
......
A
..... ....... A
...
...............
................................ ..
.... .. .......... ... ..........
........ ...... ......
..... ......
....... ..... .....
.....
..........
.... ..... ....
.
...
. 1 .... ..
... ........ ........ .
............... ..... 1 ....
... .................
.... . . .. ... . .... ... ... ... ... .
C
..... ....... .... ... .. ... ... . . ... ... .. C ..
... .................
....... .. ... ............... .
............. ...
.... 5 .. 2 .... 5 2 ...
.. ... ... .
..
... ... ... ...
... ... ... ...
... ... ... ..
.
... . ... ...
.... 4 ...3 ... 4 3 ....
..... .... .....
.....
...... .... ......
...... ...
. .. ........ ........................ ... ....
.... ........ .....
..................
.. .... ...................... .... .... . ... .... ...................... ..... .....
. .
..... . ... . .... . ..
..... .......
......
..... .......
......
B B
..... .......
......
...
................

Figure 4(a) Figure 4(b)

Then he must be in Seats 3 or 4. Suppose he is in Seat 3 as in Figure 4(a). Since Carl is not next
to Brian, he is not in Seats 2 or 4. Then Carl is in Seat 5. Since Dean is not next to Carl, he is not
in Seat 4. Then Dean is in Seat 2. It follows that Elton is in Seat 4 and his neighbors are Brian
and Carl.
Suppose now that Brian is in Seat 4 as in Figure 4(b) above. Since Carl is not next to Brian,
he is not in Seats 3 or 5. Then Carl is in Seat 2. Since Dean is not next to Carl, Dean is not in
Seat 3. Then Dean is in Seat 5. It follows that Elton is in Seat 3 and his neighbors once again are
Brian and Carl. Thus in either case Elton is sitting next to Brian and Carl.
5. Let y and z be the angles shown in the diagram below. (Both angles labeled with y are equal
because they are vertical angles, and similarly for the angles labeled with z.) From
.....
... ...
.. ......
...
. ....
....
..
... ....
....
5x
.... ....
.. ....
...
. ... y z
.......................................................................................................................................................................................................................................
... .. .... .....
...
... 3x ....
y ....
.... z ......
2x
.....
... ... .... .....
... ..
. .
.... ......
... ... ... ......
... ... ....
6x .....
... .. .... .....
... .. .... ..........
...
... ...
.. 4x .........
... ....
......
..

the triangle on the left, 3x + 4x + y = 180◦ . Then y = 180◦ − 7x. From the triangle on the right,
z+6x+2x = 180◦ and so z = 180◦ −8x. From the triangle on the top, 5x+y+z = 180◦ . Substituting
for y and z gives 5x+(180◦ −7x)+(180◦ −8x) = 180◦ . Simplifying gives 360◦ −10x = 180◦ . Adding
10x to both sides gives 360◦ = 180◦ + 10x. Subtracting 180◦ from both sides gives 180◦ = 10x.
dividing both sides by 10 gives x = 18◦ .

6. As stated in the question, the first four terms of the sequence are 1, 2, 3, and 6. The fifth term
is 2 + 3 + 6 = 11. The sixth term is 3 + 6 + 11 = 20. The seventh term is 6 + 11 + 20 = 37. The
eighth term is 11 + 20 + 37 = 68.

7. To begin, one can spend all of the $1500 to buy 15 boxes of chocolates. This gives a total of
15 coupons which may be redeemed for 5 more boxes. This gives another 5 coupons which may be
redeemed for one box of chocolate with 2 coupons left over. Finally, the one box contains a coupon
which may be redeemed with the other two for one last box of chocolate. The greatest number of
boxes one can get for $1500 is 15 + 5 + 1 + 1 = 22.

8. The original trapezoid is shown in Figure 8(a) below. The area of this trapezoid is equal to
that of the outer rectangle in Figure 8(b). This is because the small triangle in the lower right
............................................................................................................. .....................................................................................................................................
....... .. ..................... ..... .. . .. .....................
... ... ... ... .............................. .. ... ... ... ... ... ............................
... ..... ... ............................................................................................. ..... ... ... ..... ... ..........................................................................................
. .. .... .. . .. . . ....
.... . . . . . . . . . . . ............... ..
. . . . . . . . . . . ... ... .... . .. . . .. .. .
. . . .
.... . . . ... . . . . . . ................
. . . . . . . .
.. ... ......................................................................................................................... ..... ... .... ... ......................................................................................................................
... . . . ... . . . . . . . . . . . . . . . ... . . . . ... ... ...................................................................................................
. . .. .. . .. . .
.. ................................................................................................................................................. ...
... ... ... ....................................................................................................................................................................
... . . . . . ... . . . . . . . . . . . . . . . . . . . . . . . . . ..
... ........................................................................................................................................... ... ... .........................................................................................
........................................................................................................................................................................................................................................................... .................................................................................................................................................................................................................................

Figure 8(a) Figure 8(b)

corner in Figure 2(a) is congruent to the small triangle in the upper left corner in Figure 2(b).
Since the shaded area in Figure 2(b) is obviously half the area of the rectangle, the shaded area in
Figure 2(a) is half the area of the original trapezoid.

9. We may cross-multiply the given expression to obtain 3(2x − y) = 2(x + y) and hence 6x − 3y =
2x + 2y. Subtracting 2x from both sides gives 4x − 3y = 2y. Adding 3y to both sides gives 4x = 5y.
Dividing both sides by 4y gives x/y = 5/4.

10. Let M and S be the current ages of Marco and Shawana. Since Marco was three times older
than Shawana eighteen years ago, M −18 = 3(S −18). Since Marco is now twice as old as Shawana,
M = 2S. Then, by substitution, 2S − 18 = 3(S − 18) = 3S − 54. Adding 54 to the left and right
sides of this equation gives 2S + 36 = 3S. Subtracting 2S from both sides gives S = 36. Shawana
is 36 years old.

11. Let E be the point where AD and CB intersect and let F be the point on AB such that
EF ⊥ AB. Since 6 ADB = 60◦ and 6 BDA = 90◦ , it follows that 6 DAB = 30◦ . Similarly,

C... D
. ..... .........
... ............ E ....... ....
.. ....... ............ ...
.... . . .. . .
.. ... ...
.....................
.
.. .......................................................................
...
... ... . ..
. ......................................................................... .....
. . . . . .
. .
. .. ........................................................................................................................... ....
.
............................................................................................................
.
.......................................................................................................
A 6 F 6 B

6CBA = 30◦ . Thus 4EAB is isosceles and EA = EB. Furthermore, since 6 EAF = 30◦
and 6 AF E = 90◦ , it √
follows that 6 AEF = 60◦ . That is, AF E is a 30-60-90 triangle. Thus
EF : AF : AE = 1 : 3 : 2. In the same way, BF E is a 30-60-90 triangle. Thus the triangles
AEF and BEF are similar. Since AE = EB (and EF = EF ), they are in fact congruent. Then
√ EF 1 6 √
AF = F B = 6. It follows that EF : 6 = 1 : 3. That is, = √ and so EF = √ = 2 3.
6 3 3
1 √ √
Then the area of 4AEB is (12)(2 3 ) = 12 3.
2

12. Listed from least to greatest, the lowly numbers are:

1111, 1112, 1121, 1122, 1211, 1212, 1221, 1222,


2111, 2112, 2121, 2122, 2211, 2212, 2221, 2222.

The sum of the 8th and 9th numbers is 1222 + 2111 = 3333.

13. Given $80 to start with, a shopper may purchase 2 mangos, 1 mango, or no mangos at all.
We consider each case separately.
If a shopper purchases 2 mangos there will be $20 remaining. This may be spent on 1 apple or 2
bananas. There are two possibilities in this case.
If a shopper purchases 1 mango there will be $50 remaining. This may be spent on 2 apples and
1 banana, 1 apple and 3 bananas, or 5 bananas. There are three possibilities in this case.
If a shopper does not purchase a mango he or she will spend $80 on apples and bananas. This may
be spent on 4 apples, 3 apples and 2 bananas, 2 apples and 4 bananas, 1 apple and 6 bananas, or
8 bananas. There are five possibilities in this case.
In summary, considering the three cases together, there are 2 + 3 + 5 = 10 ways of purchasing fruit
in the market.

14. Join A and M with a segment as shown in the figure below. Since BC = CM the triangles
ACM and ACB have equal bases. Since the triangles both meet at A, they also have equal
M.
..
.................
..... .......
....... ..... ........
... .. ... ......
... .....
... ... ... ......
... .. ... .....
... ... .....
.. ... C ..... .....
.
.
.. ... ......... ..... .....
... .. ......... ...
...
.....
.....
... .. ..... ... .....
... ..........
. ....
.....
... ......................... .....
.. ....... ............. .... .....
.
.
.. ....... A ..............
. ...
. .
B
.............
.....
.....
.......... .. .
.. .............. .........
.......
..................................................................................................................................
.
K L

(perpendicular) heights. Then ABC and AM C both have the same area, which is 1. Now observe
that since AB = BL the triangles M AB and M BL have equal bases. Since both meet at M , they
also have equal heights. Thus they both have the same area, which is 2. By reasoning in the same
way, one shows that the triangles M CK and KAL also have area 2. Then the area of KLM is
the combined areas of ABC, M BL, M CK, and KAL. This is 1 + 2 + 2 + 2 = 7.

15. Let the five boys’ weights be a, b, c, d, and e, respectively. The weights of each possible pair
of boys are
a+b b+c c+d d+e
a+c b+d c+e
a+d b+e
a+e
If we add these expressions, the sum of the weights of all pairs of boys is 4a + 4b + 4c + 4d + 4e.
On the other hand, the sum of the weights of all pairs of boys is also 90 + 92 + 93 + 94 + 95 + 96 +
97 + 98 + 100 + 101 = 956. Then 4a + 4b + 4c + 4d + 4e = 956. Dividing throughout by 4 gives
a + b + c + d + e = 239. The total weight of all five boys is 239 kg.

16. Let h be the length of BD. The triangles ADB and ABC are similar. This is because
6 ADB = 6 ABC = 90◦ and 6 DAB = 6 BAC. Thus we also have 6 ABD = 6 BCD. Since

A ............
... .....
... .......
...
3 .....
... .....
.....
... .....
.....
...
... D .....
..
... ... .....
..... .......
...
... ..
. ... ............ ..........
.
h ... ....
... .....
4
.....
.....
... .... .....
... .... .....
... ...... .....
.....
... ..... .....
........ ..
...............................................................................................
B C

corresponding sides of similar triangles are in the same √proportion, AD/DB = BD/DC. Thus
2
3/h = h/4. Cross
√ multiplying gives h = 12. Then h = 2 3. The base AC of 4ABC is 7 and the
height BD is 2 3. The area of 4ABC is

1 √ √
(7)(2 3) = 7 3.
2
The 2015 Jamaican Mathematical Olympiad
Pracice Problem Set 5

1) How many 3-digit numbers abc have the property that a + 3b + c is a multiple of 3?

1 1
2) Let x1 , x2 , and x3 be numbers such that < x1 < x2 < x3 < . Suppose the distances between the
4 3
consecutive numbers 1/4, x1 , x2 , x3 , and 1/3 are all equal. What is x1 ?

3) In the figure below, a regular hexagon is inscribed in a circle of radius 1. What is the area of trian-
gle ABC?
.........................................
........ .
.....................................................................
.
........... ... .....
. . ... ....
... ...
.... .... ... ...
... ...
... ... ... ..
.... ..... ... ...
... ..
... .... ......
..... O ....
..................................................................................................................
• C
...... .
... ..
.. ... .. ..
...... ........
.
. .
... ... ....... .... ....
... ...
.. ... ...
........ .. ..
.. .
... .. ....... ... ....
... ... ....... ... ...
.... ... ...... ... ...
........ ....... ...........
......... ........... ..
...............................................................
......... .
....................................
A B

4) The natural numbers 24, 36, and N have the property that the product of any two of them is a multiple
of the third number. What is the smallest possible value for N ?

5) A two-digit number is called fascinating if the number obtained by reversing its digits is 75% larger
than the original number. For example, 24 is fascinating because 42 is 75% larger than 24. How many
two-digit numbers are fascinating?

6) We say that a 3-digit number is balanced to mean that its middle digit (i.e., its tens digit) is the average
of its other two digits. How many 3-digit numbers are balanced?

7) The triangle ABC below has a right angle at B. If AB = 3, BC = 4, and CA = 5, what is the radius
of the inscribed circle?
A .......
.... .........
.. ......
... ......
......
.... ......
.
.. .............................
... .......... .....
.... ..... ........
.........
..... ... ......
..... .. .......
.. . ......
..... ... ......
....... ...
.
......
......
.... ..... .. .. ......
... ......
.. ........ ....
. ...
.....................................................................................................................
B C

1 1 1 1
8) If x = √ √ +√ √ +√ √ +···+ √ √ , what is the value of x?
1+ 2 2+ 3 3+ 4 99 + 100
The 2015 Jamaican Mathematical Olympiad
Solutions for Practice Problem Set 5
1. Note that a+3b+c is a multiple of 3 if and only if a+c is a multiple of 3. Then we must consider
numbers abc such that a + c is a multiple of 3. Since we cannot have a = 0, the possibilities are:

a = 1, c = 2, 5, 8; a = 4, c = 2, 5, 8; a = 7, c = 2, 5, 8;
a = 2, c = 1, 4, 7; a = 5, c = 1, 4, 7; a = 8, c = 1, 4, 7;
a = 3, c = 0, 3, 6, 9; a = 6, c = 0, 3, 6, 9; a = 9, c = 0, 3, 6, 9;

There are 30 possibilities in all. For each one, there are 10 choices for b. So, there are 300 numbers
abc such that a + 3b + c is a multiple of 3.

2. Let d be the distance from 1/4 to x1 . Then d is also the distance from x1 to x2 , from x2 to x3 ,
and from x3 to 1/3. The total distance from 1/4 to 1/3 is 4d. But the distance from 1/4 to 1/3 is
also
1 1 4 3 1
− = − = .
3 4 12 12 12
1 1 1 12 1 13
Then 4d = 1/12. This means that d = 1/48. It follows that x1 = + d = + = + = .
4 4 48 48 48 48

3. Let D be the point opposite C, as shown in the figure below. The lines CD and AB are parallel.
This is because 6 COB = 6 OBA = 60◦ . Hence they form equal “Z-angles” between DC and AB.
Then the
...........................
........... .
..............................................................
....... ........
..
.......... ... .....
... ...
. ... ...
... ... ... ...
... ...
... ...
.. ... ... ..
. ... ...
........ O .....
.... .
..........................................................................................................

D .....
.
.. ... .. ....
. ...
. ........... C
... .. ... .... . ..
. .... ... ...
.
... .... ... ... ......... ... ...
.
... ... .. ........ .. ..
... ... ... ....... .. ... ...
.... ... .... ............ ..... .... ......
..... ... .. ........ ... .. .....
........ ......... . ..
...................................................................
.......... .
.............................
A B

triangles AOB and ACB have the same √ area. (They have√the same base, AB, and equal heights.)

But triangle AOB has base 1 and height 3/2. Its area is 3/4. Then the area of 4ABC is 3/4.

4. For any N to be considered, 24N must be a multiple of 36. This happens when N = 3, 6, 9, 12,
. . . Also, 36N must be a multiple of 24. This happens when N = 2, 4, 6, 8, 10, 12, . . . The values
of N satisfying both conditions at once are N = 6, 12, 18, . . .. The smallest of these values is
N = 6. This value also satisfies the condition that 24 × 36 is a multiple of N . So, 6 is the smallest
possible value for N .

5. Let ab be a fascinating number. Its value is 10a + b. When its digits are reversed, the new
number has value 10b + a. This is 75% larger than the original number when
75 3 7
10b + a = (10a + b) + (10a + b) = (10a + b) + (10a + b) = (10a + b)
100 4 4
Multiplying throughout by 4 gives 40b + 4a = 70a + 7b. Subtracting 7b from both sides gives
33b + 4a = 70a. Subtracting 4a from both sides gives 33b = 66a. Dividing both sides by 33 gives
b = 2a. The two-digit numbers ab with b = 2a are 12, 24, 36, and 48. These are all fascinating, as
one may verify. So, there are four such numbers in all.

6. Let abc be a 3-digit number such that b is the average of a and c. It is not possible that b = 0
because then a = 0 and c = 0 as well (and 000 is not a 3-digit number.) So we consider the cases
with b = 1, 2, 3, . . ., 9. The possibilities are as follows:
• b = 1: Then abc = 111 or 210;
• b = 2: Then abc = 123, 222, 321, or 420;
• b = 3: Then abc = 135, 234, 333, 432, 531, or 630;
• b = 4: Then abc = 147, 246, 345, 444, 543, 642, 741, or 840;
• b = 5: Then abc = 159, 258, 357, 456, 555, 654, 753, 852, or 951;
• b = 6: Then abc = 369, 468, 567, 666, 765, 864, or 963;
• b = 7: Then abc = 579, 678, 777, 876, or 975;
• b = 8: Then abc = 789, 888, or 987;
• b = 9: Then abc = 999,
There are 2 + 4 + 6 + 8 + 9 + 7 + 5 + 3 + 1 = 45 balanced numbers in all.

7. Let O be the centre of the circle, and let D, E, and F be the points on BC, CA, and AB,
respectively, where the circle is tangent to the triangle. Join O with F , with D, with E, and
with A. Since F and E are points of tangency, 6 AF O = 6 AEO = 90◦ . Also, F O = OE because

A ...
.............. a
... .... ..........
.. .. ....... E
a ....... .............................................................
..... ... .. ... ........
.... ... ... ... .........
F .............................. .. c
.......
........
O .... ..
. ... .......
........
........ .
. ...
. ........
b ... .....
..
...
....
.. .......
...
...............................................................................................................................................
B b c C
D

both are radii of the circle. By the Pythagorean theorem (used twice), (AF )2 = (AO)2 − (F O)2 =
(AO)2 − (OE)2 = (AE)2 . Since AF and AE are both positive, AF = AE. One proves similarly
that BF = BD and CD = CE. Let a = AF = AE, b = BF = BD, and c = CD = CE. We have
the following three equations in three unknowns:

a + b = 3
b + c = 4

a + c = 5

The unique solution is a = 2, b = 1, and c = 3. In particular, b = 1. But OF BD is a square


because 6 OF D = 6 F BD = 6 BDO = 90◦ and F B = BD. Thus OF = b = 1. The circle has
radius 1.

8. For n = 1, 2, 3, . . ., 99,
√ √ √ √ √ √
1 1 n+1− n n+1− n n+1− n √ √
√ √ =√ √ √ √ = = = n+1− n
n+ n+1 n+1+ n n+1− n (n + 1) − n 1
1 √ √ 1 √ √ 1 √ √
Thus √ √ = 2 − 1, √ √ = 3 − 2, √ √ = 4 − 3, and so on. Then
1+ 2 2+ 3 3+ 4

1 1 1 1
x= √ √ +√ √ +√ √ + ··· + √ √
1+ 2 2+ 3 3+ 4 99 + 100
√ √ √ √ √ √ √ √
= ( 2 − 1) + ( 3 − 2) + ( 4 − 3) + · · · + ( 100 − 99)

Writing these expressions in reverse order gives


√ √ √ √ √ √ √ √ √ √
x = ( 100 − 99) + ( 99 − 98) + · · · + ( 4 − 3) + ( 3 − 2) + ( 2 − 1)
√ √
All terms except the first and last ones cancel. Then x = 100 − 1 = 10 − 1 = 9.
The 2015 Jamaican Mathematical Olympiad
Practice Problem Set 6
r
1 1
1) What is + in simplified form?
9 16

2) Marsha has between 50 and 100 books. She told her friend that 25% of her books are novels and 1/9 of
them are cookbooks. Exactly how many books does Marsha have?

3) In the figure below, AC = 10, BD = 15, and AD = 22. What is the length of BC?
A B C D
• • • •
.........................................................................................................................................................................................................

4) The height of a certain rectangle is 12 cm and its width is three times its height. What is the area of a
square whose perimeter is 1/3 the perimeter of the rectangle?

5) Maurice started with a number, divided it by 7, added 7 to the result, and multiplied the sum by 7. He
ended up with 777. What number did Maurice start with?

6) Peter and Paul went to a scout camp. During a meeting all the scouts stood in a single row. Peter was
exactly in the middle. There were 27 scouts to Paul’s left and 13 to Paul’s right. How many scouts
stood between Peter and Paul?

7) A small boy started placing toothpicks on a table to form a rectangular grid, as shown in the figure
below. To make a grid which is 10 toothpicks wide and 8 toothpicks tall, how many toothpicks will he
need?
........................................... .....................
.... .... .... ....
... ... ... ...
.... .... .... ...
. . . ..
. ..................... . ..
...
...
...
...
...
...
. . ..
...
...
...
...
...
...
. .
..... ..... ..... .....
.. .. .. ..
... ... ... ...
.. .. .. ..
..................... ..................... .....................

8) In the figure below, the area of triangle ADB is 15, the area of triangle ACB is 12, and the area of
triangle ABE is 4. What is the area of the pentagon ABCED?

D ........
...........
... .........
... .......
....... C
... ....... .........
... ....... ........ ...
... ....... ........ ....
... ....... ........ ...
... ....... .
. . ............ .
..
....... ...
... ....... E ........ ...
... ....... ........ ...
... ....... ........ ...
... .................. ...
.
... ...
.... .
..... ..
..... ....... .
... ........ ....... ...
... ........ ....... ...
... ........ ....... ..
... ............... ...... ...
....................................................................................................................
A B

9) We say that a 3-digit number is peculiar to mean that all of its digits are odd and different from each
other. How many peculiar numbers are multiples of 3?
10) When an integer n > 1 is divided by 2, 3, 4, 5, and 6, the remainder is 1 each time. What is the smallest
number that n could be?

11) When 10002012 is written as a numeral, how many digits does it have?

12) In the figure below, ABCD is a square and ABE and BF C are equilateral triangles. If the square has
side-length 3, what is the area of the shaded region?
B
A ............................................................................................
...... ..... .......
.. .............................
..... .... .............................
... .... .....................................................................
... .. . . ... . . . . . . . . . . ...
... ... .......................................................................................
.... ... ......................................................
... ... ...........................................................
... ... ...........................................................................................................................................................
.
... ...
...
.
.................................................. ................
.........................................................................
F
...
E... ........................................................................ ............
.... ... .............................................. ......
... ... .................................................................... .......
... .........................
.... .......
... .......... .......
... ... ............
.....................................................................................
D C

13) Which power of 96 is equal to 278 ?

√ √ √
14) Suppose a, b, and c are positive real numbers such that ab = 2 3, ac = 6, and bc = 3 2. What is
abc?

15) Paul removed one number from a sequence of seven consecutive natural numbers. The sum of the
remaining numbers is 2012. Which number did Paul remove?

16) A pyramid is shown in the figure below. Its base is a square with side length 1 and its sides are equilateral
triangles. What is the height of the pyramid?

.....
...............
..... .. .......
..... ... ... ..............
..
. ...... .. .... .......
........
..... ... ...
..... ... .......
..... . ... .......
.......
. . .
.......
...
...
.
....
. ...
..
. .
... ....... ....... .......... ....... ....... ....... ....................
..
...... ....... ... ..
.......
...
.... ....... ...
... ....
. .......
.... .. .......
....
..... ..... ...
... ..............
.....................................................
...........................................................
The 2015 Jamaican Mathematical Olympiad
Solutions for Practice Problem Set 6
r r r
1 1 16 9 25 5
1. We have + = + = = .
9 16 144 144 144 12

2. Since one-fourth of Marsha’s books are novels, the number of books she has is a multiple of 4.
Since one-ninth of them are cookbooks, the number of books she has is also a multiple of 9. Then
the number of her books is a multiple of 36. The multiples of 36 are 36, 72, 108, . . . The only
multiple between 50 and 100 is 72. Marsha has 72 books.

3. From the information given, AB + BC = 10 and AB + BC + CD = 22. Subtracting equations


gives CD = 12. We also know that BC + CD = 15. Thus BC + 12 = 15. Subtracting 12 from
both sides gives BC = 3.

A B C D
• • • •
.......................................................................................................................................................................................

4. The height of the rectangle is 12 cm and its width is 36 cm. The perimeter of the rectangle
is 96 cm. Since the perimeter of the square is 1/3 as much, the perimeter of the square is 32 cm.
Then the length of each side of the square is 8 cm. The area of the square is 64 cm2 .
.......................................................................................
.... ... ...........................
... ... .... ...
...
... ... ...
12 ... ... 8 .. ..
... ... ............................
.. .
.................................................................................
8
36
The 2015 Jamaican Mathematical Olympiad
Practice Problem Set 7

1) In the table below, the letters A, B, C, . . ., G represent numbers (not necessarily distinct). If
the sum of any three consecutive numbers is 15, what is the value of F ?
.................................................................................................................................................................................................
..... ... ... ... ... ... ... ... ... ...
... ... ... ... ... ... ... ... ... ...
... 3 A B C D 8 E F G
. . . . . . . .
..............................................................................................................................................................................................................................
.

2) The positive integers greater than 1 are arranged in 5 columns as shown below:

2 3 4 5
98 7 6
10 11 12 13
17 16 15 14
.. .. .. .. ..
. . . . .

In which column will 1,000 be located?

3) In the figure below, P , Q, and R are points on a circle with centre O. If 6 OP R = 5◦ and
6 OQP = 40◦ , what is 6 OQR?

Q
...............
........... ...................................
....... . .
..... ..... .. ......... ..........
.. .....
. ...
...... .... ..... .....
.
.... ...
..... .
.
.
..... ..
..... ...
... ...... .. ..... ...
.. ..... ... ..... ..
.
... ......... ...
. ........
..
.. . R
............. ...................................................................................................
P .........................................................................
... • ..
.
.. ...
... ..
... O ..
.
.
.
... ...
... ..
... ...
....
..... . .
. .....
...... ..
....... ......
.............. ....................
........

4) In parallelogram ABCD below, P is the midpoint of AB and Q is the midpoint of BC. If the
area of ABCD is 24, what is the area of 4DP Q?
P
A •
.........................................................................
... ...
B
... ...
... ...
... ...
...
...
...
... •Q
... ...
... .
.........................................................................
D C

5) A train consists of an engine and five cars: I, II, III, IV and V. In how many ways can the
cars be arranged so that car I is closer to the engine than car II?

6) Maria has two apples, two bananas, and one mango. Each school day, from Monday to Friday,
she will eat one piece of fruit. In how many ways can she do this?
7) Find all pairs of positive integers (x, y) such that 22x − 32y = 55.

8) In the figure below, ABDE is a rectangle and the points C, D, E, and F are collinear. If the
rectangle ABDE has area 80 and the trapezoid ABCF has area 128, what is AB : CF ?

A B
.........................................................................
...... ... ......
... .. ... ......
.. ....
. ... .....
... ... ... .....
.....
... ... .. .....
... ..
. ..
. .....
..
. ..
. ..
. .....
.....
... ..
. ..
. .....
.. . .
.................................................................................................................................
. . .

F E D C

9) In the figure below, 6 C = 90◦ , AD = DB, DE ⊥ AB, AB = 20, and AC = 12. What is the
area of quadrilateral ADEC?
C
....
...... ...
..... .....
E ............... ...
...
....... ...
...... .. ...
.
......... .... ...
...
....
.... .
.
. ...
.
....... .
. ...
....
.... .
.
. ...
.
....... .
.
.
...
...
......
. .
.
. ...
..
...... .
.
. ...
........ .
.
.
...
.
...............................................................................................................................................
B A
D

10) Let S = {1, 2, 3, . . . , 24, 25}. The set T is a subset of S with the property that the sum of any
two elements in T is not divisible by 3. What is the largest number of elements that T could
have?

11) In the figure below, the sides of the heptagon T U V W XY Z (which is not necessarily regular)
have been extended to form a “7-pointed star”. Find the sum of the angles at A, B, C, D, E,
F , and G.
A
.
........
.... ......
... ...
.
.... ....
T ....... ... U
...
.........
..•................... •.......
......................... B
G ......................................................................................................
.
.
...
...
...
..... .
..
... .
....
...
...
....
..
...
...
. .. .
.
... ....
... ... ... .
.... ..
... ..... ... ..
..
... .... .•.. .....
Z ... ... •......
.
..
. ..... V
.. ...
.....
.
.. ... ... ... ... .
.. .......
.
.
. ....
..
...
.
. ... ...
. .
.. ...
...
..
........
... ... ...
F ........ ...
........ ...
...
.
. ...
. ..............
. ...... C
.•.... .
..•..
.......... .
. .........
.
......... .
.......... ... .
.......... ............... ........
............
. ..... ........ ..
Y ...... .................... ................... ..... W
...
...
...•.... ............
...... .............
.
..
... ............. ........ ....
......... X .....

E D

12) We wish to tile a patio with sides of length positive integer N . We have two types of tiles:
5 × 5 squares and 1 × 3 rectangles. Determine the values of N for which this is possible. (The
patio must be completely tiled with no tiles overlapping.)
The 2015 Jamaican Mathematical Olympiad
Solutions for Practice Problem Set 7
1. From the first three squares, 3 + A + B = 15. Thus A + B = 12. From the second, third, and
fourth squares, A + B + C = 15 as well. Then 12 + C = 15 and so C = 3. From the fourth, fifth,
................................................................................................................................................................................................................................. .............................................................................................................................................................................................................
...
.. .. .. .. .. .. .. .. .. ... .... .... .... .... .... .... .... .... ....
.... 3 A B C D 8 E F G
.... .... .... .... .... .... .... .... ... ..... 3 4.... 8
.... 3
.... 4
.... .... 8 .... 3 .... 4 .... 8 ....
.. . . . . . . . . . ... . . . . . . . . .
.................................................................................................................................................................................................. .................................................................................................................................................................................................

and sixth squares, 3 + D + 8 = 15 and so D = 4. From the fifth, sixth, and seventh squares,
4 + 8 + E = 15 and so E = 3. Finally, from the sixth, seventh, and eighth squares, 8 + 3 + F = 15
and so F = 4. The completed table is shown on the right above.

2. The entries in the first column are 9, 17, 25, 33, . . . These are the numbers of the form 8n + 1.
Eventually (i.e., when n = 125), the number 1001 occurs in the first column. The number 1000
occurs in Column 2.
2 3 4 5
9 8 7 6
10 11 12 13
17 16 15 14
.. .. .. .. ..
. . . . .

3. Since OP , OQ, and OR are radial segments, they are equal in length. Then the triangles OP Q,
OP R, and OQR are all isosceles with 6 OP Q = 6 OQP = 40◦ , 6 OP R = 6 ORP = 5◦ , and

Q
......
........ •
...................................................
.... . ... .....
..... ..... .. ......... .........
... ..... ..... ..... ....
.... ..... ..... ...
.
.... ..
....... .
... ..... ...
.
... ...... .. ..... ..
... ......... ....
.
..... ...
.......

P•
... ........ ..
. •R
......................
............................................................................................................ ...
..
...
...

......................................... ..
...
..
...
...
O ...
.
... ...
..
... ..
....
..... .
......
..... ...
....... .....
......... .......
................................

6OQR = 6 ORQ. Since the sum of the angles in any triangle is 180◦ , 6 P OR = 180◦ − 5◦ − 5◦ =
170◦ . Similarly, 6 P OQ = 180◦ − 40◦ − 40◦ = 100◦ . Then 6 QOR = 6 P OR − 6 P OQ = 170◦ −
100◦ = 70◦ . Finally, 6 QOR + 6 OQR + 6 ORQ = 180◦ . Substituting for 6 QOR and 6 ORQ gives
70◦ + 26 OQR = 180◦ . Subtracting 70◦ from both sides gives 26 OQR = 110◦ . Dividing both sides
by 2 gives 6 OQR = 55◦ .

4. First, the area of 4DAB is half of the area of ABCD. Then the area of 4DAB is 12. Also,
since P is the midpoint of AB, the area of 4DAP is half the area of 4DAB. Then the area
P
A •
...............................................................................
... . ........ ..
B
... ... .......
....... .....
... ... ....... ..
... ... .........
...
... ....
. ..
..........
......
.......... ....
...
•Q
... ... ..........
... ... ................... ...
..................................................................................
D C
of 4DAP is 6. By reasoning in the same way, one sees that the area of 4DQC is 6 as well.
Similarly, the area of 4P QB is half the area of 4P CB, and this is half the area of 4ACB, and
this is half the area of ABCD. Then the area of 4P QB is

1 1 1
× × × 24 = 3.
2 2 2

Finally, the area of 4DP Q is the area of ABCD minus the areas of 4DAP , 4DCQ, and 4P QB.
So, the area of 4DP Q is 24 − (6 + 6 + 3) = 24 − 15 = 9.

5. Car I can possibly be in the first, second, third, or fourth position in the train. (It cannot be
in the last position because then Car II could not be behind it.) We consider each possibility in
turn.
If Car I is in the first position there are 4 × 3 × 2 × 1 = 24 ways of arranging the other four cars.
(Each time, Car II will be behind Car I). This gives 24 possibilities in this case.
If Car I is in the second position any car except Car II may be in the first position. This gives 3
possibilities. For each one, there are 3 × 2 × 1 = 6 ways of arranging the last three cars. There are
3 × 6 = 18 possibilities in this case.
If Car I is in the third position there are 3 choices for the car in the first position and two for the
one in the second position. For each of these pairs of choices, there are 2 × 1 = 2 ways of arranging
the last two cars. Thus there are 6 × 2 = 12 possibilities in this case.
Finally, if Car I is in the fourth position then car II must be in the fifth position. The other three
cars will fill the first three positions and there are 3 × 2 × 1 = 6 ways of doing this.
Putting these cases together, there are 24 + 18 + 12 + 6 = 60 ways of arranging the cars so that
Car I is closer to the engine than Car II is.

6. First, Maria can decide which days she will eat the apples on. If a stands for apple and x
stands for another piece of fruit, she could eat the apples in any of the following ways:

a, a, x, x, x x, a, a, x, x x, x, a, a, x x, x, x, a, a
a, x, a, x, x x, a, x, a, x x, x, a, x, a
a, x, x, a, x x, a, x, x, a
a, x, x, x, a

There are 10 ways of choosing the days to eat an apple on. Once this is done, there are three more
days to choose fruits for. Suppose b stands for a banana and m for a mango. The fruits on these
days could be in the order of b, b, m, or b, m, b, or m, b, b. There are 3 possibilities in all. So there
are 10 ways of deciding which days to eat apples on, and for each one there are 3 ways of deciding
on the other fruits. There are 10 × 3 = 30 ways for Maria to eat her fruit.

7. We seek solutions to the equation 22x − 32y = 55. We may rewrite this as (2x )2 − (3y )2 = 55.
Since the left side is the difference of two squares, we have (2x + 3y )(2x − 3y ) = 55. Since x and y
are positive integers, 2x and 3y are both positive integers. Thus 2x + 3y and 2x − 3y are factors
of 55. Furthermore, 2x + 3y > 2x − 3y . Since the factors of 55 are 1, 5, 11, and 55, the possibilities
are:  x  x
2 + 3y = 55 2 + 3y = 11
x y
2 − 3 = 1 2x − 3y = 5
In the first case, adding equations gives 2(2x ) = 56 and so 2x = 28. However, there is no integer x
solving this equation. In the second case, adding equations gives 2(2x ) = 16 and hence 2x = 8.
This has solution x = 3. Substituting into either equation gives y = 1 as well. Then (3, 1) is the
only pair of positive integers (x, y) such that 22x − 32y = 55.

8. Since the area of ABDE is 80, we have (AB)(AE) = 80. Since the area of ABCF is 128, we
have 12 (AB + CF )(AE) = 128. Multiplying both sides by 2 gives (AB + CF )(AE) = 256. Thus

A B
..................................................................
...... .... ......
... .. ... .......
... .... ... .....
... .. .. .....
.....
.. ....
. .... .....
..
. ..
. . .....
... ..
. ...
. .....
... ..
. ..
. .....
.....
..
. .
. .
. .
.....................................................................................................................................
F E D C

(AB)(AE) + (CF )(AE) = 256. Substituting for (AB)(AE) gives 80 + (CF )(AE) = 256. Sub-
AB (AB)(AE) 80 5
tracting 80 from both sides gives (CF )(AE) = 176. Then = = = . That
CF (CF )(AE) 176 11
is, AB : CF = 5 : 11.

9. Since 4ABC is a right triangle, (AC)2 + (BC)2 = (AB)2 . Substituting for AB and AC gives
122 + (BC)2 = 202 and so 144 + (BC)2 = 400. Subtracting 144 from both sides gives (BC)2 = 256.
C
....
..... ...
...... .....
E .............. ...
...
....... ...
..... ... ...
....
....... .... ...
...
..
...... .
.
. ...
.......
. .
.
. ...
.
....... .
.
. ...
...
.
...... .
.
. ...
...
..... .
.
. ...
.
....... .
.
. ...
...
..... .
.
.
...
.
...............................................................................................................................................
B A
D

Then BC = 16. Since 6 C is a right angle, the area of 4ABC is equal to 12 (16)(12) = 96. To find
the area of 4BED, observe that the triangles BED and ABC are similar. This is because both
have a right angle and both share the angle at B. Since the sum of the angles in any triangle is 180◦ ,
it follows that 6 BED = 6 BAC as well. Since corresponding sides of similar triangles are in the
same proportion, ED/BD = CA/CB. Substituting for BD, CA, and CB gives ED/10 = 12/16.
Cross-multiplying gives 16(ED) = 120. Dividing both sides by 16 gives ED = 120/16 = 15/2. It

follows that the area of 4BED is equal to 12 (10) 15 2
= 75
2
. Finally, the area of ADEC is equal
75 117
to the area of 4ABC minus the area of 4BED. This is 96 − 2
= 2
.

10. Let
S1 = {1, 4, 7, 10, 13, 16, 19, 22, 25},
S2 = {2, 5, 8, 11, 14, 17, 20, 23},
S3 = {3, 6, 9, 12, 15, 18, 21, 24}.
Then S1 consists of all the numbers in S with remainder 1 when divided by 3, S2 consists of all
the numbers with remainder 2, and S3 consists of all the numbers divisible by 3. Note that the
sum of any two numbers in S1 is not divisible by 3. Similarly, the sum of any two numbers in S2 is
not divisible by 3. However, the sum of any number in S1 and any number in S2 is divisible by 3.
Let T be a subset of S with the property that the sum of any two elements in T is not divisible
by 3. Then T can contain all the elements in S1 or all the elements in S2 , but it cannot contain
elements from both subsets. Furthermore, suppose T contains all the elements in S1 . Then it can
also contain one element from S3 . Similarly, if T contains all the elements in S2 it may also contain
one element from S3 . However, T cannot contain two elements from S3 . For example if T contains
6 and 15 then it contains 6 + 15 = 21. But 21 is a multiple of 3. Then the possibilities are that:
a) T contains all elements in S1 and one element from S3 , or
b) T contains all elements in S2 and one element from S3 .
Since (a) gives a set with more elements, T contains 10 elements. For example, we could have
T = {1, 3, 4, 7, 10, 13, 16, 19, 22, 25}

11. Let `, m, n, p, q, r, and s be the interior angles at the vertices T , U , V , W , X, Y , and Z,


respectively. Then 6 AT U = 180◦ − ` and 6 AU T = 180◦ − m. Since the sum of

A
..
.......
... ......
... ....
.
..... ...
T ....... ... U
....
...........................•......
.
G .................................................................... ...........................
.......................... B
...•`
. ....
..
m .. ...
...
... .. .. ..
....
...
... ..
...
... . ... ...
....
.
.
....
....
.
.... ...
...
.

. ...
... .....
.. ... ..
... .
......
.•.. s
Z ............... n• .... V
.....

...
. .
.
.. ....... ... ... ..
.. ....
.. ......
...
... ... ... ...
..
... ...
... ... .. ...
F ..........
........ ... ...
. ...
.
. .......... .
. .......... C
........ ....
.•....r p . ..
.......... q .........•. ...............
..........
........ .
... ........ ........
.......... .........
. .... ....... ..
Y ...... .................... .................. ...... W
...
...
...•.... ............
........ ............... ...
..
... ............. ........ ..
........ X ....
E D

the angles in any triangle is 180◦ , 6 A+ 6 AT U + 6 AU T = 180◦ . Substituting for 6 AU T and 6 AT U


gives 6 A + (180◦ − `) + (180◦ − m) = 180◦ . Simplifying gives 6 A + 180◦ = ` + m. Reasoning in a
similar way, we obtain
6 B + 180◦ = m + n; 6 C + 180◦ = n + p; 6 D + 180◦ = p + q;
6 E + 180◦ = q + r; 6 F + 180◦ = r + s; 6 G + 180◦ = s + `.
Adding these seven equations together gives
6 A + 6 B + 6 C + 6 D + 6 E + 6 F + 6 G + 1260◦
= (` + m) + (m + n) + (n + p) + (p + q) + (q + r) + (r + s) + (s + `)
= 2(` + m + n + p + q + r + s)
But ` + m + n + p + q + r + s is the sum of the interior angles of the heptagon T U V W XY Z. This
is equal to (7 − 2)180◦ = 5(180◦ ) = 900◦ . Thus
6 A+6 B+6 C +6 D+6 E+6 F +6 G
= 2(` + m + n + p + q + r + s) − 1260◦ = 2(900◦ ) − 1260◦ = 540◦
The University of the West Indies, Mona Campus
The 2014 Jamaican Mathematical Olympiad
First Round
Test for Grades 9, 10, and 11

Part A

This part consists of four multiple-choice questions. For each one, mark the letter for the correct answer
(a), (b), (c), (d), or (e) in the answer book provided. Each question in this part is worth 5 marks.

1) Ava had a bag of sweeties. She gave half of them to Beth, Beth gave one-third of her share to Celine,
and Celine gave one-fourth of her share to Davia. Davia received 3 sweeties from Celine. How many
sweeties did Ava have in her bag?
(a) 72 (b) 64 (c) 108 (d) 84 (e) 12

2) Which of the fractions below has the greatest value?

7 66 555 4444 33333


(a) (b) (c) (d) (e)
8 77 666 5555 44444

3) Suppose m and n are integers and P = 2m and Q = 3n . Which of the following expressions is equal to
12mn ?
(a) P 2 Q (b) P n Qm (c) P n Q2m (d) P 2m Qn (e) P 2n Qm

4) Each of the four squares in the figure below are congruent. In each of them the midpoints of the sides
are marked with a solid dot and regions with areas S1 , S2 , S3 , and S4 , respectively, are shaded. Which
expression below is true?

.... •
.................................................................................
........
.................
... •
................................................................................................................
................................................................................ ... •
.................................................................................
....... ................................................ •
.............................................................................................
..
.... ............................
... .... ............................................................................................ ... .. .......... ... .. ........................................................ ....
.. ................................................... ... .. ................................................................... ... ... ......... .. ... ..............................
... ... . ... ... ............................................................. .... .... . ................ ....
. .... .........................................................
..
...
... . . . . . . . . . . . . .. .......... .
.... ............................................................................................ .... .... ......................................................................... ... ... ................ .... ... .............................................. ....
.. ...................................................................... ... .. ................................ .. ... ... ............... .. ... ........................................... ..
..................................................................................... ... S
...................................
.......................2 ............ ... .. .....................
.............................. ... .. .............................
........................................... ...
• S
.................................................................................
... ................................................1 .................................. . .. • • ...
...
..............................................
..................................
...
... • • ....
... .. .
.
..
.........................
...........................
...
...
• • ....
... S
...............4
. . . . . . . . . . .
.............................................
. . • ...
...
... ..................................................................... .... ... ............................... ..... ... ...
...................................... ... ... ...........................................
........................................... ....
.
.... ................................................ ... .... ................... ... ... .
.. .... ...
................................
..
...
.............................
.........................
...........................
. ....
...
..
...
.......................
...................
.............
...
....
... . .
.. S
... ......................................3
. . . . . . . ...........
.. .
..
...
...
...
.......................................... ...
........................................... ..
............................... ..
.... ................ ... .... ......... .. ... .............................................. .. ... ........................................ ..
...................................................................................... ................................................................................... ............................................................................................................... ...............................................................................................................
..
• • • •

(a) S3 < S4 < S1 = S2


(b) S3 < S1 = S4 < S2
(c) S4 < S3 < S1 < S2
(d) S3 < S1 = S2 = S4
(e) S3 < S4 < S1 < S2
Part B

This part consists of eight written-answer questions. For each one, give your solution in the answer book
provided. Each question in this part is worth 10 marks. To score full marks, you must provide an answer
which is both correct and completely justified.

5) A grocer makes a display of cans in which the top row has one can and each lower row has two more
cans than the row above it. Suppose the display contains 36 cans. How many rows does it contain?

6) For how many two-digit numbers ab is it true that a and b are odd numbers and a 6= b?

7) The figure below shows two circles with radii 3 and 2, respectively. They are tangent to each other and
tangent to the larger circle around them both. What is the area of the shaded region?
...............................
........................................
.............................................................
.....................................................
.
. ..........................................................................................................
...........................................................................................
................. ........................................
............. ...................................................
........ ...........................................
........... ...............
..............
..........
......
........ ........ .......
.. 3
..................................
.. 2
.......................
..
..
... • .
.....
.
• .
......
.....
.............
. . .. .
...... .............
........ ........
.............. ......................................................................................
.......................................
.
................
........................................................................................................................
.........................................................
............................................................................
...............................................
...........................................
.........................................
......................

8) A 3 × 3 × 3 cube is formed by gluing together 27 standard cubical dice. (On a standard die, the sum of
the numbers on any pair of opposite sides is 7.) What is the smallest possible sum of all the numbers
showing on the surface of the 3 × 3 × 3 cube?

9) During a recent football match, the average age of the 11 players on the Strikers was 22. During the
match one of them was injured and had to leave the field. The average age of the remaining players
was 21. How old was the injured player?

10) Marcus has some $10 and $20 coins in a box. The number of $10 coins is three times the number of
$20 coins. If he removes 8 coins of each type from the box, the number of $10 coins will be five times
the number of $20 coins. What is the total value of the coins in his box?

11) In the figure below, four equal circles are inscribed in a square. The points P and Q are the centres of
two of the circles. If each side of the square has length 2a, what is the length of P Q?

...............................................................................................................
... ..... .... ....
... .. .... ..
.....
....... ...... .....
...
•P
.... ..
.. . .. .
....... ....... ....... ..
........ ... ... .... .......
. .. ..
... .......... ............. ................. ........... .....
... ............................. ..... ............................. ...
... .... .. . .... ..
....... .... ......
....... .... ......... ....
....
. .... ..
• .... .
..
.
Q ........ ....... ..
......
... .... ... .... ..
..
.
.... ........... ........... ............ ........... ....
............. .................................... .......................

p
3
12) What is p/q if

p = (1)(2)(4) + (2)(4)(8) + (3)(6)(12) + · · · + (2014)(4028)(8056)


q = (1)(3)(9) + (2)(6)(18) + (3)(9)(27) + · · · + (2014)(6042)(18126)?
The University of the West Indies, Mona Campus
Presents

The 2014 Jamaican Mathematical Olympiad


First Round
Solutions for Grades 9, 10, and 11

1. Let S be the number of sweeties Ava had in her bag. Beth received 12 S = S2 sweeties from Ava.
  
Celine received 13 S2 = S6 sweeties from Beth. Davia received 14 S6 = 24 S
sweeties from Celine.
S
So, 24 = 3. Multiplying both sides by 24 gives S = 72. Ava had 72 sweeties in her bag.

2. The fractions given in the problem are respectively equivalent to

7 6 5 4 3
, , , , and .
8 7 6 5 4

To compare 7/8 and 6/7, we may cross multiply and note that 49 > 48. Dividing both sides by 56
gives 49/56 > 48/56. Simplifying both sides gives 7/8 > 6/7. One similarly proves that 6/7 > 5/6,
5/6 > 4/5 and 4/5 > 3/4. The fraction with the greatest value is 7/8.

3. We have 12mn = (4 · 3)mn = 4mn 3mn = 22 )mn 3mn = 22mn 3mn = (2m )2n (3n )m = P 2n Qm

4. Let A be the area of a square. One may verify that S1 = 12 A, S2 = 12 A, and S4 = 12 A. (Lines
have been added to the first, second, and fourth squares below to suggest how to show this.) So,
...................................................................................
....

...................... ... •
......................................................................................................................................
........................................................................................................... ... •
...........................................................................
.
.......... ................................................. •
..........................................................................................................
....
... .......................... ... ... ........................................................................ ... ... .......... .. ... ............................................................ ...
....................................... ... ... ................................................................... ... ... ............ .. ... ..............................................
...
... .. .
.................................................................... .. ... ............................................................................ ... ... . .................... ....
. ... ..............................................
...
...
.. . . . . . . . . . . . . . .
... ............................................................................................ .... ... . . .. .
... ......................................................... ... ... .................. .... ... ............................................................ ...
.. ............................................................................. ...
.................................................................................................................................................................
..
...
.................................... S
.......................................2 ......
... ... .................
......................
.. ... .................................
............................................... ..
• .................................................................................... • • .. ............................................. ...
.. • • ...
.. ................................... ...
.. • • ...
.. ......................................................... • ...
..
.... .......................................................................................... .... ....
...................................... ... .. ................................... .... .. ............................................
S ........................
. . . . . . . . . .
.....
.
.
............................. ..... ............................................. ....
... ............................................1
... .......................................................
. . . ... ...
...
...
...
...................
. . . . . . . .
........................
. . ....
... ...
S.... . .
...
.
..............................
. . . . . . .............
...
... ... S
................................ ...
.
4
................................ ...
. . . . . . . . . . . . .
.
.
... .............................
................................ .. ... ...................... ... .. ...................................3 . . .
. ... .. ............................................. ..
... .................. . . ... ... ............... ... ... .......................................... ... ... ................................ ..
.. ................ . .. ............ . ............................................................ . .... ................................................
.
.......................................................................
• ...........................................................................
• •
............................................................................. .......................................................................................................

S1 = S2 = S4 . In the third square, however, the total area below the diagonal, shaded and
unshaded, is 12 A. The shaded part is only half of this area. So, S3 = 14 A. Then S3 < S1 = S2 = S4 .

5. The grocer’s display has one can in the top row, 3 cans in the row below it, 5 in the next one
down, and so on. The grocer has 6 rows in his display because 1 + 3 + 5 + 7 + 9 + 11 = 36.

6. Let ab be a two-digit number where a and b are odd and a 6= b. There are 5 possibilities for a
(1, 3, 5, 7, or 9). For each one, there are 4 possibilities for b (it can be any odd digit except a).
There are 5 × 4 = 20 such numbers in all.
7. The centres of the three circles lie on the same line segment, and this may be extended to be a
diameter for the largest circle. Clearly, the diameter of the outer circle is 10 and its radius is 5.
......................
..........................................
.............................................
.....................................................................
.. ............................................................................................................
.. . . . . . . . . . . . . . . . . . . . . . . . . ..
...................................................................................................................................
............. ........................ .............
........ ..........................................
. .......... ................... ...........
.....
...........
...... 3 3 ..
2 2 ....
.....................................................................................................
..
.... • .
..
.......
• .
..
..... ....... ......
......... . . ............................ . . ... .............
.............................................
.......... .
.................. ......................................
.........................................................................................................................................
.........................................................
...........................................................................
................................................
.......................................................
.............................................

The area of the outer circle is π(5)2 = 25π. The areas of the inner circles are π(3)2 = 9π and
π(2)2 = 4π, respectively. Their combined area is 9π + 4π = 13π. The area of the shaded region is
the area of the outer circle minus the combined area of the inner circles. This is 25π − 13π = 12π.

8. When the 27 dice are glued together, 8 of them will be in a corner of the cube. Each one will
have three faces exposed. The smallest numbers that could be showing on them are 1, 2, and 3.
.................................................
....... .......................................................................... ....
....................................
........................................................................... .................................................. ...
........................................................................................................ ........................ ....
... .................................. ... ...................................
... .................................. ... ...................................
. ................ . .... .........
......................................................................................................................................... .......................
........................................... ............................................................. ................. ...
.................................. ..................................................................... ...
. . .. .. . . .. . ...
................................................................................................................... ...................... .......
. . .. .. .
..... .............................................
. .. . .. . ... .......................
.. ................................ ... ............
...................................................................................

This gives a minimum sum of 6 for each corner die. There will also be 12 dice on the edges of the
cube that lie between corner pieces. (These are the shaded dice in the figure above.) Each one has
two sides exposed. The smallest numbers that could be showing on them are 1 and 2. This gives
a minimum sum of 3 for each die on an edge. There will also be six dice, one on each face of the
cube, which have one side exposed. The smallest possible number on the exposed side is 1. The
smallest possible sum of the numbers on the surface of the 3 × 3 × 3 cube is 8(6) + 12(3) + 6 = 90.

9. Let x be the age of the injured player, and let a1 , a2 , a3 , . . ., a10 be the ages of the other ten
Strikers. Since the average age of the whole team is 22,

a1 + a2 + a3 + · · · + a10 + x
= 22.
11
Multiplying both sides by 11 gives

a1 + a2 + a3 + · · · + a10 + x = 242. (1)

After the injured player left, the average age of the 10 remaining players was 21. So,
a1 + a2 + a3 + · · · + a10
= 21.
10
Multiplying both sides by 10 gives

a1 + a2 + a3 + · · · + a10 = 210. (2)

Subtracting Equation (2) from (1) gives x = 32. The injured player was 32 years old.
10. Let x be the number of $10 coins and y the number of $20 coins in the box. Then x = 3y.
After removing 8 coins of each type from the box, we would have x − 8 = 5(y − 8) = 5y − 40.
Adding 8 throughout gives x = 5y − 32. Since x = x we have 3y = 5y − 32. It follows that 2y = 32
and so y = 16. Also, x = 3y = 3(16) = 48. There are 48 $10 coins and 16 $20 coins in the box.
Their value, in dollars, is 48(10) + 16(20) = 480 + 320 = 800.

11. Let ABCD be the vertices of the square, and let O be the centre of a third circle as shown
below. Since the points O and Q are equally distant from the side AD, OQ is parallel to AD.

A ........................................................................................................................... B
........ ... ...
....
......
...
.... O .......................................... P ...
.
. ..... • . .• . ...
...
..
..... .....
.. ..
.....
......... ... .. .... ... ..
... ........ .... ......... ................. ..... ..
... ............................... ....... ............................... .....
... ...... .. .......... ..... ..... ..
...... ... ..... ... .. .....
. .......... ...... ....
.... ..
Q ........ • .
.. ..
........ ...
....
.......
.. ........ ...
. ..
. .... .
. ..... ....
..
. . . .
..............................................................................................
D C

Similarly, OP is parallel to AB. Then 4P OQ has a right angle at O. It is clear that 2a, the
length of a side of the square, is equal to twice the diameter of one of the circles. It follows that
each circle has diameter a and radius a/2. Then OP = a/2 + a/2 = a and OQ = a/2 √ + a/2 = a.
2 2 2 2 2 2
By the Pythagorean theorem, (P Q) = (P O) + (OQ) = a + a = 2a . Thus P Q = 2 a.

12. First,

p = (1)(2)(4) + (2 · 1)(2 · 2)(2 · 4) + (3 · 1)(3 · 2)(3 · 4) + · · · + (2014 · 1)(2014 · 2)(2014 · 4)


= (1)(2)(4) + 23 (1)(2)(4) + 33 (1)(2)(4) + · · · + 20143 (1)(2)(4) = (13 + 23 + · · · + 20143 )(8)

In the same way, q = (13 + 23 + 33 + · · · + 20143 )(27). Then

p (13 + 23 + 33 + · · · + 20143 )(8) 8


= 3 3 3 3
= .
q (1 + 2 + 3 + · · · + 2014 )(27) 27
p
3
p
3
Thus p/q = 8/27 = 2/3.
THE UNIVERSITY OF THE WEST INDIES, MONA
Presents
The 2013 Jamaican Mathematical Olympiad

Test for Grades 9, 10, and 11

NAME:

GRADE:

SCHOOL:

PRINCIPAL:

YEAR OF BIRTH:

STUDENT PHONE:

CONTACT TEACHER:

CONTACT PHONE:

EXAMINATION QUESTIONS

1) Consider the following five expressions:


2 − (−4); (−2) × (−3); 2 − 8; 0 − (−6); (−12) ÷ (−2)
How many of them are not equal to 6?
(a) 0 (b) 1 (c) 2 (d) 4 (e) 5

2) What is the value of (−1)1 + (−1)2 + (−1)3 + · · · + (−1)2012 ?


(a) −2012 (b) −1 (c) 0 (d) 1 (e) 2012

3) In the figure below, ABCDE is a pentagon with AB = AC = AD = AE. If 6 EAB =


90◦ , 6 EAD = 2x◦ , 6 BAC = 2x◦ , and 6 ADC = 5x◦ , what is the value of x?

2x .
E ...................................................................................................A
... .. .
........ .... ....
... ........
........ . .
...
...
.................
..... .
.. .. .. .
... .............
... ....
2x◦
..... ◦ ..
D 5x ..... ... ....
.....
..... ... ...
..... ..... ..
..... ..
. ...
..... . ..
..... .... ...
..... .. .
............
.............. ....
.......
C B

(a) 15 (b) 12 (c) 14 (d) 10 (e) 20


4) Western High School has 1,000 students and 570 of them are girls. One fourth of the
students at Western High ride the bus to school, and exactly 313 of the boys do not
ride the bus. How many girls ride the bus to school?
(a) 7 (b) 63 (c) 153 (d) 180 (e) 133

5) Two numbers P and Q are positive real numbers such that P is 40% larger than Q.
What is the ratio P : Q?
(a) 40 : 1 (b) 5 : 7 (c) 7 : 5 (d) 5 : 2 (e) 5 : 3

6) In the figure below, ADCB is a semicircle with centre S and diameter AB. The angle
CAB is 20◦ . DS ⊥ AC, and E is the point where AC and DB intersect. What is the
measure of 6 AED?
D .............
............................
........
....... ........ ......
...... ..... ........... .....
.......
. . .
. .
. .....
. .
.....
. . .
...
...
.......
....... .. E
...
..
.................
C
. ... ..
.................
..
. ..
... ............ ........ . ...
...
.
..
... ....... ...... ...
... ...... ................ ......
.. ...... ....
.
..
.. .................. ...
... .. ...... ..
.............
.............................................................................................................
• .
A B
S

(a) 55◦ (b) 60◦ (c) 50◦ (d) 30◦ (e) 40◦

7) If 3k = 930 , what is the value of k?


(a) 15 (b) 30 (c) 40 (d) 60 (e) 90

8) For how many prime numbers p is it true that p4 + 1 is also prime?


(a) None (b) 1 (c) 2 (d) 3 (e) Infinitely many

9) In the figure below, ABCD is a rectangle and E, F , and G are points lying on its sides
such that DG is half of DC, CF is one-third of CB, and BE is one-fourth of BA.
What is the ratio of the area of the quadrilateral EF GA to the area of ABCD?
E
A .......................................................................................................... B
... ..... ...
...
... ...... ...
... ....
... .....
... .... ... ..
... ..
..... .... ...
... ..... ......
.... ...
...
...
.....
..... ................... F
... .... .......
..... .....
... .... .........
..............................................................................................
D C
G

1 7 2 3 3
(a) (b) (c) (d) (e)
2 12 3 4 5

10) If x2 yz 3 = 73 and xy 2 = 79 , What is xyz equal to?


(a) 74 (b) 76 (c) 78 (d) 79 (e) 710
11) At a book shop, each of the paperbacks cost $500 and each of the hard-covered books
cost $700. Jordan spent exactly $10,100 on books. What is the smallest number of
books he could have bought?
(a) 19 (b) 18 (c) 17 (d) 16 (e) 15

12) We say that a point (x, y) is a grid point to mean that at least one of its coordinates is
an integer. How many grid point lie on the circle x2 + y 2 = 50?
(a) 16 (b) 30 (c) 48 (d) 60 (e) 100

13) How many four-digit numbers are divisible by 3, have the digit 2 in the thousands place,
and the digit 8 in the ones place?
(a) 34 (b) 30 (c) 19 (d) 20 (e) 33

14) In the figure below, ABCD is a rectangle with AB = 2x and AD = x. The diagonals
AC and BD intersect at E. The point F on CD has the property that the area of
triangle BF C is half the area of the pentagon BAEDC. What is the length of F C?

2x
A .............................................................................................................................
... ......... .....
B
........ ............. ...
.. ........ ............ ....
... ........ ...................
. ...
..
... ....... E
....... ............. ..........
..... .. ...
............ .... ...
x ..
...
. . . ... ..... ...
. .
...........
... ...
.. ....... .. ..
.. .........
. . ...
... ..... .. . .
.. ....... ...
.. ...
...... .. ..
. .......... .
. ..
... .............. .....
. ..... ...
................................................................................................................................
D C
F

x 5x 3x 3x
(a) (b) x (c) (d) (e)
2 4 2 4

15) Jacob had a pitcher of juice and poured 2/3 of it into a jar. From the jar, he drank 5/8
of the juice, poured 1/8 into a glass for later, and poured the rest of it back into the
pitcher. What fraction of the original juice was in the pitcher then?
1 1 2 5 1
(a) (b) (c) (d) (e)
3 2 9 4 4

16) In the figure below, ABCD is a rectangle with AB = 4 and BC = 3. The point E on
AC has the property that EB is perpendicular to AC. What is the length of EB?

A .........................................................................
.. ........ ...
B
... ....... .... ..
...... ... ...
..
... ...... ..... ...
...... . ...
..
... ...... ...... ...
.......... ...
.. ..... .
.. ......
... E ...... .....
.. ...... ..
......................................................................
D C

(a) 2.4 (b) 2.2 (c) 2.3 (d) 2.1 (e) 2.5
17) Mark has 10 cards. Each card is labeled with one of the following numbers: 3, 8, 13,
18, 23, 28, 33, 48, 53, 68. Each card has a different number. What is the minimum
number of cards Mark has to choose so that the value of the cards totals exactly 100?
(a) 2 (b) 3 (c) 4 (d) 5 (e) Impossible

18) How many triples (a, b, c) of positive integers satisfy the simultaneous equations
(
ab + bc = 44
ac + bc = 23

(a) 0 (b) 1 (c) 2 (d) 3 (e) 4

19) In the figure below, six circles which are tangent to their neighbors are inscribed in a
parallelogram. If each circle has radius 3 cm, what is the area, in square centimetres,
of the parallelogram?
...................................................................................................................
... ... .... ... .... ... ....
... ... ..... ...... ....
.....
.... ..
......
..
...... ......
.... .. .
. ... ...
..............................................................................................................
... .. ...... .
. ...... .
. ...
..... ...
...... . . ...
... . ..... . ..... ......
....... .... .
. ...... .... .
. ...... .
..
.... .....
.................................................................................................

√ √ √
(a) 108 (b) 8(4 + 3 3) (c) 15(2 + 3) (d) 12(9 + 5 3) (e)216

20) If n is a positive integer we define n! = 1 × 2 × 3 × · · · × (n − 1) × n. (The symbol n! is


called n factorial.) Suppose n is a positive integer and n! has the prime factorization
215 × 36 × 53 × 72 × 11 × 13. What is n?
(a) 15 (b) 14 (c) 13 (d) 17 (e) 16

21) In the figure below, ABCD is a square, E and F are the midpoints of AB and BC,
respectively, and G is the point where AF and EC intersect. What is the ratio of the
area of the quadrilateral AGCD to the area of ABCD?

E
A ...........................................................................
... .......... ... .. B
....... ...
...
... ........ ..... ..
........ ... ...
.... ........ .
.........
..
... ... ........ ....
.
G ... .........
.. ... .
...
..
... ....
... ..
F
... ... ...
.. ... ..
... ... ..
.. ....
...................................................................
D C

1 5 2 3 5
(a) (b) (c) (d) (e)
2 8 3 4 9
22) We say that a number is primo to mean any two consecutive digits in this number gives
another number which is divisible by 17 or 23. How many numbers with 2012 digits
are primo?
(a) 5 (b) 6 (c) 7 (d) 9 (e) More than 9

23) Suppose x, y, and z are real numbers with x 6= 0, y 6= 0, and z 6= 0. If x + y + z = 5,


1 1 1
xyz = 4, and + + = 2, what is x2 + y 2 + z 2 equal to?
x y z
25
(a) 9 (b) 4 (c) 25 (d) (e) 16
16

24) In the figure below four small, equal circles are circumscribed by a larger one. What
is the ratio of the sum of the areas of the four smaller circles to the area of the larger
one?
........................................
.............. .... .......
...... .... ... .....
........ .... .. .....
.... .... .. ....
. . ..
.
.
.
.
. .................... .
..................... .....
.......... ......
. ........ ........
......
. .......
........ ........................ .....
. .. .. ...
.... .... .... ..
.....
....... .. . .
....................... . ......
. ..... ....
............ ............. ....... .
..
.
... .............. .................... ....
... .. .. .
.... . ...
..... ..... .. ....
...... .. .
... ..........
....... ...... . .
................. .............................
.........

√ √ √ 1 √ √
(a) 3 − 2 2 (b) 2 − 2 (c) 4(3 − 2 2) (d) (3 − 2) (e) 2 2 − 2
2

25) How many positive integers have the property that each of their digits is either 1 or 3
and the sum of their digits is 10?
(a) 34 (b) 28 (c) 35 (d) 56 (e) 55

END OF QUESTIONS

You may mail your completed question paper to:

Mathematical Olympiad
P.O. Box 94
Mona Post Office
Kingston 7

You may also deliver your entry by hand or by courier directly to the Department of Math-
ematics at the UWI, Mona Campus. In all cases, an entry must be received by December
10, 2012 in order to be considered.

For more information, a copy of this question paper, or the latest updates, please visit
http://myspot.mona.uwi.edu/mathematics/ (see the link to the Olympiad Resource Centre).
THE UNIVERSITY OF THE WEST INDIES, MONA
Presents

The 2013 Jamaican Mathematical Olympiad


Qualifying Round
Solutions for Grades 9, 10, and 11

1. Using ordinary laws of algebra, 2 − (−4) = 2 + 4 = 6; (−2) × (−3) = −(−6) = 6; 2 − 8 = −6;


0 − (−6) = 0 + 6 = 6; (−12) ÷ (−2) = −(−6) = 6. Then exactly one of these expressions, the third one, is
not equal to 6.

2. Observe that (−1)1 = −1, (−1)2 = 1, (−1)3 = −1, (−1)4 = 1, and so on. In general, −1 raised to any
odd power is −1 and −1 raised to any even power is 1. Then the given sum is

(−1) + 1 + (−1) + 1 + (−1) + 1 + · · · + (−1) + 1.

This is a sum of 2012 terms, and 1006 of them are −1 while the other 1006 are 1. Then the total sum is
−1006 + 1006 = 0.

3. From the information given, AD = AC. Thus the triangle ADC is isosceles and 6 ADC = 6 ACD = 5x◦ .
Since the sum of the angles in 4ADC is 180◦ , 6 DAC = 180◦ − 10x◦ . Since 6 EAB is a right angle,
2x + (180 − 10x) + 2x = 90. Then 180 − 6x = 90 and it follows that x = 15.

2x◦
...
. A
E ................................................................................................................
... .. ... ..
........ ........
... ........
...
... ....
........
. ........ ..............................
.. ...
(180◦ − 10x◦ )
... ..... .
........ ... ...
... ........ ... ...............
... .......
... ............... ..
..........
.... ◦ ..
.
...
... ....
... 2x◦
D 5x
.... .. ...
... .. ...
.... ... ...
.... ..
. ...
... ..
.... ... ...
... ◦ .. ...
....
.... 5x ..... ...
...
.... .... . ...
...... ...
............. ...
...............
...................
C B

4. From the information given, Western High School has 430 boys and 570 girls. Since 313 boys do not ride
the bus, 117 of them do. Since 250 students ride the bus to school, this means that 133 of them are girls.

5. Since P is equal to Q plus 40% of Q, P = Q + 0.40Q = (1 + 0.4)Q = 1.4Q. Then

P 1.4Q 1.4 14 7
= = = =
Q Q 1 10 5

That is, P : Q = 7 : 5.

6. Let F be the point where AC and DS intersect. From the information given, 6 F AS = 20◦ and
6 AF S = 90◦ . Since the sum of the angles in 4AF S is 180◦ , 6 ASF = 70◦ . Since 6 DSB is the supplement
..........
D .................... ..............................
............ ........
........ ............... .......
..
.......... .
. ...... ......
.
...... ... ....... ......
.....
..... ... ...... .....
... . .
. .
.......
.....
. ...
... ◦ ........ ..
.
....
..
.....
. .... .
...... 35 ..
......
......... ......
C
.
...
. ...
.
..
......
.. .
. ....
........ .. ....
... E
. ... ◦ ........................
. . ...
...
...
...
.......... ..................
. ......
......
x ...
...
.. ... ............. ...... ...
.
.... . . . .... .
........ .....
.
.. .. F ......
.......
...
...
.
. .. .. ... . .. ..
.... ...
............ .... ...
.
......
.
..
...... ...
.... .......
....... ... .
.. ..
...
...... ◦ ◦ ...
◦ ◦ ...... .
... .
....
.. . ....... ...
.
.. ................. 20 ... 70
•110 35
.
. . .
...................................................................................................................................................................................................
A B
S
of 6 DSA = 6 F SA, we have 6 DSB = 110◦ . The triangle DSB is isosceles with SB = SD. Since the sum
of the angles in 4DSB is 180◦, it follows that 6 SDB = 6 SBD = 35◦ . Since 6 DF E = 90◦ and the sum of
the angles in 4DF E is 180◦ , we have x = 55.

7. Since 9 = 32 , we have 930 = (32 )30 = 360 . If 3k = 930 then 3k = 360 . Thus k = 60.

8. If p = 2 then p4 + 1 = 24 + 1 = 17. This is prime. Suppose now that p is a prime number greater than 2.
Then p must be odd. (If p were even it would have 1, 2, and p as factors and would not be prime.) Then p4
is odd and hence p4 + 1 is even. Since p4 + 1 is greater than 2, it is cannot be prime. Then p = 2 is the only
prime number such that p4 + 1 is also prime.

9. Suppose AB = BC = x and AD = BC = y. Then the area of the rectangle ABCD  is xy, and
DG = GC = x/2, F C = y/3, F B = 2y/3, and EB = x/4. The area of 4ADG is 12 (y) x2 = xy
4 . The area
of 4GCF is
x
E 4
A ..................................................................................................................................................
........... ...................
B
. .. ...................
................. .................
......................... .................
...............
................... .............. 2y
............................ .............
............
.............................. .......... 3
y ........................
.....................................
.........
........
.......................................... ......
.............................. ..........
......................
. . F
.............................................. . .. .. .. .
.....................
.
................................................... .. . .. .. .
...................................... ..............................
..................................
.............................................................................................................................................................................................................................
D C
x G x
2 2
1 x
 y
 xy 1 2y
 x
 xy
2 2 3 = 12 . The area of 4F BE is 2 3 4 = 12 . Then the total shaded area is

xy xy xy 3xy xy xy 5xy
+ + = + + =
4 12 12 12 12 12 12
5xy 7xy
Since the area of the rectangle is xy, the area of the quadrilateral EF GA is xy − 12 = 12 . Then the ratio
of area of EF GA to that of ABCD is
7xy/12 7
=
xy 12

10. Multiply both equations together to obtain (x2 yz 3 )(xy 2 ) = (73 )(79 ) and hence x3 y 3 z 3 = 712 . Taking
the cube root of both sides (or raising both sides to the 1/3 power), xyz = 74 .

11. Suppose Jordan bought p paperbacks and h hardbound books. Then 500p + 700h = 10,100. Dividing
throughout by 100, 5p + 7h = 101. Thus 5p = 101 − 7h. Since 5p is obviously a positive multiple of 5, so is
101 − 7h. This happens when h = 3, 8, and 13. In the first case h = 3 and p = 16, and Jordan would have
bought 19 books in all. In the second case h = 8 and p = 9, and he would have bought 17 books in all. In
the last case h = 13 and p = 2, and he would have bought 15 books in all. Then the smallest number of
books Jordan could have bought is 15.

12. If x2 + y 2 = 50 then −8 < x < 8 and −8 < y < 8. We list all possible grid points below. The first
column has all points where x is an integer and the second one has all points where y is an integer,
√ √
(0, ± 50) (± 50, 0)
(±1, ±7) (±7, ±1)
√ √
(±2, ± 46) (± 46, ±2)
√ √
(±3, ± 41) (± 41, ±3)
√ √
(±4, ± 34) (± 34, ±4)
(±5, ±5) (±5, ±5)
√ √
(±6, ± 14) (± 14, ±6)
(±7, ±1) (±1, ±7)

The first column contains 30 points and the second one contains 30 points. However, the pairs (±1, ±7),
(±5, ±5), and (±7, ±1) occur in each column. This gives 12 duplicate points in all. Then there are 60 − 12 =
48 grid points in all.

13. Any number n meeting the stated criteria has the form 2ab8, where a and b are digits. Furthermore,
the fact that n is a multiple of 3 means that the sum of its digits, 2 + a + b + 8 = a + b + 10, is a multiple
of 3. We list the possible choices for a and b below.

a = 0; b = 2, 5, 8; a = 5; b = 0, 3, 6, 9;
a = 1; b = 1, 4, 7; a = 6; b = 2, 5, 8;
a = 2; b = 0, 3, 6, 9; a = 7; b = 1, 4, 7;
a = 3; b = 2, 5, 8; a = 8; b = 0, 3, 6, 9;
a = 4; b = 1, 4, 7; a = 9; b = 2, 5, 8;

There are 33 possible numbers in all.

14. The area of the pentagon BAEDC is the area of the rectangle ABCD minus the area of the triangle
ADE. The area of ABCD is 2x2 . Triangle ADE has base AD and its height is the perpendicular distance

2x
A ........................................................................................................................................................ B
.......................... ........ .
.................................. ............ ....
....... .....
................................. ....... ..... ...
................................................. E .. .
......... ..........
. ....
... ..
........................................................... .............. .......... ...
...
x ..................................................... ........ ...
....................................................... ...........
............
.............................................. .... .....
............................. .......
. .......
.......... ...
............................... .
....... ........... ...
....
..................... .
..... ........ ...
.................................................................................................................................................
D C
F

from E to AD. Since its base and height are each x, its area is 12 x2 . Then the area of the pentagon BAEDC
is 2x2 − 12 x2 = 32 x2 . Given that the triangle F CB has half this area, its area is 34 x2 . But its area is also
1 1 1 1 3 2 3
2 (F C)(BC) = 2 (F C)x = 2 x(F C). Then 2 x(F C) = 4 x and it follows that F C = 2 x.

15. When Jacob poured 2/3 of the juice from a pitcher into a jar, 1/3 of it remained in the pitcher. Of the
juice in the jar, he drank 5/8 of it and poured 1/8 of it into a glass for later. This accounts for 58 + 18 = 68 = 34
of it. Thus he poured 14 of it back into the pitcher. At the end, the amount of juice in the pitcher, as a
fraction of the original, was  
1 1 2 1 1 2 1 3 1
+ = + = + = = .
3 4 3 3 6 6 6 6 2

16. The triangles ABC, AEB, and BEC are all right triangles with right angles at B, E, and E, respectively.
By the Pythagorean theorem, AC = 5. Since ABC and AEB share an angle at A, their third angles are

4
A ...............................................................................
..............
B
.....
... ......... ... ..
... ...... ... ...
... ...... ..... ....
...... ... ....
... ......
...... ... ...
... ...... ..... 3
... ....... ....
... ......
...... ..
... E ...... ....
... ...... ...
....................................................................................
D C

also equal. (The sum of the angles in any triangle is 180◦ ). Thus ABC and AEB are similar. Similarly, since
ABC and BCE share an angle at C, their third angles are also equal. Thus ABC and BCE are similar.
Then the triangles ABC, AEB, and BEC are all similar to each other. Since corresponding sides of similar
triangles are proportional,
EC BC EC 3 9
= ; = ; EC = .
BC AC 3 5 5
In the same way,
AE AB AE 4 16
= ; = ; AE = .
AB AC 4 5 5
Finally,
EC EB 9/5 EB 144 12
= ; = ; (EB)2 = ; EB = = 2.4
EB EA EB 16/5 25 5

17. On five of Mark’s cards the one’s digit of the number shown is 3, and on the other five it is 8. In order
to have numbers on the cards adding up to 100, their one’s digits must add up to a number ending in 0. If
the last digits are 3, 3, 3, 3, and 8 their sum is 20. In this case five cards would be needed. If the last digits
are 3, 3, 8, 8, and 8 their sum is 30. In this case five cards would also be needed. It is not possible to use
four or fewer cards and have the sum of the one’s digits end in 0 (try it). So if it is possible at all to have
the numbers on the cards sum to 100, at least five of them are needed.
In fact, it is possible to use exactly five cards. Note that any of these combinations sum to 100:

3, 13, 23, 33, 28; 3, 13, 23, 53, 8; 13, 33, 8, 18, 28; 3, 23, 8, 18, 48; 3, 13, 8, 28, 48.

So the minimum number of cards Mark has to choose is five.

18. Subtracting the second equation from the first one gives ab − ac = 44 − 23, or a(b − c) = 21. Since a
and b − c are integers and a is positive, there are four possibilities to consider: a = 1, b − c = 21; or a = 3,
b − c = 7; or a = 7, b − c = 3; or a = 21, b − c = 1. We consider each one in turn.
Case 1. If a = 1 then the second of the original equations reduces to c(1 + b) = 23. The only possibilities
are c = 23, 1 + b = 1, or c = 1, 1 + b = 23. In the first instance we would have b = 0, which is impossible.
The second one gives b = 22, c = 1 (and a = 1). This is a valid solution, as one may verify.
Case 2. If a = 3 then the second of the original equations reduces to c(3 + b) = 23. The only possibilities
are c = 23, 3 + b = 1, or c = 1, 3 + b = 23. In the first instance we would have b = −2, which is impossible.
The second one gives b = 20, c = 1 (and a = 3). However, this is not a valid solution.
Case 3. If a = 7 then the second of the original equations reduces to c(7 + b) = 23. The only possibilities
are c = 23, 7 + b = 1, or c = 1, 7 + b = 23. In the first instance we would have b = −6, which is impossible.
The second one gives b = 16, c = 1 (and a = 7). However, this is not a valid solution.
Case 4. If a = 21 then the second of the original equations reduces to c(21 + b) = 23. The only possibilities
are c = 23, 21+b = 1, or c = 1, 21+b = 23. In the first instance we would have b = −20, which is impossible.
The second one gives b = 2, c = 1 (and a = 21). This is a valid solution.
In summary, there are exactly two triples (a, b, c) of positive integers solving the original system. These are
(1, 22, 1) and (21, 2, 1).

19. Mark the centres of the six circles and let H, G, and L be three of them, as shown in the figure below.
Let EF be perpendicular to DC and pass through G. Then EF also passes through I, the point of tangency
between the two circles as shown. The area of the parallelogram is (DC)(EF ). To find DC and EF , note

F B
A .......................................................................................................................................................................................................................................
...... ........
... .... ......
.... .... .........
.
......
.
... ..... .....
... ......
....
... ... ... .. ... .....
... ... ... ... ... ... ... ......
... .... .......
. . .
... ..
. .
....
....
... ... ..... ....
... .. . . ......
......
..... H •
..................................
...
...
•I
....
.........
• .
.
..
.
.
.....
• . ..
.. ..
.. .....
.... . .. .. .
.
... ... . .....
. . ...
. .. ...
... ... .. .. ... .. . . .. .
.... . . ...
...... ... ............................. .... .... ..
......... . .. ........ .................................... .....................................
... ........................................ .
. ................................... ..................................... ......
... . .
... .. . .... ... . .... ... . ....
... . ... .
... .. . .... .. . .... .. . ....
... .. ...
... ....
... ..
... ..
.
.... .
.
.....
.
...
.....
.
..
. L .....
......
.......................................................................................................................................
... ..
.....
.....
G . •
.....
. ..
...
.
...
.
•.
... .
..
....
.
• . .......
... ....... ... ...
.. ...
... ..
. ... .
. .... .
.
.
. .... .
.
........ .. ....
. .
.. .. . .. . .. . .
...
..... .... ....
.
.. ....
. ..
. .. ....
. ..
. ..
. .
......... ....
. .... . .... . ....... ...
........... . .... ... .... ... .... ..........
.............. ... ............. ...... . ...... ...... . ......
......... ....................................................................................................................................................................................................................
D C
E J K

that whenever three circles are mutually tangent their centres form an equilateral triangle. It follows that
the angles in the parallelogram ABCD are 60◦ (at A and C) and 120◦ (at B and D). Also, GHI, GDE,
and CLK are√all 30-60-90 triangles.
√ Since each circle has radius 3, GE = LK = 3 and GL = EK = 12.
Also, DE = 3 and √ KC = 3 3. (In any 30-60-90 triangle we have, in ratio, shortest side : middle side :
longest side = 1 : 3 : 2.) Then
√ √ √ √
DC =3 + 12 + 3 3 = 12 + 4 3 = 4(3 + 3)
√ √ √ √
Similarly, EG = 3, GI = 3 3, and IF = 3. Thus EF = 3 + 3 3 + 3 = 6 + 3 3 = 3(2 + 3). Then
√ √
(DC)(EF ) = 4(3 + 3)3(2 + 3)
√ √ √
= 12(6 + 3 3 + 2 3 + 3) = 12(9 + 5 3)

20. Since n! contains 13 as a factor, n ≥ 13. Note that

13! = 1 × 2 × 3 × 4 × 5 × 6 × 7 × 8 × 9 × 10 × 11 × 12 × 13
= 2 × 3 × 22 × 5 × (2 × 3) × 7 × 23 × 32 × (2 × 5) × 11 × (22 × 3) × 13
= 210 × 35 × 52 × 7 × 11 × 13

Since 14 = 2 × 7, 15 = 3 × 5, and 16 = 24 ,

16! = 13! × 14 × 15 × 16 = 210 × 35 × 52 × 7 × 11 × 13 × (2 × 7) × (3 × 5) × (24 )
= 215 × 36 × 53 × 72 × 11 × 13

This is exactly the same as the n! in the statement of the problem. Thus n = 16.
21. Let the square ABCD have side length s and area s2 . Then EBC is a right
 E
A B
triangle with EB = 12 s and BC = s. Its area is 12 12 s (s) = 14 s2 . Join B and G with
........................................................................................
... .......... ... .
.... ..
.......
... ........ ... .... ..
... ....... .... .... ....
... ....... ... ...... ...
.............
a line segment as shown. We prove that the area of 4GBC is two-thirds the area ...
...
.........
...........
... ......... ...
...
.
... ... .........
of 4EBC. It is shown below that 6 GBE = 6 GBF = 45◦ . Assuming so for now, G F ...
...
...
...
...
..
...
... .
... ....
the triangles GBE and GBF are congruent by the side-angle-side theorem. In- ...
...
... ..
... ..
... ... ...
deed, EB = F B = 12 s, 6 EBG = 6 F BG, and the side BG is common to both tri- ... .....
................................................................................
D C
angles. Then 4GBE and 4GBF have the same area. Furthermore, 4GBF
and 4GF C have the same area. This is because they have equal bases (BF and F C,
respectively) and equal heights (the perpendicular distance from G to BC). Thus the triangle EBF is
divided into three smaller triangles, GEB, GBF , and GF C, with equal area. It follows that the area of
4GBC is two-thirds that of 4EBC.
Since the area of 4EBC is 14 s2 , the area of 4GBC is therefore 23 14 s2 ) = 16 s2 . Reasoning in the same
way, the area of 4GBA is also 16 s2 . Then the area of the quadrilateral ABCG is

1 2 1 2 1
s + s = s2 .
6 6 3

Since the area of the square ABCD is s2 , The area of the quadrilateral AGCD is s2 − 13 s2 = 23 s2 . Finally,
the ratio of the area of AGCD to the area of ABCD is

2s2 /3 2
=
s2 3

To show that 6 GBE = 6 GBF = 45◦ , join E to F with a line segment. Then
E
4EBF is isosceles with EB = F B. Since 6 EBF = 90◦ and the sum of the A .............................................................................................
... .......... ...... .. .
B
....... ....... ... ..
... .... ....
angles in 4EBF is 180◦ , 6 EF B = 6 F EB = 45◦ . Also, 4ABF and 4CBE are ....
...
.......
....... .... .............
....... ... ......
......... ... ...
..
..
... .......... .... ....
congruent by the side-angle-side theorem. Indeed, EB = F B, 6 ABF = 6 CBE, ...
...
............ .... ..
... ................
... ...
... G ... ... F
and BC = BA. Thus 6 CEB = 6 AF B. Subtracting 45◦ from both sides, 6 CEF ...
...
...
... ...
... ...
..
... ... ..
= 6 AF E. This implies that the triangle GEF is isosceles with GE = GF . Then the ...
...
... ...
... ..
... .....
triangles BEG and BF G are congruent by the side-side-side theorem. Thus ..............................................................................
D C
6 GBE = 6 GBF , and each one is one-half of a 90◦ angle.

22. The multiples of 17 and 23 which are two-digit numbers are:

17 23
34 46
51 69
68 92
85

Suppose a number with 2012 digits is primo. If one of the digits is 8 then the only digits that can follow are
5, 1, and 7 (in that order). After that, there can be no more digits. This means that the digit 8 can occur
only as the 2009th, 2010th, 2011th, or 2012th digit.
Suppose now that n has 2012 digits and is primo. Its first digit cannot be 1. Otherwise, its second digit
is 7 and there is no possible third digit. Similarly, its first digit cannot be 5. Otherwise its first three digits
are 517 and there is no possible fourth digit. Similarly, its first digit cannot be 7 (no second digit is possible)
and its first digit cannot be 8 (no 8 can come before the 2009th digit). Then the first digit of n must be 2,
3, 4, 6, or 9.
Suppose the first digit of n is 2. Then its first four digits are 2346. Since the next digit cannot be 8,
it must be 9 and the one following must be 2. Thus the number has the form 234692346923469 · · ·2346xyz.
In other words, the digits repeat in cycles of five and 6 occurs as the 4th, 9th, 14th 19th, . . ., 2004th, and
2009th digits. After that, x could be 8 or 9. This gives the two possible numbers

234692346923469 · · ·2346851
234692346923469 · · ·2346923

Reasoning in the same way, if the first digit of n is 3 it has the form 346923469234692 · · ·346xyzw. The
digits occur in cycles of 5 again but this time 6 occurs as the 3rd, 8th, 13th 18th, . . ., 2003rd, and 2008th
digits. Once again, x could be 8 or 9. This gives two more possible numbers:

346923469234692 · · ·3468517
346923469234692 · · ·3469234

If the first digit of n is 4 it has the form 469234692346923 · · ·46xyzwu. In this case, x cannot be 8 because
it falls in the 2008th position. Thus x is 9 and the only possible number is

469234692346923 · · ·4692346.

If the first digit of n is 6 it has the form 692346923469234 · · ·692346x. In this case, x could be 8 or 9 again.
This gives two more possible numbers:

692346923469234 · · ·6923468
692346923469234 · · ·6923469

If the first digit of n is 9 it has the form 923469234692346 · · ·92346xy. In this case, x could be 8 or 9 again.
This gives two more possible numbers:

923469234692346 · · ·9234685
923469234692346 · · ·9234692

All possibilities have now been considered. There are nine different numbers which are primo and have 2012
digits.

1 1 1
23. It is given that + + = 2. Finding common denominators,
x y z

yz xz xy xy + xz + yz xy + xz + yz
+ + = 2; = 2; = 2; xy + xz + yz = 8.
xyz xyz xyz xyz 4

Also (x + y + z)2 = (5)2 = 25. But

(x + y + z)2 = x2 + xy + xz + xy + y 2 + yz + xz + yz + z 2
= x2 + y 2 + z 2 + 2(xy + xz + yz)
= x2 + y 2 + z 2 + 2(8)
= x2 + y 2 + z 2 + 16

Then x2 + y 2 + z 2 + 16 = 25 and so x2 + y 2 + z 2 = 9.

24. Let A and B be the endpoints of a diameter and P , Q, and R the centres of small circles as shown in
the figure below. Let r be the radius of a small circle. Then each small circle has area πr2 , and the sum of
....................................................
............... .... .......
....... .... ... ......
.
........ .... ... .....
...
....
.
....
.. Q ..
..
...
....
.
.
...
. ...
.

.
.. .......
. .... ..
... .. ................ .....
..
. ...
.
... ................................ ...... .. .......... . . .
. .
. ...... ...
........... .......
.. ............. . ..
.. ......
.... .
......
...... .
.. ... ....
. ... ............. .. .... .....
... .... . . .. .
...................................................................................................................................
A• ...
.....
• ..... ...
...
• •B
.
..
....
......
........
P ....
.
...................................
.....
R .
.....
.......
..
... ........ .......... ..... ..
..
.............................. ..
... ........... ... ... .
...
....
. .. ...
... .. ...
....
.... ... ... .....
...... .... ..
...... ... ... ........
........ .... .... ......
................... . ...........................
...............

their areas is 4πr2 . To find the area of the large, circumscribing circle we first determine AB. We know that
AP = r, P Q = 2r, QR = 2r, and and RB = r. Also, the triangle P QR is a right triangle with a right angle
at Q. By the Pythagorean theorem, (P Q)2 + (QR)2 = (P R)2 . Thus
√ √
(2r)2 + (2r)2 = (P R)2 ; 4r2 + 4r2 = (P R)2 ; 8r2 = (P R)2 ; P R = 8r2 = 2 2r.
√ √
Then AB = AP √ + P R + RB = r + 2 2r + r = 2(1 + 2)r. It follows that the radius of the circumscribing
circle is (1 + 2)r, and its area is
√ √ √
π(1 + 2)2 r2 = π(1 + 2 2 + 2)r2 = π(3 + 2 2)r2 .
Then the ratio of the sum of the areas of the four smaller circles to the area of the larger one is
√ √
4πr2 4 3−2 2 4(3 − 2 2) √
√ = √ √ = = 4(3 − 2 2)
π(3 + 2 2)r2 3+2 2 3−2 2 9−8

25. Suppose all the digits in a number are either 1 or 3 and they sum to 10. Then there can be ten 1s, or
seven 1s and one 3, or four 1s and two 3s, or one 1 and three 3s. We consider each possibility in turn.
Ten 1s. If a number has ten digits and they are all 1, it must be 1,111,111,111. There is only one possibility
in this case.
Seven 1s and One 3. Suppose a number has eight digits and seven of them are equal to 1 and the other
is 3. Then the number must be one of these:
31, 111, 111, 13, 111, 111, 11, 311, 111, ··· 11, 111, 131, 11, 111, 113
There are eight possibilities in this case. In fact, this is exactly “eight choose one”.
Four 1s and Two 3s. Suppose a number has six digits and four of them are equal to 1 and the others are 3.
Then the number must be one of these:
331, 111, 133, 111, 113, 311, 111, 331, 111, 133,
313, 111, 131, 311, 113, 131, 111, 313,
311, 311, 131, 131, 113, 113,
311, 131, 131, 113,
311, 113
There are 15 possibilities in all. In fact, this is exactly “six choose two”.
One 1 and Three 3s. Suppose a number has four digits and one of them is equal to 1 and the others are 3.
Then the number must be one of these:
1, 333, 3, 133, 3, 313, 3, 331.
There are four possibilities in all. In fact, this is exactly “four choose one”.
Putting all of the cases together, there are 1 + 8 + 15 + 4 = 28 numbers with the property that each of their
digits is 1 or 3 and the sum of the digits is 10.
THE UNIVERSITY OF THE WEST INDIES, MONA
Presents
The 2013 Jamaican Mathematical Olympiad

Test for Grades 7 and 8

NAME:

GRADE:

SCHOOL:

PRINCIPAL:

YEAR OF BIRTH:

STUDENT PHONE:

CONTACT TEACHER:

CONTACT PHONE:

EXAMINATION QUESTIONS

1) Daniel walked for 35 minutes after his dinner. He finished at 7:10 pm. At what time, pm,
did he start?
(a) 6:30 (b) 6:35 (c) 6:40 (d) 6:25 (e) 6:45

2) Which of the following quantities is largest?


1 1 1 1 1 1 1 1 1
(a) (b) + (c) × (d) − (e) ÷
4 4 4 4 4 4 4 4 4

3) Brenda bought some bananas at the supermarket. She gave half of them to her friend Ana,
gave 3 to Catalina, and had 4 left for herself. How many bananas did Brenda buy?
(a) 11 (b) 4 (c) 7 (d) 14 (e) 3

4) In the figure below, 6 RP Q = 20◦ and 6 RQU = 120◦ . What is the measure, in degrees,
of 6 SRT ?
S .......
.....
..... .....
........ T
..... ................
............
.
.. .
.... ..... ..
........ ....
......... R ....
........ .....
.........
.
.....
. ................................................................................ U
P
Q

(a) 60 (b) 140 (c) 100 (d) 80 (e) 120


5) The figure below shows the net of a cube. When it is folded into a cube, what letter will be
on the side opposite the one with the X?
...........................
... ...
... ..
...
...
A ...
..
.............................................................................
... .. ... ..
... ... ... ...
B ..
..
X
...
...
...
...
C ...
..
...............................................................................................
.... ...
...
...
.. ...
...
..
D ...
.. E ...
.
................................................

(a) A (b) B (c) C (d) D (e) E


.................. ................
... ... ..... ....
6) What number do we need to place inside . .
.................. so that 0.1 × 0.2 × 0.3 × 0.4 × ..................... = 0.12?
(a) 500 (b) 50 (c) 5 (d) 0.5 (e) 0.05

7) In the figure below, the rectangle ABCD has been divided into 10 equal squares. If the
perimeter of ABCD is 21 cm, what is the perimeter, in centimetres, of each small square?
A ..........................................................................................................................................................
... ... ... ... ... ...
B
... ... ... ... ... ..
... ... ... ... ... ...
... ... ... ... ... ..
..................................................................................................................................................
.. .. .. .. .. ..
... .. .. .. .. ...
... ... ... ... ... ...
... ... ... ... ... ...
... ... ... ... ... ..
.....................................................................................................................................................
D C

(a) 6 (b) 3 (c) 2.1 (d) 8.4 (e) 12

8) In a certain country, 800 pennies have the value of 100 ducats and 100 pennies have the
value of 250 talars. How many ducats have the value of 100 talars?
(a) 50 (b) 25 (c) 10 (d) 5 (e) 2

9) The figure below shows part of a magic square. The sum in each row, column, and diagonal
is the same. What is x + y?
.........................................................................
..... ..... ..... ...
...
... 16 ...
.
...
.
y ....
..
..............................................................................
.. . . ...
... .... .... ...
...
... x ..
...
..
...10
............................................................................
...
.
... ... ... ...
... ... ... ...
... ... ... ...
8 ... ... 12 ... .
..........................................................................

(a) 34 (b) 35 (c) 36 (d) 37 (e) 38

10) Three points were chosen to divide a line segment into four equal parts. Then two other
points were chosen to divide the same segment into three equal parts. Thus the segment is
divided into six parts. If the segment has length 1, how many different numbers represent
the lengths of these six parts?
(a) 6 (b) 5 (c) 3 (d) 4 (e) 2
11) In the figure below, ABCDE is a pentagon with AB = AC = AD = AE. If 6 BAE = 90◦ ,
6 EAD = 2x◦ , 6 BAC = 2x◦ , and 6 ACD = 5x◦ , what is the value of x?


2x ..
B ...................................................................................................A
... ....... .... ....
... ........
... ........ ... ...
...
... ............ . ..
.. .........
..
.. ............
... .... 2x◦
......... ◦ ... .....
C ..... 5x ...
..... ...
..... ... ..
..... ..
. ..
..... .
.. ...
..... .
.. ...
.....
..... .... ...
........ ..
.............
............... ....
.....
D E

(a) 10 (b) 12 (c) 14 (d) 15 (e) 20

12) Five boys, Alex, Bob, Carl, Doug, and Ed, met recently. Several handshakes took place.
Alex and Bob each shook hands with one person, and Carl, Doug, and Ed each shook hands
with two persons. If Alex shook hands with Ed, which of the following handshakes definitely
did not take place?
(a) Bob and Ed
(b) Ed and Carl
(c) Bob and Carl
(d) Ed and Doug
(e) Bob and Doug

13) In the subtraction problem below, different letters stand for different digits. Assuming that
the subtraction is correct, what is the value of W × X × Y × Z?

4 W X Y
− ...Y 5 3 Z
............................................................

2 0 0 9

(a) 0 (b) 96 (c) 90 (d) 135 (e) 120

14) In the figure below, the sector AOB of a circle with centre O is shaded. If the shaded area
is 15% of the area of the circle, what is the measure of 6 AOB?

..............................
B
......... ................
......
..... ..................................
....
. ......................
... .......................................................
...
. .
.........................
...........................................
A
.. ........................... ...
.... .............. ..
...
....
...
... O .
..
.
... ...
... ...
..... ...
......
.. . .. .....
........
................................

(a) 15◦ (b) 36◦ (c) 54◦ (d) 90◦ (e) 150◦

15) In three years, Stephen will be three times older than he was three years ago. In four years,
he will be n times older than he was four years ago. What is n?
(a) two (b) three (c) four (d) five (e) six
16) Jose, Andres, and Miguel went fishing and caught fewer than 100 fish. The number of fish
that Jose caught was exactly three times the number of fish Andres caught and four times
the number of fish Miguel caught. What is the largest number of fish that Jose could have
caught?
(a) 50 (b) 48 (c) 72 (d) 66 (e) 60

17) The figure below is made of four squares and a line segment from A to B. The siddes of
each square have length 1. What is the length of the segment AB?

.................................................. B
... ........... ..
. ...
.. .......
...................................................................
... . . .
...
. ...
.
... ....... ..... . .
.. ...
.............................................
A

√ √ √ √ √ √
(a) 5 (b) 13 (c) 5+ 2 (d) 5 (e) 3+ 2

18) How many numbers between 100 and 1000 have the product of their digits equal to 24?
(a) 21 (b) 15 (c) 18 (d) 12 (e) 24

19) What is the measure of the angle formed by the hands of a clock at 4:40 pm?
(a) 105◦ (b) 100◦ (c) 90◦ (d) 80◦ (e) 20◦

20) What is the greatest number of triangles that one can form if their vertices are points in the
figure below, none of them are right triangles, and no two of them are congruent?

• • •
• • •
• • •

(a) 1 (b) 2 (c) 3 (d) 4 (e) 5

21) In the figure below, ABCD is a rectangle with area S. Each of the points E, F , G, and H
lie on a side of the rectangle and divide that side in the ratio of 1 : 2. What is the area of
the parallelogram EF GH?

A ..........................................................................E
...................................... B
. .
.. ........... ..... ...
... ........... .... ...
. ........... .... ...
................ ....
H .... ..... .... ....
.... ...
... ..... .... ..
... .... ......
....
...
... ....
.... .
. .............
.
........... ....
...
F
... .... ...........
.. ........... ..
.......................................................................................................
D C
G

2S 5S 3S 2S 4S
(a) (b) (c) (d) (e)
5 9 5 3 9
22) The numbers x and y are the two smallest positive integers for which the product of 360
and x is a perfect square and the product of 360 and y is a perfect cube. What is the sum
of x and y?
(a) 80 (b) 610 (c) 115 (d) 165 (e) 85

23) How many numbers between 1 and 1000 do not contain the digit 1?
(a) 728 (b) 648 (c) 720 (d) 512 (e) 800

24) In the figure below, a square of area 40 is inscribed in a semicircle. What is the area of the
semicircle?
...........................
..............................................................
...... ............................................ .....
.
....... ......................................................................... .........
. ...
... ........................................................................... ...
... ............................................. ...
... ......................................................... ..
... . . ...
... ..........................................................
. . . . . . . . . . . . .
......................................................................................
.

(a) 20π (b) 40π (c) 30π (d) 25π (e) 50π

1
25) What is the last digit of the number in decimal notation?
52012
(a) 2 (b) 4 (c) 6 (d) 8 (e) 5

END OF QUESTIONS

You may mail your completed question paper to:

Mathematical Olympiad
P.O. Box 94
Mona Post Office
Kingston 7

You may also deliver your entry by hand or by courier directly to the Department of Mathematics
at the UWI, Mona Campus. In all cases, an entry must be received by December 10, 2012 in
order to be considered.

For more information, a copy of this question paper, or the latest updates, please visit
http://myspot.mona.uwi.edu/mathematics/ (see the link to the Olympiad Resource Centre).
THE UNIVERSITY OF THE WEST INDIES, MONA
Presents

The 2013 Jamaican Mathematical Olympiad


Qualifying Round
Solutions for Grades 7 and 8

1. To subtract 35 minutes from 7:10 pm we may subtract 10 minutes and then 25 minutes more. Subtracting
10 minutes gives 7:00 pm, and subtracting 25 minutes from 7:00 pm gives 6:35 pm. Daniel started at 6:35 pm.

2. Observe that
1 1 2 1 1 1 1 1 1 1 1 1 4
+ = = , × = , − = 0, and ÷ = × = 1.
4 4 4 2 4 4 16 4 4 4 4 4 1
1 1
Then the largest of the five quantities given is ÷ = 1.
4 4

3. From the information given, Brenda gave half of her bananas to Anna and shared the other half between
Catalina and herself. Since their half consisted of 3 + 4 = 7 bananas, Brenda bought 14 bananas at the
supermarket.

4. Let 6 SRT = x◦ . By vertical angles, 6 P RQ = x◦ as well. It is given that 6 RQU = 120. Since 6 RQP is
its supplement, 6 RQP = 60◦ . Since the sum of the angles in 4P RQ is 180◦ , it follows that x = 100.

S ....
...
.... ........
T
........
....
◦ .....................
....
....
.... .......... ...
.. . x
R .........
..........
........ ......
........
.........
.. .
◦ .........
........
........
x ....
.
.............. ....
....
.
........ ....
.
.............. ...
....
............ ◦ ◦ .... ◦
.. ..
.......... 20 60
.... 120
................................................................................................................................................................................................................ U
P
Q

5. It is clear that the sides marked with B and C are adjacent to the one with X. Once the folds between
these sides are made, it is also clear that the sides marked with A and D are also adjacent to the one with X.
When the last fold is made, the side opposite the one with X will be marked with E.
.............................
... ...
... ...
...
....
A ...
..
....................................................................................
... ... ... ...
... ... ... ...
B
...
....
X ....
...
...
....
C ....
..
...........................................................................................................
... ... ...
... ... ...
... D
....
E ...
...
...
...
.....................................................

.................. .................
.
6. The given equation reduces to 0.0024 × ...................... = 0.12. Dividing both sides by 0.0024 gives ... ..
................
.
= 50.
7. Let s be the side length (in centimetres) of one of the squares. Then AB = 5s, BC = 2s, CD = 5s, and
DA = 2s. Thus 5s + 2s + 5s + 2s = 21. Then 14s = 21 and hence s = 3/2. The perimeter of each small
square is s + s + s + s = 4s. Then the perimeter of each small square is 4 32 ) = 6.
s
A ..............................................................................................................................................................
... ... ... ... ... ...
B
... ... ... ... ... ...
s ... ... ... ... ... ...
... ... ... ... ... ...
...
... ... ... ... ...
....................................................................................................................................................................
... ... ... ... ... ..
... ... ... ... ... ....
... ... ... ... ... ...
... ... ... ... ... ...
... ... ... ... ... ...
.............................................................................................................................................................
D C

8. It is given that 800 pennies have the value of 100 ducats, and 100 pennies have the value of 250 talars.
Multiplying both of these latter quantities by 8, 800 pennies also have the value of 2000 talars. Then 100
ducats and 2000 talars have the same value. Dividing both of these quantities by 20, 5 ducats have the value
of 100 talars.


9. Equating the two diagonals see Figure 9(a) , 8+x+y = 16+x+12. Thus 8+x+y = x+28. Subtracting 
x from both sides, 8 + y = 28 and so y = 20. Substituting this value into the table see Figure 9(b) , we see
................................................................................. .................................................................................
... ... ... ... ... ... ... ...
... ... ... ... ... ... ... ...
... 16 ... ... y .... ... 16 ... ... 20 ...
... ... ... .. ... ... ... ..
.................................................................................... ......................................................................................
... ... ... ...
... ... ... ... ...
... ... ... ... ... ... ...
... x ... ... 10 ... ... x ... ... 10 ...
... ... ... .... ... ... ... ..
.................................................................................... .....................................................................................
... ... ... ... ... ... ... ...
... ... ... .... ... ... ... ...
8 ...
...
...
...
12
...
...
...
..
8 ...
...
...
...
12 ...
...
...
..
................................................................................. ................................................................................

Fig. 9(a) Fig. 9(b)

from the third column that each row, column, and diagonal sums to 42. Then, from the upper left diagonal,
16 + x + 12 = 42. This implies that x = 14. Finally, x + y = 14 + 20 = 34.

10. We may consider the segment of length 1 to be the part of the number line from 0 to 1, as shown below.
The three points that divide the segment into four equal parts lie at 1/4, 1/2, and 3/4. These are marked

• • • • • •
.................................................................................................................................................................................................................................. •
1 1 1 2 3
1 0
4 3 2 3 4
on the number line. The two points that divide the segment into three equal parts lie at 1/3 and 2/3. These
are also marked on the number line. Then the six segments that these points produce are from 0 to 1/4,
1/4 to 1/3, 1/3 to 1/2, 1/2 to 2/3, 2/3 to 3/4, and 3/4 to 1. Their respective lengths are:

1 1 1 1 1 1 1 1 2 1 1 3 2 1 3 1
−0= ; − = ; − = ; − = ; − = ; 1− = .
4 4 4 3 12 2 3 6 3 2 6 4 3 12 4 4

There are three different numbers, 1/4, 1/12, and 1/6, that represent the lengths of these segments.
11. From the information given, AD = AC. Thus the triangle ADC is isosceles and 6 ADC = 6 ACD = 5x◦ .
Since the sum of the angles in 4ADC is 180◦ , 6 DAC = 180◦ − 10x◦ . Since 6 EAB is a right angle,
2x + (180 − 10x) + 2x = 90. Then 180 − 6x = 90 and it follows that x = 15.

2x◦
...
. A
E ................................................................................................................
.. ..
... .. ........ ....
... ........ ... ..
...
... ....
...........
........ ..............................
.. ..
.
(180◦ − 10x◦ )
... .... . .
........ ... ...
... ........ ... ..............
... ........ ...
... ..............
...........
.... ◦ ..
...
.
... ....
... 2x◦
5x .. ...
D ....
... .
. ...
.... ... ...
.... ..
. ...
... .. ...
.... .
... ◦ ... ...
....
.... 5x ..
... ...
... . ...
.... ... ...
...... ...
.............
............... .
.................
C .
B

12. Suppose Bob and Ed shook hands. Then Alex and Bob would each have shaken hands with one person
(Ed in both cases) and Ed would have shaken hands with two people. From the information given in the
problem, they could not have shaken hands with anyone else. Then Carl and Doug would have to shake
hands with two people, but they could not shake with Alex, Bob, or Ed. Since this is impossible, Bob
definitely did not shake hands with Ed.

13. Note that 4W XY − Y 53Z = 2009 if and only if 4W XY = Y 53Z + 2009. Thus the original problem
(shown on the left below) is equivalent to the addition problem on the right. From the thousands column

4 W X Y Y 5 3 Z
− Y 5 3 Z + 2 0 0 9
....................................................................... .......................................................................

2 0 0 9 4 W X Y

in the addition problem, it is clear that Y is either 1 or 2. Thus, from the ones column there, Z + 9 is either
11 or 12. In either case, a 1 will carry into the tens column. Then 1 + 3 + 0 = X, and so X = 4. Also,
5 + 0 = W , and so W = 5. Then Y + 2 = 4 and so Y = 2. Finally, from the ones column again, Z + 9 = 12
and so Z = 3. Thus W × X × Y × Z = 5 × 4 × 2 × 3 = 120.

14. Let r be the radius of the circle and x the measure of 6 AOB. The area B
of the shaded sector is proportional to the measure of its central angle. That .................................................
........
...... ..................................
.
....... ......................
is, ... ..................................
....
. ...................................................
.
... . . . . . . . . . . . ..
...............................
Area of sector x ...
.. .................................................
.................................................................... ....
A
= ...
.. x ....................................... ..
. ...
Area of circle 360 ....
..
................. r .
...
...
O ..
.
... .
.
Since the area of the sector is 15% of the area of the circle, we also have ...
... ..
.
.
... ...
... ...
.... ...
..... ....
.
Area of sector 15 3 ......
.......
........... .......
..... . ..
.
= = ..............................
Area of circle 100 20
x 3
Thus = . Cross multiplying, we obtain 20x = 1080 and hence x = 54.
360 20

15. Let a be Stephen’s current age. In three years his age will be a + 3, and three years ago it was a − 3.
Then a + 3 = 3(a − 3). Thus
a + 3 = 3a − 9; 12 = 2a; 6 = a.
That is, Stephen is 6 years old. In four years his age will be 10, and four years ago he was 2. Then 10 = n(2)
and hence n = 5.
16. Let the number of fish that Jose, Andres, and Miguel caught be J, A, and M , respectively. From the
information given, J = 3A and J = 4M . Since J is a multiple of both 3 and 4, it is a multiple of 12. Thus
J = 12n for some n. It follows that A = 4n and M = 3n as well. The total number of fish they caught was
12n + 4n + 3n = 19n. Since they caught fewer than 100 fish, the most they could have caught was 95 fish,
which occurs when n = 5. In this case, Jose would have caught 60 fish (and Andre and Miguel would have
caught 20 and 15 fish, respectively).

17. Extend some of the sides of the squares to a point C as shown in the√figure below. By the Pythagorean
theorem, (AB)2 = (AC)2 + (CB)2 = 32 + 22 = 9 + 4 = 13. Then AB = 13.

..............................................
... ... .......... ...
B
.... ........ ...
. ........ .
.......................................................................
..
... ..
....
......
. .
.
. ...
.. ........... ... .... ..
.................................................... .... .... ..
A C

18. Suppose abc is a three-digit number with a × b × c = 24. Then a, b, c ∈ {1, 2, 3, 4, 6, 8}. If the largest
digit of abc is 8 then the other two digits must be 3 and 1. There are six numbers of this type: 138, 183,
318, 381, 813, and 831. If the largest digit of abc is 6, its digits could be 6, 4, and 1, or 6, 2, and 2. There
are six numbers of the first type: 146, 164, 416, 461, 614, and 641. There are three numbers of the second
type: 226, 262, and 622. If the largest digit of abc is 4 then its digits must be 4, 3, and 2. There are six
numbers of this type: 234, 243, 324, 342, 423, and 432. The largest digit of abc cannot be 3. This is because
the other two digits would have to be less than or equal to 3 and have a product of 8. This is impossible.
Similarly, the largest digit of abc cannot be 2 or 1. Therefore, there are 6 + 6 + 3 + 6 = 21 numbers abc such
that a × b × c = 24.

19. Each 60 minutes the minute hand on a clock travels one full circle, which is 360◦ . Thus it travels at the
rate of 6◦ per minute. The hour hand also travels one-twelfth of a full circle. This is 30◦ . Thus the hour

0◦
. .
................................... ...................................
................ ........ ................ ........
..........
. ... . ......
..... ..........
. .......
.....
...
.
.... ... ....
...
....
...
. ....
...
.
.......
..
.
.
... .........
...
..
.......
..
.
.
.........
...
....
. ... ...
... .... ...
...
... .. ... ... ..
.
....... •........... ........
...
.......
.....•....
.
........
........ .......
... ...
... ........ ... ... ....
...
. ... ...
...
...
. ....
.
.
...
... .
........ ....... .
..
..
...
... .
....
.... ........
..
..

... ◦ . ......... ...


...
....
.....
........ . .....
....
.... 120◦ 240 .....
....... . ......
..
....
...
. .
..
............... .
...... ................. .........
................................... ...................................
. . 140◦

Fig. 19(a) Fig. 19(b)

hand travels at the rate of 0.5◦ per minute. As shown in Figure 19(a), at 4:00 the minute hand is at 0◦ and
the hour hand is at 120◦ . (We will use compass directions in this problem with north being at the 12.) After
40 minutes, the minute hand has traveled 40 × 6◦ = 240◦ and ends at 240◦. The hour hand has traveled
40 × 0.5◦ = 20◦ and ends at 140◦. Then the angle between the hands is 100◦ .
20. It is possible to form four triangles in such a way that none of them are right triangles and none are
congruent to another. These are shown in Figures 20(a), 20(b), 20(c), and 20(d) below. All other triangles
which can be formed are either right triangles or congruent to one of these.

•.........................................•............ • •................................•........ • •..............................•............................•..


.. •......................... • •
....... .... .... ... ... . ... ........
....... ...
........ .... .... ... ... ... ... .......
........
.......... ....
.... ..... ... ... ... .......
.......... ... .. ... ......
• • • • •
.... ...
.... ...
.... ...
• • ...
... • .
...
..
. • • ...
...
...

..... •
.... ... ...
... ...
...
... ..
. ....
...... ... ... ... ......
....... ...... ... ....
.....
• • • • • • • •
. • • ...
• •

Fig. 20(a) Fig. 20(b) Fig. 20(c) Fig. 20(d)

21. Let AB = DC = ` and AD = BC = w. Then `w = S. The area of E


A ................................................................................................................................................................................................................................................................................................ B
1 1 w 2 2` ............................................................................. ........................................
4AEH is 2 (AH)(AE). Since AH = 3 (AD) = 3 and AE = 3 (AB) = 3 ,
........................................................................
.........................................
...........................
...................................
...............................
........................ .....................
H ........... ...................
.......................
the area of 4AEH is ..............
......................
.................
.............
..............
....................... ........
............................
   ......................................
.................................... . . . .. . .
.. ...........
........................................
................
.
................. F
1 w 2` 1 S .......................................
........................................................ . . .. . . .. . .. .. .. .. .
............................................................................
= (`w) = . . .. .
....... . . . . . . . . . . . . . . . . . . ..
. .
...................................................................................................................................................................................................................................................................................
2 3 3 9 9 D C
G
` 2w
Similarly, EB = 3 and BF = 3 . Then the area of 4EBF is
  
1 1 ` 2w 1 S
(EB)(BF ) = = (`w) = .
2 2 3 3 9 9

In the same way, the areas of 4HDG and 4GCF are each S9 . Then the total shaded area is S
9 + S9 + S9 + S9 =
4S
9 . It follows that the area of the parallelogram EF GH is

4S 9S 4S 5S
S− = − = .
9 9 9 9

22. Suppose m is a perfect square. Then m = n2 for some positive integer n. Let n have the prime
factorization n = pa q b · · · rc , where p, q, . . ., r are distinct prime numbers and a, b, . . ., c are positive
integers. Then
m = n2 = (pa q b · · · rc )(pa q b · · · rc ) = p2a q 2b · · · r2c
Thus each prime number in the factorization of m has an even exponent. On the other hand, suppose m is
a number and each of its prime factors has an even exponent. Then m is a perfect square. Indeed, if each
exponent ie even then each is a multiple of 2. Thus m can be written as

m = p2a q 2b · · · r2c

for some prime numbers p, q, . . ., r and some positive integers a, b, . . ., c. Let n = pa q b · · · rc . Then n is
an integer and n2 = m, as one may verify. Thus m is a perfect square. We may therefore conclude that
a number m is a perfect square if and only if every exponent in its prime factorization is a multiple of 2.
Reasoning in the same way, we may conclude that a number m is a perfect cube if and only if every exponent
in its prime factorization is a multiple of 3.
The prime factorization of 360 is 23 32 5. Let x be the smallest number such that 360x is a perfect square.
Clearly, 360x must have an even numbers of 2’s, 3’s and 5’s in its prime factorization. The smallest value
for x that makes 360x a perfect square is x = 2(5) = 10. Let y be the smallest number such that 360y is a
perfect cube. The number of 2’s, 3’s and 5’s in the prime factorization of 360y must all be multiples of 3.
The smallest value for y that makes 360y a perfect cube is y = 3(52 ) = 75. Thus, x + y = 10 + 75 = 85.
23. A number n between 1 and 1000 could have one, two, or three digits. If n has one digit it could be 2, 3,
4, . . ., or 9. There are 8 possibilities in all. If n has two digits, it has the form ab, where a ∈ {2, 3, 4, . . . , 9}
and b ∈ {0, 2, 3, 4, . . . , 9}. There are 8 possibilities for a and 9 for b. Then there are 8 × 9 = 72 possibilities
for ab in all. If n has three digits, it has the form abc, where a ∈ {2, 3, 4, . . . , 9}, b ∈ {0, 2, 3, 4, . . . , 9}, and
c ∈ {0, 2, 3, 4, . . . , 9}. There are 8 possibilities for a and 9 each for b and c. Then there are 8 × 9 × 9 = 648
possibilities for abc in all. Considering all three cases, there are 8 + 72 + 648 = 728 numbers between 1 and
1000 that do not contain the digit 1.

24. Let the square be ABCD and √ let O be


√ the centre of the semicircle,
√ as shown below.
√ Since the square
has area 40, each side has length 40 = 2 10. It follows that AD = 2 10 and OD = 10. Note that OA
.................................
.................
A .......... ..........
B
.........................................................................................
.........................................................................
..
......... ................................................................................................................................................. ..........
.... .. . . . . . . . . . . . . . . . . . . . . . . . . .. .....
.... ...................................................................................................................... .....
.... ......................................................................................... ....
.... ........................................................................................
....
....
. .
. ........................................................
...
...
.... ....................................................................................................................
. ...
... .........................................................................................
. ...
...
..
. ..................................................................................................................
. ...
..
. √ ......................................................................................
. . ...
.... 2 10 ................................................................................................................................................. ...
.
. ...........................................................
. ..
.... .......................................................................................................................
...
..
... .................................................................................................................. ..
..
... .........................................................................................

...........................................................................................................................................................................................................

D 10 O C
is a radius of the semicircle. By the Pythagorean theorem,
√ 2 √ 2
(OA)2 = (AD)2 + (OD)2 = 2 10 + 10 = 40 + 10 = 50.

√ √ 1 π √ 2 π
Then OA = 50 = 5 2. Thus the area of the semicircle is π(OA)2 = 5 2 = (50) = 25π.
2 2 2

25. First, observe that


1 1 22012 22012
= × =
52012 52012 22012 102012
2012
In decimal notation, this is the number 2 with the decimal point moved 2012 places to the left. The last
digit of this expression is the same as the last digit of 22012 itself. The powers of 2 are 2, 4, 8, 16, 32, 64,
128 256, 512, etc. Their last digits are 2, 4, 8, 6, 2, 4, 8, 6, 2, 4, 8, 6, . . .. This sequence repeats in cycles of
four. Thus the 4th, 8th, 12th, 16th, . . ., 2012th terms are all 6. Then last digit of 1/52012 is 6.
THE UNIVERSITY OF THE WEST INDIES, MONA
Presents
The 2012 Jamaican Mathematical Olympiad

Test for Grades 9, 10, and 11

NAME:

GRADE:

SCHOOL:

PRINCIPAL:

YEAR OF BIRTH:

STUDENT PHONE:

CONTACT TEACHER:

CONTACT PHONE:

EXAMINATION QUESTIONS

1) A rosebush in a garden has three branches. On each branch there is a cluster of three roses, and in each
rose there are two bees. How many bees are in the rosebush?
(a) 8 (b) 18 (c) 24 (d) 27 (e) 54

2) In a certain mathematics class, 17 students are in the Spanish class and 13 are in the French class. If
four of them study both languages, how many study at least one foreign language?
(a) 26 (b) 17 (c) 30 (d) 34 (e) 13

3) A square of area 81 cm2 is cut into three equal rectangles which are placed end-to-end, as shown below.
What is the perimeter of the new rectangle?
...................................................................................................................
.... . . ..
... .. .. ...
.... ..
......................................................................................................................

(a) 36 cm (b) 81 cm (c) 60 cm (d) 72 cm (e) 54 cm

4) Marsha says that 25% of her books are novels and 1/9 of them are cookbooks. How many books does
Marsha have if we know that she has between 50 and 100 books?
(a) 93 (b) 56 (c) 64 (d) 50 (e) 72

5) If 4x = 9 and 9y = 256 then xy is equal to:


1
(a) 2 (b) 4 (c) −1 (d) log36 256 (e)
4
6) In the figure below, triangles P QR and LM N are equilateral. If 6 QSM is 55◦ , what is the value of x?

L......
. ...
... ......
...
.
..
.
....
...
... S
... .....................
......
............ .... Q
.. .
.. .............. ◦ ....
.. ....................... ......
.
. 55 ..
.
. .. .
.... ........... ...
.. ...
......... .. ...
.... ...
............... ..
P .....
.....
.....
..
...
... ...
.....
...
..... .... ... .....
..... ..
..... ... ......
.......
. ... ...
... ......... . ...... M
◦ .......................................
.
..
...
.....
.....
.. x
..... ............ . .......
. .. .
.
...
... ...................
... .............. ..... ..
..
............ .....
N ..... ....
..... ..
.....

(a) 125◦ (b) 105◦ (c) 100◦ (d) 115◦ (e) 120◦

7) The sum of four consecutive odd numbers is 48. What is the largest of these numbers?
(a) 13 (b) 15 (c) 17 (d) 19 (e) 21

8) What is the maximum number of digits a number can have if every pair of consecutive digits is a perfect
square?
(a) 5 (b) 10 (c) 3 (d) 6 (e) 4

9) In the figure below, ABCD is a rectangle with AB = 5 and AD = 3. The points E and F divide AC
into three equal parts. What is the area of triangle BEF ?

A •................................................................................................................................•....... B
...... ...
.... ......
...... ...
... ...... ...
•E
.. ...... ...
... ...... ...
......
... ...... ...
... ...... ...
......
... ...... ...

•F
... ...... ...
...... ...
... ...... ...
... ......
...... ...
... ...... ..
... ......
...... ....
... ........
D• •C
.................................................................................................................

√ √
34 5 68 3 5
(a) (b) (c) (d) (e)
3 3 3 2 2

10) When 10002012 is written as a numeral, how many digits does it have?
(a) 2013 (b) 6036 (c) 6037 (d) 8048 (e) 2016

11) Suppose a and b are positive real numbers greater than 1. Which of the following fractions has the
greatest value?
a a 2a 2a 3a
(a) (b) (c) (d) (e)
b−1 b+1 2b + 1 2b − 1 3b + 1
12) The diagram below
√ shows a cuboid with four of its vertices marked X, Y , Z, and A, respectively. If
XY = 8, XZ = 55, and Y Z = 9, what is the length of XA?
X
•...............................................................................................................................
.......... .. .
.......... .......... ....
....................................................................................................................... • ...
...
... .. .... ...
...
.
. .
.
.
.
.
Y ...
...
.... ..... .... ...
.... ... ...
... .. ... ...
... .. ... ...
... .
. ...
... ..... .... ...
... ... ...
... .. ... ...
... .
.... ....... ....... ....... ....... ....... ....... ..................
. .
.
... ....... .. . .....
...
..
........................................................................................................................................
Z • •
A
√ √ √
(a) 90 (b) 10 (c) 10 2 (d) 120 (e) 11

13) When 1001 is divided by a certain 1-digit number the remainder is 5. What is the remainder when the
same 1-digit number divides 2012?
(a) 0 (b) 1 (c) 2 (d) 3 (e) 5

a2 − b2 ab − b2
14) The difference − is equal to:
ab ab − a2
a a2 − 2b a2
(a) (b) (c) a2 (d) a − 2b (e)
b ab b2

15) In the figure below, ABC and ACD are right triangles with right angles at B and C, respectively. If
AB = 64 and AD = 100, what is AC?

C
...........
........ .........
.........
............... .. ...
... ...
.
......... ... ....
................ ... ...
.......
...... ... ....
.............
. ... ...
............ . ..
.....................................................................................................................................
.
A D
B
√ √
(a) 32 3 (b) 72 (c) 80 (d) 84 (e) 50 3

16) The numbers 257 and 338 have the property that when their digits are put in reverse order the new
numbers, 752 and 833 respectively, are larger. How many 3-digit numbers have this property?
(a) 124 (b) 252 (c) 280 (d) 288 (e) 360


17) How many right triangles have one side of length 60 and integer values for the lengths of the hypotenuse
and the other side?
(a) 0 (b) 1 (c) 2 (d) 3 (e) 6

18) Suppose M and N are positive real numbers with M 6= N . If logM N = logN M , what is M N ?
1 1
(a) (b) 1 (c) 2 (d) 10 (e)
2 10

19) The numbers a, b, and c are prime numbers with a > b > c. If a + b + c = 78 and a − b − c = 40, what
is abc?
(a) 438 (b) 590 (c) 1062 (d) 1239 (e) 2006
20) In the figure below, the quadrilateral ABCD is inscribed in a circle with center O. It is known that
6 OAB = 35◦ , 6 OBC = 40◦ , and 6 OCD = 55◦ . What is 6 ODA?

...............................
A..•.................................... .......
.....
....
.. ............ ....
.... ... ............
............. .
.. ...
. ............ .....
. ... ...........
..
.... .... •B
...
... ..
... ..
. ... ..
.... .... • .. ...
... .. ..... ....
... ..
... ... O ...
...
.
...
...
..... ... ..
D• .. ...
.......... . .
... ......... ..
..
.
.... .......
..... ........ ... ...
....... . ... ......
......... ............... ...................

......................

(a) 40◦ (b) 35◦ (c) 100◦ (d) 50◦ (e) 55◦

21) Which of the following integers is odd?


13 1 5 13 + 72
(a) (13 + 7)5 (b) 5(72 − 13) (c) + − (d) (e) 2(14 + 7)13
7 2 14 2

22) Let a, b, c, and d be real numbers such that |a − b| = 2, |b − c| = 3, and |c − d| = 4. What is the sum of
all the possible values of |a − d|?
(a) 18 (b) 9 (c) 15 (d) 24 (e) 12

23) In the figure below, two circles of radius 6 overlap in such a way that each passes through the centre of
the other. What is the area of the shaded region?
.............................. ........................................
...........
.....
......
....... ...
...
....
.....
......
............
... ....
...... .........
.......
......
.....
...
. . .......
........
.........
..........
........... ...
.... ...
...
...
...
...
....
..
............
.............
..............
.............
.............. ...
.
. ...
. ...
.
..
. ...............
................
...............
.................
.................
..
.. ...
...
...
...
....
..
. ................
.
.................
.....
................
.................
................
.
... ..
..
... .................
................
•.................
..
................•
.................
...
...
...
...
...
...
... ................
.................
................
...
.................
................ ... ..
..
...
... .................
...
................
...............
................
...
............... .
.
..
.
.
.
.
...
...
..............
.............
..............
.............
............
...
... ...
..
...
..
...
...
...........
..........
.........
........
.......
...
... ...
... ...
...
....
.....
......
......
.....
....
...
...
....
..... .
...... ........
.
....
....
.
....
.
........ ... ......
................. ....................... ....................... .......................
... ....

√ √
(a) 24π (b) 24π − 18 3 (c) 9π − 18 (d) 18π − 36 (e) 36π 3

24) A number less than 3568 is odd, has remainder 2 when divided by 3, and has remainder 4 when divided
by 5. What is the sum of the digits of the largest number which meets these conditions?
(a) 14 (b) 17 (c) 18 (d) 20 (e) 21

25) What is the greatest number of elements that can be chosen from the set S = {1, 2, 3, . . . , 24, 25} so
that the sum of any two elements is not divisible by 3?
(a) 9 (b) 10 (c) 5 (d) 4 (e) 8

END OF QUESTIONS

You may mail your completed question paper to:

Mathematical Olympiad
P.O. Box 94
Mona Post Office
Kingston 7
You may also deliver your entry by hand or by courier directly to the Department of Mathematics at the
UWI, Mona. In all cases, an entry must be received by February 27, 2012 in order to be considered.

For more information and the latest updates, please visit


http://myspot.mona.uwi.edu/mathematics/ (see the link to the Olympiad Resource Centre).

For extra copies of this question paper, please visit


http://myspot.mona.uwi.edu/mathematics/ (see the link to the Olympiad Resource Centre).
THE UNIVERSITY OF THE WEST INDIES, MONA
Presents

The 2012 Jamaican Mathematical Olympiad

Solutions for Grades 9, 10, and 11


1. There are 3 branches in the rosebush, 3 × 3 = 9 roses in the rosebush, and 2 × 3 × 3 = 18
bees in the rosebush.

2. Since 4 students study both languages, they should not be counted twice. Then the
number who study at least one language is 17 + 13 − 4 = 26.

3. Since the square has area 81 cm2 , each side has length 9 cm. When the square is cut into
three equal rectangles, each one is 3 cm × 9 cm. When the strips are placed end-to-end, the
resulting rectangle is 27 cm × 3 cm. Its perimeter is 27 cm + 3 cm + 27 cm + 3 cm = 60 cm.
........................................................
.. ...
... ... ... 9 9 9
.. .. .. ..... ...................................................................................................................................................
.. . . .... ..
... .. .. ... ... ...
.. .. .. ... 3 .. ... 3
.. ... ..................................................................................................................................................
... ... ... ...
.. .. .. ...
..
.......................................................
. 9 9 9

4. Since one-fourth of Marsha’s books are novels, the number of her books is a multiple
of 4. Since one-ninth of them are cookbooks, the number is also a multiple of 9. Then the
number of her books is also a multiple of 36. The 36-multiples are 36, 72, 108, . . . The
only one between 50 and 100 is 72. Then Marsha has 72 books.

5. From the information given, 4xy = (4x )y = 9y = 256. On the other hand, we also have
44 = 4 × 4 × 4 × 4 = 256. Then 4xy = 44 and it follows that xy = 4.
6. Let T , U , and V be the points of intersection as shown in the figure below. From the
information given, the angles at Q, M , and R are each 60◦ . Since the sum of the angles in

L
....
... ......
.... .........
. .....
... .....
.
.
... .....
..... ...........
Q
.
. ..... ............ ....
.
.. .....
S ............ ...
.. ..... .. ............... ◦
.
..
..... ........
...................
.
. .. ..
60 ......
.... .... ... ... ..... ..... ◦ .
.
.
... ............ ..... 55 ...
............ ..... ...
............... ...
... .. . . ...
................ .....
.... .
.
.
............ .. .....
...
............ .. .....
..... 65◦....
................ ...
P .....
.....
.....
...
.
..
.....
..... ....
..... ...
.
.....
.....
.....
...
...
.......
. T
..........
.....
..... ...
.
.
.... .........
.
..... ..
. ... ◦ ......
....... . .....
......
.. ..... ... 65 .....
... .......... ... .....
.. . ..... ◦ ................. M
.. ..... .. 55................
. .
...... . ....
.. ..... .............
.. ◦ ..............
...
...
.....
.....
..... x . ... .. ................
............. .....
◦ .. U
. ................. . 55 ....
.. ............ .......... 65◦
.
... ............ ...
... ............. .....
...
..................... .....
...
N V .....
.....
.....
.....
.
.
...
..... ....
..... ..
....

any triangle is 180◦ , 6 QT S = 65◦ . By vertical angles, 6 M T U = 65◦ and it follows


that 6 M U T = 55◦ . By vertical angles again, 6 RU V = 55◦ and it follows again that
6 RV U = 65◦ . Finally, since 6 P V M and 6 RV M are supplementary, x + 65 = 180. Then
x = 115.

7. Let n be the first of the four numbers. Then the other ones are n + 2, n + 4, and n + 6.
From the information given,

n + (n + 2) + (n + 4) + (n + 6) = 48
4n + 12 = 48
4n = 36
n=9

The consecutive odd numbers are 9, 11, 13, and 15. The largest one is 15.

8. The two-digit perfect squares are 16, 25, 36, 49, 64, and 81. So suppose every pair of
consecutive digits of a number is a perfect square. If the digit 1 is present the next digit
must be 6, when 2 is present the next digit must be 5, and so on. Note that whenever 5,
7, or 9 occurs there cannot be a next digit. Among other things, this means that the first
digit of the number cannot be 5, 7, or 9. We consider each of the possibilities 1, 2, 3, 4, 6,
and 8 in turn. If the first digit of the number is 1 it has to be 1649. If its first digit is 2 it
has to be 25. If its first digit is 3 it has to be 3649. If its first digit is 4 it has to be 49. If
its first digit is 6 it has to be 649. And if its first digit is 8 it has to be 81649. The longest
such number is 81649 and it has five digits.
9. From the information given, the rectangle ABCD has area 15. It follows that the
triangle ABC has area 15/2. Furthermore, the triangles BAE, BEF , and BF C all have

A •..............................................................................................................................•
....... B
.... ...........
. .. .
.......... ....... ....
...... ..........
..
.. ...... ................... ...
... ....
..
... • ..........
......
...... ....
...
..
. ...
...
..
...
E ......
......
...... .... .....
.
...
...
... ........ ...
..
.. • ......
......
...
..
...
..
F ......
...... ....
. .
..............................................................................................................
D• •C

equal bases and equal heights. Thus they have equal areas. Then the area of BEF is 1/3
the area of ABC, and this is
1 15 15 5
× = = .
3 2 6 2

10. The easiest approach is to note that 1000 = 103 . Then 10002012 = (103 )2012 = 106036.
When written as a numeral, this number has a 1 followed by 6036 zeroes. It has 6037
digits in all.

a c
> then, multiplying
11. Suppose first that a, b, c, and d are any positive numbers. If
b d
both sides by the positive number bd, we have ad > bc. On the other hand, if ad > bc
1 a c
then, multiplying both sides by , we have > . Thus we may say that
bd b d
a c
> if and only if ad > bc
b d
a a
We now compare the five quantities (a)–(e). First, > because b − 1 < b + 1
b−1 b+1
a 2a
and when a is divided by a smaller quantity the quotient is larger. Second, >
b−1 2b + 1
if and only if

a(2b + 1) > (b − 1)(2a); 2ab + a > 2ab − 2a; a > −2a.

a 2a
Since a is positive, this is true. Third, > if and only if
b−1 2b − 1

a(2b − 1) > (b − 1)(2a); 2ab − a > 2ab − 2a; −a > −2a; a < 2a.

a 3a
Since a is positive, this is true. Finally, > if and only if
b−1 3b + 1

a(3b + 1) > (b − 1)(3a); 3ab + a > 3ab − 3a; a > −3a.


a
Since a is positive, this is true. In summary, the largest of the five quantities is .
b−1

12. Let a, b, and c be the dimensions of the cuboid as shown. Since XZ is perpendicular
to ZA, we have (XZ)2 + (ZA)2 = (XA)2 by the Pythagorean theorem. Furthermore,

X b
a...............................•........................................................................................................................................
........ ......... ....
..........
........ . ... b .......... ..
...........................................................................................................................................•
............ a ..
...
.
... .. ... . ... ..
... .. . ..
... .. ... ... Y ..... ...
... . .
. . ..
... ... .. ..
.
.
. ..
... . ... .
. ... c
... .
.. .
. ..
... .
. . .
.... .
. ..
.
. .
. ...
... .
.. .
. ..
.. .... .
c ...
... .
.
.
.. c .
.
.
.
..
...
... ... ... ..
. ..
... . .. .. .
. ..
... .. . ... .
. ...
... .. .. .... ..
... .
....... ....... ....... ....... ....... ........ ........ ....... ....... ....... ....
.
... .... . .. ... ...
.
... . ..
.... .
.......
...
. ....... .... ... ..........
........... ....... . . ............ a
• ........................................................................................................................................

Z
b A

(XZ)2 = a2 + c2 and (ZA)2 = b2 . Thus (XA)2 = a2 + b2 + c2 . Now note that a2 + b2 =


(XY )2 = 64, a2 + c2 = (XZ)2 = 55, and b2 + c2 = (Y Z)2 = 81. Then, adding all three
2 2 2 2 2 2
√ + 2b + 2c = 64 + 55 + 81 = 200. Thus a + b + c = 100 and it follows
equations, 2a
that XA = 100 = 10.

13. In order to have a remainder of 5, the one-digit number dividing 1001 must be 6, 7,
8, or 9. If 1001 is divided by 6 the remainder is 5; if 1001 is divided by 7 the remainder
is 0 (in other words, 7 divides evenly into 1001); if 1001 is divided by 8 the remainder is 1;
and if 1001 is divided by 9 the remainder is 2. Therefore, the number dividing 1001 is 6.
When 2012 is divided by 6, the remainder is 2.

ab − b2 b(a − b) −b(b − a) b
14. First, 2
= = = − . Then
ab − a a(b − a) a(b − a) a
 
a2 − b2 ab − b2 a2 − b2 b a2 − b2 b a2 − b2 b2 a2 a
− = − − = + = + = = .
ab ab − a2 ab a ab a ab ab ab b

15. The triangles ABC and ACD are similar. This is because both are right triangles

......
C
........ ........
......... ... ...
.....
........... ... ...
......
.
............... .. ....
.. ..
........... ... ...
........... . ...
......
..........................................................................................................
.
....
A D
B
with a common angle at A. Since the sum of the angles in any triangle is 180◦ ,

90◦ + 6 A + 6 ACB = 90◦ + 6 A + 6 ACD.


It follows that 6 ACB = 6 ACD. Thus the two triangles have all of their corresponding
angles equal. Since corresponding sides in similar triangles are in the same proportion,
AB AC 64 AC
= ; = ; (AC)2 = 6400; AC = 80
AC AD AC 100

16. When the digits of a 3-digit number abc are put in reverse order the new number,
cba, is larger if and only if c > a. The possibilities for a and c are:

1b2, 1b3, 1b4, 1b5, 1b6, 1b7, 1b8, 1b9,


2b3, 2b4, 2b5, 2b6, 2b7, 2b8, 2b9,
3b4, 3b5, 3b6, 3b7, 3b8, 3b9,
4b5, 4b6, 4b7, 4b8, 4b9,
5b6, 5b7, 5b8, 5b9,
6b7, 6b8, 6b9,
7b8, 7b9,
8b9,

There are 36 possibilities listed, and for each one there are 10 choices for b. Then there
are 360 such numbers in all.

17. Let h be the hypotenuse of the triangle and a the length of the other side. By the
√ 2
Pythagorean theorem, a2 + 60 = h2 and hence a2 + 60 = h2 . By the laws of algebra,
2 2
h − a = 60 and so (h + a)(h − a) = 60. Since h and a are positive integers with h > a,
h + a and h − a are also positive integers and h + a > h − a. Since h + a and h − a are
factors of 60, the possibilities are:

h + a = 60, h − a = 1; h + a = 20, h − a = 3; h + a = 12, h − a = 5;


h + a = 30, h − a = 2; h + a = 15, h − a = 4; h + a = 10, h − a = 6.

When h + a =√30 and h − a = 2 then h = 16 and a = 14. This gives a right triangle with
sides 14 and 60 and hypotenuse 16. When h + a = √ 10 and h − a = 6 then h = 8 and
a = 2. This gives a right triangle with sides 2 and 60 and hypotenuse 8. In the other
pairs of equations, the solutions for
√ h and a are not integers.
√ Thus there are two right
triangles meeting our criteria: 14, 60, and 16, and 2, 60 and 8.

18. Let p = logM N so that p = logN M as well. Then M p = N and N p = M . Thus


2
M p = (M p )p = N p = M . It follows that p2 = 1 and hence p = 1 or p = −1. The choice
p = 1 gives M 1 = N and so M = N . Since this is not true, p = −1. Thus M −1 = N and
 
1 1
this means that N = . Then M N = M = 1.
M M
19. Since a + b + c = 78 and a − b − c = 40, we may add equations to obtain 2a = 118
and hence a = 59. It follows that b + c = 19. The only solutions with b and c prime and
b > c is b = 17 and c = 2. Then abc = (59)(17)(2) = 2006.

20. Let 6 ODA = x◦ . Note that OA = OB = OC = OD because OA, OB, OC, and OD
are all radial segments. Then the triangles OAB, OBC, OCD, and ODA are all isosceles.

..........................................
........... ........
....... .......
A...•................................... ......
.....
.....
. .... ... ...... ................. .....
. .. .. .....
. ............ .....
.
.. ... .... ............. ...
.. ...... . ... ............ ...
. .. .. ◦ ...
. ...
.
.. .
.
. ◦ ......
....
.35
.
.............
.. ............. ..
..
...
..
x .
.
.
.
. .....
.... ◦
. .. .................
.. •B
.............
...
.
. .
.
...
.....
..... 35 ... .. .
.......... ..... ....
. .. .
.. .. .... .........
.. ... ..... ......... .. ...
... .... .................. ◦ ... ..
...
...
• ........... 40 ...
. ..
...
.
... ......... ...
.. ...... .... ...
...
...
.
.
.... .
. ...... ..... O
..
... ..... .
.
.
.
... .. ◦ ................ .. ...
.. ...
... ...
... .. x .......
.. ...
... ...
.. ...
...
... ... ........... .. ... ..
........... ◦ ... ..
... ...
D • 55 ...........
.... .........
..... ..........
..
...
40.
◦......
.. ......
...
..... ........ ◦ .... .... .....
.....
......
........
........ 55 ... .... .....
.....
......
.......
........
........ .... ..... ...........
......... .. . . .....
............... .......................................
................. •
C

Thus 6 OAB = 6 OBA = 35◦ , 6 OBC = 6 OCB = 40◦ , 6 OCD = 6 ODC = 55◦ , and
6 ODA = 6 OAD = x◦ . Since the sum of the angles in any quadrilateral is 360◦ ,

2(35) + 2(40) + 2(55) + 2x = 360


260 + 2x = 360
2x = 100
x = 50

That is, 6 ODA = 50◦ .

21. We consider each expression in order. First, (13 + 7)5 = 205 , and any power of an
even number is even. Second, 5(72 − 13) = 5(49 − 13) = 5(36) = 180. This is even. Third,

13 1 5 26 7 5 26 + 7 − 5 28
+ − = + − = = = 2.
7 2 14 14 14 14 14 14

13 + 72 13 + 49 62
This is even. Fourth, = = = 31. This is odd. Finally, 2(14 + 7)13 is
2 2 2
even because 2 times any integer is even. The one and only odd integer among the choices
13 + 72
given is .
2
22. From the information given, a − b = ±2, b − c = ±3, and c − d = ±4. Since
a − d = (a − b) + (b − c) + (c − d), there are 8 possibilities for a − d:

a − d = 2 + 3 + 4 = 9; a − d = −2 + 3 + 4 = 5;
a − d = 2 + 3 − 4 = 1; a − d = −2 + 3 − 4 = −3;
a − d = 2 − 3 + 4 = 3; a − d = −2 − 3 + 4 = −1;
a − d = 2 − 3 − 4 = −5; a − d = −2 − 3 − 4 = −9.

Then the possible values for |a − d| are 1, 3, 5, and 9. The sum of these values is 18.

23. Let O1 and O2 be the centres of the respective circles and let A and B be the points
where the circles intersect. (See Figure 23(a) below.) Note that AO1 and O1 O2 are radii

A A
............................... ....................................... .................................
.............................. .....................
......... .........
...................................
.... ...... .......
...... ................ ........
..... .......... ........
..... .................. ..... ........................................................
.......................................... .
. ..... .. ..... . .
..
.......... ...
.. . . . . . . . . . . . . . . . . . ..
.. C
..............................................
.. . .... .. ....
...
..
.
. .
......................... ...
................. ...
. . . ...
B .............................................................................................................................................................
.... .... A
.............................
. ... ............. .... ...
.
..
. .
.
.............................
. ... ..
................... ... ... .... ................................................................................................................................................................................ ....
. . . . . . . . . . . .. ..
. ... . ... . ................................................................... ...
.... .............................................. .. ........................ ...
... ... .... ....................................................
...................................... ..
.. .
............................... .. ................. .. .. . ........... ..
...
.. O • 1 ................................................ •O
................................................................
2 ...
. .......................
.................
•O .
...
.
2 ...
. ....
...
• ..
...
...
...
. . . . .
.............................
.... .... . . . . . ......
. . . . . .... ...
. . .
............... . . . .
....
...
. ...
O ...
.
. ............... .. . . . ... 2 .
... ......................................... .
... ............. ... ...
.. ... ..
...
... .......................... ... .............. ... ... ... ...
...
..... ......................... .... .............. ... .... ..... ....
..... .. . . .. .
...................
. .. .
.. .... ............. ... ..... ...
...... ........... ..... ...........
......
... ......
......
...
......... .. ...... ........
....................................
........
....................................
............................... .......................................
B B
Fig. 23(c)
Fig. 23(a) Fig. 23(b)
of the circle with centre O1 , and O1 O2 and O2 A are radii of the circle with centre O2 . It
follows that triangle AO1 O2 is equilateral. Similarly, the triangle BO1 O2 is equilateral.
Thus 6 AO2 B = 60◦ + 60◦ = 120◦ .
The shaded region in Figure 23(a) consists of two sectors as shaded in Figure 23(b).
By rotating one sector 90◦ , this may be viewed as the upper portion of an “ice-cream cone”
as in Figure 23(c). The area of the entire “ice-cream cone” is 1/3 of the area of the circle
with centre O2 . (This is because 6 AO2 B is 1/3 of an entire circle.) Thus its area is

1 1
π(62) = (36π) = 12π
3 3
To find the area of 4BO2 A, let C be the midpoint of BA. Then BO2 C and AO2 C √ are
each 30-60-90 triangles. It follows that BO2 = AO2 = 6, CO2 = 3, and BC = AC = 3 3.
Then the area of BO2 A is

1 1 √  √
× BA × CO2 = 6 3 (3) = 9 3.
2 2
 √
Thus the area of one sector as in Figure 23(b) is 12π − 9 3. The total shaded area as
 √
in Figure 23(a) is 24π − 18 3.
24. If a number has remainder 4 when divided by 5 then its last digit is either 4 or 9. If
the number is odd then its last digit must be 9. If the number is also less than 3568 then
it is one of these numbers (counting backwards):

3559, 3549, 3539, 3529, 3519, 3509, . . .

The largest number on this list which has remainder 2 when divided by 3 is 3539. The
sum of its digits is 20.

25. Let
S1 = {1, 4, 7, 10, 13, 16, 19, 22, 25},
S2 = {2, 5, 8, 11, 14, 17, 20, 23},
S3 = {3, 6, 9, 12, 15, 18, 21, 24}.
Then S1 consists of all the numbers in S with remainder 1 when divided by 3, S2 consists
of all the numbers with remainder 2, and S3 consists of all the numbers divisible by 3. Note
that the sum of any two numbers in S1 is not divisible by 3. Similarly, the sum of any two
numbers in S2 is not divisible by 3. However, the sum of any number in S1 and any number
in S2 is divisible by 3. Suppose T is a set with the greatest number of elements that can
be chosen from S so that the sum of any two elements is not divisible by 3. Then T can
contain all the elements in S1 or all the elements in S2 , but it cannot contain elements
from both subsets. Furthermore, suppose T contains all the elements in S1 . Then it can
also contain one element from S3 . Similarly, if T contains all the elements in S2 it may
also contain one element from S3 . However, T cannot contain two elements from S3 . For
example if T contains 6 and 15 then it contains 6 + 15 = 21, and this is a multiple of 3.
Then the possibilities are that:
a) T contains all elements in S1 and one element from S3 , or
b) T contains all elements in S2 and one element from S3 .
Since (a) gives a set with more elements, T contains 10 elements. For example, we could
have T = {1, 3, 4, 7, 10, 13, 16, 19, 22, 25}
THE UNIVERSITY OF THE WEST INDIES, MONA
Presents

The 2012 Jamaican Mathematical Olympiad

Test for Grades 7 and 8

NAME:

GRADE:

SCHOOL:

PRINCIPAL:

YEAR OF BIRTH:

STUDENT PHONE:

CONTACT TEACHER:

CONTACT PHONE:

EXAMINATION QUESTIONS

1) Which number is half way between 2006 and 6002?


(a) 1998 (b) 3996 (c) 4002 (d) 4004 (e) 4006

2012 + 2012
2) The fraction is equal to:
2012 + 2012 + 2012
2 1 2013 1
(a) 5(2012) (b) (c) (d) (e)
3 2012 4025 1007

3) A square of area 64 cm2 is cut into four equal rectangles, as indicated below. What is the perimeter of
one of the rectangles?
................................................................................
.... .. .. .. ...
... ... ... ... ...
... ... ... ... ...
... ... ... ... ...
... ... ... ... ...
...
... ... ... ... ...
... ... ... ... ...
... ... ... ...
... ... ... ... .....
... ... ... ... ...
... ... ... ... ...
... ... ... ... ....
... ... ... ... ...
... ... ... ... ...
...................................................................................

(a) 18 cm (b) 8 cm (c) 16 cm (d) 10 cm (e) 20 cm

4) What is the ones digit (or units digit) in the product of all the prime numbers less than 2012?
(a) 0 (b) 2 (c) 4 (d) 6 (e) 8
5) How many times between 00:00 and 23:59 does an electronic watch show all of the following four digits,
2, 0, 1, 2, in any order?
(a) 2 (b) 12 (c) 5 (d) 4 (e) 6

6) In the figure below, a triangle is inscribed in a regular hexagon. What proportion of the area of the
hexagon is shaded?
........................................................... .
...
... .
... .... .......
...

..............................
...
... ... .. ...
... .. ... ...
.. .. .... ...
................................................
.. .
. ...
...
. ... ...
... ...
.
. .. ...
...........................................................................
... . .
... ...
.. ... ... ...
..... ..
. . .
.. ... ...
... .
..................... ... ..
.................................................................
... .
.. . ..
... .. ... ...
.
........................................................................................................................
... ... .
.
... ... ... ...
... .. ... ...
... .... ... ....
... .. .............................
...............................................................................
... .. .. ...
...... .. ...
.......................................................

1 1 1 2 1
(a) (b) (c) (d) (e)
6 4 3 5 12

7) Two litres of fruit juice containing 10% sugar are mixed with 3 litres of another juice containing 15%
sugar. What percentage of the mixture consists of sugar?
(a) 13% (b) 5% (c) 25% (d) 12.5% (e) 12.75%

8) In the figure below, AC = 10, BD = 15, and AD = 22. What is the length of BC?
A B C D
• • •
......................................................................................................................................................................................................... •

(a) 2 (b) 5 (c) 1 (d) 4 (e) 3

9) In a certain year, there were four Mondays and four Fridays in January. Which day of the week was
January 1 of that year?
(a) Sunday (b) Tuesday (c) Wednesday (d) Thursday (e) Saturday

10) The positive real numbers a, b, c, d, and e satisfy the conditions ab = 2, bc = 3, cd = 4, and de = 5.
What is e/a?
4 5 15 3
(a) (b) (c) 120 (d) (e)
5 6 8 2

11) In the figure below, the sides of a triangle have been extended to form three angles as shown. What is
the value of x?
.
...
....
....
.
....
. ◦
.....
... ...
... ....
106
.... ...
..... .
....
..
.... ...
.... ...
... ...
....... ...
...
.
.... ...
.... ...
◦ ....... . ...
142 .
.... ...
..
...........................................................................................................................................
..
◦ .......
... x

(a) 112 (b) 144 (c) 108 (d) 120 (e) 68


12) There are three married couples attending a party. In how many ways can they form a three-person
group in which there will not be a married couple?
(a) 1 (b) 2 (c) 6 (d) 8 (e) 20

13) A small box of chocolates costs $100. There is a coupon inside each of the boxes of chocolate. With
three coupons, you can get an additional box of chocolates free. What is the greatest number of boxes
you can get for $1,500?
(a) 15 (b) 18 (c) 20 (d) 21 (e) 22

14) How many right triangles have one side of length 60 and integer values for the lengths of the hypotenuse
and the other side?
(a) 0 (b) 1 (c) 2 (d) 3 (e) 6

15) In the figure below, a square with side length 12 cm has been divided into three rectangles with equal
perimeter. What is the area of the shaded rectangle?
................................................................................................
... ...
... ...
... ...
... ...
.................................................................................................
..............................................................................
.. ... ...
.............................................................................................
...
...
..
....
..
....
............................................................................................................................
...
...
..
....
..
....
.............................................................................................
...
...
....
...
....
...
.............................................................................................
...
...
...
...
...
...
............................................................................................................................
... ... ...
... ... ...
. .
...............................
....................................................................................................

(a) 72 cm2 (b) 54 cm2 (c) 40 cm2 (d) 48 cm2 (e) 36 cm2

16) What is the sum of the digits of the square of 111, 111, 111?
(a) 18 (b) 27 (c) 81 (d) 63 (e) 45

17) In a room there are stools and chairs. On each stool and each chair there is a child. Each stool has
three legs, each chair has four legs, and each child has two legs. All together there are 39 legs. How
may chairs are in the room?
(a) 4 (b) 3 (c) 6 (d) 9 (e) 5

18) How many isosceles triangles may be found in the figure below?
...........
...............
...... .. ... ......
...... .... .... ..........
.......... .. ... ......
.... ... ... ......
..... ... ......
...... ..
.. ......
..... ... ...
...............................................................................................................
.. . . .. .
. .
... ...... . .... .. .
... ....... .. . ...... ..
.
... ....... ....
. ...
.
. .
..... ....
.
... ...... .. ... ..... .
....... .
..... . .
.
... . ... ....... ...
... .. ........... .......... ..... ..
...
...
. ..... . ...
.. .. .
...
... ... ... ..
... .... ...... ........... . .
... ... .......... ...... .... ...
... .. ....... ...... .. ...
...........
.............
..................................................................

(a) 15 (b) 13 (c) 25 (d) 20 (e) 10

19) Eighteen years ago, Marco was three times older than his niece Shawana. Now he is twice as old as she
is. How old is Shawana?
(a) 18 (b) 24 (c) 28 (d) 36 (e) 72
20) The diagram below
√ shows a cuboid with four of its vertices marked X, Y , Z, and A, respectively. If
XY = 8, XZ = 55, and Y Z = 9, what is the length of XA?
X

..........
.........................................................................................
...........
.
. .
.......... ...
.................................................................................................... •
... .. ....
.
. .
. ....
.... ... Y .... ..
.... .
. ...
... .... .... ....
... ... ..
.... ... ... ...
... . ... ....
... .
. .
.... .... ....... ....... ....... .......... ....... ....... ....
.
... ....... ....... .
.... ...................
.................................................................................................
Z • •
A
√ √ √
(a) 90 (b) 10 (c) 120 (d) 11 (e) 10 2

21) How many four-digit numbers abcd are multiples of 3, 4, and 5 and satisfy the conditions that a is the
double of c and b is 6?
(a) 5 (b) 1 (c) 4 (d) 0 (e) 2

2x − y 2 x
22) If = , what is ?
x+y 3 y
1 4 5 6
(a) (b) (c) 1 (d) (e)
5 5 4 5

23) In the figure below, the quadrilateral ABCD is inscribed in a circle with center O. It is known that
6 OAB = 35◦ , 6 OBC = 40◦ , and 6 OCD = 55◦ . What is 6 ODA?

...............................
A..•.................................... .......
.....
....
. .. .. ............ ...
... ... ............. ...
. . ...............
... ... .................
.. ...
.
.... ....
•B
......
....
... ...
... .. ... ..
.... ....
... ...
• ...
..... ...
...
... ...
.....
O ...
...
...
..
...... .. ..
D• ........... .. ..
.... ........ ... ...
..... ......... .. ....
...... ......... ... ....
........ . ... .......
.................................................

C

(a) 50◦ (b) 40◦ (c) 100◦ (d) 55◦ (e) 35◦

24) How many positive integers less than 1,000 are neither multiples of 5 nor of 7?
(a) 326 (b) 686 (c) 692 (d) 682 (e) 658

25) What is the greatest number of elements that can be chosen from the set S = {1, 2, 3, . . . , 24, 25} so
that the sum of any two elements is not divisible by 3?
(a) 4 (b) 8 (c) 10 (d) 5 (e) 9

END OF QUESTIONS

You may mail your completed question paper to:

Mathematical Olympiad
P.O. Box 94
Mona Post Office
Kingston 7

You may also deliver your entry by hand or by courier directly to the Department of Mathematics at the
UWI, Mona. In all cases, an entry must be received by February 27, 2012 in order to be considered.

For more information and the latest updates, please visit


http://myspot.mona.uwi.edu/mathematics/ (see the link to the Olympiad Resource Centre).

For extra copies of this question paper, please visit


http://myspot.mona.uwi.edu/mathematics/ (see the link to the Olympiad Resource Centre).
THE UNIVERSITY OF THE WEST INDIES, MONA
Presents

The 2012 Jamaican Mathematical Olympiad


Solutions for Grades 7 and 8

1. If m is half way between 2006 and 6002 (as shown in the figure below) then m − 2006 =
6002 − m. Thus 2m = 8008 and so m = 4004.

• •
................................................................................................................................................................ •
2006 m 6002

2. We have
2012 + 2012 2012(1 + 1) 2012(2) 2
= = =
2012 + 2012 + 2012 2012(1 + 1 + 1) 2012(3) 3

3. Since the area of the square is 64 cm2 , each side has length 8 cm. Then each smaller
rectangle has width 2 cm and height 8 cm. Thus each smaller rectangle has perimeter (in
centimetres) 2 + 8 + 2 + 8 = 20.
2 2 2 2
.......................................................................................................
.... .... .... .... ...
... ... ... ... ....
.. .. .. .. ...
... ... ... ... ...
.. .. .. .. ....
... ... ... ... ...
.. .. .. .. ...
... ... ... ... ...
.. .. .. .. ...
... ... ... ...
8 ..
...
..
...
..
...
..
...
...
...
.. .. .. .. ...
... ... ... ... ...
.. .. .. .. ...
... ... ... ...
.. .. .. .. .....
... ... ... ... ...
.. .. .. .. ...
... ... ... ... ...
.. .. .. .. .
..................................................................................................

4. The product of all the prime numbers less than 2012 has the form 2 × 3 × 5 × 7 × 11 ×
· · · × 2011. (One may verify that 2011 is, indeed, prime.) Rearranging terms, this is

(2 × 5) × (3 × 7 × 11 × · · · × 2011) = 10 × (3 × 7 × 11 × · · · × 2011)

The last digit of this product is clearly 0.

5. First, consider the times of the form ab: cd, where a, b, c, and d are the numbers 2, 0, 1,
and 2 in some order. There are 6 ways of replacing two of the letters with 2, as follows:

22: cd, 2b: 2d, 2b: c2, a2: 2d, a2: c2, ab: 22.
For each of these ways, we can choose the remaining letters to be 0 and 1 in two different
ways. This gives 6 × 2 = 12 possibilities in all. They are:

22: 01, 20: 21, 20: 12, 02: 21, 02: 12, 01: 22,
22: 10, 21: 20, 21: 02, 12: 20, 12: 02, 10: 22.

Since each of these displays occurs on an electronic watch exactly once between 00:00 and
23:59, the answer is 12.

6. Consider a triangle with the same base as the given triangle and with one vertex being
the centre of the circle. (For example, consider the shorter, darker triangle in the figure
below.) The area of the shorter triangle is 1/6 the area of the hexagon. The taller triangle
......................................................
... ... ...
... ........ ...
... ............ ...
..... . .................... ...
...
... . .. ................ ...
.
.. ............... ...
.
.. ..................... ...
.......................
....
...
... . .. .. . .
..................................
...............................................
• . ...
..
... ..
.................................................................... ...
... ..
.. .................................... ... ...
... ................................................................... ...
... .......................................................................................................................................................... ....
... .................................................................................................................. ...
...............................................................................................................
..................................................................................................................

has twice as much area because it has the same base as the shorter one and twice its height.
Then the proportion of the area of the hexagon which is contained in the taller triangle is
 
1 2 1
2 = =
6 6 3

7. The total amount of sugar (in litres) in the mixture is 2 × 0.10 + 3 × 0.15 = 0.20 + 0.45 =
0.65. The mixture itself has 5 litres of juice all together. The percentage of sugar in the
juice is
0.65
× 100 = (0.13) × 100 = 13.
5
The mixture contains 13% sugar.

8. Let AB = x, BC = y, and CD = z. From the information given, x + y = 10 and


x + y + z = 22. It follows that z = 12. We also have y + z = 15 and it follows that y = 3.
Thus BC = 3.
A B C D
• • •
........................................................................................................................................................................... •
x y z

9. Suppose January 1 was on a Sunday. Then there would be five Sundays in January
(the 1st, 8th, 15th, 22nd, and 29th), five Mondays, (the 2nd, 9th, 16th, 23rd, and 30th), and
five Tuesdays (the 3rd, 10th, 17th, 24th, and 31st) in that month. Since there were only four
Mondays, January 1st was not on Sunday. Similarly, January 1 was not on Monday or there
would have been five Mondays again. Also, January 1 was not on Wednesday, Thursday,
or Friday or there would have been five Fridays in that January. Finally, it was not on
Saturday or else there would have been five Mondays again. So the only possibility is that
January 1 was on Tuesday. This works because when January 1 is on Tuesday there are five
Tuesdays, five Wednesdays and five Thursdays in January. On the other hand, there are
four Fridays, four Saturdays, four Sundays, and four Mondays in that month.

2 3 2 3 3a
10. From the first two equations, b = and b = . Then = and it follows that c = .
a c a c 2
4 5 4 5 4e
From the next two equations, d = and d = . Then = and it follows that c = .
c e c e 5
4e 3a e 15
Since c = c we have = . Then 8e = 15a and so = .
5 2 a 8

11. Let a, b, and c be the measures of the interior angles, as shown in the figure below.
Since the three angles were formed by extending the sides of the triangle, 142 + a = 180,
.
.....
.....
..
......
.
..... ◦
..... ...
..... .....
106
..
......
. ...
..... ◦ .....
.....
.....
b ...
...
.
........ ...
.....
. ...
.
... .
◦ .......... ◦..
◦ ........
142
..
...
. .
. a c ..
..................................................................................................................
.
◦ ......
x ...

106 + b = 180, and c + x = 180. From the first two equations, a = 38 and b = 74.
Furthermore, we know that a + b + c = 180. This means that 38 + 74 + c = 180 and so
c = 68. Since x + c = 180, it follows that x = 112.

12. Suppose one married couple is denoted by A1 and A2 , another by B1 and B2 , and the
third by C1 and C2 . The only way to form a three-person group with no married couple
present is to choose one of the A’s, one of the B’s, and one of the C’s for the group. There
are two ways to choose one of the A’s, two ways for the B’s, and two for the C’s. Thus
there are 2 × 2 × 2 = 8 ways of choosing in all. That is, there are eight ways of forming a
three-person group in which there will not be a married couple.

13. To begin, one can spend all of the $1500 to buy 15 boxes of chocolates. This gives a
total of 15 coupons which may be redeemed for 5 more boxes. This gives another 5 coupons
which may be redeemed for one box of chocolate with 2 coupons left over. Finally, the
one box contains a coupon which may be redeemed with the other two for one last box of
chocolate. The greatest number of boxes one can get for $1500 is 15 + 5 + 1 + 1 = 22.
14. Let h be the hypotenuse of the triangle and a the length of the other side. By the
√ 2
Pythagorean theorem, a2 + 60 = h2 and hence a2 + 60 = h2 . By the laws of algebra,
h2 − a2 = 60 and so (h + a)(h − a) = 60. Since h and a are positive integers with h > a,
h + a and h − a are also positive integers and h + a > h − a. Since h + a and h − a are
factors of 60, the possibilities are:

h + a = 60, h − a = 1; h + a = 20, h − a = 3; h + a = 12, h − a = 5;


h + a = 30, h − a = 2; h + a = 15, h − a = 4; h + a = 10, h − a = 6.

When h + a = √30 and h − a = 2 then h = 16 and a = 14. This gives a right triangle
with sides 14 and 60 and hypotenuse 16. When √ h + a = 10 and h − a = 6 then h = 8 and
a = 2. This gives a right triangle with sides 2 and 60 and hypotenuse 8. In the other pairs
of equations, the solutions
√ for h and a are not√integers. Thus there are two right triangles
meeting our criteria: 14, 60, and 16, and 2, 60 and 8.

15. The upper rectangle has width 12 and its height may be denoted by x, as shown in the
figure below. (All dimensions are in centimetres.) The lower rectangles obviously have

12
................................................................................
.... ...
... ....
x ..
...
...
.
x
.................................................................................
......................................... ...
....................................... ...
............................ ...
........................................ ...
............................ ....
.......................................
12 − x .........................................
.......................................
....
... 12 − x
............................ ...
........................................ ...
............................ .
..........................................................................................................

6 6

equal height and it is given that they have equal perimeter. This implies that they have
equal width as well, which is 6 in both cases. Since the upper and lower rectangles have the
same perimeter,
12 + x + 12 + x = (12 − x) + 6 + (12 − x) + 6
24 + 2x = 36 − 2x
4x = 12
x=3
This means that each lower rectangle has height 9. It follows that the area of the shaded
rectangle is 9 × 6 = 54.

16. Using the usual multiplication algorithm,

111, 111, 111 × 111, 111, 111 = 12, 345, 678, 987, 654, 321

The sum of the digits in this product is 1 + 2 + 3 + · · · + 8 + 9 + 8 + 7 + · · · + 1 = 81.


17. Let c be then number of chairs and s the number of stools in the room. For each
chair there are 6 legs: four for the chair and two for the child sitting on it. For each stool
there are 5 legs: three for the stool and two for the child sitting on it. The total number of
legs is 6c + 5s, and this is equal to 39. The possibilities for c are 1, 2, 3, 4, 5, and 6. (If
there were 7 or more chairs there would be 42 or more legs, which is too many.) The only
possibility that gives an integer number of stools is c = 4. In this case s = 3 and there are
6(4) + 3(5) = 24 + 15 = 39 legs in the room.

18. There are 5 triangles congruent to the one highlighted in Figure 18(a), five congruent
to the one in Figure 18(b), and so on. There are 25 isosceles triangles in all.
....... ..
.............. .
.....
.
................. .......... .................
.
................ .
.....
.
.................

.............. .. ....
...... .. .. ......
...... .... ..... ...........
.. ....
...... .. .. ......
...... .... ..... ..........
... .... ......
...
. ........ ............
.. ....
...... .. .. ......
...... .... ..... ...........
.. ....
...... .. .. ......
...... .... ..... ...........
... ....
.. . ....
...... .. .. .....
..... .... ..... ...........
......
. ...... ... .... ......
...... ... .... .. ...... . ... .... ......
...... ... .... ......
...... ... ....
...................... ......................................................
...... ...... ...... ...... ......
...... ... . ...... ... . .. ...... ... . ...... ... . ...... . ... . ..
.
............................................................................................. ...........
............................................................................................ .............................................................................................
.
..................................................... .
.............................................................................................
.. ...
............................................................................................
........... . . .......... ........... . .... ............... ........... . . .......... .......
........... . . . .......... ........... . .... ...............
....... ............
. . . . . . .
... ....
... ....... .... ... ... ... ....... .... ... ....... .... ... ..
. ... ....... .... ... ... ... ....... ....
.
... ...... .. . ... ....... ... ..
... ...... .. .
... ..... ... .......... ...
... ......... ... . ... .......... . ... .......... ... . ... ......... .... . ... .......... .
...... ........ ... ...... ........ ... ...... ........ ... ...... ........ ... ...... ........ ...
...
... .... ..................... .... ....
... ... .............. ... ...
...
... .... ..................... .... ....
... ... ............. ... ...
...
... .... ..................... .... ....
... ... ............. ... ...
...
.........
... .... ..................... .... ....
... ... .............. ... ...
...
.. ...
... .... ..................... .... ....
... ... ............. ... ...
... ... ....... ...... ... ... ... ... ....... ...... ... ... ... ... ....... ...... ... ... ... ... ....... ...... ... ...
... ... ... ....... .. ...... ... ...
............................
............. ..... .. .. ............. ..... .. .. ............. ..... .. .. ............. ..... .. .. ............. ..... .. ..
.......................................................... .......................................................... .......................................................... .......................................................... ..........................................................

Fig. 18(a) Fig. 18(b) Fig. 18(c) Fig. 18(d) Fig. 18(e)

19. Let M and S be the current ages of Marco and Shawana. Since Marco was three times
older than Shawana eighteen years ago, M − 18 = 3(S − 18). Since Marco is now twice as
old as Shawana, M = 2S. Then, by substitution,

2S − 18 = 3(S − 18); 2S − 18 = 3S − 54; 36 = S.

20. Let a, b, and c be the dimensions of the cuboid as shown. Since XZ is perpendicular
to ZA, we have (XZ)2 + (ZA)2 = (XA)2 by the Pythagorean theorem. Furthermore,

X b
a..............................•.................................................................................................................................................
... ..
. ........ ....
..........
........ . ... b .......... ..
..............................................................................................................................................•............ a ...
..
.
... . .. ... . ... ...
.. .
...
... ... ... .... Y ...... ..
..
... . . .. ...
... ... .. ... .
.
.
. ...
... .. .. .
. .. c
... .. .
. ...
... .
. . ..... .
. ..
.
.
. .
. ..
... .
.. . .
. ...
... .... .
c ... .
. .
.
.
.. c .
.
.
.
.
..
...
... ... . ...
... .
.
. ...
... . .
. ..
... .. ..
. ..
... .
. ..
... .. ..
. ...
... .
... ... ...... ....... ....... ....... ....... ........... ......... ....... ....... ....... ......
.
.
.
... . ..
... . .. ...
...
...
. .... ... ......
.. ..
...... ....... ..
.... ..........
.. . ..........
................................................................................................................................................. a
Z
• •
b A

(XZ)2 = a2 + c2 and (ZA)2 = b2 . Thus (XA)2 = a2 + b2 + c2 . Now note that a2 + b2 =


(XY )2 = 64, a2 + c2 = (XZ)2 = 55, and b2 + c2 = (Y Z)2 = 81. Then, adding all three
2 2 2 2 2 2
√ 2a + 2b + 2c = 64 + 55 + 81 = 200. Thus a + b + c = 100 and it follows that
equations,
XA = 100 = 10.
21. Since b = 6 the number in question has the form a6cd. Since the number is a multiple
both of 4 and 5, it must be a multiple of 20. Thus it must have the form a600, a620, a640,
a660, or a680. We consider each possibility in turn. Since a is the double of c, the number in
the first case would be 0600. However, this is not a four-digit number and must be discarded.
The number in the second case would be 4620. This is also a 3-multiple and so it satisfies
all stated criteria. The number in the third case would be 8640. This is also 3-multiple and
it also satisfies all stated criteria. The number in the last two cases is impossible because
no single digit a can be the double of 6 or 8. Then there are two numbers that satisfy all
stated criteria: 4620 and 8640.

22. We may cross-multiply the given expression as follows:

2x − y 2 x 5
= ; 3(2x − y) = 2(x + y); 6x − 3y = 2x + 2y; 4x = 5y; = .
x+y 3 y 4

23. Let 6 ODA = x◦ . Note that OA = OB = OC = OD because OA, OB, OC, and OD
are all radial segments. Then the triangles OAB, OBC, OCD, and ODA are all isosceles.

............................
................. .........
......... .......
....... .......
A• . . .. . ... .....
..............................
......
.....
.....
..
.. .. ..... ............. .....
. .. ............
.. ... ... ...... ... .
............. ....
...
.. .. ..... .
.
. . . .
. ..... ◦ ............. ...
..
. .
.
. ◦ 35 ..... ....
..............
. ............. .....
.

...
...
. x .
.
.
.
.
. ....
......
.
. .
◦ .
•B
............
........
... ...
.....
35 .......... .. ..
.. ..
..... ......... ..... .....
.... .....
.....
.... ..
..... .........
. .. .
....
. . .....
..... ..................
........ ◦ ..... ....
...
..
.... • ... .
.....
.
40 .... ..
.
... .
...
...
.
..... . ..
. ......... .....
O ..... ...
.
.
... .. .. .
. ..... ... ... ..
.
◦ ............... ...
...
. .
... ...
... ... x
... .. ............
...
...
...
...
...
...
...
...... ....... ... .
. .
...
.......... ◦ .. ... ..
D • .............55
... ........
..... ........ 40
...
...
...
◦ ......
.. ..
....
..
...
..... ........ ◦ ... .... .
.....
.
.....
......
........ 55
........ ... ...
.....
.....
......
.......
........
........ .... ....
. .
.....
. ......
......... . . ...
............... .........................................

.................

Thus 6 OAB = 6 OBA = 35◦ , 6 OBC = 6 OCB = 40◦ , 6 OCD = 6 ODC = 55◦ , and
6 ODA = 6 OAD = x◦ . Since the sum of the angles in any quadrilateral is 360◦ ,

2(35) + 2(40) + 2(55) + 2x = 360


260 + 2x = 360
2x = 100
x = 50

That is, 6 ODA = 50◦ .


24. We first determine the number of positive integers less than 1000 which are in fact
multiples of 5 or 7 (or both). The 5-multiples less than 1000 are 5, 10, 15, 20, . . ., 995,
and there are 199 such numbers in all. The 7-multiples less than 1000 are 7, 14, 21, 28, . . .,
994, and there are 142 such numbers in all. However, some numbers are multiples both of
5 and 7, and these are the multiples of 35. The 35-multiples less than 1000 are 35, 70, 105,
140, . . ., 980, and there are 28 such numbers in all. Then the number of positive integers
less than 1000 which are multiples of 5 or 7 (or both) is 199 + 142 − 28 = 313. Then the
number which are not multiples of 5 or 7 is 999 − 313 = 696.

25. Let
S1 = {1, 4, 7, 10, 13, 16, 19, 22, 25},
S2 = {2, 5, 8, 11, 14, 17, 20, 23},
S3 = {3, 6, 9, 12, 15, 18, 21, 24}.
Then S1 consists of all the numbers in S with remainder 1 when divided by 3, S2 consists
of all the numbers with remainder 2, and S3 consists of all the numbers divisible by 3. Note
that the sum of any two numbers in S1 is not divisible by 3. Similarly, the sum of any two
numbers in S2 is not divisible by 3. However, the sum of any number in S1 and any number
in S2 is divisible by 3. Suppose T is a set with the greatest number of elements that can be
chosen from S so that the sum of any two elements is not divisible by 3. Then T can contain
all the elements in S1 or all the elements in S2 , but it cannot contain elements from both
subsets. Furthermore, suppose T contains all the elements in S1 . Then it can also contain
one element from S3 . Similarly, if T contains all the elements in S2 it may also contain
one element from S3 . However, T cannot contain two elements from S3 . For example if
T contains 6 and 15 then it contains 6 + 15 = 21, and this is a multiple of 3. Then the
possibilities are that:
a) T contains all elements in S1 and one element from S3 , or
b) T contains all elements in S2 and one element from S3 .
Since (a) gives a set with more elements, T contains 10 elements. For example, we could
have T = {1, 3, 4, 7, 10, 13, 16, 19, 22, 25}
THE 2010 JAMAICAN MATHEMATICAL OLYMPIAD

Presented by The University of the West Indies


In Collaboration with Sterling Asset Management Ltd

Qualifying Round
Test for Grades 9, 10, and 11

NAME:

GRADE:

SCHOOL:

PRINCIPAL:

DATE OF BIRTH:

ADDRESS:

PHONE:

E-MAIL:

EXAMINATION QUESTIONS

Note: The natural numbers are the numbers {1, 2, 3, . . .} These are also called the
counting numbers or positive integers.

1) If four-fifths of a number is 60, what is the number?


(a) 32 (b) 48 (c) 75 (d) 84 (e) 90

2) How many triangles may be found in the diagram below?


..............................
....... ...... ......
... .. ... .. ... ..
...................................................................
.
... ... ... ..
... ... ... ... ... ... ... ...
... ... ... ... ... ... ... ....
.............................................................................

(a) 12 (b) 13 (c) 14 (d) 15 (e) 16


3) Consider the sequence: doh, re, mi, fah, sol, la, ti, doh, re, mi, fah, sol, la, ti, doh, re,
mi, . . .. What is the 2010th term of this sequence?
(a) doh (b) re (c) mi (d) sol (e) ti

4) A certain farmer raises chickens and rabbits. If she has 25 animals and they have a
total of 84 legs, how many chickens are on the farm?
(a) 5 (b) 8 (c) 17 (d) 25 (e) 62

5) In the diagram below, ABC is a triangle and P , Q, and R divide AC into 4 equal
segments. If the area of 4BQC is 24, what is the area of 4BAR?
B
..
.........
...... .....

.
.......... ...
..... ...
...... ...
..........
. ...
...
........ ...
......
. ..
.........................................................................................
• • • • •
A P Q R C

(a) 12 (b) 16 (c) 32 (d) 36 (e) 48

6) What is the last digit in the number 32010 ?


(a) 1 (b) 3 (c) 5 (d) 7 (e) 9

7) Suppose N is a natural number such that N 2 < 3, 456 and (N + 1)2 > 3, 456. What
is N ?
(a) 58 (b) 59 (c) 345 (d) 1728 (e) 3455

8) In a certain town half of the people are men, one-third of the men play dominoes, and
one fourth of the male domino players enjoy golf. If 1,200 people live in the town,
how many are men who play dominoes and enjoy golf?
(a) 20 (b) 50 (c) 300 (d) 400 (e) 600

9) In the diagram below, ABC is a right triangle and DB = DC. If 6 DAC = 25◦ , what
is 6 BDC?

B
...........

.
..............
D ........... ..
.............................
• ...
.
........ ...........
....
............ ............. ...
.. ..
..........................................................................................
.
A• . .
•C
(a) 25◦ (b) 40◦ (c) 45◦ (d) 50◦ (e) 60◦
10) How many three-digit numbers are divisible by both 3 and 11?
(a) 25 (b) 26 (c) 27 (d) 33 (e) 105

11) If a > b > 0 and a2 + 6b2 = 7ab, what is a/b?



(a) 1 (b) 6 (c) 7 (d) 13 (e) 42

12) How many rectangles may be found in the diagram below?


.............................................................................................
... . ...
.. .................................................
.. . . . ...
......................................................................................................
.. .. ... .. ...
.. .. . .
... ............................................... ....
.. ... ..
.........................................................................................

(a) 8 (b) 9 (c) 16 (d) 17 (e) 18

13) What is the sum of the digits in the number 102010 − 2010?
(a) 9 (b) 2009 (c) 18,007 (d) 18,079 (e) 18,086

14) The figure below consists of four congruent rectangles. The length of each rectangle
is three times its width. The area of each rectangle is 12 cm2 . What is the perimeter
of the figure?
...........
... ....
.. ..
... ...
.. ...
.. ..................................
.. ... ..
..
. .
..............................................................................
.. ... ..
.................................. ....
... ..
... ...
.. ..
.. ...
...........

(a) 6 (b) 12 (c) 16 (d) 48 (e) 64

15) How many digits are in the number (999, 999, 999, 876)2 − (124)2?
(a) 12 (b) 24 (c) 25 (d) 144 (e) 999,876

16) How many natural numbers less than 75 have exactly 4 divisors? (Note: 1 and n are
always divisors of n.)
(a) 12 (b) 15 (c) 18 (d) 19 (e) 22
17) Suppose ABC is a triangle with 6 ABC = 90◦ , and let D be the point on AC such
that BD ⊥ AC. If AC = 13 and AB = 12, what is BD?
........
B
........ .........
........ ... ..
......
......... .. ....
...... .. ..
............... ... .....
........ .
..........................................................................................
.
.
A C
D

(a) 4 (b) 56/12 (c) 60/13 (d) 5 (e) 13/2

18) In how many ways can we choose natural numbers a and b so that their sum is less
than 50 and the equation x2 + ax + b has exactly one real root?
(a) 0
(b) 3
(c) 5
(d) 6
(e) infinitely many ways

19) Suppose the sequence a1 , a2 , a3 , a4 , . . . of natural numbers has the property that
a1 < a2 and an+2 = an + an+1 for all n ≥ 1. If a6 = 46, what is a3 ?
(a) 2 (b) 8 (c) 10 (d) 18 (e) 23

20) In the diagram below, 4ABC is inscribed in a circle with centre O. If 6 OAB = 40◦
and 6 OBC = 30◦ , what is 6 OCA?
........
A
...........................
.....
..... • .
.............
..... ... .....
.... .
..
....... .... .....
... ... ... ...
..... ... ..
...
.. ..... ... ..
.....
... ........
..........
•O ... ...
... ...
C• .............................. . .
... ......................... ........
....
.....
...... ..
•B
..
......
...
............ .................
......

(a) 10◦ (b) 15◦ (c) 20◦ (d) 25◦ (e) 30◦

21) If 52010 is divided by 3, what will the remainder be?


(a) 0 (b) 1 (c) 2 (d) 3 (e) 4

22) A total of 4 packages must be delivered to a building with 5 apartments, called A, B,


C, D, and E. If no apartment is receiving more than two packages, in how many ways
can they be delivered?
(a) 35 (b) 45 (c) 60 (d) 180 (e) 200
23) A regular hexagon is inscribed in one circle and circumscribed about another. If the
inner circle has area A1 and the outer circle has area A2 , what is A2 /A1 ?
......
............ ............
..................................................................
............... .............
.. ............ .... ...
... .... .... ..
.........
............ ........
........ .. ....
........ . .
.
.......
. . .. ............
... ... . .
... ....
.... .....
. .
.... ......
............... ............
...........................................................
............ ............
.....

√ √ √
(a) 3/4 (b) 3/2 (c) 2/ 3 (d) 4/3 (e) 3

24) Suppose N is a natural number such that 1 + 2 + 3 + · · · + N < 1, 500 but 1 + 2 + 3 +


· · · + N + (N + 1) > 1, 500. What is N ?
(a) 27 (b) 54 (c) 55 (d) 63 (e) 1,500

25) Suppose S1 and S2 are squares with areas A1 and A2 and perimeters P1 and P2 ,
respectively. If A1 /A2 = 5, what is P1 /P2 ?
√ √
(a) 1/5 (b) 5 (c) 5 (d) 5 5 (e) 25

26) In the following multiplication, different letters represent different digits. What is the
value of E?
A B
× 5 D
......................................
A B
B E 5
...................................................
B A 2 B

(a) 2 (b) 3 (c) 4 (d) 5 (e) 6

27) Suppose a and b are real numbers and a2 + b2 = 10. What is the largest possible value
of 2a2 + 3b2 ?
(a) 2 (b) 3 (c) 6 (d) 20 (e) 30
28) Circles of radius 2, 6, and 4 are constructed with centres P , Q, and R, respectively.
Assume that P , Q, and R lie in a straight line and the circles are tangent. Let SU
be the line tangent to the first and third circles. Let T be the point on SU such that
QT ⊥ SU . What is the length of QT ?
.................................
...... ..... U
..... ..........
..... T ....
S ...............................................................................................
.
.
• •
.......................................
.....
...
..

.....................................
...
......
....
...
..
............................................................................................................................................................
• • •
... .. .
.......P....... ... . . . .
....
. ....
..... .. Q .
.. ... R ..... .
... ... ........... ..
.... ... ......................
.....
...... ..
........
......... .........
................

(a) 5/2 (b) 26/9 (c) 3 (d) 11/3 (e) 4

29) How many ordered pairs (m, n) of natural numbers satisfy the relation m + n < 20?
(a) 20 (b) 60 (c) 171 (d) 190 (e) 400

30) A six-digit number is called curious if its digits are 1, 2, 3, 4, 5, and 6 (in some order)
and its first k digits are divisible by k for k = 1, 2, 3, . . ., 6. So, if the number is
d1 d2 d3 d4 d5 d6 then d1 must be divisible by 1, d1 d2 must be divisible by 2, d1 d2 d3 must
be divisible by 3, and so on. How many six-digit numbers are curious?
(a) 0 (b) 2 (c) 3 (d) 6 (e) 16

END OF QUESTIONS

Mail completed question paper to:

Mathematical Olympiad
P.O. Box 94
Mona Post Office
Kingston 7

In order to qualify, all entries must be postmarked no later than February 15, 2010. To be
guaranteed consideration, all entries must be received by February 22, 2010.

All high school students in Jamaica are eligible to become a National Mathematics Cham-
pion at their grade level. However, only students who were born on January 1, 1994, or
thereafter are eligible to be named to the Jamaican Mathematical Olympiad Team.

Please visit http://myspot.mona.uwi.edu/mathematics/ (follow the links below the group


picture) for more information and the latest updates.

Extra copies of this question paper may be downloaded from the Olympiad website
http://myspot.mona.uwi.edu/mathematics/ (follow the links below the group picture).
THE 2010 JAMAICAN MATHEMATICAL OLYMPIAD

Presented by The University of the West Indies


In Collaboration with Sterling Asset Management Ltd

Qualifying Round
Solutions for Grades 9, 10, and 11

1) Let x be the number. Then


4 4x 60
x = 60; = ; 4x = 300; x = 75
5 5 1
The number is 75.

2) There are 12 small triangles in this figure. In addition there are 4 larger triangles, with each
one made up of four smaller triangles as shown below.
............................................................ . . ..........................
...... ......
......................................... ......................................... .........................................

...............................................................................................................................
..... ..... .....
... ... ... ... ... ...
.. ..
... ... ... ... ... ...
.. ..
... ..
... ... ... ... ... ...
.. ..

. . . .
.
..
.. . ......
.. ........ ........ ....
.................................................................... .
.. .. ........ ........ ....
.. . ......
...................................................................... .
.. .. ........ ........ ....
... ..
......................................................................

.................................................................................................................................
. . .
. . . . . .
... .. ... .. ... .. ... ..
.
.......................
.. ... .. ... .. ... .. ...
. .
.................................................................................
.
... .. ... .. ... .. ... ..
.. ... .. ... .. ... .. ...
. .
.......................
.
...............................................................................
.
....
... .. ... .. ... .. ... ..
.. ... .. ... .. ... .. ...
. . .
...............................................................................

There are 16 triangles in all.

3) This sequence repeats every 7 terms. Then the 7th, 14th, 21st, 28th, 35th, and so on, terms
are all ti. This continues up to the 2009th term, which is also ti. Then the 2010th term is doh.

4) Let c be the number of chickens and r the number of rabbits on the farm. Then c + r = 25.
Since each chicken has 2 legs and each rabbit has 4 legs, 2c + 4r = 84. Multiplying the first
equation by 2 gives the system 
2c + 2r = 50
2c + 4r = 84
Subtracting the first equation from the second, 2r = 34 and so r = 17. Then c + 17 = 25 and
so c = 8. There are 8 chickens on the farm.

5) Since P , Q, and R divide AC into 4 equal parts, 4BAP , 4BP Q, 4BQR, and 4BRC have
B
• ...
................
............ .....
.......... .. ... ...
................... .... .... ......
... .. . . ..
...... .... .. .... .....
...... ..... .... ...
...... ..... ..
... ...
.
.......... ........ ...
... ...
..
.. . .
• ........................................................................................................
• • • •
A P Q R C
equal areas. We will write a(BAP ) = a(BP Q) = a(BQR) = a(BRC) in this case. (So,
a(BAP ) stands for the area of 4BAP , and so on.) Then

a(BQR) + a(BRC) = a(BQC); 2 × a(BQR) = 24; a(BQR) = 12.

Then we have a(BAR) = a(BAP ) + a(BP Q) + a(BQR) = 12 + 12 + 12 = 36.


6) The last digit (in fact, the only digit) of 31 is 3; the last digit of 32 is 9; the last digit of 33 is
7; and the last digit of 34 is 1. Continuing, the last digits of 35 , 36 , 37 , 38 , 39 , . . ., are 3, 9,
7, 1, 3, . . .. In fact, the last digits of the powers of 3 form the sequence 3, 9, 7, 1, 3, 9, 7, 1,
3, 9, . . .. This repeats every 4 terms and continues forever. Note that the 4th, 8th, 12th, and
so on, terms are all 1, and this means that eventually the 2008th term is 1. Then the 2009th
term is 3 and the 2010th term is 9. Therefore, the last digit of 32010 is 9.

7) By trial and eror we see that 582 = 3, 364 and 592 = 3, 481. Then 582 < 3, 456 < 592 . It
follows that N = 58.

8) The number of men in the town is 12 × 1, 200 = 600. The number of men who play dominoes
is 13 × 600 = 200. The number of male dominoe players who enjoy golf is 14 × 200 = 50. There
are 50 men who play dominoes and enjoy golf.

9) The sum of the angles in any triangle is 180◦ . Since 6 BAC = 6 DAC = 25◦ ,
25◦ + 90◦ + 6 CBA = 180◦ ; 115◦ + 6 CBA = 180◦ ; 6 CBA = 65◦
This means that 6 CBD = 65◦ as well. Since 4DBC is isosceles, we also have 6 BCD = 65◦ .
Furthermore, the sum of the angles in any triangle is 180◦ . Then
6 BDC + 65◦ + 65◦ = 180◦ ; 6 BDC + 130◦ = 180◦ ; 6 BDC = 50◦

10) If a number is divisible by 3 and 11 then it is divisible by 33. In other words, it is a multiple
of 33. The smallest multiple of 33 which has three digits is 132. This is 4 × 33. The next
multiples of 33 are 165, 198, 231, and so on. These are 5 × 33, 6 × 33, 7 × 33, and so on.
The highest multiple of 33 which less than 1000 is 990. This is 30 × 33. Then the number of
three-digit numbers divisible by both 3 and 11 is the same as the number of numbers in the
set {4, 5, 6, 7, . . . , 30}. This is 27.

11) If a2 + 6b2 = 7ab then a2 − 7ab + 6b2 = 0. Thus (a − b)(a − 6b) = 0. Then, possibly, a − b = 0
and so a = b. Otherwise, a − 6b = 0 and so a = 6b. Since a and b re positve and a > b, we
must have a = 6b. Then
a 6b
= =6
b b

12) First, there are 4 small rectangles as illustrated below. Also, these rectangles can be paired
..................................................................................................... ..................................................................................................... ................................................................................................. .................................................................................................
... ... ..... ... ... .....
.................................... ...................................
... .. ... ... .. ... . .
... ................................................... .. ... ................................................... .. ... ..................................................... ... ... ..................................................... ...
... .... . . .. ... .... . . .. ... .. ... ... .. ...
.... .. .... ..
..........................................................................................................
.... ... ... .. ...
..........................................................................................................
.... ... ... .. ...
...
....................
... .
.........................................................................................................
. ..
....
................... ... .
....................................................................................................
... .....
...
...
...
... ....
..................................................
....
... ....
...
.
...
...
...
... ...
..................................................
....
... ....
...
.
...
...
...
..
................
...
..................................................
....
..
. ....
...
...
...
...
................
... ....
..................................................
....
.
...
...
................................................................................................. ................................................................................................. ... ..
............................................................................................
... ..
............................................................................................

to make larger rectangles in 4 different ways. Finally, the four small rectangles together make a
single larger rectangle. Next, there are 4 larger rectangles as shown below. Also, these rectangles
............................................................................................................................................ ..................................
.................................................................................................. ................................................................................................. .................................................................................................
........ ............................................................. ...... ..
.. ...
. ... ... ... ... ... ... ...

...................................
.... . ... ... .
..................................................... .... ... .
..................................................... ....
...........................................................
....................................................................................................................................................... ...
... ... .. ...
.......................................................................................................
...
.
...
... ... ..
..................................
. .
.
.
............................................................................................................
...
.
...
..................................
.. ... ..
. .
.
.
........................................................................................................
...
.
.... .... ....
.. ... ... ... .. ... .... ... .... ... ... ...
.. ...
.
.... ....
.. . .... ....

................................... ...................................
.... ...................................................... ...
... ... ................................................... ... ... ................................................... ... ... ................................................... ...
... ... .. ... ... ... ... ... ... ... ... ...
.. . .. .. . ... ... . ... ... .
................................................................................................ ............................................................................................. ............................................................................................. .............................................................................................

can be paired to make larger rectangles in 4 different ways. Finally, there is also the outermost
rectangle. A total of 4 + 4 + 1 + 4 + 4 + 1 = 18 rectangles may be found in the diagram.
13) In its decimal form, the number 102010 consists of a 1 followed by a 0 written 2010 times.
Subtracting 2010 from this gives the number represented by a 9 written 2006 times followed
by 7990. The sum of the digits in this number is 2006(9) + 7 + 9 + 9 + 0 = 18, 079.

14) Let x be the width of a rectangle and 3x be its length. Since the area of each rectangle
is 12 cm2 , we have 3x2 = 12. Then x2 = 4 and so x = 2. It follows that segments in the figure
have the lengths shown. The perimeter of the figure is 48.
2
............
..... ....
.. ...
6 ...... ..... 4 6
3x ... .....................................
.....................................
.. ... 4 ..... ..... ..
... 2
x ...
....................................
x ...................................................................................
... .. ..
2 ...................................... ... 4
3x 6
... ...
.. ..
4 ..... ...... 6
..............
2

15) Since (999, 999, 999, 876)2 − (124)2 is the difference of two squares, it may be factored as

(999, 999, 999, 876 + 124)(999, 999, 999, 876 − 124) = (1, 000, 000, 000, 000)(999, 999, 999, 752)

The number 999,999,999,752 obviously has 12 digits. If it is multiplied by 1,000,000,000,000


then the effect will be to add 12 zeroes to the end of this number. Therefore, the total number
of digits in (999, 999, 999, 876)2 − (124)2 is 24.

16) Since the only divisor of 1 is 1, it does not have exactly 4 divisors. Suppose now that n is
a natural number and consider its prime factorization. If this factorization contains three
distinct primes, then n has more than 4 divisors. To see this, suppose p, q, and r are prime
factors of n. Then 1, p, q, r, and pqr are all divisors of n. So, if n has exactly 4 divisors it
must have only 1 or 2 primes in its factorization. Suppose n has two primes, p and q, in its
factorization. Let p represent the smaller one and q the larger one. Then 1, p, q, and pq are
factors of n. If n has exactly 4 factors then these are the only ones. Then pq = n. In this
case, it is possible that p = 2, 3, 5, or 7. (Otherwise, if p ≥ 11 then q ≤ 75/11. This means
that q ≤ 7 and so q < p.) If p = 2 then q could be 3, 5, 7, 11, 13, 17, 23, 29, 31, or 37. There
are 10 choices in this case. If p = 3 then q could be 5, 7, 11, 13, 17, or 23. There are 6 choices
in this case. If p = 5 then q could be 7, 11, or 13. There are 3 choices in this case. If p = 7
then there are no possible choices available. (The next prime is 11 and 7 × 11 is too large.)
Then the total number of choices for primes p and q is 10 + 6 + 3 = 19.
Suppose now that n has only one prime, p, in its factorization. In order to have exactly 4
divisors they would have to be 1, p, p2 , and p3 . Also, we would have p3 = n in this case. The
possiblities are p = 2, n = 8, or p = 3, n = 27, or p = 4, n = 64. There are 3 choices in this
case. Adding these to the case we analyzed earlier, there are 22 numbers less than 75 which
have exactly 4 divisors.
17) By the Pythagorean theorem, (AB)2 + (BC)2 = (AC)2 . Substituting AC = 13 and AB = 12,

122 + (BC)2 = 132 ; 144 + (BC)2 = 169; (BC)2 = 25; BC = 5.

Also 4BDC is a right triangle and 4ABC and 4BDC have a common angle at C. Since the
sum of the angles of any triangle is 180◦ , it follows that 6 DBC = 6 BAC. In this case, the
corresponding angles in 4ABC and 4BDC are equal. Therefore, these triangles are similar.
Then
BD AB BD 12 60
= ; = ; 13BD = 60; BD =
BC AC 5 13 13

18) The equation x2 + ax + b has exactly one real root when a2 − 4b = 0. The values for a and b
that satisfy this relation are a = 2, b = 1; a = 4, b = 4; a = 6, b = 9; a = 8, b = 16; a = 10,
b = 25; a = 12, b = 36. Any other choices for a and b will have the property that a + b > 50.
Then we can choose a and b in exactly 6 ways.

19) Let a1 = m and a2 = n. Then

a3 = a1 + a2 = m + n
a4 = a2 + a3 = m + 2n
a5 = a3 + a4 = 2m + 3n
a6 = a4 + a5 = 3m + 5n

Therefore, m and n are natural numbers such that m < n and 3m + 5n = 46. In this case,
5n = 46 − 3m and so 46 − 3m must be a 5-multiple. The only m-values for which 46 − 3m is
a 5-multiple are m = 2, m = 7, and m = 12. Then there are three candidates for m and n:
m = 2 and n = 8; m = 7 and n = 5; m = 12 and n = 2. Since we must also have m < n, the
only possible solution is m = 2 and n = 8. In this case, a3 = m + 2n = 2 + 2(8) = 2 + 16 = 18.

20) Since OA, OB, and OC are radial segments, they are equal in length. Then 4AOB, 4BOC,

..........
..................... A
.....
...... • ..
...............
........ .. ....
. .... ..... .. .. ...
...
.
........ .... .... ....
.
... .... . ... ...
... ..... .. ... ...
..... .... ... ..
... .....
... ......... ........................
... ....................
•O .
......
... ...
.. ..
....... .... ....
C•
.......................... ..
... ......................... ............
...
.....
......
.. .. . . ..
•B
.. ..........
....
..
........ . .... . .....
.......................

and 4COA are isosceles triangles. This means that 6 OAB = 6 OBA, 6 OBC = 6 OCB, and
6 OCA = 6 OAC. Also, the sum of the angles in 4ABC is 180◦ . Then

6 OAB + 6 OBA + 6 OBC + 6 OCB + 6 OCA + 6 OAC = 180◦


26 OAB + 26 OBC + 26 OCA = 180◦
6 OAB + 6 OBC + 6 OCA = 90◦
40◦ + 30◦ + 6 OCA = 90◦
70◦ + 6 OCA = 90◦
6 OCA = 20◦
21) By the binomial formula,
52010 = (6 − 1)2010 = 62010 + c1 62009 (−1) + c2 62008 (−1)2 + c3 62007 (−1)3 + · · ·
+ c2008 62 (−1)2008 + c2009 6(−1)2009 + (−1)2010

Here, each ck represents the kth binomial coefficient 2010!/ k!(2010 − k)! . Also, each term
except the last one, (−1)2010 , is a multiple of 6. Then each term except the last one is divisible
by 3. Then the remainder when 52010 is divided by 3 will be (−1)2010 = 1.

22) We must consider three possibilities: two apartments receive two samples each; one apartment
receives two samples and two receive one each; or four apartments receive one sample each. In
the first case, there are 10 ways of choosing the two apartments to receive the samples: AB,
AC, AD, AE, BC, BD, BE, CD, CE, or DE. In the second case, there are 5 ways to choose
one apartment to receive two samples. For each such choice, there are six ways to choose
two more apartments to receive one sample each. Then there are a total of 5 × 6 = 30 ways
to distribute the samples in this case. In the third case, there are 5 ways to choose the four
apartments to receive the samples: ABCD, ABCE, ABDE, ACDE, or BCDE. Then there are
10 + 30 + 5 = 45 ways of delivering the samples.

23) Let r1 be the radius of the inner circle and r2 the radius of the outer circle. Then A1 = πr12

A ............................. B A................................C B
............................................. .....................................
...
................ .. ......................... ... ..
............... ..... ..... ...... ... ...
...
... ...
.
... .... ... .. ... ... ... ..
.. .. ... ... ...
... ....
............
r . .
r
2 ............. 1 ............... r ...
2 ...... r ...
... 1 ....
.
.
.
..... ... ..... ... ..... ... .. .
......... ........ ... .. ...
........ .......... ... .... ....
... ....
... ....
O ..
... ..
. ... ... ...
... .. ..
... ...... ... ... ... ... ...
.............. .
...
................. .........
....... .......... ......... ...... ......
.............. ............................
....................
O

and A2 = πr22 . Also, as seen in the enlarged diagram, 6 AOB is 60◦ . This is because this angle
is one-sixth of a full circle and so 6 AOB = 360◦ /6 = 60◦ . Therefore, 6 AOC = 30◦ . Then
√ √
r1 3 3 r2 3r 2

= cos 30 = ; r1 = ; r12 = 2
r2 2 2 4
Finally,  
3r22
π
A2 πr 2 4 3πr22 3
= 12 = = = .
A1 πr2 πr22 4πr22 4

24) By trial and error, one may verify that 1+2+3+· · ·+54 = 1, 485 and 1+2+3+· · ·+54+55 =
1540. Then N = 54. Alternatively, there is a “fast formula” for finding a sum of the form
1 + 2 + 3 + · · · + n. Let S represent this sum. Then

S = 1 + 2 + 3 + ··· + n
S = n + (n − 1) + (n − 2) + · · · + 1
Adding equations, 2S = (n+1)+(n+1)+(n+1)+· · ·+(n+1), where the term n+1 occurs on
the right side n times. Then 2S = n(n+1) and so S = n(n+1)/2. In this problem, we still need
some amount of trial and error. However, one verifies that 1 + 2 + 3 + · · · + 54 = 54(54 + 1)/2 =
54(55)/2 = 1485. Similarly, 1 + 2 + 3 + · · · + 55 = 55(55 + 1)/2 = 55(56)/2 = 1540.
25) Let `1 and `2 be the side lengths of S1 and S2 , respectively. Then A1 = `21 and A2 = `22 . Also,
P1 = 4`1 and P2 = 4`2 . We have
A1 `21 √
= 5; = 5; `21 = 5`22 ; `1 = 5`2 .
A2 `22
(We choose the positive square root above because the length `1 cannot be negative.) Then

P1 4`1 `1 5`2 √
= = = = 5.
P2 4`2 `2 `2

26) The first step in this multiplication indicates that D × AB = AB. Then D = 1. Also, adding
the two partial answers indicates that A + 5 gives an answer ending in 2. Since A is a digit
from 0 to 9, the only possibility is A = 7. Then 7 + 5 = 12. After the 2 is recorded the tens
digit, 1, is carried to the hundreds place. Then 1 + E = 7 and so E = 6. To complete the
analysis, it is clear from the multiplication of the tens digits that B = 3.

27) Since squares of real numbers are always positive or 0, we have b2 ≥ 0. Similarly, a2 ≥ 0.
Since a2 = 10 − b2 we have 10 − b2 ≥ 0 and so 10 ≥ b2 . That is, b2 ≤ 10. Thus 0 ≤ b2 ≤ 10.
Since a2 + b2 = 10,
2a2 + 3b2 = 2a2 + 2b2 + b2 = 2(a2 + b2 ) + b2 = 2(10) + b2 = 20 + b2 .

Then 2a2 + 3b2 = 20 + b2 ≤ 30. Furthermore, 2a2 + 3b2 = 30 when a = 0 and b = ± 10.
Then the largest possible value for 2a2 + 3b2 is 30.

28) Let A be the point where the lines SU and P R meet. Since AU is tangent to the first and
..................................
....... ..... U ...............
..... .....
. ...... T . •
....................................
S ................................................................................................... ..... .............

................................. ..
• .. ... .. ... ...
............................. ..... .... .......... ... ..... ... ..
............................ . .
...............................................................................................................................................................................................................................................................................
• • • •
... . . .. .
.......P.............. Q ......
. ..
.
A .... ...
...
.. ....
.
. .....
R ...
..
.. ....... .
...
...
.....
.
... .....................
...... .....
......... ......
..........................

third circles, 6 ASP and 6 AU R are right angles. Then4ASP , 4AT Q, and 4AU R are right
triangles. Also, they all have the same angle at A. Since the sum of the angles in any triangle
is 180◦ , it follows that 6 AP S, 6 AQT , and 6 ARU are equal. Therefore, the triangles AP S,
AQT , and ARU are similar. Since similar triangles have proportional sides,
AP AR
=
SP UR
The circles at P , Q, and R have radii 2, 6, and 4, respectively. Then AR = AP +2+6+6+4 =
AP + 18. By substitution,
AP AP + 18
= ; 4AP = 2AP + 36; 2AP = 36; AP = 18.
2 4
To solve for T Q, note that AQ = AP + 2 + 6 = 18 + 2 + 6 = 26. Then
TQ SP TQ 2 TQ 1 26
= ; = ; = ; 9T Q = 26; TQ =
AQ AP 26 18 26 9 9
29) Note first that m may be any of the numbers 1, 2, 3, . . ., 18. (We cannot have m = 19 because
19 + 1 is not less than 20.) If m = 1 then n may be any one of 1, 2, 3, . . ., 18. There are 18
choices in all. If m = 2 then n may be any one of 1, 2, 3, . . ., 17. There are 17 choices in all.
If m = 3 then n may be any one of 1, 2, 3, . . ., 16. There are 16 choices in all. In this way,
when m = 4, 5, 6, . . ., 18, there are 14, 13, 12, . . ., 1 choices for n. Then the total number of
pairs (m, n) is 1 + 2 + 3 + · · · + 18 = 171.

30) Suppose the number is d1 d2 d3 d4 d5 d6 . Then the digits must be 1, 2, 3, 4, 5, and 6 in some
order. Also, we must have d1 d2 divisible by 2, d1 d2 d3 d4 divisible by 4, and d1 d2 d3 d4 d5 d6
divisible by 6. This means that d2 , d4 , and d6 must be even. Then the number has the form

O E O E O E,

Where O represents an odd digit and E represents an even digit. Also, d1 d2 d3 d4 d5 must be
divisible by 5. This means that its last digit must be 0 or 5. Since 0 is not possible, d5 = 5.
Then the number looks like
O E O E 5 E.
Note that the first and third digits are 1 and 3 in some order. If d2 = 4 then d1 d2 d3 must
be 143 or 341. However, neither is divisible by 3. If d2 = 6 then d1 d2 d3 must be 163 or 361.
However, neither is divisible by 3. Therefore d2 = 2, and the number has the form

O 2 O E 5 E.

If d4 = 4 then d1 d2 d3 d4 is either 1234 or 3214. However, neither one is divisible by 4.


Therefore, d4 = 6. By process of elimination, d6 = 4. Then the number has the form

O 2 O 6 5 4.

There are two possibilities remaining: 123,654 or 321,654. In fact, both are curious. Then
there are exactly 2 six-digit curious numbers.
THE 2010 JAMAICAN MATHEMATICAL OLYMPIAD

Presented by The University of the West Indies


In Collaboration with Sterling Asset Management Ltd

Qualifying Round
Test for Grades 7 and 8

NAME:

GRADE:

SCHOOL:

PRINCIPAL:

DATE OF BIRTH:

ADDRESS:

PHONE:

E-MAIL:

EXAMINATION QUESTIONS

Note: The natural numbers are the numbers {1, 2, 3, . . .} These are also called the
counting numbers or positive integers.

1) If 3/4 of a number is 48, what is the number?


(a) 12 (b) 16 (c) 36 (d) 64 (e) 100

2) How many rectangles may be found in the diagram below?


.........................................................................
.. ... ... ... ...
...
.. .... .... .... ....
.. ... ... ... ...
.. ... ... ... ...
... ... ... ... ...
.. ... ... ... ...
.. ... ... ... ...
... . . . .
.....................................................................

(a) 4 (b) 5 (c) 9 (d) 10 (e) 12


3) The average of three real numbers is 2. If two of the numbers are 5/3 and 7/4, what
is the third one?
(a) 6 (b) 9/16 (c) 31/12 (d) 9/5 (e) 2

4) In the diagram below, ABC is an isosceles triangle. Lines AC and BD are parallel.
If 6 ABC = 40◦ , what is 6 CBD?
B D
• •
.........................................................................................
... ...
... ...
.... .....
.
.. ...
... ...
.. .
.......................................................................................
• •
A C

(a) 40◦ (b) 60◦ (c) 65◦ (d) 70◦ (e) 140◦

5) In a certain community there are 5 houses. In each house there are 3 children, and
each child has 6 toys. How many toys are in the community?
(a) 6 (b) 14 (c) 18 (d) 30 (e) 90

6) What is the next number in the sequence 3, 4, 7, 12, 19, 28, , . . .?


(a) 29 (b) 36 (c) 39 (d) 45 (e) 100

7) In the multiplication below, different letters stand for different digits. What is B?
A A
×A
B C 9

(a) 1 (b) 3 (c) 5 (d) 7 (e) 9

8) In the diagram below, ABC is a triangle and P is the midpoint of BC. Also, Q
and R divide AC into three equal parts. If the area of 4ABC is 36, what is the area
of 4P RC?
B•..
............
.. .........
.........
...
...
.........
.........
P
.
.
.
.. • .........
.........
.........
..
. ...
........................................................................................................
A• • • •C
Q R

(a) 4 (b) 6 (c) 9 (d) 12 (e) 18

9) What is the last digit in the number 72010 ?


(a) 1 (b) 3 (c) 5 (d) 7 (e) 9
10) A number of straws are used to form a triangular grid in a manner illustrated below.
(Only a part of the grid is shown.) If the final grid has 12 straws on each side, how
many straws are required to make the grid?
....
... ...
..... .....
.......................
... ... ... ...
... ... ... ..
.
.................................................
.. .. .. ... .. ..
... .... .... ... .... ...
.........................................................

(a) 36 (b) 78 (c) 108 (d) 234 (e) 600

11) In the diagram below, ABCD is a parallelogram. Also, P is the midpoint of AB and
Q is the midpoint of BC. If the area of ABCD is 24, what is the area of 4DP Q?
P
A •..................................•
...............................•
.... B
...
... ...
... ...
... ...
...
... •Q
...
...
... ...
... .
..................................................................
D• •C

(a) 6 (b) 9 (c) 12 (d) 14 (e) 15

12) A student wishes to write down two distinct letters of the English alphabet in such a
way that they are in alphabetical order. In how many ways can this be done?
(a) 25 (b) 26 (c) 210 (d) 325 (e) 351

13) In the diagram below, ABDE is a rectangle. Also, BCD is an isosceles triangle and
AB = BC. If 6 BCD = 40◦ , what is 6 AEC?
A.........................................B
....
... ...............
... ...........
... ... ...........
.. .......
... ... ........... C
.. ... .....................
......................................................
E D

(a) 10◦ (b) 45◦ (c) 70◦ (d) 80◦ (e) 90◦

14) How many positive divisors does 2010 have? (Note: The divisors of a natural number n
include 1 and n itself.)
(a) 12 (b) 16 (c) 22 (d) 24 (e) 201
15) Suppose ABC is a triangle with 6 ABC = 90◦ , AC = 5, and BC = 3. If D is the
point on AC such that BD ⊥ AC, what is CD?

.......
B
...... ......
...... .... ......
....
....... ..... ......
...... .. ...
..... ... ....
........
. .. ....
...
... . ....
....
. .
... ....
..
............................................................................................
A C
D

(a) 1 (b) 9/5 (c) 2 (d) 4 (e) 8

16) A two-digit number is called reflexive if it is 27 more than the number obtained by
reversing its digits. For example, 41 is reflexive because it is 27 more than 14; 81 is not
reflexive because it is 63 more than 18. How many two-digit numbers are reflexive?
(a) 1 (b) 3 (c) 5 (d) 7 (e) 9

17) The figure below consists of four rectangles. The length of each rectangle is two times
its width and each one has perimeter 18. What is the perimeter of the figure?
.................
... ...
... ....
..
.. ....................................
.. ... ..
... ... ...
. . ..
...........................................................................
.. ... ..
... . .
....
....................................
.. ....
... ...
.. .
.................

(a) 18 (b) 36 (c) 48 (d) 60 (e) 72

18) Suppose a and b are real numbers (not necessarily positive). If a2 + b2 = 4, what is
the largest possible value of 2a2 − 3b2 ?
(a) 0 (b) 2 (c) 3 (d) 4 (e) 8

19 If A + 1 = B − 2 = C + 3 = D − 4 = E + 5, what is A + B + C + D + E?
(a) 5A (b) 5A + 1 (c) 5A + 2 (d) 5A + 3 (e) 5A + 4

20 How many three-digit numbers are odd, divisible by 3, and less than 456?
(a) 58 (b) 59 (c) 60 (d) 116 (e) 117
21) In the figure below, 6 C = 90◦ , AD = DB, DE ⊥ AB, AB = 20, and AC = 12. What
is the area of quadrilateral ADEC?
C .
.........
...... .....
E.............. ....
...
.
...... ....
.......... .... ...
..
..... .
. ...
...
.
...... .
. ....
...
.... .
.
. ...
..
..... .
. ....
...
.... .
. ...
...
.... .
.
. ...
....
... .
. ...
.... . ..
................................................................................................................................
... .
B D A

(a) 12 (b) 75/2 (c) 117/2 (d) 84 (e) 96

22) In how many ways can 18 be written as a sum of perfect squares?


(a) 1 (b) 8 (c) 9 (d) 10 (e) 12

23) How many natural numbers less than 1,000 are divisible by 4, 5, and 6?
(a) 3 (b) 8 (c) 16 (d) 19 (e) 36

24) Consider the following succession of figures:


? ? ? ? ? ? ?
? ? ? ? ?
? ? ? ? ? ? ? ? ? ?
? ? ? ? ? ···
? ? ? ? ? ? ? ? ? ?
? ? ? ? ?
? ? ? ? ? ? ?
How many stars will be in the 8th figure?
(a) 128 (b) 137 (c) 153 (d) 190 (e) 256

25) How many natural numbers have the property that their largest divisor, other than
themselves, is 35?
(a) 3 (b) 4 (c) 5 (d) 7 (e) 35

END OF QUESTIONS

Mail completed question paper to:

Mathematical Olympiad
P.O. Box 94
Mona Post Office
Kingston 7
In order to qualify, all entries must be postmarked no later than February 15, 2010. To be
guaranteed consideration, all entries must be received by February 22, 2010.

All high school students in Jamaica are eligible to become a National Mathematics Cham-
pion at their grade level. However, only students who were born on January 1, 1994, or
thereafter are eligible to be named to the Jamaican Mathematical Olympiad Team.

Please visit http://myspot.mona.uwi.edu/mathematics/ (follow the links below the group


picture) for more information and the latest updates.

Extra copies of this question paper may be downloaded from the Olympiad website
http://myspot.mona.uwi.edu/mathematics/ (follow the links below the group picture).
THE 2010 JAMAICAN MATHEMATICAL OLYMPIAD

Presented by The University of the West Indies


In Collaboration with Sterling Asset Management Ltd

Qualifying Round
Solutions for Grades 7 and 8

1) Let x be the number. Then


3 3x 48 192
x = 48; = ; 3x = 48(4) = 192; x= = 64
4 4 1 3
The number is 64.

2) There are 4 small rectangles in this diagram. There are 3 rectangles made up of two
smaller ones together, as shown below:
............................................................................................................................ ..........................
.................................................................................... ... ...
.........................
...........................................................................
...... ... ....
....... ...... ........ ..... ..... ..
.. ..
...
.....
...
.....
... ...
.....
...
.....
... ..
...
..
..
...
..
... .. ..
...... ..... ........ ...... ...... ...
.. .. ... ... .. ... ...
...
... ....
... .... .... .. ...
..
....... ...... ........ ..... ..... ..
.. ... ...
. ... ... .. .. .. ..
. ..
......................... .........................
.. ... ... ... ... ... ... ... ... ..
...................................................................................................... ...
.. ... ... ...
.........................................................................
... .
....
.
....
.
....
.
....
..................................................................
...
.

There are 2 rectangles made up of three smaller ones together, as shown below:
................................................................................................................................ ...................................
..........................................................................
...... ...... ..... ........ ..... ...
... ... ...
..
...
.. .....
.. ....
..
....... ...... ...... ........ ...... ..
.... .. ...
....
...
.... .. ...
....
..
...
....... ..... ..... ....... ..... .. ... .. ..
. .. ..

....................................
... ... ... ... ..
....................................................................................................................... .
....
.
....
.
....
.
....
.......................................................................
...
.

And there is the 1 outer rectangle. Then there are 4 + 3 + 2 + 1 = 10 rectangles in all.

3) Let x be the third number. Since the average of these numbers is 2,


5 7
+ +x 5 7 20 21 41 72 31
3 4 = 2; + + x = 6; + + x = 6; +x= ; x= .
3 3 4 12 12 12 12 12

4) First, 6 BAC = 6 BCA because 4ABC is isosceles. Since the sum of the angles in
any triangle is 180◦ ,
6 BAC + 6 BCA + 40◦ = 180◦

26 BCA + 40◦ = 180◦


26 BCA = 140◦
6 BCA = 70◦

Since the lines AC and BD are parallel, 6 CBD = 6 BCA. Therefore, 6 CBD = 70◦ .
5) There are 5 × 3 = 15 children in the community. Since each child has 6 toys, there
are 15 × 6 = 90 toys in the community.

6) The differences between terms are 1, 3, 5, 7, 9, . . .. In this case, the next difference
will be 11. Then the next term is 28 + 11 = 39.

7) According to the first step in this multiplication, A × A is a number ending in 9. The


only possibilities are A = 3 and A = 7. If A = 3 then the product is 3 × 33 = 99. But
this is not a three-digit number. Then the product must be 7 × 77 = 539. Therefore,
B = 5.

8) Since Q and R divide AC into three equal parts, the area of 4BRC is one-third of
the area of 4BAC. Then the area of 4BRC is 12. Since P divides BC into two
equal parts, the area of 4P RC is half the area of 4BRC. Then the area of 4P RC
is 6.
B•..
................
B•...
.. .............
.. ........................ .. ........................
... ...... .......... P ... ...... .......... P
... ...... ......... ... ...... .........
..
.
.
.. ....... •
....... .........
...
....... ................. ..
.
.
.. •
....... ..........
....... .... ..........
....... ... .........
.. .... ...... .. ...... .......
...........................................................................................................
• • •C ............................................................................................................
A A• • • •C
Q R Q R

9) The last digit (in fact, the only digit) of 71 is 7; the last digit of 72 is 9; the last digit
of 73 is 3; and the last digit of 74 is 1. Continuing, the last digits of 75 , 76 , 77 , 78 , 79 ,
. . ., are 7, 9, 3, 1, 7, . . .. In fact, the last digits of the powers of 7 form the sequence
7, 9, 3, 1, 7, 9, 3, 1, 7, 9, . . .. This repeats every 4 terms and continues forever. Note
that the 4th, 8th, 12th, and so on, terms are all 1, and this means that the 2008th
term is 1. Then the 2009th term is 7 and the 2010th term is 9. Therefore, the last
digit of 72010 is 9.

10) There will be 12 straws at the base of the grid, 11 straws in the row above it, 10
straws in the row above that one, and so on. Then the total number of straws on the
base or parallel to it is 12 + 11 + 10 + · · · + 1 = 78. There are also straws forming the
sides of the triangles. On one side, there will be 12 straws on the boundary. Parallel
to them, there will be lines with 11 straws, 10 straws, 9 straws, and so on, down to
one straw. Then there will be 12 + 11 + 10 + · · · + 1 = 78 straws on this boundary
or in a parallel direction. Finally, there will also be a total of 78 straws on the third
boundary or in lines parallel to it. Then there will be a total of 78 + 78 + 78 = 234
straws making the grid.
11) First, the area of 4DAB is half of the area of ABCD. Then the area of 4DAB is 12.
P
A •...................................•..............................•
...............
.... B
...
... . .......
... ... ....... .....
.. ....... ..
... ... ........
...
... ....
..
. .............
...
.......... ....
...
•Q
... ... ......
....... .................... ..
.....................................................................
D• •C
Also, since P is the midpoint of AB, the area of 4DAP is half the area of 4DAB.
Then the area of 4DAP is 6. By reasoning in the same way, one sees that the area
of 4DQC is 6 as well. Similarly, the area of 4P QB is half the area of 4P CB, and
this is half the area of 4ACB, and this is half the area of ABCD. Then the area of
4P QB is
1 1 1
× × × 24 = 3.
2 2 2
Finally, the area of 4DP Q is the area of ABCD minus the areas of 4DAP , 4DCQ,
and 4P QB. That is, the area of 4DP Q is 24 − (6 + 6 + 3) = 24 − 15 = 9.

12) The first letter can be any one from A to Y. (The first letter cannot be Z because there
is no letter after Z in the alphabet.) If the first letter is A, the second letter can be
any of B, C, D, . . . Z. There are 25 possibilities in all. If the first letter is B, the second
letter can be any of C, D, E, . . . Z. There are 24 possibilities in all. If the third letter
is C, there are 23 possibilities for the second letter. Continuing in this way, there are
22 possibilities after a D, 21 possibilities after an E, and so on, up to one possibility
for Y. The total number of pairs of letters is 25 + 24 + 23 + · · · + 2 + 1 = 325. (Note:
there is a “fast formula” for adding the numbers 1 + 2 + 3 + · · · + n. This states that
the sum is n(n + 1)/2. In our case, 1 + 2 + 3 + · · · + 25 = 25(25 + 1)/2 = 25(13) = 325.)

13) Since BCD is an isosceles triangle, 6 CBD = 6 CDB. Also the sum of three angles in
any triangle is 180◦ . Since 6 BCD = 40◦ it follows that 6 CBD = 6 CDB = 70◦ .
A.........................................B
....
... ...............
.. ... ...........
... ...........
.. ....
.. ... ............................................... C
... ..... .
...
...
. ......
................................................................................................
E D

Since 6 BDE = 90◦ we also have 6 EDC = 160◦ . Since the triangle EDC is isosceles,
6 DEC = 6 DCE. It follows that 6 CED = 10◦ . Since 6 AED = 90◦ we must have
6 AEC = 80◦ .

14) The prime factorization of 2010 is 2 × 3 × 5 × 67. To produce a divisor for 2010, we
have two choices for the number of 2’s which may be used (none or one), two choices
for the number of 3s (none or one), two choices for the number of 5s (none or one), and
two choices for the number of 67’s (none or one). There are a total of 2×2×2×2 = 16
possible divisors. They are: 1, 2, 3, 5, 6, 10, 15, 30, 67, 134, 201, 335, 402, 670, 1005,
and 2010.
15) First, 4ABC and 4BDC are right triangles and both have the angle at C in common.
Since the sum of the angles in any triangle is 180◦ , it follows that 6 DBC = 6 BAC.
Then 4ABC and 4BDC are similar. Since corresponding sides of similar triangles are
proportional, BC/AC = CD/BC. Since BC = 3 and AC = 5, we have 3/5 = CD/3.
By cross-multiplying, we have 5CD = 9 and so CD = 9/5.

16) Let ab be a two-digit number, where a and b are each one of the numbers 0, 1, 2, . . ., 9.
Then the value of ab is 10a+b. When the digits are reversed we obtain the number ba,
with a value of 10b+a. In order to be reflexive, we must have (10a+b)−(10b+a) = 27.
This means that 10a + b − 10b − a = 27 and so 9a − 9b = 27. In turn, this means
that 9(a − b) = 27 and so a − b = 3. The possibilities for a and b, respectively, are 3
and 0, 4 and 1, 5 and 2, 6 and 3, 7 and 4, 8 and 5, and 9 and 6. That is, the reflexive
numbers are 30, 41, 52, 63, 74, 85, and 96. There are 7 of them in all.

17) Let x be the width of a single rectangle and 2x be its length. Then its perimeter is
2x + x + 2x + x = 6x. Equating this to 18 gives x = 3. Then each small rectangle has
........3 ............
... ...
.. .3
6 .. . .... 6
2x .... .......................................
... ...
..................................... .. ... 3
.. ... 3 . ..
x ...
....................................
x ..........................................................................
... ... ... 3
3 .. ... ...
2x ................................... ...
. ...
6 3 ..... ....
6
...................
3

width 3 and length 6. It follows that the lengths of the segments in the boundary of
the figure are as shown. The perimeter of the figure is 48

18) First, since a2 + b2 = 4 we have a2 = 4 − b2 . Then

2a2 − 3b2 = 2(4 − b2 ) − 3b2 = 8 − 2b2 − 3b2 = 8 − 5b2 .

Also, b2 ≥ 0 because all perfect squares are positive or 0. Then 5b2 ≥ 0 and it follows
that 8 − 5b2 ≤ 8. On the other hand, if a = 2 and b = 0 then a2 + b2 = 22 + 02 = 4.
Also, 2a2 − 3b2 = 2(22 ) − 3(02 ) = 8 − 0 = 8. Then the greatest possible value for
2a2 − 3b2 is 8.

19 Since A + 1 = B − 2 we have B = A + 3. Since A + 1 = C + 3 we have C = A − 2. Since


A + 1 = D − 4 we have D = A + 5. Finally, since A + 1 = E + 5 we have E = A − 4.
Then

A + B + C + D + E = A + (A + 3) + (A − 2) + (A + 5) + (A − 4)
= 5A + (3 − 2 + 5 − 4)
= 5A + 2
20 The first few three-digit numbers which are odd and divisible by 3 are 105, 111, 117,
. . .. In general, if n is odd and divisible by 3 then the next such number is n + 6.
Since 105 is the first odd number divisible by 3, the others we seek are of the form
105 + 6k for some k. For these to be less than 456 we must have 105 + 6k ≤ 456 and
thus 6k ≤ 351. Since k is a natural number, the solution is k ≤ 58. Starting with 105
(when k = 0) and ending with 453 (when k = 58), there are 59 such numbers in total.

21) Since 4ABC is a right triangle, (AC)2 +(BC)2 = (AB)2 by the Pythagorean theorem.
Since AC = 12 and AB = 20,

122 + (BC)2 = 202 ; 144 + (BC)2 = 400; (BC)2 = 256; BC = 16.

Also, the area of 4ABC is 12 × height × width = 12 × 16 × 12 = 96. Second, 4EBD


is a right triangle and 4ABC and 4EBD have the angle at B in common. Since
the sum of the angles in any triangle is 180◦ , it follows that 6 BED = 6 BAC. Then
4ABC and 4EBD are similar. Since corresponding sides of similar triangles are
proportional, DE/BD = AC/BC. This means that

DE 12 3 30 15
= = ; 4DE = 30; DE = =
10 16 4 4 2

Since the area of 4BDE is 12 × height × width, its area is 12 (10) 15
2 = 75
2 . Finally,
if we subtract the area of 4BDE from that of 4ABC we obtain the area of ADEC.
This area is 96 − 75 192 75 117
2 = 2 − 2 = 2 .

22) The perfect squares less than 18 are 1, 4, 9, and 16. If 16 is the largest summand then
18 can be written in one way: 16 + 1 + 1. If 9 is the largest summand then 18 can be
written in four ways: 9+9, 9 +4+ 4+1, 9+ 4+1+ 1+1+ 1+1, and 9 + 1+1 +· · · +1.
If 4 is the largest summand then 18 can be written in four ways: 4 + 4 + 4 + 4 + 1 + 1,
4 + 4 + 4 + 1 + 1 + 1 + 1 + 1 + 1, 4 + 4 + 1 + 1 + 1 + · · · + 1, 4 + 1 + 1 + 1 + · · · + 1. If
1 is the largest summand then 18 can be written in exactly one way (using eighteen
1s). In total there are 10 ways of writing 18 as a sum of perfect squares.

23) The least common multiple of 4, 5, and 6 is 60. So in order for a number to be divisible
by 4, 5, and 6 it must be a 60-multiple. The 60-multiples less than 1000 are 60, 120,
180, . . ., 960. There are 16 such numbers in total.

24) The figures have 2, 3, 4, . . . rows and 3, 5, 7, . . . columns. That is, the nth figure has
n + 1 rows and 2n + 1 columns. When n = 8 the figure has 9 rows and 17 columns,
for a total of 153 stars.
25) To say that 35 is a divisor of a number n is to say that n is a 35-multiple. Then we
can write n = 35m for some natural number m. The cases with m = 1, 2, 3, 4,and 5
are indicated in the table below.
................................................................................................................................
..... ... ... ...
.. ... ... ...
..
... .....
Largest
..... .....
..
..
m n ...
.
...
Divisor
.
...
.
.........................................................................................................................................
.. ... ... ...
.. ... ... ...
... .... .... ....
.. 1 35
........................................................................................................................................
7
... ... ... ...
.. ... ... ...
.. ... ... ...
2
... 70 . . 35
........................................................................................................................................
.
.. ... ... ...
... ... ... ...
..
3 .. 105 .... 35
....
..........................................................................................................................................
...
.. .. .. ...
.. ... ... .
... ... ... ....
4 .. 140 ... 70 ...
.....................................................................................................................................
.
.. ... ... ...
.. .. .. ...
... ... ... ...
5 .. 175 ... 35 ...
....................................................................................................................................
.

If m = 2, 3, or 5 then 35 is the largest divisor of n. If m is 6 or more then n = 5(7)m.


In this case 7m is a factor of n and 7m > 35 (because m > 5). Then the only
possibility that 35 is the largest factor of n, other than n itself, is when n = 70, 105,
or 175. There are three such numbers.
THE 2010 JAMAICAN MATHEMATICAL OLYMPIAD

Presented by The University of the West Indies


In Collaboration with Sterling Asset Management Ltd

Qualifying Round
Test for Grades 7 and 8

NAME:

GRADE:

SCHOOL:

PRINCIPAL:

DATE OF BIRTH:

ADDRESS:

PHONE:

E-MAIL:

EXAMINATION QUESTIONS

Note: The natural numbers are the numbers {1, 2, 3, . . .} These are also called the
counting numbers or positive integers.

1) If 3/4 of a number is 48, what is the number?


(a) 12 (b) 16 (c) 36 (d) 64 (e) 100

2) How many rectangles may be found in the diagram below?


.........................................................................
.. ... ... ... ...
...
.. .... .... .... ....
.. ... ... ... ...
.. ... ... ... ...
... ... ... ... ...
.. ... ... ... ...
.. ... ... ... ...
... . . . .
.....................................................................

(a) 4 (b) 5 (c) 9 (d) 10 (e) 12


3) The average of three real numbers is 2. If two of the numbers are 5/3 and 7/4, what
is the third one?
(a) 6 (b) 9/16 (c) 31/12 (d) 9/5 (e) 2

4) In the diagram below, ABC is an isosceles triangle. Lines AC and BD are parallel.
If 6 ABC = 40◦ , what is 6 CBD?
B D
• •
.........................................................................................
... ...
... ...
.... .....
.
.. ...
... ...
.. .
.......................................................................................
• •
A C

(a) 40◦ (b) 60◦ (c) 65◦ (d) 70◦ (e) 140◦

5) In a certain community there are 5 houses. In each house there are 3 children, and
each child has 6 toys. How many toys are in the community?
(a) 6 (b) 14 (c) 18 (d) 30 (e) 90

6) What is the next number in the sequence 3, 4, 7, 12, 19, 28, , . . .?


(a) 29 (b) 36 (c) 39 (d) 45 (e) 100

7) In the multiplication below, different letters stand for different digits. What is B?
A A
×A
B C 9

(a) 1 (b) 3 (c) 5 (d) 7 (e) 9

8) In the diagram below, ABC is a triangle and P is the midpoint of BC. Also, Q
and R divide AC into three equal parts. If the area of 4ABC is 36, what is the area
of 4P RC?
B•..
............
.. .........
.........
...
...
.........
.........
P
.
.
.
.. • .........
.........
.........
..
. ...
........................................................................................................
A• • • •C
Q R

(a) 4 (b) 6 (c) 9 (d) 12 (e) 18

9) What is the last digit in the number 72010 ?


(a) 1 (b) 3 (c) 5 (d) 7 (e) 9
10) A number of straws are used to form a triangular grid in a manner illustrated below.
(Only a part of the grid is shown.) If the final grid has 12 straws on each side, how
many straws are required to make the grid?
....
... ...
..... .....
.......................
... ... ... ...
... ... ... ..
.
.................................................
.. .. .. ... .. ..
... .... .... ... .... ...
.........................................................

(a) 36 (b) 78 (c) 108 (d) 234 (e) 600

11) In the diagram below, ABCD is a parallelogram. Also, P is the midpoint of AB and
Q is the midpoint of BC. If the area of ABCD is 24, what is the area of 4DP Q?
P
A •..................................•
...............................•
.... B
...
... ...
... ...
... ...
...
... •Q
...
...
... ...
... .
..................................................................
D• •C

(a) 6 (b) 9 (c) 12 (d) 14 (e) 15

12) A student wishes to write down two distinct letters of the English alphabet in such a
way that they are in alphabetical order. In how many ways can this be done?
(a) 25 (b) 26 (c) 210 (d) 325 (e) 351

13) In the diagram below, ABDE is a rectangle. Also, BCD is an isosceles triangle and
AB = BC. If 6 BCD = 40◦ , what is 6 AEC?
A.........................................B
....
... ...............
... ...........
... ... ...........
.. .......
... ... ........... C
.. ... .....................
......................................................
E D

(a) 10◦ (b) 45◦ (c) 70◦ (d) 80◦ (e) 90◦

14) How many positive divisors does 2010 have? (Note: The divisors of a natural number n
include 1 and n itself.)
(a) 12 (b) 16 (c) 22 (d) 24 (e) 201
15) Suppose ABC is a triangle with 6 ABC = 90◦ , AC = 5, and BC = 3. If D is the
point on AC such that BD ⊥ AC, what is CD?

.......
B
...... ......
...... .... ......
....
....... ..... ......
...... .. ...
..... ... ....
........
. .. ....
...
... . ....
....
. .
... ....
..
............................................................................................
A C
D

(a) 1 (b) 9/5 (c) 2 (d) 4 (e) 8

16) A two-digit number is called reflexive if it is 27 more than the number obtained by
reversing its digits. For example, 41 is reflexive because it is 27 more than 14; 81 is not
reflexive because it is 63 more than 18. How many two-digit numbers are reflexive?
(a) 1 (b) 3 (c) 5 (d) 7 (e) 9

17) The figure below consists of four rectangles. The length of each rectangle is two times
its width and each one has perimeter 18. What is the perimeter of the figure?
.................
... ...
... ....
..
.. ....................................
.. ... ..
... ... ...
. . ..
...........................................................................
.. ... ..
... . .
....
....................................
.. ....
... ...
.. .
.................

(a) 18 (b) 36 (c) 48 (d) 60 (e) 72

18) Suppose a and b are real numbers (not necessarily positive). If a2 + b2 = 4, what is
the largest possible value of 2a2 − 3b2 ?
(a) 0 (b) 2 (c) 3 (d) 4 (e) 8

19 If A + 1 = B − 2 = C + 3 = D − 4 = E + 5, what is A + B + C + D + E?
(a) 5A (b) 5A + 1 (c) 5A + 2 (d) 5A + 3 (e) 5A + 4

20 How many three-digit numbers are odd, divisible by 3, and less than 456?
(a) 58 (b) 59 (c) 60 (d) 116 (e) 117
21) In the figure below, 6 C = 90◦ , AD = DB, DE ⊥ AB, AB = 20, and AC = 12. What
is the area of quadrilateral ADEC?
C .
.........
...... .....
E.............. ....
...
.
...... ....
.......... .... ...
..
..... .
. ...
...
.
...... .
. ....
...
.... .
.
. ...
..
..... .
. ....
...
.... .
. ...
...
.... .
.
. ...
....
... .
. ...
.... . ..
................................................................................................................................
... .
B D A

(a) 12 (b) 75/2 (c) 117/2 (d) 84 (e) 96

22) In how many ways can 18 be written as a sum of perfect squares?


(a) 1 (b) 8 (c) 9 (d) 10 (e) 12

23) How many natural numbers less than 1,000 are divisible by 4, 5, and 6?
(a) 3 (b) 8 (c) 16 (d) 19 (e) 36

24) Consider the following succession of figures:


? ? ? ? ? ? ?
? ? ? ? ?
? ? ? ? ? ? ? ? ? ?
? ? ? ? ? ···
? ? ? ? ? ? ? ? ? ?
? ? ? ? ?
? ? ? ? ? ? ?
How many stars will be in the 8th figure?
(a) 128 (b) 137 (c) 153 (d) 190 (e) 256

25) How many natural numbers have the property that their largest divisor, other than
themselves, is 35?
(a) 3 (b) 4 (c) 5 (d) 7 (e) 35

END OF QUESTIONS

Mail completed question paper to:

Mathematical Olympiad
P.O. Box 94
Mona Post Office
Kingston 7
In order to qualify, all entries must be postmarked no later than February 15, 2010. To be
guaranteed consideration, all entries must be received by February 22, 2010.

All high school students in Jamaica are eligible to become a National Mathematics Cham-
pion at their grade level. However, only students who were born on January 1, 1994, or
thereafter are eligible to be named to the Jamaican Mathematical Olympiad Team.

Please visit http://myspot.mona.uwi.edu/mathematics/ (follow the links below the group


picture) for more information and the latest updates.

Extra copies of this question paper may be downloaded from the Olympiad website
http://myspot.mona.uwi.edu/mathematics/ (follow the links below the group picture).
THE 2010 JAMAICAN MATHEMATICAL OLYMPIAD

Presented by The University of the West Indies


In Collaboration with Sterling Asset Management Ltd

Qualifying Round
Solutions for Grades 7 and 8

1) Let x be the number. Then


3 3x 48 192
x = 48; = ; 3x = 48(4) = 192; x= = 64
4 4 1 3
The number is 64.

2) There are 4 small rectangles in this diagram. There are 3 rectangles made up of two
smaller ones together, as shown below:
............................................................................................................................ ..........................
.................................................................................... ... ...
.........................
...........................................................................
...... ... ....
....... ...... ........ ..... ..... ..
.. ..
...
.....
...
.....
... ...
.....
...
.....
... ..
...
..
..
...
..
... .. ..
...... ..... ........ ...... ...... ...
.. .. ... ... .. ... ...
...
... ....
... .... .... .. ...
..
....... ...... ........ ..... ..... ..
.. ... ...
. ... ... .. .. .. ..
. ..
......................... .........................
.. ... ... ... ... ... ... ... ... ..
...................................................................................................... ...
.. ... ... ...
.........................................................................
... .
....
.
....
.
....
.
....
..................................................................
...
.

There are 2 rectangles made up of three smaller ones together, as shown below:
................................................................................................................................ ...................................
..........................................................................
...... ...... ..... ........ ..... ...
... ... ...
..
...
.. .....
.. ....
..
....... ...... ...... ........ ...... ..
.... .. ...
....
...
.... .. ...
....
..
...
....... ..... ..... ....... ..... .. ... .. ..
. .. ..

....................................
... ... ... ... ..
....................................................................................................................... .
....
.
....
.
....
.
....
.......................................................................
...
.

And there is the 1 outer rectangle. Then there are 4 + 3 + 2 + 1 = 10 rectangles in all.

3) Let x be the third number. Since the average of these numbers is 2,


5 7
+ +x 5 7 20 21 41 72 31
3 4 = 2; + + x = 6; + + x = 6; +x= ; x= .
3 3 4 12 12 12 12 12

4) First, 6 BAC = 6 BCA because 4ABC is isosceles. Since the sum of the angles in
any triangle is 180◦ ,
6 BAC + 6 BCA + 40◦ = 180◦

26 BCA + 40◦ = 180◦


26 BCA = 140◦
6 BCA = 70◦

Since the lines AC and BD are parallel, 6 CBD = 6 BCA. Therefore, 6 CBD = 70◦ .
5) There are 5 × 3 = 15 children in the community. Since each child has 6 toys, there
are 15 × 6 = 90 toys in the community.

6) The differences between terms are 1, 3, 5, 7, 9, . . .. In this case, the next difference
will be 11. Then the next term is 28 + 11 = 39.

7) According to the first step in this multiplication, A × A is a number ending in 9. The


only possibilities are A = 3 and A = 7. If A = 3 then the product is 3 × 33 = 99. But
this is not a three-digit number. Then the product must be 7 × 77 = 539. Therefore,
B = 5.

8) Since Q and R divide AC into three equal parts, the area of 4BRC is one-third of
the area of 4BAC. Then the area of 4BRC is 12. Since P divides BC into two
equal parts, the area of 4P RC is half the area of 4BRC. Then the area of 4P RC
is 6.
B•..
................
B•...
.. .............
.. ........................ .. ........................
... ...... .......... P ... ...... .......... P
... ...... ......... ... ...... .........
..
.
.
.. ....... •
....... .........
...
....... ................. ..
.
.
.. •
....... ..........
....... .... ..........
....... ... .........
.. .... ...... .. ...... .......
...........................................................................................................
• • •C ............................................................................................................
A A• • • •C
Q R Q R

9) The last digit (in fact, the only digit) of 71 is 7; the last digit of 72 is 9; the last digit
of 73 is 3; and the last digit of 74 is 1. Continuing, the last digits of 75 , 76 , 77 , 78 , 79 ,
. . ., are 7, 9, 3, 1, 7, . . .. In fact, the last digits of the powers of 7 form the sequence
7, 9, 3, 1, 7, 9, 3, 1, 7, 9, . . .. This repeats every 4 terms and continues forever. Note
that the 4th, 8th, 12th, and so on, terms are all 1, and this means that the 2008th
term is 1. Then the 2009th term is 7 and the 2010th term is 9. Therefore, the last
digit of 72010 is 9.

10) There will be 12 straws at the base of the grid, 11 straws in the row above it, 10
straws in the row above that one, and so on. Then the total number of straws on the
base or parallel to it is 12 + 11 + 10 + · · · + 1 = 78. There are also straws forming the
sides of the triangles. On one side, there will be 12 straws on the boundary. Parallel
to them, there will be lines with 11 straws, 10 straws, 9 straws, and so on, down to
one straw. Then there will be 12 + 11 + 10 + · · · + 1 = 78 straws on this boundary
or in a parallel direction. Finally, there will also be a total of 78 straws on the third
boundary or in lines parallel to it. Then there will be a total of 78 + 78 + 78 = 234
straws making the grid.
11) First, the area of 4DAB is half of the area of ABCD. Then the area of 4DAB is 12.
P
A •...................................•..............................•
...............
.... B
...
... . .......
... ... ....... .....
.. ....... ..
... ... ........
...
... ....
..
. .............
...
.......... ....
...
•Q
... ... ......
....... .................... ..
.....................................................................
D• •C
Also, since P is the midpoint of AB, the area of 4DAP is half the area of 4DAB.
Then the area of 4DAP is 6. By reasoning in the same way, one sees that the area
of 4DQC is 6 as well. Similarly, the area of 4P QB is half the area of 4P CB, and
this is half the area of 4ACB, and this is half the area of ABCD. Then the area of
4P QB is
1 1 1
× × × 24 = 3.
2 2 2
Finally, the area of 4DP Q is the area of ABCD minus the areas of 4DAP , 4DCQ,
and 4P QB. That is, the area of 4DP Q is 24 − (6 + 6 + 3) = 24 − 15 = 9.

12) The first letter can be any one from A to Y. (The first letter cannot be Z because there
is no letter after Z in the alphabet.) If the first letter is A, the second letter can be
any of B, C, D, . . . Z. There are 25 possibilities in all. If the first letter is B, the second
letter can be any of C, D, E, . . . Z. There are 24 possibilities in all. If the third letter
is C, there are 23 possibilities for the second letter. Continuing in this way, there are
22 possibilities after a D, 21 possibilities after an E, and so on, up to one possibility
for Y. The total number of pairs of letters is 25 + 24 + 23 + · · · + 2 + 1 = 325. (Note:
there is a “fast formula” for adding the numbers 1 + 2 + 3 + · · · + n. This states that
the sum is n(n + 1)/2. In our case, 1 + 2 + 3 + · · · + 25 = 25(25 + 1)/2 = 25(13) = 325.)

13) Since BCD is an isosceles triangle, 6 CBD = 6 CDB. Also the sum of three angles in
any triangle is 180◦ . Since 6 BCD = 40◦ it follows that 6 CBD = 6 CDB = 70◦ .
A.........................................B
....
... ...............
.. ... ...........
... ...........
.. ....
.. ... ............................................... C
... ..... .
...
...
. ......
................................................................................................
E D

Since 6 BDE = 90◦ we also have 6 EDC = 160◦ . Since the triangle EDC is isosceles,
6 DEC = 6 DCE. It follows that 6 CED = 10◦ . Since 6 AED = 90◦ we must have
6 AEC = 80◦ .

14) The prime factorization of 2010 is 2 × 3 × 5 × 67. To produce a divisor for 2010, we
have two choices for the number of 2’s which may be used (none or one), two choices
for the number of 3s (none or one), two choices for the number of 5s (none or one), and
two choices for the number of 67’s (none or one). There are a total of 2×2×2×2 = 16
possible divisors. They are: 1, 2, 3, 5, 6, 10, 15, 30, 67, 134, 201, 335, 402, 670, 1005,
and 2010.
15) First, 4ABC and 4BDC are right triangles and both have the angle at C in common.
Since the sum of the angles in any triangle is 180◦ , it follows that 6 DBC = 6 BAC.
Then 4ABC and 4BDC are similar. Since corresponding sides of similar triangles are
proportional, BC/AC = CD/BC. Since BC = 3 and AC = 5, we have 3/5 = CD/3.
By cross-multiplying, we have 5CD = 9 and so CD = 9/5.

16) Let ab be a two-digit number, where a and b are each one of the numbers 0, 1, 2, . . ., 9.
Then the value of ab is 10a+b. When the digits are reversed we obtain the number ba,
with a value of 10b+a. In order to be reflexive, we must have (10a+b)−(10b+a) = 27.
This means that 10a + b − 10b − a = 27 and so 9a − 9b = 27. In turn, this means
that 9(a − b) = 27 and so a − b = 3. The possibilities for a and b, respectively, are 3
and 0, 4 and 1, 5 and 2, 6 and 3, 7 and 4, 8 and 5, and 9 and 6. That is, the reflexive
numbers are 30, 41, 52, 63, 74, 85, and 96. There are 7 of them in all.

17) Let x be the width of a single rectangle and 2x be its length. Then its perimeter is
2x + x + 2x + x = 6x. Equating this to 18 gives x = 3. Then each small rectangle has
........3 ............
... ...
.. .3
6 .. . .... 6
2x .... .......................................
... ...
..................................... .. ... 3
.. ... 3 . ..
x ...
....................................
x ..........................................................................
... ... ... 3
3 .. ... ...
2x ................................... ...
. ...
6 3 ..... ....
6
...................
3

width 3 and length 6. It follows that the lengths of the segments in the boundary of
the figure are as shown. The perimeter of the figure is 48

18) First, since a2 + b2 = 4 we have a2 = 4 − b2 . Then

2a2 − 3b2 = 2(4 − b2 ) − 3b2 = 8 − 2b2 − 3b2 = 8 − 5b2 .

Also, b2 ≥ 0 because all perfect squares are positive or 0. Then 5b2 ≥ 0 and it follows
that 8 − 5b2 ≤ 8. On the other hand, if a = 2 and b = 0 then a2 + b2 = 22 + 02 = 4.
Also, 2a2 − 3b2 = 2(22 ) − 3(02 ) = 8 − 0 = 8. Then the greatest possible value for
2a2 − 3b2 is 8.

19 Since A + 1 = B − 2 we have B = A + 3. Since A + 1 = C + 3 we have C = A − 2. Since


A + 1 = D − 4 we have D = A + 5. Finally, since A + 1 = E + 5 we have E = A − 4.
Then

A + B + C + D + E = A + (A + 3) + (A − 2) + (A + 5) + (A − 4)
= 5A + (3 − 2 + 5 − 4)
= 5A + 2
20 The first few three-digit numbers which are odd and divisible by 3 are 105, 111, 117,
. . .. In general, if n is odd and divisible by 3 then the next such number is n + 6.
Since 105 is the first odd number divisible by 3, the others we seek are of the form
105 + 6k for some k. For these to be less than 456 we must have 105 + 6k ≤ 456 and
thus 6k ≤ 351. Since k is a natural number, the solution is k ≤ 58. Starting with 105
(when k = 0) and ending with 453 (when k = 58), there are 59 such numbers in total.

21) Since 4ABC is a right triangle, (AC)2 +(BC)2 = (AB)2 by the Pythagorean theorem.
Since AC = 12 and AB = 20,

122 + (BC)2 = 202 ; 144 + (BC)2 = 400; (BC)2 = 256; BC = 16.

Also, the area of 4ABC is 12 × height × width = 12 × 16 × 12 = 96. Second, 4EBD


is a right triangle and 4ABC and 4EBD have the angle at B in common. Since
the sum of the angles in any triangle is 180◦ , it follows that 6 BED = 6 BAC. Then
4ABC and 4EBD are similar. Since corresponding sides of similar triangles are
proportional, DE/BD = AC/BC. This means that

DE 12 3 30 15
= = ; 4DE = 30; DE = =
10 16 4 4 2

Since the area of 4BDE is 12 × height × width, its area is 12 (10) 15
2 = 75
2 . Finally,
if we subtract the area of 4BDE from that of 4ABC we obtain the area of ADEC.
This area is 96 − 75 192 75 117
2 = 2 − 2 = 2 .

22) The perfect squares less than 18 are 1, 4, 9, and 16. If 16 is the largest summand then
18 can be written in one way: 16 + 1 + 1. If 9 is the largest summand then 18 can be
written in four ways: 9+9, 9 +4+ 4+1, 9+ 4+1+ 1+1+ 1+1, and 9 + 1+1 +· · · +1.
If 4 is the largest summand then 18 can be written in four ways: 4 + 4 + 4 + 4 + 1 + 1,
4 + 4 + 4 + 1 + 1 + 1 + 1 + 1 + 1, 4 + 4 + 1 + 1 + 1 + · · · + 1, 4 + 1 + 1 + 1 + · · · + 1. If
1 is the largest summand then 18 can be written in exactly one way (using eighteen
1s). In total there are 10 ways of writing 18 as a sum of perfect squares.

23) The least common multiple of 4, 5, and 6 is 60. So in order for a number to be divisible
by 4, 5, and 6 it must be a 60-multiple. The 60-multiples less than 1000 are 60, 120,
180, . . ., 960. There are 16 such numbers in total.

24) The figures have 2, 3, 4, . . . rows and 3, 5, 7, . . . columns. That is, the nth figure has
n + 1 rows and 2n + 1 columns. When n = 8 the figure has 9 rows and 17 columns,
for a total of 153 stars.
25) To say that 35 is a divisor of a number n is to say that n is a 35-multiple. Then we
can write n = 35m for some natural number m. The cases with m = 1, 2, 3, 4,and 5
are indicated in the table below.
................................................................................................................................
..... ... ... ...
.. ... ... ...
..
... .....
Largest
..... .....
..
..
m n ...
.
...
Divisor
.
...
.
.........................................................................................................................................
.. ... ... ...
.. ... ... ...
... .... .... ....
.. 1 35
........................................................................................................................................
7
... ... ... ...
.. ... ... ...
.. ... ... ...
2
... 70 . . 35
........................................................................................................................................
.
.. ... ... ...
... ... ... ...
..
3 .. 105 .... 35
....
..........................................................................................................................................
...
.. .. .. ...
.. ... ... .
... ... ... ....
4 .. 140 ... 70 ...
.....................................................................................................................................
.
.. ... ... ...
.. .. .. ...
... ... ... ...
5 .. 175 ... 35 ...
....................................................................................................................................
.

If m = 2, 3, or 5 then 35 is the largest divisor of n. If m is 6 or more then n = 5(7)m.


In this case 7m is a factor of n and 7m > 35 (because m > 5). Then the only
possibility that 35 is the largest factor of n, other than n itself, is when n = 70, 105,
or 175. There are three such numbers.
THE 2010 JAMAICAN MATHEMATICAL OLYMPIAD

Presented by The University of the West Indies


In Collaboration with Sterling Asset Management Ltd

Qualifying Round
Test for Grades 9, 10, and 11

NAME:

GRADE:

SCHOOL:

PRINCIPAL:

DATE OF BIRTH:

ADDRESS:

PHONE:

E-MAIL:

EXAMINATION QUESTIONS

Note: The natural numbers are the numbers {1, 2, 3, . . .} These are also called the
counting numbers or positive integers.

1) If four-fifths of a number is 60, what is the number?


(a) 32 (b) 48 (c) 75 (d) 84 (e) 90

2) How many triangles may be found in the diagram below?


..............................
....... ...... ......
... .. ... .. ... ..
...................................................................
.
... ... ... ..
... ... ... ... ... ... ... ...
... ... ... ... ... ... ... ....
.............................................................................

(a) 12 (b) 13 (c) 14 (d) 15 (e) 16


3) Consider the sequence: doh, re, mi, fah, sol, la, ti, doh, re, mi, fah, sol, la, ti, doh, re,
mi, . . .. What is the 2010th term of this sequence?
(a) doh (b) re (c) mi (d) sol (e) ti

4) A certain farmer raises chickens and rabbits. If she has 25 animals and they have a
total of 84 legs, how many chickens are on the farm?
(a) 5 (b) 8 (c) 17 (d) 25 (e) 62

5) In the diagram below, ABC is a triangle and P , Q, and R divide AC into 4 equal
segments. If the area of 4BQC is 24, what is the area of 4BAR?
B
..
.........
...... .....

.
.......... ...
..... ...
...... ...
..........
. ...
...
........ ...
......
. ..
.........................................................................................
• • • • •
A P Q R C

(a) 12 (b) 16 (c) 32 (d) 36 (e) 48

6) What is the last digit in the number 32010 ?


(a) 1 (b) 3 (c) 5 (d) 7 (e) 9

7) Suppose N is a natural number such that N 2 < 3, 456 and (N + 1)2 > 3, 456. What
is N ?
(a) 58 (b) 59 (c) 345 (d) 1728 (e) 3455

8) In a certain town half of the people are men, one-third of the men play dominoes, and
one fourth of the male domino players enjoy golf. If 1,200 people live in the town,
how many are men who play dominoes and enjoy golf?
(a) 20 (b) 50 (c) 300 (d) 400 (e) 600

9) In the diagram below, ABC is a right triangle and DB = DC. If 6 DAC = 25◦ , what
is 6 BDC?

B
...........

.
..............
D ........... ..
.............................
• ...
.
........ ...........
....
............ ............. ...
.. ..
..........................................................................................
.
A• . .
•C
(a) 25◦ (b) 40◦ (c) 45◦ (d) 50◦ (e) 60◦
10) How many three-digit numbers are divisible by both 3 and 11?
(a) 25 (b) 26 (c) 27 (d) 33 (e) 105

11) If a > b > 0 and a2 + 6b2 = 7ab, what is a/b?



(a) 1 (b) 6 (c) 7 (d) 13 (e) 42

12) How many rectangles may be found in the diagram below?


.............................................................................................
... . ...
.. .................................................
.. . . . ...
......................................................................................................
.. .. ... .. ...
.. .. . .
... ............................................... ....
.. ... ..
.........................................................................................

(a) 8 (b) 9 (c) 16 (d) 17 (e) 18

13) What is the sum of the digits in the number 102010 − 2010?
(a) 9 (b) 2009 (c) 18,007 (d) 18,079 (e) 18,086

14) The figure below consists of four congruent rectangles. The length of each rectangle
is three times its width. The area of each rectangle is 12 cm2 . What is the perimeter
of the figure?
...........
... ....
.. ..
... ...
.. ...
.. ..................................
.. ... ..
..
. .
..............................................................................
.. ... ..
.................................. ....
... ..
... ...
.. ..
.. ...
...........

(a) 6 (b) 12 (c) 16 (d) 48 (e) 64

15) How many digits are in the number (999, 999, 999, 876)2 − (124)2?
(a) 12 (b) 24 (c) 25 (d) 144 (e) 999,876

16) How many natural numbers less than 75 have exactly 4 divisors? (Note: 1 and n are
always divisors of n.)
(a) 12 (b) 15 (c) 18 (d) 19 (e) 22
17) Suppose ABC is a triangle with 6 ABC = 90◦ , and let D be the point on AC such
that BD ⊥ AC. If AC = 13 and AB = 12, what is BD?
........
B
........ .........
........ ... ..
......
......... .. ....
...... .. ..
............... ... .....
........ .
..........................................................................................
.
.
A C
D

(a) 4 (b) 56/12 (c) 60/13 (d) 5 (e) 13/2

18) In how many ways can we choose natural numbers a and b so that their sum is less
than 50 and the equation x2 + ax + b has exactly one real root?
(a) 0
(b) 3
(c) 5
(d) 6
(e) infinitely many ways

19) Suppose the sequence a1 , a2 , a3 , a4 , . . . of natural numbers has the property that
a1 < a2 and an+2 = an + an+1 for all n ≥ 1. If a6 = 46, what is a3 ?
(a) 2 (b) 8 (c) 10 (d) 18 (e) 23

20) In the diagram below, 4ABC is inscribed in a circle with centre O. If 6 OAB = 40◦
and 6 OBC = 30◦ , what is 6 OCA?
........
A
...........................
.....
..... • .
.............
..... ... .....
.... .
..
....... .... .....
... ... ... ...
..... ... ..
...
.. ..... ... ..
.....
... ........
..........
•O ... ...
... ...
C• .............................. . .
... ......................... ........
....
.....
...... ..
•B
..
......
...
............ .................
......

(a) 10◦ (b) 15◦ (c) 20◦ (d) 25◦ (e) 30◦

21) If 52010 is divided by 3, what will the remainder be?


(a) 0 (b) 1 (c) 2 (d) 3 (e) 4

22) A total of 4 packages must be delivered to a building with 5 apartments, called A, B,


C, D, and E. If no apartment is receiving more than two packages, in how many ways
can they be delivered?
(a) 35 (b) 45 (c) 60 (d) 180 (e) 200
23) A regular hexagon is inscribed in one circle and circumscribed about another. If the
inner circle has area A1 and the outer circle has area A2 , what is A2 /A1 ?
......
............ ............
..................................................................
............... .............
.. ............ .... ...
... .... .... ..
.........
............ ........
........ .. ....
........ . .
.
.......
. . .. ............
... ... . .
... ....
.... .....
. .
.... ......
............... ............
...........................................................
............ ............
.....

√ √ √
(a) 3/4 (b) 3/2 (c) 2/ 3 (d) 4/3 (e) 3

24) Suppose N is a natural number such that 1 + 2 + 3 + · · · + N < 1, 500 but 1 + 2 + 3 +


· · · + N + (N + 1) > 1, 500. What is N ?
(a) 27 (b) 54 (c) 55 (d) 63 (e) 1,500

25) Suppose S1 and S2 are squares with areas A1 and A2 and perimeters P1 and P2 ,
respectively. If A1 /A2 = 5, what is P1 /P2 ?
√ √
(a) 1/5 (b) 5 (c) 5 (d) 5 5 (e) 25

26) In the following multiplication, different letters represent different digits. What is the
value of E?
A B
× 5 D
......................................
A B
B E 5
...................................................
B A 2 B

(a) 2 (b) 3 (c) 4 (d) 5 (e) 6

27) Suppose a and b are real numbers and a2 + b2 = 10. What is the largest possible value
of 2a2 + 3b2 ?
(a) 2 (b) 3 (c) 6 (d) 20 (e) 30
28) Circles of radius 2, 6, and 4 are constructed with centres P , Q, and R, respectively.
Assume that P , Q, and R lie in a straight line and the circles are tangent. Let SU
be the line tangent to the first and third circles. Let T be the point on SU such that
QT ⊥ SU . What is the length of QT ?
.................................
...... ..... U
..... ..........
..... T ....
S ...............................................................................................
.
.
• •
.......................................
.....
...
..

.....................................
...
......
....
...
..
............................................................................................................................................................
• • •
... .. .
.......P....... ... . . . .
....
. ....
..... .. Q .
.. ... R ..... .
... ... ........... ..
.... ... ......................
.....
...... ..
........
......... .........
................

(a) 5/2 (b) 26/9 (c) 3 (d) 11/3 (e) 4

29) How many ordered pairs (m, n) of natural numbers satisfy the relation m + n < 20?
(a) 20 (b) 60 (c) 171 (d) 190 (e) 400

30) A six-digit number is called curious if its digits are 1, 2, 3, 4, 5, and 6 (in some order)
and its first k digits are divisible by k for k = 1, 2, 3, . . ., 6. So, if the number is
d1 d2 d3 d4 d5 d6 then d1 must be divisible by 1, d1 d2 must be divisible by 2, d1 d2 d3 must
be divisible by 3, and so on. How many six-digit numbers are curious?
(a) 0 (b) 2 (c) 3 (d) 6 (e) 16

END OF QUESTIONS

Mail completed question paper to:

Mathematical Olympiad
P.O. Box 94
Mona Post Office
Kingston 7

In order to qualify, all entries must be postmarked no later than February 15, 2010. To be
guaranteed consideration, all entries must be received by February 22, 2010.

All high school students in Jamaica are eligible to become a National Mathematics Cham-
pion at their grade level. However, only students who were born on January 1, 1994, or
thereafter are eligible to be named to the Jamaican Mathematical Olympiad Team.

Please visit http://myspot.mona.uwi.edu/mathematics/ (follow the links below the group


picture) for more information and the latest updates.

Extra copies of this question paper may be downloaded from the Olympiad website
http://myspot.mona.uwi.edu/mathematics/ (follow the links below the group picture).
THE 2010 JAMAICAN MATHEMATICAL OLYMPIAD

Presented by The University of the West Indies


In Collaboration with Sterling Asset Management Ltd

Qualifying Round
Solutions for Grades 9, 10, and 11

1) Let x be the number. Then


4 4x 60
x = 60; = ; 4x = 300; x = 75
5 5 1
The number is 75.

2) There are 12 small triangles in this figure. In addition there are 4 larger triangles, with each
one made up of four smaller triangles as shown below.
............................................................ . . ..........................
...... ......
......................................... ......................................... .........................................

...............................................................................................................................
..... ..... .....
... ... ... ... ... ...
.. ..
... ... ... ... ... ...
.. ..
... ..
... ... ... ... ... ...
.. ..

. . . .
.
..
.. . ......
.. ........ ........ ....
.................................................................... .
.. .. ........ ........ ....
.. . ......
...................................................................... .
.. .. ........ ........ ....
... ..
......................................................................

.................................................................................................................................
. . .
. . . . . .
... .. ... .. ... .. ... ..
.
.......................
.. ... .. ... .. ... .. ...
. .
.................................................................................
.
... .. ... .. ... .. ... ..
.. ... .. ... .. ... .. ...
. .
.......................
.
...............................................................................
.
....
... .. ... .. ... .. ... ..
.. ... .. ... .. ... .. ...
. . .
...............................................................................

There are 16 triangles in all.

3) This sequence repeats every 7 terms. Then the 7th, 14th, 21st, 28th, 35th, and so on, terms
are all ti. This continues up to the 2009th term, which is also ti. Then the 2010th term is doh.

4) Let c be the number of chickens and r the number of rabbits on the farm. Then c + r = 25.
Since each chicken has 2 legs and each rabbit has 4 legs, 2c + 4r = 84. Multiplying the first
equation by 2 gives the system 
2c + 2r = 50
2c + 4r = 84
Subtracting the first equation from the second, 2r = 34 and so r = 17. Then c + 17 = 25 and
so c = 8. There are 8 chickens on the farm.

5) Since P , Q, and R divide AC into 4 equal parts, 4BAP , 4BP Q, 4BQR, and 4BRC have
B
• ...
................
............ .....
.......... .. ... ...
................... .... .... ......
... .. . . ..
...... .... .. .... .....
...... ..... .... ...
...... ..... ..
... ...
.
.......... ........ ...
... ...
..
.. . .
• ........................................................................................................
• • • •
A P Q R C
equal areas. We will write a(BAP ) = a(BP Q) = a(BQR) = a(BRC) in this case. (So,
a(BAP ) stands for the area of 4BAP , and so on.) Then

a(BQR) + a(BRC) = a(BQC); 2 × a(BQR) = 24; a(BQR) = 12.

Then we have a(BAR) = a(BAP ) + a(BP Q) + a(BQR) = 12 + 12 + 12 = 36.


6) The last digit (in fact, the only digit) of 31 is 3; the last digit of 32 is 9; the last digit of 33 is
7; and the last digit of 34 is 1. Continuing, the last digits of 35 , 36 , 37 , 38 , 39 , . . ., are 3, 9,
7, 1, 3, . . .. In fact, the last digits of the powers of 3 form the sequence 3, 9, 7, 1, 3, 9, 7, 1,
3, 9, . . .. This repeats every 4 terms and continues forever. Note that the 4th, 8th, 12th, and
so on, terms are all 1, and this means that eventually the 2008th term is 1. Then the 2009th
term is 3 and the 2010th term is 9. Therefore, the last digit of 32010 is 9.

7) By trial and eror we see that 582 = 3, 364 and 592 = 3, 481. Then 582 < 3, 456 < 592 . It
follows that N = 58.

8) The number of men in the town is 12 × 1, 200 = 600. The number of men who play dominoes
is 13 × 600 = 200. The number of male dominoe players who enjoy golf is 14 × 200 = 50. There
are 50 men who play dominoes and enjoy golf.

9) The sum of the angles in any triangle is 180◦ . Since 6 BAC = 6 DAC = 25◦ ,
25◦ + 90◦ + 6 CBA = 180◦ ; 115◦ + 6 CBA = 180◦ ; 6 CBA = 65◦
This means that 6 CBD = 65◦ as well. Since 4DBC is isosceles, we also have 6 BCD = 65◦ .
Furthermore, the sum of the angles in any triangle is 180◦ . Then
6 BDC + 65◦ + 65◦ = 180◦ ; 6 BDC + 130◦ = 180◦ ; 6 BDC = 50◦

10) If a number is divisible by 3 and 11 then it is divisible by 33. In other words, it is a multiple
of 33. The smallest multiple of 33 which has three digits is 132. This is 4 × 33. The next
multiples of 33 are 165, 198, 231, and so on. These are 5 × 33, 6 × 33, 7 × 33, and so on.
The highest multiple of 33 which less than 1000 is 990. This is 30 × 33. Then the number of
three-digit numbers divisible by both 3 and 11 is the same as the number of numbers in the
set {4, 5, 6, 7, . . . , 30}. This is 27.

11) If a2 + 6b2 = 7ab then a2 − 7ab + 6b2 = 0. Thus (a − b)(a − 6b) = 0. Then, possibly, a − b = 0
and so a = b. Otherwise, a − 6b = 0 and so a = 6b. Since a and b re positve and a > b, we
must have a = 6b. Then
a 6b
= =6
b b

12) First, there are 4 small rectangles as illustrated below. Also, these rectangles can be paired
..................................................................................................... ..................................................................................................... ................................................................................................. .................................................................................................
... ... ..... ... ... .....
.................................... ...................................
... .. ... ... .. ... . .
... ................................................... .. ... ................................................... .. ... ..................................................... ... ... ..................................................... ...
... .... . . .. ... .... . . .. ... .. ... ... .. ...
.... .. .... ..
..........................................................................................................
.... ... ... .. ...
..........................................................................................................
.... ... ... .. ...
...
....................
... .
.........................................................................................................
. ..
....
................... ... .
....................................................................................................
... .....
...
...
...
... ....
..................................................
....
... ....
...
.
...
...
...
... ...
..................................................
....
... ....
...
.
...
...
...
..
................
...
..................................................
....
..
. ....
...
...
...
...
................
... ....
..................................................
....
.
...
...
................................................................................................. ................................................................................................. ... ..
............................................................................................
... ..
............................................................................................

to make larger rectangles in 4 different ways. Finally, the four small rectangles together make a
single larger rectangle. Next, there are 4 larger rectangles as shown below. Also, these rectangles
............................................................................................................................................ ..................................
.................................................................................................. ................................................................................................. .................................................................................................
........ ............................................................. ...... ..
.. ...
. ... ... ... ... ... ... ...

...................................
.... . ... ... .
..................................................... .... ... .
..................................................... ....
...........................................................
....................................................................................................................................................... ...
... ... .. ...
.......................................................................................................
...
.
...
... ... ..
..................................
. .
.
.
............................................................................................................
...
.
...
..................................
.. ... ..
. .
.
.
........................................................................................................
...
.
.... .... ....
.. ... ... ... .. ... .... ... .... ... ... ...
.. ...
.
.... ....
.. . .... ....

................................... ...................................
.... ...................................................... ...
... ... ................................................... ... ... ................................................... ... ... ................................................... ...
... ... .. ... ... ... ... ... ... ... ... ...
.. . .. .. . ... ... . ... ... .
................................................................................................ ............................................................................................. ............................................................................................. .............................................................................................

can be paired to make larger rectangles in 4 different ways. Finally, there is also the outermost
rectangle. A total of 4 + 4 + 1 + 4 + 4 + 1 = 18 rectangles may be found in the diagram.
13) In its decimal form, the number 102010 consists of a 1 followed by a 0 written 2010 times.
Subtracting 2010 from this gives the number represented by a 9 written 2006 times followed
by 7990. The sum of the digits in this number is 2006(9) + 7 + 9 + 9 + 0 = 18, 079.

14) Let x be the width of a rectangle and 3x be its length. Since the area of each rectangle
is 12 cm2 , we have 3x2 = 12. Then x2 = 4 and so x = 2. It follows that segments in the figure
have the lengths shown. The perimeter of the figure is 48.
2
............
..... ....
.. ...
6 ...... ..... 4 6
3x ... .....................................
.....................................
.. ... 4 ..... ..... ..
... 2
x ...
....................................
x ...................................................................................
... .. ..
2 ...................................... ... 4
3x 6
... ...
.. ..
4 ..... ...... 6
..............
2

15) Since (999, 999, 999, 876)2 − (124)2 is the difference of two squares, it may be factored as

(999, 999, 999, 876 + 124)(999, 999, 999, 876 − 124) = (1, 000, 000, 000, 000)(999, 999, 999, 752)

The number 999,999,999,752 obviously has 12 digits. If it is multiplied by 1,000,000,000,000


then the effect will be to add 12 zeroes to the end of this number. Therefore, the total number
of digits in (999, 999, 999, 876)2 − (124)2 is 24.

16) Since the only divisor of 1 is 1, it does not have exactly 4 divisors. Suppose now that n is
a natural number and consider its prime factorization. If this factorization contains three
distinct primes, then n has more than 4 divisors. To see this, suppose p, q, and r are prime
factors of n. Then 1, p, q, r, and pqr are all divisors of n. So, if n has exactly 4 divisors it
must have only 1 or 2 primes in its factorization. Suppose n has two primes, p and q, in its
factorization. Let p represent the smaller one and q the larger one. Then 1, p, q, and pq are
factors of n. If n has exactly 4 factors then these are the only ones. Then pq = n. In this
case, it is possible that p = 2, 3, 5, or 7. (Otherwise, if p ≥ 11 then q ≤ 75/11. This means
that q ≤ 7 and so q < p.) If p = 2 then q could be 3, 5, 7, 11, 13, 17, 23, 29, 31, or 37. There
are 10 choices in this case. If p = 3 then q could be 5, 7, 11, 13, 17, or 23. There are 6 choices
in this case. If p = 5 then q could be 7, 11, or 13. There are 3 choices in this case. If p = 7
then there are no possible choices available. (The next prime is 11 and 7 × 11 is too large.)
Then the total number of choices for primes p and q is 10 + 6 + 3 = 19.
Suppose now that n has only one prime, p, in its factorization. In order to have exactly 4
divisors they would have to be 1, p, p2 , and p3 . Also, we would have p3 = n in this case. The
possiblities are p = 2, n = 8, or p = 3, n = 27, or p = 4, n = 64. There are 3 choices in this
case. Adding these to the case we analyzed earlier, there are 22 numbers less than 75 which
have exactly 4 divisors.
17) By the Pythagorean theorem, (AB)2 + (BC)2 = (AC)2 . Substituting AC = 13 and AB = 12,

122 + (BC)2 = 132 ; 144 + (BC)2 = 169; (BC)2 = 25; BC = 5.

Also 4BDC is a right triangle and 4ABC and 4BDC have a common angle at C. Since the
sum of the angles of any triangle is 180◦ , it follows that 6 DBC = 6 BAC. In this case, the
corresponding angles in 4ABC and 4BDC are equal. Therefore, these triangles are similar.
Then
BD AB BD 12 60
= ; = ; 13BD = 60; BD =
BC AC 5 13 13

18) The equation x2 + ax + b has exactly one real root when a2 − 4b = 0. The values for a and b
that satisfy this relation are a = 2, b = 1; a = 4, b = 4; a = 6, b = 9; a = 8, b = 16; a = 10,
b = 25; a = 12, b = 36. Any other choices for a and b will have the property that a + b > 50.
Then we can choose a and b in exactly 6 ways.

19) Let a1 = m and a2 = n. Then

a3 = a1 + a2 = m + n
a4 = a2 + a3 = m + 2n
a5 = a3 + a4 = 2m + 3n
a6 = a4 + a5 = 3m + 5n

Therefore, m and n are natural numbers such that m < n and 3m + 5n = 46. In this case,
5n = 46 − 3m and so 46 − 3m must be a 5-multiple. The only m-values for which 46 − 3m is
a 5-multiple are m = 2, m = 7, and m = 12. Then there are three candidates for m and n:
m = 2 and n = 8; m = 7 and n = 5; m = 12 and n = 2. Since we must also have m < n, the
only possible solution is m = 2 and n = 8. In this case, a3 = m + 2n = 2 + 2(8) = 2 + 16 = 18.

20) Since OA, OB, and OC are radial segments, they are equal in length. Then 4AOB, 4BOC,

..........
..................... A
.....
...... • ..
...............
........ .. ....
. .... ..... .. .. ...
...
.
........ .... .... ....
.
... .... . ... ...
... ..... .. ... ...
..... .... ... ..
... .....
... ......... ........................
... ....................
•O .
......
... ...
.. ..
....... .... ....
C•
.......................... ..
... ......................... ............
...
.....
......
.. .. . . ..
•B
.. ..........
....
..
........ . .... . .....
.......................

and 4COA are isosceles triangles. This means that 6 OAB = 6 OBA, 6 OBC = 6 OCB, and
6 OCA = 6 OAC. Also, the sum of the angles in 4ABC is 180◦ . Then

6 OAB + 6 OBA + 6 OBC + 6 OCB + 6 OCA + 6 OAC = 180◦


26 OAB + 26 OBC + 26 OCA = 180◦
6 OAB + 6 OBC + 6 OCA = 90◦
40◦ + 30◦ + 6 OCA = 90◦
70◦ + 6 OCA = 90◦
6 OCA = 20◦
21) By the binomial formula,
52010 = (6 − 1)2010 = 62010 + c1 62009 (−1) + c2 62008 (−1)2 + c3 62007 (−1)3 + · · ·
+ c2008 62 (−1)2008 + c2009 6(−1)2009 + (−1)2010

Here, each ck represents the kth binomial coefficient 2010!/ k!(2010 − k)! . Also, each term
except the last one, (−1)2010 , is a multiple of 6. Then each term except the last one is divisible
by 3. Then the remainder when 52010 is divided by 3 will be (−1)2010 = 1.

22) We must consider three possibilities: two apartments receive two samples each; one apartment
receives two samples and two receive one each; or four apartments receive one sample each. In
the first case, there are 10 ways of choosing the two apartments to receive the samples: AB,
AC, AD, AE, BC, BD, BE, CD, CE, or DE. In the second case, there are 5 ways to choose
one apartment to receive two samples. For each such choice, there are six ways to choose
two more apartments to receive one sample each. Then there are a total of 5 × 6 = 30 ways
to distribute the samples in this case. In the third case, there are 5 ways to choose the four
apartments to receive the samples: ABCD, ABCE, ABDE, ACDE, or BCDE. Then there are
10 + 30 + 5 = 45 ways of delivering the samples.

23) Let r1 be the radius of the inner circle and r2 the radius of the outer circle. Then A1 = πr12

A ............................. B A................................C B
............................................. .....................................
...
................ .. ......................... ... ..
............... ..... ..... ...... ... ...
...
... ...
.
... .... ... .. ... ... ... ..
.. .. ... ... ...
... ....
............
r . .
r
2 ............. 1 ............... r ...
2 ...... r ...
... 1 ....
.
.
.
..... ... ..... ... ..... ... .. .
......... ........ ... .. ...
........ .......... ... .... ....
... ....
... ....
O ..
... ..
. ... ... ...
... .. ..
... ...... ... ... ... ... ...
.............. .
...
................. .........
....... .......... ......... ...... ......
.............. ............................
....................
O

and A2 = πr22 . Also, as seen in the enlarged diagram, 6 AOB is 60◦ . This is because this angle
is one-sixth of a full circle and so 6 AOB = 360◦ /6 = 60◦ . Therefore, 6 AOC = 30◦ . Then
√ √
r1 3 3 r2 3r 2

= cos 30 = ; r1 = ; r12 = 2
r2 2 2 4
Finally,  
3r22
π
A2 πr 2 4 3πr22 3
= 12 = = = .
A1 πr2 πr22 4πr22 4

24) By trial and error, one may verify that 1+2+3+· · ·+54 = 1, 485 and 1+2+3+· · ·+54+55 =
1540. Then N = 54. Alternatively, there is a “fast formula” for finding a sum of the form
1 + 2 + 3 + · · · + n. Let S represent this sum. Then

S = 1 + 2 + 3 + ··· + n
S = n + (n − 1) + (n − 2) + · · · + 1
Adding equations, 2S = (n+1)+(n+1)+(n+1)+· · ·+(n+1), where the term n+1 occurs on
the right side n times. Then 2S = n(n+1) and so S = n(n+1)/2. In this problem, we still need
some amount of trial and error. However, one verifies that 1 + 2 + 3 + · · · + 54 = 54(54 + 1)/2 =
54(55)/2 = 1485. Similarly, 1 + 2 + 3 + · · · + 55 = 55(55 + 1)/2 = 55(56)/2 = 1540.
25) Let `1 and `2 be the side lengths of S1 and S2 , respectively. Then A1 = `21 and A2 = `22 . Also,
P1 = 4`1 and P2 = 4`2 . We have
A1 `21 √
= 5; = 5; `21 = 5`22 ; `1 = 5`2 .
A2 `22
(We choose the positive square root above because the length `1 cannot be negative.) Then

P1 4`1 `1 5`2 √
= = = = 5.
P2 4`2 `2 `2

26) The first step in this multiplication indicates that D × AB = AB. Then D = 1. Also, adding
the two partial answers indicates that A + 5 gives an answer ending in 2. Since A is a digit
from 0 to 9, the only possibility is A = 7. Then 7 + 5 = 12. After the 2 is recorded the tens
digit, 1, is carried to the hundreds place. Then 1 + E = 7 and so E = 6. To complete the
analysis, it is clear from the multiplication of the tens digits that B = 3.

27) Since squares of real numbers are always positive or 0, we have b2 ≥ 0. Similarly, a2 ≥ 0.
Since a2 = 10 − b2 we have 10 − b2 ≥ 0 and so 10 ≥ b2 . That is, b2 ≤ 10. Thus 0 ≤ b2 ≤ 10.
Since a2 + b2 = 10,
2a2 + 3b2 = 2a2 + 2b2 + b2 = 2(a2 + b2 ) + b2 = 2(10) + b2 = 20 + b2 .

Then 2a2 + 3b2 = 20 + b2 ≤ 30. Furthermore, 2a2 + 3b2 = 30 when a = 0 and b = ± 10.
Then the largest possible value for 2a2 + 3b2 is 30.

28) Let A be the point where the lines SU and P R meet. Since AU is tangent to the first and
..................................
....... ..... U ...............
..... .....
. ...... T . •
....................................
S ................................................................................................... ..... .............

................................. ..
• .. ... .. ... ...
............................. ..... .... .......... ... ..... ... ..
............................ . .
...............................................................................................................................................................................................................................................................................
• • • •
... . . .. .
.......P.............. Q ......
. ..
.
A .... ...
...
.. ....
.
. .....
R ...
..
.. ....... .
...
...
.....
.
... .....................
...... .....
......... ......
..........................

third circles, 6 ASP and 6 AU R are right angles. Then4ASP , 4AT Q, and 4AU R are right
triangles. Also, they all have the same angle at A. Since the sum of the angles in any triangle
is 180◦ , it follows that 6 AP S, 6 AQT , and 6 ARU are equal. Therefore, the triangles AP S,
AQT , and ARU are similar. Since similar triangles have proportional sides,
AP AR
=
SP UR
The circles at P , Q, and R have radii 2, 6, and 4, respectively. Then AR = AP +2+6+6+4 =
AP + 18. By substitution,
AP AP + 18
= ; 4AP = 2AP + 36; 2AP = 36; AP = 18.
2 4
To solve for T Q, note that AQ = AP + 2 + 6 = 18 + 2 + 6 = 26. Then
TQ SP TQ 2 TQ 1 26
= ; = ; = ; 9T Q = 26; TQ =
AQ AP 26 18 26 9 9
29) Note first that m may be any of the numbers 1, 2, 3, . . ., 18. (We cannot have m = 19 because
19 + 1 is not less than 20.) If m = 1 then n may be any one of 1, 2, 3, . . ., 18. There are 18
choices in all. If m = 2 then n may be any one of 1, 2, 3, . . ., 17. There are 17 choices in all.
If m = 3 then n may be any one of 1, 2, 3, . . ., 16. There are 16 choices in all. In this way,
when m = 4, 5, 6, . . ., 18, there are 14, 13, 12, . . ., 1 choices for n. Then the total number of
pairs (m, n) is 1 + 2 + 3 + · · · + 18 = 171.

30) Suppose the number is d1 d2 d3 d4 d5 d6 . Then the digits must be 1, 2, 3, 4, 5, and 6 in some
order. Also, we must have d1 d2 divisible by 2, d1 d2 d3 d4 divisible by 4, and d1 d2 d3 d4 d5 d6
divisible by 6. This means that d2 , d4 , and d6 must be even. Then the number has the form

O E O E O E,

Where O represents an odd digit and E represents an even digit. Also, d1 d2 d3 d4 d5 must be
divisible by 5. This means that its last digit must be 0 or 5. Since 0 is not possible, d5 = 5.
Then the number looks like
O E O E 5 E.
Note that the first and third digits are 1 and 3 in some order. If d2 = 4 then d1 d2 d3 must
be 143 or 341. However, neither is divisible by 3. If d2 = 6 then d1 d2 d3 must be 163 or 361.
However, neither is divisible by 3. Therefore d2 = 2, and the number has the form

O 2 O E 5 E.

If d4 = 4 then d1 d2 d3 d4 is either 1234 or 3214. However, neither one is divisible by 4.


Therefore, d4 = 6. By process of elimination, d6 = 4. Then the number has the form

O 2 O 6 5 4.

There are two possibilities remaining: 123,654 or 321,654. In fact, both are curious. Then
there are exactly 2 six-digit curious numbers.
The 2010 Jamaican Mathematical Olympiad
Presented by
The University of the West Indies
in Collaboration with
Sterling Asset Management Ltd.

First Round Examination


Test for Grades 7 and 8

Part A

This part consists of 7 multiple-choicequestions. For each one, write the letter for
the correct answer (a), (b), (c), (d), or (e) in the answer book provided. Each question
in this part is worth 5 marks.

1) What is 50% of 40% of 200?


(a) 180 (b) 50 (c) 90 (d) 25 (e) 40

2) In the diagram below, 6 BOC = 90◦ and 6 AOB is three times 6 COD. What is the
measure of 6 COD?
...
B •........
...
...
... C
...
... .........
• .....
.........
... ...............
• • •
.....................................................................................
A O D

(a) 22.5◦ (b) 45◦ (c) 30◦ (d) 40◦ (e) 15◦

3) Suppose today is Saturday and we count this as Day 1. Which day of the week will
Day 100 be?
(a) Thursday (b) Monday (c) Sunday (d) Tuesday (e) Saturday

4) Consider the sequence of numbers 1, 11, 111, 1111, 11111, . . . If the first 30 numbers
in this sequence are added, what will the tens digit of the sum be?
(a) 1 (b) 9 (c) 4 (d) 2 (e) 3
5) The figure below is made from five identical squares placed side-by-side. If the figure
has a total area of 45 cm2 , what is its perimeter?
..................................................................
.....................................................................
..................................................................
....
..
....
..
....
..
...
....
..................................................................................
... . .
...........................................................................
.
......................
..
.. .... ......................
.
....
...
..
......................
...
..
........................
..
.... ......................
.
...
.........................
...
..

(a) 60 cm (b) 36 cm (c) 12 cm (d) 30 cm (e) 72 cm

6) Consider the numbers between 201 and 699 which are multiples of 5. How many of
them are also multiples of 4?
(a) 99 (b) 19 (c) 24 (d) 40 (e) 25

7) How many prime numbers between 10 and 99 are still prime if you reverse their digits?
(a) 0 (b) 2 (c) 5 (d) 8 (e) 9

Part B

This part consists of three written-answer questions. For each one, give a complete
solution in the answer book provided. Each question in this part is worth 10 marks.

8) The following figure is a magic square with some of its entries filled in. When it is
completed, the sum of the four numbers in each row, column, and diagonal will be
the same. What are the values of A and B?
...................................................................................................
... ... ... ... ...
... 7 ... 12
..
.. A ... . . ...
..................................................................................................
.. ... ... ... ...
.. ... ... ... ...
...
.. 4 ...
...
... 9
...
...
...
...
.
...............................................................................................
... ... . . .
.. ... .... .... ....
..
... 5 ...
. 16 ...
.
...
.
...
.
..................................................................................................
.. ...
... ... ..... ..... .....
8 ..
.. 11 ...
.
...
. B...
.
...
.
..............................................................................................
9) In the diagram below, ABDE is a rectangle and the points C, D, E, and F are
collinear. If the rectangle ABDE has area 80 and the trapezoid ABCF has area 128,
what is AB : CF ?
A B

......
... ...

............................................................
.. ....
... ....
...... ... .. ....
.... ... ... ....
.... .
. .. ....
.... .
. .. ....
.
.... ..
. .
.
...
....
.
... .
. .
. ....
.
... .
. .
.
• ..........................................................................................................................
• • •
F E D C

10) Anna, Marissa, and Nakeisha wish to share out 10 oranges in such a way that each of
them receives at least one. In how many ways can this be done?
THE 2010 JAMAICAN MATHEMATICAL OLYMPIAD

Presented by The University of the West Indies


In Collaboration with Sterling Asset Management Ltd

Qualifying Round
Solutions for Grades 7 and 8

1) Let x be the number. Then


3 3x 48 192
x = 48; = ; 3x = 48(4) = 192; x= = 64
4 4 1 3
The number is 64.

2) There are 4 small rectangles in this diagram. There are 3 rectangles made up of two
smaller ones together, as shown below:
............................................................................................................................ ..........................
.................................................................................... ... ...
.........................
...........................................................................
...... ... ....
....... ...... ........ ..... ..... ..
.. ..
...
.....
...
.....
... ...
.....
...
.....
... ..
...
..
..
...
..
... .. ..
...... ..... ........ ...... ...... ...
.. .. ... ... .. ... ...
...
... ....
... .... .... .. ...
..
....... ...... ........ ..... ..... ..
.. ... ...
. ... ... .. .. .. ..
. ..
......................... .........................
.. ... ... ... ... ... ... ... ... ..
...................................................................................................... ...
.. ... ... ...
.........................................................................
... .
....
.
....
.
....
.
....
..................................................................
...
.

There are 2 rectangles made up of three smaller ones together, as shown below:
................................................................................................................................ ...................................
..........................................................................
...... ...... ..... ........ ..... ...
... ... ...
..
...
.. .....
.. ....
..
....... ...... ...... ........ ...... ..
.... .. ...
....
...
.... .. ...
....
..
...
....... ..... ..... ....... ..... .. ... .. ..
. .. ..

....................................
... ... ... ... ..
....................................................................................................................... .
....
.
....
.
....
.
....
.......................................................................
...
.

And there is the 1 outer rectangle. Then there are 4 + 3 + 2 + 1 = 10 rectangles in all.

3) Let x be the third number. Since the average of these numbers is 2,


5 7
+ +x 5 7 20 21 41 72 31
3 4 = 2; + + x = 6; + + x = 6; +x= ; x= .
3 3 4 12 12 12 12 12

4) First, 6 BAC = 6 BCA because 4ABC is isosceles. Since the sum of the angles in
any triangle is 180◦ ,
6 BAC + 6 BCA + 40◦ = 180◦

26 BCA + 40◦ = 180◦


26 BCA = 140◦
6 BCA = 70◦

Since the lines AC and BD are parallel, 6 CBD = 6 BCA. Therefore, 6 CBD = 70◦ .
5) There are 5 × 3 = 15 children in the community. Since each child has 6 toys, there
are 15 × 6 = 90 toys in the community.

6) The differences between terms are 1, 3, 5, 7, 9, . . .. In this case, the next difference
will be 11. Then the next term is 28 + 11 = 39.

7) According to the first step in this multiplication, A × A is a number ending in 9. The


only possibilities are A = 3 and A = 7. If A = 3 then the product is 3 × 33 = 99. But
this is not a three-digit number. Then the product must be 7 × 77 = 539. Therefore,
B = 5.

8) Since Q and R divide AC into three equal parts, the area of 4BRC is one-third of
the area of 4BAC. Then the area of 4BRC is 12. Since P divides BC into two
equal parts, the area of 4P RC is half the area of 4BRC. Then the area of 4P RC
is 6.
B•..
................
B•...
.. .............
.. ........................ .. ........................
... ...... .......... P ... ...... .......... P
... ...... ......... ... ...... .........
..
.
.
.. ....... •
....... .........
...
....... ................. ..
.
.
.. •
....... ..........
....... .... ..........
....... ... .........
.. .... ...... .. ...... .......
...........................................................................................................
• • •C ............................................................................................................
A A• • • •C
Q R Q R

9) The last digit (in fact, the only digit) of 71 is 7; the last digit of 72 is 9; the last digit
of 73 is 3; and the last digit of 74 is 1. Continuing, the last digits of 75 , 76 , 77 , 78 , 79 ,
. . ., are 7, 9, 3, 1, 7, . . .. In fact, the last digits of the powers of 7 form the sequence
7, 9, 3, 1, 7, 9, 3, 1, 7, 9, . . .. This repeats every 4 terms and continues forever. Note
that the 4th, 8th, 12th, and so on, terms are all 1, and this means that the 2008th
term is 1. Then the 2009th term is 7 and the 2010th term is 9. Therefore, the last
digit of 72010 is 9.

10) There will be 12 straws at the base of the grid, 11 straws in the row above it, 10
straws in the row above that one, and so on. Then the total number of straws on the
base or parallel to it is 12 + 11 + 10 + · · · + 1 = 78. There are also straws forming the
sides of the triangles. On one side, there will be 12 straws on the boundary. Parallel
to them, there will be lines with 11 straws, 10 straws, 9 straws, and so on, down to
one straw. Then there will be 12 + 11 + 10 + · · · + 1 = 78 straws on this boundary
or in a parallel direction. Finally, there will also be a total of 78 straws on the third
boundary or in lines parallel to it. Then there will be a total of 78 + 78 + 78 = 234
straws making the grid.
11) First, the area of 4DAB is half of the area of ABCD. Then the area of 4DAB is 12.
P
A •...................................•..............................•
...............
.... B
...
... . .......
... ... ....... .....
.. ....... ..
... ... ........
...
... ....
..
. .............
...
.......... ....
...
•Q
... ... ......
....... .................... ..
.....................................................................
D• •C
Also, since P is the midpoint of AB, the area of 4DAP is half the area of 4DAB.
Then the area of 4DAP is 6. By reasoning in the same way, one sees that the area
of 4DQC is 6 as well. Similarly, the area of 4P QB is half the area of 4P CB, and
this is half the area of 4ACB, and this is half the area of ABCD. Then the area of
4P QB is
1 1 1
× × × 24 = 3.
2 2 2
Finally, the area of 4DP Q is the area of ABCD minus the areas of 4DAP , 4DCQ,
and 4P QB. That is, the area of 4DP Q is 24 − (6 + 6 + 3) = 24 − 15 = 9.

12) The first letter can be any one from A to Y. (The first letter cannot be Z because there
is no letter after Z in the alphabet.) If the first letter is A, the second letter can be
any of B, C, D, . . . Z. There are 25 possibilities in all. If the first letter is B, the second
letter can be any of C, D, E, . . . Z. There are 24 possibilities in all. If the third letter
is C, there are 23 possibilities for the second letter. Continuing in this way, there are
22 possibilities after a D, 21 possibilities after an E, and so on, up to one possibility
for Y. The total number of pairs of letters is 25 + 24 + 23 + · · · + 2 + 1 = 325. (Note:
there is a “fast formula” for adding the numbers 1 + 2 + 3 + · · · + n. This states that
the sum is n(n + 1)/2. In our case, 1 + 2 + 3 + · · · + 25 = 25(25 + 1)/2 = 25(13) = 325.)

13) Since BCD is an isosceles triangle, 6 CBD = 6 CDB. Also the sum of three angles in
any triangle is 180◦ . Since 6 BCD = 40◦ it follows that 6 CBD = 6 CDB = 70◦ .
A.........................................B
....
... ...............
.. ... ...........
... ...........
.. ....
.. ... ............................................... C
... ..... .
...
...
. ......
................................................................................................
E D

Since 6 BDE = 90◦ we also have 6 EDC = 160◦ . Since the triangle EDC is isosceles,
6 DEC = 6 DCE. It follows that 6 CED = 10◦ . Since 6 AED = 90◦ we must have
6 AEC = 80◦ .

14) The prime factorization of 2010 is 2 × 3 × 5 × 67. To produce a divisor for 2010, we
have two choices for the number of 2’s which may be used (none or one), two choices
for the number of 3s (none or one), two choices for the number of 5s (none or one), and
two choices for the number of 67’s (none or one). There are a total of 2×2×2×2 = 16
possible divisors. They are: 1, 2, 3, 5, 6, 10, 15, 30, 67, 134, 201, 335, 402, 670, 1005,
and 2010.
15) First, 4ABC and 4BDC are right triangles and both have the angle at C in common.
Since the sum of the angles in any triangle is 180◦ , it follows that 6 DBC = 6 BAC.
Then 4ABC and 4BDC are similar. Since corresponding sides of similar triangles are
proportional, BC/AC = CD/BC. Since BC = 3 and AC = 5, we have 3/5 = CD/3.
By cross-multiplying, we have 5CD = 9 and so CD = 9/5.

16) Let ab be a two-digit number, where a and b are each one of the numbers 0, 1, 2, . . ., 9.
Then the value of ab is 10a+b. When the digits are reversed we obtain the number ba,
with a value of 10b+a. In order to be reflexive, we must have (10a+b)−(10b+a) = 27.
This means that 10a + b − 10b − a = 27 and so 9a − 9b = 27. In turn, this means
that 9(a − b) = 27 and so a − b = 3. The possibilities for a and b, respectively, are 3
and 0, 4 and 1, 5 and 2, 6 and 3, 7 and 4, 8 and 5, and 9 and 6. That is, the reflexive
numbers are 30, 41, 52, 63, 74, 85, and 96. There are 7 of them in all.

17) Let x be the width of a single rectangle and 2x be its length. Then its perimeter is
2x + x + 2x + x = 6x. Equating this to 18 gives x = 3. Then each small rectangle has
........3 ............
... ...
.. .3
6 .. . .... 6
2x .... .......................................
... ...
..................................... .. ... 3
.. ... 3 . ..
x ...
....................................
x ..........................................................................
... ... ... 3
3 .. ... ...
2x ................................... ...
. ...
6 3 ..... ....
6
...................
3

width 3 and length 6. It follows that the lengths of the segments in the boundary of
the figure are as shown. The perimeter of the figure is 48

18) First, since a2 + b2 = 4 we have a2 = 4 − b2 . Then

2a2 − 3b2 = 2(4 − b2 ) − 3b2 = 8 − 2b2 − 3b2 = 8 − 5b2 .

Also, b2 ≥ 0 because all perfect squares are positive or 0. Then 5b2 ≥ 0 and it follows
that 8 − 5b2 ≤ 8. On the other hand, if a = 2 and b = 0 then a2 + b2 = 22 + 02 = 4.
Also, 2a2 − 3b2 = 2(22 ) − 3(02 ) = 8 − 0 = 8. Then the greatest possible value for
2a2 − 3b2 is 8.

19 Since A + 1 = B − 2 we have B = A + 3. Since A + 1 = C + 3 we have C = A − 2. Since


A + 1 = D − 4 we have D = A + 5. Finally, since A + 1 = E + 5 we have E = A − 4.
Then

A + B + C + D + E = A + (A + 3) + (A − 2) + (A + 5) + (A − 4)
= 5A + (3 − 2 + 5 − 4)
= 5A + 2
20 The first few three-digit numbers which are odd and divisible by 3 are 105, 111, 117,
. . .. In general, if n is odd and divisible by 3 then the next such number is n + 6.
Since 105 is the first odd number divisible by 3, the others we seek are of the form
105 + 6k for some k. For these to be less than 456 we must have 105 + 6k ≤ 456 and
thus 6k ≤ 351. Since k is a natural number, the solution is k ≤ 58. Starting with 105
(when k = 0) and ending with 453 (when k = 58), there are 59 such numbers in total.

21) Since 4ABC is a right triangle, (AC)2 +(BC)2 = (AB)2 by the Pythagorean theorem.
Since AC = 12 and AB = 20,

122 + (BC)2 = 202 ; 144 + (BC)2 = 400; (BC)2 = 256; BC = 16.

Also, the area of 4ABC is 12 × height × width = 12 × 16 × 12 = 96. Second, 4EBD


is a right triangle and 4ABC and 4EBD have the angle at B in common. Since
the sum of the angles in any triangle is 180◦ , it follows that 6 BED = 6 BAC. Then
4ABC and 4EBD are similar. Since corresponding sides of similar triangles are
proportional, DE/BD = AC/BC. This means that

DE 12 3 30 15
= = ; 4DE = 30; DE = =
10 16 4 4 2

Since the area of 4BDE is 12 × height × width, its area is 12 (10) 15
2 = 75
2 . Finally,
if we subtract the area of 4BDE from that of 4ABC we obtain the area of ADEC.
This area is 96 − 75 192 75 117
2 = 2 − 2 = 2 .

22) The perfect squares less than 18 are 1, 4, 9, and 16. If 16 is the largest summand then
18 can be written in one way: 16 + 1 + 1. If 9 is the largest summand then 18 can be
written in four ways: 9+9, 9 +4+ 4+1, 9+ 4+1+ 1+1+ 1+1, and 9 + 1+1 +· · · +1.
If 4 is the largest summand then 18 can be written in four ways: 4 + 4 + 4 + 4 + 1 + 1,
4 + 4 + 4 + 1 + 1 + 1 + 1 + 1 + 1, 4 + 4 + 1 + 1 + 1 + · · · + 1, 4 + 1 + 1 + 1 + · · · + 1. If
1 is the largest summand then 18 can be written in exactly one way (using eighteen
1s). In total there are 10 ways of writing 18 as a sum of perfect squares.

23) The least common multiple of 4, 5, and 6 is 60. So in order for a number to be divisible
by 4, 5, and 6 it must be a 60-multiple. The 60-multiples less than 1000 are 60, 120,
180, . . ., 960. There are 16 such numbers in total.

24) The figures have 2, 3, 4, . . . rows and 3, 5, 7, . . . columns. That is, the nth figure has
n + 1 rows and 2n + 1 columns. When n = 8 the figure has 9 rows and 17 columns,
for a total of 153 stars.
25) To say that 35 is a divisor of a number n is to say that n is a 35-multiple. Then we
can write n = 35m for some natural number m. The cases with m = 1, 2, 3, 4,and 5
are indicated in the table below.
................................................................................................................................
..... ... ... ...
.. ... ... ...
..
... .....
Largest
..... .....
..
..
m n ...
.
...
Divisor
.
...
.
.........................................................................................................................................
.. ... ... ...
.. ... ... ...
... .... .... ....
.. 1 35
........................................................................................................................................
7
... ... ... ...
.. ... ... ...
.. ... ... ...
2
... 70 . . 35
........................................................................................................................................
.
.. ... ... ...
... ... ... ...
..
3 .. 105 .... 35
....
..........................................................................................................................................
...
.. .. .. ...
.. ... ... .
... ... ... ....
4 .. 140 ... 70 ...
.....................................................................................................................................
.
.. ... ... ...
.. .. .. ...
... ... ... ...
5 .. 175 ... 35 ...
....................................................................................................................................
.

If m = 2, 3, or 5 then 35 is the largest divisor of n. If m is 6 or more then n = 5(7)m.


In this case 7m is a factor of n and 7m > 35 (because m > 5). Then the only
possibility that 35 is the largest factor of n, other than n itself, is when n = 70, 105,
or 175. There are three such numbers.
The 2010 Jamaican Mathematical Olympiad
Presented by
The University of the West Indies
in Collaboration with
Sterling Asset Management Ltd.

First Round Examination


Test for Grades 9, 10 and 11

Part A

This part consists of 7 multiple-choicequestions. For each one, write the letter for
the correct answer (a), (b), (c), (d), or (e) in the answer book provided. Each question
in this part is worth 5 marks.

1) What is 30% of 40% of 400?


(a) 280 (b) 120 (c) 19,200 (d) 48 (e) 160

2) Yesterday, Mrs. Brown went to town. After she spent one-third of her money on
groceries, she deposited $1,000 in her bank account. If she had $200 remaining, how
much money did she start with?
(a) $1,800 (b) $2,600 (c) $3,200 (d) $1,500 (e) $3,600

3) In the diagram below, 6 BOC = 60◦ and 6 COD is two-thirds of 6 BOA. What is the
measure of 6 COD?
..
B •........ ...
.....•
... .....
...
..... ... .... C
... .....
... ........
. .. .
........................................................................................
• • •
A O D

(a) 45◦ (b) 22.5◦ (c) 48◦ (d) 40◦ (e) 30◦

4) In November, a woman told her friend that her son was 100 months old. In which
month was her son born?
(a) March (b) July (c) February (d) November (e) September

5) Shannon picked 17 clovers this morning. If all of them had three or four leaves and
she had a total of 63 leaves, how many four-leaf clovers did she pick?
(a) 1 (b) 17 (c) 63 (d) 5 (e) 12
6) In the diagram below, both circles have centre O and their radii are in the ratio of
3 : 1. Also, AC is a diameter of the larger circle and BC is tangent to the smaller
circle. If AB is 12, what is the radius of the larger circle?
...........................
............ .......
....... ......
...
...... .....
...
. .....
B ............
• .. ....
...
.
... ............. ...
.. .......... ...
... ..........
. ........
.
..... .................. ...
.. .... .... .......... ..
.... .... ... ........... ...
.. .... . ... .
A•......................................................................................................................
• •C
... ... .
. .
... .... O ........ ...
... ......
..............
.... ....
.
... .
... ...
... ...
....
..... .
.......
..... ...
...... .....
....... ......
........... .......
..............................

(a) 12 (b) 24 (c) 6 (d) 18 (e) 30

7) Suppose a girl tells only lies on Monday, Tuesday, and Wednesday, and tells only the
truth on the other days of the week. On how many days of the week can she say, “I
am lying today and I will tell the truth tomorrow”?
(a) 2 (b) 1 (c) 7 (d) 0 (e) 3

Part B

This part consists of three written-answer questions. For each one, give a complete
solution in the answer book provided. Each question in this part is worth 10 marks.

8) Find the smallest natural number N with the properties that when N is divided by 7
its remainder is 4, and when N is divided by 12 its remainder is 5.

9) Suppose a square ABCD is circumscribed about a circle with centre O and radius 1.
Let P and Q be the points such that AP = AQ and P Q is tangent to the circle. Find
the length of the segment OP .
A ...................P B
•..... ..........•............
......................................... ..........
...... ....
..... ..
• .
... ........ ... ..
... ..
........
Q• .....
...
......
.....
... ..
..
......
• ..
......
..... O ..
... ... ....
... ..
.. ...... ..... ..
.. ........ ..... ...
..................................................................................
D
• •C

10) How many natural numbers N have the property that N 2 − 72 is a perfect square?
THE 2010 JAMAICAN MATHEMATICAL OLYMPIAD

Presented by The University of the West Indies


In Collaboration with Sterling Asset Management Ltd

Qualifying Round
Solutions for Grades 9, 10, and 11

1) Let x be the number. Then


4 4x 60
x = 60; = ; 4x = 300; x = 75
5 5 1
The number is 75.

2) There are 12 small triangles in this figure. In addition there are 4 larger triangles, with each
one made up of four smaller triangles as shown below.
............................................................ . . ..........................
...... ......
......................................... ......................................... .........................................

...............................................................................................................................
..... ..... .....
... ... ... ... ... ...
.. ..
... ... ... ... ... ...
.. ..
... ..
... ... ... ... ... ...
.. ..

. . . .
.
..
.. . ......
.. ........ ........ ....
.................................................................... .
.. .. ........ ........ ....
.. . ......
...................................................................... .
.. .. ........ ........ ....
... ..
......................................................................

.................................................................................................................................
. . .
. . . . . .
... .. ... .. ... .. ... ..
.
.......................
.. ... .. ... .. ... .. ...
. .
.................................................................................
.
... .. ... .. ... .. ... ..
.. ... .. ... .. ... .. ...
. .
.......................
.
...............................................................................
.
....
... .. ... .. ... .. ... ..
.. ... .. ... .. ... .. ...
. . .
...............................................................................

There are 16 triangles in all.

3) This sequence repeats every 7 terms. Then the 7th, 14th, 21st, 28th, 35th, and so on, terms
are all ti. This continues up to the 2009th term, which is also ti. Then the 2010th term is doh.

4) Let c be the number of chickens and r the number of rabbits on the farm. Then c + r = 25.
Since each chicken has 2 legs and each rabbit has 4 legs, 2c + 4r = 84. Multiplying the first
equation by 2 gives the system 
2c + 2r = 50
2c + 4r = 84
Subtracting the first equation from the second, 2r = 34 and so r = 17. Then c + 17 = 25 and
so c = 8. There are 8 chickens on the farm.

5) Since P , Q, and R divide AC into 4 equal parts, 4BAP , 4BP Q, 4BQR, and 4BRC have
B
• ...
................
............ .....
.......... .. ... ...
................... .... .... ......
... .. . . ..
...... .... .. .... .....
...... ..... .... ...
...... ..... ..
... ...
.
.......... ........ ...
... ...
..
.. . .
• ........................................................................................................
• • • •
A P Q R C
equal areas. We will write a(BAP ) = a(BP Q) = a(BQR) = a(BRC) in this case. (So,
a(BAP ) stands for the area of 4BAP , and so on.) Then

a(BQR) + a(BRC) = a(BQC); 2 × a(BQR) = 24; a(BQR) = 12.

Then we have a(BAR) = a(BAP ) + a(BP Q) + a(BQR) = 12 + 12 + 12 = 36.


6) The last digit (in fact, the only digit) of 31 is 3; the last digit of 32 is 9; the last digit of 33 is
7; and the last digit of 34 is 1. Continuing, the last digits of 35 , 36 , 37 , 38 , 39 , . . ., are 3, 9,
7, 1, 3, . . .. In fact, the last digits of the powers of 3 form the sequence 3, 9, 7, 1, 3, 9, 7, 1,
3, 9, . . .. This repeats every 4 terms and continues forever. Note that the 4th, 8th, 12th, and
so on, terms are all 1, and this means that eventually the 2008th term is 1. Then the 2009th
term is 3 and the 2010th term is 9. Therefore, the last digit of 32010 is 9.

7) By trial and eror we see that 582 = 3, 364 and 592 = 3, 481. Then 582 < 3, 456 < 592 . It
follows that N = 58.

8) The number of men in the town is 12 × 1, 200 = 600. The number of men who play dominoes
is 13 × 600 = 200. The number of male dominoe players who enjoy golf is 14 × 200 = 50. There
are 50 men who play dominoes and enjoy golf.

9) The sum of the angles in any triangle is 180◦ . Since 6 BAC = 6 DAC = 25◦ ,
25◦ + 90◦ + 6 CBA = 180◦ ; 115◦ + 6 CBA = 180◦ ; 6 CBA = 65◦
This means that 6 CBD = 65◦ as well. Since 4DBC is isosceles, we also have 6 BCD = 65◦ .
Furthermore, the sum of the angles in any triangle is 180◦ . Then
6 BDC + 65◦ + 65◦ = 180◦ ; 6 BDC + 130◦ = 180◦ ; 6 BDC = 50◦

10) If a number is divisible by 3 and 11 then it is divisible by 33. In other words, it is a multiple
of 33. The smallest multiple of 33 which has three digits is 132. This is 4 × 33. The next
multiples of 33 are 165, 198, 231, and so on. These are 5 × 33, 6 × 33, 7 × 33, and so on.
The highest multiple of 33 which less than 1000 is 990. This is 30 × 33. Then the number of
three-digit numbers divisible by both 3 and 11 is the same as the number of numbers in the
set {4, 5, 6, 7, . . . , 30}. This is 27.

11) If a2 + 6b2 = 7ab then a2 − 7ab + 6b2 = 0. Thus (a − b)(a − 6b) = 0. Then, possibly, a − b = 0
and so a = b. Otherwise, a − 6b = 0 and so a = 6b. Since a and b re positve and a > b, we
must have a = 6b. Then
a 6b
= =6
b b

12) First, there are 4 small rectangles as illustrated below. Also, these rectangles can be paired
..................................................................................................... ..................................................................................................... ................................................................................................. .................................................................................................
... ... ..... ... ... .....
.................................... ...................................
... .. ... ... .. ... . .
... ................................................... .. ... ................................................... .. ... ..................................................... ... ... ..................................................... ...
... .... . . .. ... .... . . .. ... .. ... ... .. ...
.... .. .... ..
..........................................................................................................
.... ... ... .. ...
..........................................................................................................
.... ... ... .. ...
...
....................
... .
.........................................................................................................
. ..
....
................... ... .
....................................................................................................
... .....
...
...
...
... ....
..................................................
....
... ....
...
.
...
...
...
... ...
..................................................
....
... ....
...
.
...
...
...
..
................
...
..................................................
....
..
. ....
...
...
...
...
................
... ....
..................................................
....
.
...
...
................................................................................................. ................................................................................................. ... ..
............................................................................................
... ..
............................................................................................

to make larger rectangles in 4 different ways. Finally, the four small rectangles together make a
single larger rectangle. Next, there are 4 larger rectangles as shown below. Also, these rectangles
............................................................................................................................................ ..................................
.................................................................................................. ................................................................................................. .................................................................................................
........ ............................................................. ...... ..
.. ...
. ... ... ... ... ... ... ...

...................................
.... . ... ... .
..................................................... .... ... .
..................................................... ....
...........................................................
....................................................................................................................................................... ...
... ... .. ...
.......................................................................................................
...
.
...
... ... ..
..................................
. .
.
.
............................................................................................................
...
.
...
..................................
.. ... ..
. .
.
.
........................................................................................................
...
.
.... .... ....
.. ... ... ... .. ... .... ... .... ... ... ...
.. ...
.
.... ....
.. . .... ....

................................... ...................................
.... ...................................................... ...
... ... ................................................... ... ... ................................................... ... ... ................................................... ...
... ... .. ... ... ... ... ... ... ... ... ...
.. . .. .. . ... ... . ... ... .
................................................................................................ ............................................................................................. ............................................................................................. .............................................................................................

can be paired to make larger rectangles in 4 different ways. Finally, there is also the outermost
rectangle. A total of 4 + 4 + 1 + 4 + 4 + 1 = 18 rectangles may be found in the diagram.
13) In its decimal form, the number 102010 consists of a 1 followed by a 0 written 2010 times.
Subtracting 2010 from this gives the number represented by a 9 written 2006 times followed
by 7990. The sum of the digits in this number is 2006(9) + 7 + 9 + 9 + 0 = 18, 079.

14) Let x be the width of a rectangle and 3x be its length. Since the area of each rectangle
is 12 cm2 , we have 3x2 = 12. Then x2 = 4 and so x = 2. It follows that segments in the figure
have the lengths shown. The perimeter of the figure is 48.
2
............
..... ....
.. ...
6 ...... ..... 4 6
3x ... .....................................
.....................................
.. ... 4 ..... ..... ..
... 2
x ...
....................................
x ...................................................................................
... .. ..
2 ...................................... ... 4
3x 6
... ...
.. ..
4 ..... ...... 6
..............
2

15) Since (999, 999, 999, 876)2 − (124)2 is the difference of two squares, it may be factored as

(999, 999, 999, 876 + 124)(999, 999, 999, 876 − 124) = (1, 000, 000, 000, 000)(999, 999, 999, 752)

The number 999,999,999,752 obviously has 12 digits. If it is multiplied by 1,000,000,000,000


then the effect will be to add 12 zeroes to the end of this number. Therefore, the total number
of digits in (999, 999, 999, 876)2 − (124)2 is 24.

16) Since the only divisor of 1 is 1, it does not have exactly 4 divisors. Suppose now that n is
a natural number and consider its prime factorization. If this factorization contains three
distinct primes, then n has more than 4 divisors. To see this, suppose p, q, and r are prime
factors of n. Then 1, p, q, r, and pqr are all divisors of n. So, if n has exactly 4 divisors it
must have only 1 or 2 primes in its factorization. Suppose n has two primes, p and q, in its
factorization. Let p represent the smaller one and q the larger one. Then 1, p, q, and pq are
factors of n. If n has exactly 4 factors then these are the only ones. Then pq = n. In this
case, it is possible that p = 2, 3, 5, or 7. (Otherwise, if p ≥ 11 then q ≤ 75/11. This means
that q ≤ 7 and so q < p.) If p = 2 then q could be 3, 5, 7, 11, 13, 17, 23, 29, 31, or 37. There
are 10 choices in this case. If p = 3 then q could be 5, 7, 11, 13, 17, or 23. There are 6 choices
in this case. If p = 5 then q could be 7, 11, or 13. There are 3 choices in this case. If p = 7
then there are no possible choices available. (The next prime is 11 and 7 × 11 is too large.)
Then the total number of choices for primes p and q is 10 + 6 + 3 = 19.
Suppose now that n has only one prime, p, in its factorization. In order to have exactly 4
divisors they would have to be 1, p, p2 , and p3 . Also, we would have p3 = n in this case. The
possiblities are p = 2, n = 8, or p = 3, n = 27, or p = 4, n = 64. There are 3 choices in this
case. Adding these to the case we analyzed earlier, there are 22 numbers less than 75 which
have exactly 4 divisors.
17) By the Pythagorean theorem, (AB)2 + (BC)2 = (AC)2 . Substituting AC = 13 and AB = 12,

122 + (BC)2 = 132 ; 144 + (BC)2 = 169; (BC)2 = 25; BC = 5.

Also 4BDC is a right triangle and 4ABC and 4BDC have a common angle at C. Since the
sum of the angles of any triangle is 180◦ , it follows that 6 DBC = 6 BAC. In this case, the
corresponding angles in 4ABC and 4BDC are equal. Therefore, these triangles are similar.
Then
BD AB BD 12 60
= ; = ; 13BD = 60; BD =
BC AC 5 13 13

18) The equation x2 + ax + b has exactly one real root when a2 − 4b = 0. The values for a and b
that satisfy this relation are a = 2, b = 1; a = 4, b = 4; a = 6, b = 9; a = 8, b = 16; a = 10,
b = 25; a = 12, b = 36. Any other choices for a and b will have the property that a + b > 50.
Then we can choose a and b in exactly 6 ways.

19) Let a1 = m and a2 = n. Then

a3 = a1 + a2 = m + n
a4 = a2 + a3 = m + 2n
a5 = a3 + a4 = 2m + 3n
a6 = a4 + a5 = 3m + 5n

Therefore, m and n are natural numbers such that m < n and 3m + 5n = 46. In this case,
5n = 46 − 3m and so 46 − 3m must be a 5-multiple. The only m-values for which 46 − 3m is
a 5-multiple are m = 2, m = 7, and m = 12. Then there are three candidates for m and n:
m = 2 and n = 8; m = 7 and n = 5; m = 12 and n = 2. Since we must also have m < n, the
only possible solution is m = 2 and n = 8. In this case, a3 = m + 2n = 2 + 2(8) = 2 + 16 = 18.

20) Since OA, OB, and OC are radial segments, they are equal in length. Then 4AOB, 4BOC,

..........
..................... A
.....
...... • ..
...............
........ .. ....
. .... ..... .. .. ...
...
.
........ .... .... ....
.
... .... . ... ...
... ..... .. ... ...
..... .... ... ..
... .....
... ......... ........................
... ....................
•O .
......
... ...
.. ..
....... .... ....
C•
.......................... ..
... ......................... ............
...
.....
......
.. .. . . ..
•B
.. ..........
....
..
........ . .... . .....
.......................

and 4COA are isosceles triangles. This means that 6 OAB = 6 OBA, 6 OBC = 6 OCB, and
6 OCA = 6 OAC. Also, the sum of the angles in 4ABC is 180◦ . Then

6 OAB + 6 OBA + 6 OBC + 6 OCB + 6 OCA + 6 OAC = 180◦


26 OAB + 26 OBC + 26 OCA = 180◦
6 OAB + 6 OBC + 6 OCA = 90◦
40◦ + 30◦ + 6 OCA = 90◦
70◦ + 6 OCA = 90◦
6 OCA = 20◦
21) By the binomial formula,
52010 = (6 − 1)2010 = 62010 + c1 62009 (−1) + c2 62008 (−1)2 + c3 62007 (−1)3 + · · ·
+ c2008 62 (−1)2008 + c2009 6(−1)2009 + (−1)2010

Here, each ck represents the kth binomial coefficient 2010!/ k!(2010 − k)! . Also, each term
except the last one, (−1)2010 , is a multiple of 6. Then each term except the last one is divisible
by 3. Then the remainder when 52010 is divided by 3 will be (−1)2010 = 1.

22) We must consider three possibilities: two apartments receive two samples each; one apartment
receives two samples and two receive one each; or four apartments receive one sample each. In
the first case, there are 10 ways of choosing the two apartments to receive the samples: AB,
AC, AD, AE, BC, BD, BE, CD, CE, or DE. In the second case, there are 5 ways to choose
one apartment to receive two samples. For each such choice, there are six ways to choose
two more apartments to receive one sample each. Then there are a total of 5 × 6 = 30 ways
to distribute the samples in this case. In the third case, there are 5 ways to choose the four
apartments to receive the samples: ABCD, ABCE, ABDE, ACDE, or BCDE. Then there are
10 + 30 + 5 = 45 ways of delivering the samples.

23) Let r1 be the radius of the inner circle and r2 the radius of the outer circle. Then A1 = πr12

A ............................. B A................................C B
............................................. .....................................
...
................ .. ......................... ... ..
............... ..... ..... ...... ... ...
...
... ...
.
... .... ... .. ... ... ... ..
.. .. ... ... ...
... ....
............
r . .
r
2 ............. 1 ............... r ...
2 ...... r ...
... 1 ....
.
.
.
..... ... ..... ... ..... ... .. .
......... ........ ... .. ...
........ .......... ... .... ....
... ....
... ....
O ..
... ..
. ... ... ...
... .. ..
... ...... ... ... ... ... ...
.............. .
...
................. .........
....... .......... ......... ...... ......
.............. ............................
....................
O

and A2 = πr22 . Also, as seen in the enlarged diagram, 6 AOB is 60◦ . This is because this angle
is one-sixth of a full circle and so 6 AOB = 360◦ /6 = 60◦ . Therefore, 6 AOC = 30◦ . Then
√ √
r1 3 3 r2 3r 2

= cos 30 = ; r1 = ; r12 = 2
r2 2 2 4
Finally,  
3r22
π
A2 πr 2 4 3πr22 3
= 12 = = = .
A1 πr2 πr22 4πr22 4

24) By trial and error, one may verify that 1+2+3+· · ·+54 = 1, 485 and 1+2+3+· · ·+54+55 =
1540. Then N = 54. Alternatively, there is a “fast formula” for finding a sum of the form
1 + 2 + 3 + · · · + n. Let S represent this sum. Then

S = 1 + 2 + 3 + ··· + n
S = n + (n − 1) + (n − 2) + · · · + 1
Adding equations, 2S = (n+1)+(n+1)+(n+1)+· · ·+(n+1), where the term n+1 occurs on
the right side n times. Then 2S = n(n+1) and so S = n(n+1)/2. In this problem, we still need
some amount of trial and error. However, one verifies that 1 + 2 + 3 + · · · + 54 = 54(54 + 1)/2 =
54(55)/2 = 1485. Similarly, 1 + 2 + 3 + · · · + 55 = 55(55 + 1)/2 = 55(56)/2 = 1540.
25) Let `1 and `2 be the side lengths of S1 and S2 , respectively. Then A1 = `21 and A2 = `22 . Also,
P1 = 4`1 and P2 = 4`2 . We have
A1 `21 √
= 5; = 5; `21 = 5`22 ; `1 = 5`2 .
A2 `22
(We choose the positive square root above because the length `1 cannot be negative.) Then

P1 4`1 `1 5`2 √
= = = = 5.
P2 4`2 `2 `2

26) The first step in this multiplication indicates that D × AB = AB. Then D = 1. Also, adding
the two partial answers indicates that A + 5 gives an answer ending in 2. Since A is a digit
from 0 to 9, the only possibility is A = 7. Then 7 + 5 = 12. After the 2 is recorded the tens
digit, 1, is carried to the hundreds place. Then 1 + E = 7 and so E = 6. To complete the
analysis, it is clear from the multiplication of the tens digits that B = 3.

27) Since squares of real numbers are always positive or 0, we have b2 ≥ 0. Similarly, a2 ≥ 0.
Since a2 = 10 − b2 we have 10 − b2 ≥ 0 and so 10 ≥ b2 . That is, b2 ≤ 10. Thus 0 ≤ b2 ≤ 10.
Since a2 + b2 = 10,
2a2 + 3b2 = 2a2 + 2b2 + b2 = 2(a2 + b2 ) + b2 = 2(10) + b2 = 20 + b2 .

Then 2a2 + 3b2 = 20 + b2 ≤ 30. Furthermore, 2a2 + 3b2 = 30 when a = 0 and b = ± 10.
Then the largest possible value for 2a2 + 3b2 is 30.

28) Let A be the point where the lines SU and P R meet. Since AU is tangent to the first and
..................................
....... ..... U ...............
..... .....
. ...... T . •
....................................
S ................................................................................................... ..... .............

................................. ..
• .. ... .. ... ...
............................. ..... .... .......... ... ..... ... ..
............................ . .
...............................................................................................................................................................................................................................................................................
• • • •
... . . .. .
.......P.............. Q ......
. ..
.
A .... ...
...
.. ....
.
. .....
R ...
..
.. ....... .
...
...
.....
.
... .....................
...... .....
......... ......
..........................

third circles, 6 ASP and 6 AU R are right angles. Then4ASP , 4AT Q, and 4AU R are right
triangles. Also, they all have the same angle at A. Since the sum of the angles in any triangle
is 180◦ , it follows that 6 AP S, 6 AQT , and 6 ARU are equal. Therefore, the triangles AP S,
AQT , and ARU are similar. Since similar triangles have proportional sides,
AP AR
=
SP UR
The circles at P , Q, and R have radii 2, 6, and 4, respectively. Then AR = AP +2+6+6+4 =
AP + 18. By substitution,
AP AP + 18
= ; 4AP = 2AP + 36; 2AP = 36; AP = 18.
2 4
To solve for T Q, note that AQ = AP + 2 + 6 = 18 + 2 + 6 = 26. Then
TQ SP TQ 2 TQ 1 26
= ; = ; = ; 9T Q = 26; TQ =
AQ AP 26 18 26 9 9
29) Note first that m may be any of the numbers 1, 2, 3, . . ., 18. (We cannot have m = 19 because
19 + 1 is not less than 20.) If m = 1 then n may be any one of 1, 2, 3, . . ., 18. There are 18
choices in all. If m = 2 then n may be any one of 1, 2, 3, . . ., 17. There are 17 choices in all.
If m = 3 then n may be any one of 1, 2, 3, . . ., 16. There are 16 choices in all. In this way,
when m = 4, 5, 6, . . ., 18, there are 14, 13, 12, . . ., 1 choices for n. Then the total number of
pairs (m, n) is 1 + 2 + 3 + · · · + 18 = 171.

30) Suppose the number is d1 d2 d3 d4 d5 d6 . Then the digits must be 1, 2, 3, 4, 5, and 6 in some
order. Also, we must have d1 d2 divisible by 2, d1 d2 d3 d4 divisible by 4, and d1 d2 d3 d4 d5 d6
divisible by 6. This means that d2 , d4 , and d6 must be even. Then the number has the form

O E O E O E,

Where O represents an odd digit and E represents an even digit. Also, d1 d2 d3 d4 d5 must be
divisible by 5. This means that its last digit must be 0 or 5. Since 0 is not possible, d5 = 5.
Then the number looks like
O E O E 5 E.
Note that the first and third digits are 1 and 3 in some order. If d2 = 4 then d1 d2 d3 must
be 143 or 341. However, neither is divisible by 3. If d2 = 6 then d1 d2 d3 must be 163 or 361.
However, neither is divisible by 3. Therefore d2 = 2, and the number has the form

O 2 O E 5 E.

If d4 = 4 then d1 d2 d3 d4 is either 1234 or 3214. However, neither one is divisible by 4.


Therefore, d4 = 6. By process of elimination, d6 = 4. Then the number has the form

O 2 O 6 5 4.

There are two possibilities remaining: 123,654 or 321,654. In fact, both are curious. Then
there are exactly 2 six-digit curious numbers.
Puerto Rican Mathematical Olympiad

Round I

2007/2008

Luis F. Cáceres, University of Puerto Rico, Mayaguez Campus

Jonathan Ho Fung, University of Puerto Rico, Mayaguez Campus

Arturo Portnoy, University of Puerto Rico, Mayaguez Campus

Translated into English by

Raymond McEachin, University of the West Indies, Mona Campus


Preface

Each year, many countries around the world participate in an International Math-
ematical Olympiad. This is a mathematics problem-solving competition in which high
school students tackle significant problems over a two-day period. Many countries also
participate in one of several Regional Olympiads held each year. For example, approxi-
mately 17 countries are expected to participate in the Central American and Caribbean
Regional Mathematical Olympiad to be held next year. This will take place in Mayaguez,
Puerto Rico, from May 26-June 1, 2010. Teams of high school students from throughout
the region will compete for individual honors and participate in several cultural events
during this Olympiad. The University of the West Indies, Mona Campus, has accepted
an invitation to sponsor a Jamaican Mathematical Olympiad Team to participate in this
event. During the 2009/2010 school year, the Department of Mathematics will be orga-
nizing activities which allow us to select national champions at each grade level and form
a national team to participate in this exciting event.
In order to assist us in preparing for this opportunity, the Alliance for the Improvement
of Mathematics Learning (AFAMaC) has kindly provided us with many materials. This
alliance is based in Puerto Rico and represents a joint effort between the Department of
Mathematical Sciences, University of Puerto Rico, Mayaguez Campus, and the Puerto Rico
Department of Education. The materials contained in these pages consist of the questions
and answers used in the first two stages of the Puerto Rican Mathematical Olympiad held
during the 2007/2008 school year.
Each stage of the Olympiad was held at Levels 1 and 2. Level 1 was open to students
in Grades 4–6 and Level 2 was open to students in Grades 7–12. This year, here in
Jamaica, we will only be inviting high school students to participate in the Mathematical
Olympiad. However, we are including in this manual the Puerto Rican questions for Grades
4–6 because they still have a great value. For students and teachers who have no experience
with Olympiad questions, they provide a good introduction to the style and content of the
questions likely to be used in future competitions. And although students will be sure
to get many of them right, many of them will surely generate much discussion and real
thought.

i
In the process of translating the questions into English, a few changes were made in
order to take into account the new context in which they would be used. For example,
almost all of the original questions were multiple choice; in this manual only a very few are.
In some cases, the British units of measure (e.g. feet) were changed to metric units (e.g.,
metres). Also, although the questions are the same (except for any accidental changes
that may have occurred), the solutions are sometimes presented in a different style. In
all cases, the changes were made to make the material seem as natural as possible in the
Jamaican context. Any errors in the translation, the resulting English grammar, or the
mathematical content of the material are entirely my own responsibility. Students and
teachers who have questions on the solutions given or have found mistakes in this manual
are invited to let me know of these matters.
On behalf of the Department of Mathematics, University of the West Indies, Mona
Campus, I thank the AFAMaC for assisting us in many ways as we prepare for this year’s
events. In particular, I thank them for their kind permission to use their past questions
and circulate them freely in Jamaica. I also wish all students good luck and much success
in their mathematical studies this year.

Raymond McEachin
September 21, 2009

ii
First Round Questions
Level I (Grades 4–6)

1) How many rectangles are in the figure below?


.........................................................................................................
.. .. .. .. ...
.. .. .. ..
... ... ... ... .....
.. .. .. .. ...
.. .. .. .. ...
... ... ... ... ....
.. .. .. .. ...
.. .. .. .. ...
... ... ... ...
.. .. .. .. ...
... ... ... ... ...
.. .. .. .. ...
.. .. .. .. ...
... ... ... ... ...
.. .. .. ..
.. .. .. .. .....
... ... ... ... ...
.. .. .. .. .
.......................................................................................

2) What are the next two numbers in the sequence 1, 4, 2, 8, 3, 12, 4, 16, 5, . . . ?

3) How many three-digit numbers can be formed using only the digits 0 and 5?

4) If 20 boxes of papayas weigh 1600 lbs and half a pound of papaya is removed from
each box, how much will the remaining papayas weigh?

5) What is the maximum number of points of intersection that a circle and a square may
have?

6) If ten trees are planted in a row with 15 m between each tree, what is the distance
between the first and last trees?

7) At tea party, Maria, Ana, and Rosa wanted to exchange gifts in such a way that each
would give one and receive one (with no one giving a gift to herself). In how many
ways can this be done?

8) On a certain farm, there are equal numbers of dogs and chickens. If you count the
total number of legs on the farm (not including any other animals), how many could
there possibly be?
a) 4 d) 24
b) 14 e) 32
c) 16

9) When Mario was 27 years old his brother Pedro was 3. When Mario was three times
as old as Pedro, how old was Mario?

1
10) In the sum below, different letters stand for different digits, AH is a two-digit number,
and HEE is a three-digit number. Find H + E.
A H
+ A
...........................................

H E E
11) What is the least number of colours needed to paint a cube in such a way that no two
adjacent sides have the same colour?

12) In the diagram below, the rectangle ABCD has area 36 square units. The points E,
F , and G are the midpoints of the sides on which they lie. What is the area of the
triangle EF G?
B F C
.....................................................................................................................................
.... .....
. ...... ...
......
.. ...... ...... ...
...
.. ..
........ ...... .....
.... ......
..
... .........
. ......
..
...... ....
.. ......
. ........ ..
. ...... ....
.. ..........
... ....... ...... ...
......................................................................................................................................
E ..
...
...
...
G
.. ...
..
... .....
.. ...
.. ...
... ....
..
................................................................................................................................

A D

13) If a cube 5 cm on each side is formed of cubes 1 cm on each side, how many of the
smaller cubes cannot be seen from the outside?

14) Luis wrote down all of the numbers from 1 to 100. What is the total number of digits
Luis wrote down?

15) If the base of a triangle is increased by 10% and its altitude is decreased by 10%, by
how much is its area changed? [You may give your answer as a percent.]

16) One light bulb flashes every 2 minutes and another flashes every 3 12 minutes. If both
flash at midnight, when is the first time after 1 am that they flash together again?

17) A square sheet of paper is folded down the middle to form two rectangles as shown
below. If each rectangle has perimeter 18 cm, what is the perimeter of the original
square?
........................................................................................
.. .. ...
... . ...
.. ...
.. ... ....
... . ...
.. ...
.. ... ...
... .
.. ...
... ... ...
.. . ...
.. ...
... ...
.. . .....
.. ...
... ... ...
.. . ...
.. ...
.. ...
......................................................................................

2
18) At the Balboa Theatre, the price of admission is $7. They have four shows each day.
For the first and second shows, admission is half price. One Thursday, the fourth show
had twice as many patrons as the third, and the third had twice as many as the first
and second shows combined. If they received $1183 that day, how many patrons were
at the fourth show?

19) From an equilateral triangle with perimeter 75 cm, an equilateral triangle with sides
5 cm is removed from each corner. What is the perimeter of the resulting region?
.....
... ...
... ....
... ...
. .....
.....................
............................
...
.....................................................
.. ...
. ...
... ...
...
.............................................................
. ...
... ...
.
.. ...
.............................................................................................
... ...
. . ...
. .. ...
..................
.. ...
...................................................................................................................
..
. ...
... ...
...
. ..
.. ..
...................................................... .....
.
. .... .. .....
.................................................
.. ..
..
. ... ... ...
.................................................................................................................

20) Susana works in a building with 5 offices. In that building, there are three plants: a
cactus, an azalea, and an orchid. Each day, Susana changes the plants in the offices.
In how many ways can she arrange the plants so that not all three of them are placed
in the same office?

3
First Round Questions
Level II (Grades 7–12)

1) On a typical piano, there are 87 keys and 1/3 of them are black while the rest are
white. How many are white?

2) Conchita worked hard and doubled her savings. With this money, she paid off $600
she owed and had $200 left. How much savings did she start with?

3) Antonio is the uncle of Rosa; Ana is the niece of Pedro; Fabiola is the sister of Antonio;
Antonio is the father of Pedro. How are Rosa and Pedro related?

4) Consider 5 whole numbers. How many of them are odd if their product is odd?

5) In the figure below, the letters A, B, C, . . ., G represent numbers (not necessarily


distinct). If the sum of any three consecutive numbers is 18, what is the value of F ?

...........................................................................................................................................................................................................
.. .. .. .. .. .. .. .. .. ...
..
.. 3 A B C D 8 E F G ..
.
..
.
..
.
..
.
..
.
..
.
..
.
..
.
......................................................................................................................................................................................
..

6) How many positive divisors does 5 × 4 × 3 × 2 have? (The divisors of a number include
1 and the number itself.)

7) In the diagram below, the horizontal lines are parallel. What is the measure of 6 x?

.........................................................................................................................................................................................................................
... . ..
... ◦ .............
...
...
40 ..
..
...
.....
... .....
... .....
... ...
......
... . .
.... .. ......
x ... .............
....... ..
.. . . ...
..... ...
.....
...
...... ...
...
....
.
.... ... ◦
...
. .... .
..... ........
... ....... 100
.
... . ..
........................................................................................................................................................................................................................

8) What is the sum of all the digits in the number 102007 − 2007?

9) Maria gets channels 2 through 42 on her television. When scrolling through the
channels they cycle so that after channel 42 comes channel 2 again. If she is watching
Channel 15 and advances through 518 channels, which one will she be watching?

10) Pablo bought an item on sale. After a 15% discount its price was $106.25. What was
its original price?

11) Simon wrote 3 on the chalkboard. The he erased it and wrote its square, 9, in its
place. Then he erased it and wrote its square, 81, in its place. If he repeats this
process 2007 times, what will be the last digit of the final number?

4
12) How many ways are there of ordering the letters L, A, P , I, and Z so that the first
and last letters are vowels?

13) There are some marbles in a bag. Maria said, “There are only 3 marbles in the bag
and all of them are black”. Luis said, “There are only two black marbles and two red
marbles in the bag”. Jorge said, “There are only black marbles in the bag”. If only
one of them is wrong, how many marbles are in the bag?
14) In the diagram below, DE is parallel to AB. If the area of 4DEC is 3/4 the area of
4ABC and AC is 1 m, what is the length of DC?
.......
C .........
... .......
... .......
... ......
... ......
... ......
... ......
... ......
... ......
......
... ......
... .
................................................... E
D ...
...
......
....
...............................................................

A B

15) Some vandals stole all the tires off the cars and motorcycles in a certain street. The
police arrested them and determined that 44 vehicles (cars and motorcycles) were
vandalized. If a total of 144 tires were stolen, how many motorcycles were in the
street?

16) The diagonals of a rhombus are in the proportion 3 : 4 and their sum is 56 cm. What
is the perimeter of the rhombus?

17) The rows and columns of an 8 × 8 square board are numbered from 1 to 8 as shown
below. In each square, Maurice placed as many checkers as the sum of the row and
column it was in. How many checkers did Maurice place all together?
1 2 3 4 5 6 7 8
....................................................................................................................................................
... ... ... ... ... ... ... ... ...
1 .. . . . . .
....................................................................................................................................................
. .
... ... ... ... ... ... ... ... ....
2 .. .. .. .. .. .. .. .. ..
.. . . . . . . .
............................................................................................................................................................
.. .. .. .. .. .. .. .. ...
3 .. .. .. .. .. ..
....................................................................................................................................................
.. .. .
.. .. .. .. .. .. .. .. ..
.. .. .. .. .. .. .. .. ...
4 ... ... ... ... ... ...
.............................................................................................................................................
... ... .
.. .. .. .. .. .. .. .. ...
... ... ... ... ... ... ... ...
5 .. . . . . .
...................................................................................................................................................
. . ...
... ... ... ... ... ... ... ... ...
6 .. .. .. .. .. .. .. .. ...
.. . . . . . . . .
...........................................................................................................................................................
.. .. .. .. .. .. .. .. ...
7 .. . . . . . .
...........................................................................................................................................................
. .
.. .. .. .. .. .. .. .. ...
.. .. .. .. .. .. .. .. ...
8 ... . . . . . .
..........................................................................................................................................................
. .

18) Consider all the four-digit numbers which can be formed using the digits 3, 4, 6, and 7
without repetition. How many of these numbers are divisible by 44?

5
19) In the triangle below, ABC is an equilateral triangle and DEF G is a square. If AB
is 1 m, what is the length of DE?
C
.....
... ...
... .....
.... ...
. ...
... ...
.... .
..........................................
.
.
G
....
.
... ...... F
.. ...
. ... ....
.. . ... ....
.... ... .. ...
... .. ... ...
... .
.
. .... ...
... .
. ... ...
. . .
..................................................................................
.
. .
A D E B

20) If x is the solution to the equation

x+1 x+2 x + 100


+ + ···+ = 100
1 2 100

then what is x?

21) In the figure below a small square is inscribed inside a circle and a larger square is
circumscribed about the circle. What is the ratio between the areas of the larger and
smaller squares?
..............................................................................................................................
... ........ ........ ...
.. ...... ...... .
......
.. ...... . ....
... ............................................................................................ .
.. .... ... ... .... ....
.. ... .. .. .... ...
... ... .. ... ... ...
..... .. ... ......
..... ... .... .....
.... .. ......
.. .. .... ...
... ... ... ...
..
....
..
..
...
... .....
..... ... .. ..
..... . ..... .....
... ... ... .. .... .....
.. .... ... ... ... ..
.. ... .. .. ... ...
... ..... .. .. .
.. ........................................................................................ ....
.. ..... .
... ...
... ...... .
...... ...
....... ....
.. .... ....... ..
..............................................................................................................................

22) On a certain island, there are only two types of people: those who always tell the
truth and those who always lie. Three inhabitants of the island were talking. Andrea
said, “Barbara always tells the truth”. Barbara said, “Andrea and Carlos always tell
the truth”, and Carlos said, “Andrea lies”. It follows that:
a) The three of them tell the truth;
b) Andrea and Barbara tell the truth and Carlos lies;
c) Andrea tells the truth and Barbara and Carlos lie;
d) Andrea and Barbara lie and Carlos tells the truth;
e) The three of them lie.

23) Suppose 50 distinct numbers are chosen from {1, 2, 3, . . . , 100}. If their sum is 3000,
what is the least number of even numbers that are possible among the 50?

24) Consider a certain set of 20 consecutive numbers greater than 50. What is the largest
number of primes possible in the set?

6
25) A student passed a certain number of examinations with an average of 23. After he
sat one more exam, his average was 22.25. What are the possible scores on his last
exam if his score on every exam was between 18 and 30 (including 18 and 30)?

26) An equilateral triangle has the same perimeter


√ as a rectangle with dimensions b and h
(with b > h). If the area of the triangle is 3 times that of the rectangle, what is the
value of b/h?

27) Suppose Q is a cube and S is a sphere whose center is at a vertex of Q and radius is
equal to a side length of Q. What is the volume of the intersection of Q and S as a
proportion of the volume of Q?

28) Let P (x) = x3 +ax2 +bx+c and suppose the sum of two of the roots of the polynomial
is 0. Which of the following relationships is certain?
a) a × b × c = 0 d) b2 = a × c
b) c = a × b e) None of the above
c) c = a + b

29) Let ABC be an isosceles triangle with AB = BC 6= AC, and let P be a point on AB.
In how many ways can a point Q be chosen so that 4AP Q is similar to 4ABC?

30) Alberto, Barbara, Clara, and David are playing a game with 40 cards from a standard
deck of cards. After one deal Alberto exclaimed, “How unusual, I have no spades in
my hand!” If 10 of the 40 cards were spades, what was the probability that Barbara
doesn’t have any spades in her hand either? (You may give your answer in terms of
factorials and other expressions without simplifying.)

7
First Round Solutions
Level I (Grades 4–6)

1) There are four basic rectangles not made up of smaller ones. There are three more
rectangles made up of two smaller ones each, two rectangles made up of three smaller
ones each, and the outer rectangle (which is in fact a square) made up of the four
smaller ones. Thus there are 4 + 3 + 2 + 1 = 10 rectangles in all.

2) The first, third, fifth, etc., terms are 1, 2, 3, etc. The second, fourth, sixth, etc., terms
are 4, 8, 12, etc. Following these progressions separately, the next two terms are 20
and 6.

3) Reading from right to left, the there are two choices for the last digit, two for the middle
digit, and one for the first digit (which must be a 5). Then there are 1 × 2 × 2 = 4
such numbers in total, which are 500, 505, 550, and 555.

4) If half a pound is removed from each box then a total of 10 lbs is removed from the
20 boxes. The remaining papayas will weigh 1590 lbs.

5) In the diagram below, the square intersects the circle in 8 points. This is the maximum
because no side of a square can cut a circle in more than two points. Since squares
have four sides, no square can intersect a circle in more than eight points.
..............
...............................................................
....... ......
...
....
... ........
...... ..
...... .........
...... ..
.... ....
....... ......
..........................................................
....................

6) If there are ten trees then there are nine spaces between them. Since each space is
15 m, the distance between the first and last trees is 135 m.

7) Suppose Maria gives a gift to Ana. Then Ana cannot give one to Maria or else Rosa
must give a gift to herself. So Ana must give a gift to Rosa and Rosa must give one
to Maria. If Maria doesn’t give a gift to Ana then she must give one to Rosa. In this
case, Rosa will give one to Ana and Ana will give one to Maria. Thus there are two
ways of exchanging gifts at the party.

8) If there are n dogs and n chickens then there are 4n legs on the dogs and 2n legs on
the chickens. The total number of legs is 6n. The only possible number of legs is 24
because that is the only 6-multiple among the choices offered.

9) After n years, Mario was 27 + n and Pedro was 3 + n. Mario would have been three
times as old as Pedro when 27 + n = 3(3 + n). In this case,

27 + n = 9 + 3n; 18 = 2n; 9=n

1
Thus after 9 years Mario was three times as old as Pedro, and this happened when
Mario was 36 and Pedro was 12.

10) Note that AH can be no larger than 99 and A can be no larger than 9. Then HEE
must be a number between 100 and 108. In particular, H = 1. Also, A = 9 because
if it were less than 9 then the sum of AH and A would be less than or equal to 89.
Then A = 9, AH = 91, and HEE = 100. Thus H + E = 1.

11) Suppose one side, say the top, is red and an adjacent side, say the front, is blue. A
side adjacent to both of them must be another colour such as green. However, with
these three colours the rest of the cube can be coloured as shown. (The back of the
cube would be blue.) The least number of colours needed is 3.

..
R R
..
........................................................ ......................................................
....... . ....... .
....... . ....... .... ....... . ....... ....
.....
................................................
. ... .....
..................................................
. ...
..... ...
.. ..... ...
... .. ..... ...
.
.. .... ... .......
. .... ..................
...
.. ... ... G ... ... ... G
..
..
B ..
... ..
... ..
..
B ..
... ...
...
... ... ...
. ... ... ...
.
.. ... ............. .. ... .............
................................................ ................................................
.
R

12) The area of rectangle BCGE is half the area of ABCD and the area of triangle EF G
is half the area of BCGE. Then the area of the triangle EF G is 9 square units.
B .............................................................F
.................................................................
C
... ......... ...
...... .. ...... ...
.. ...... . ...........
..
.. ....
....... ... ......
......
...
...
... ......
. .
. . ...
...... ......
.. ..
. .
. ... ........ ...
... .
..... ........ .
... .
..... .
. ....... .....
... ........... ..
. .
.................................................................................................................................................
E ...
..
...
...
G
.. ....
... ...
.. ...
.. ...
...
.. ...
.. .
..................................................................................................................................

A D

13) If you remove all the small cubes which can be seen from the outside, there would
remain a 3 × 3 × 3 cube which could not have been seen from the outside. Thus there
are 27 smaller cubes which cannot be seen from the outside.
..........................................................................................................
....................................................................................................................................................... .....
......... ....... ........
.
...
..
..
..
.................................................................................................................................................................................................................................................. .... ...
..
..... ..... .... ..... .... .... ... ...
........................................................................................................................... ..... .. ..............
................................................................................................................................ ... ..
.. ... ... ... ... ... ... .... ........................ ....
.. ................. ... ... .................................................................................
... .. .. .. .. ..
... ... ................................................................................... ....
.. . .
...............................................................................................................................
. . . ....... .. ... .. ...................................................................... .......... ....
... .... ......................... .......... . ....... ........ ............... ...
... ... ... ... ... ... ... ... ... .................................................................... ... ... ...
.. ... .. ... ... .. ... .. ............... .... .... .. .. .. .. ... ....................
... ... ... ... ... ... ......................... ... ... ... .. .. .. .
... ..
. .... .. ... ... .. .. ... .......................... .... ...
........................................................................................................................... ....
.
..... . .
... ................. ... . ..
. . .................................................................. ... .. ..
.... ... ... ... ... ... ... ... . ... ... ... ... . ... ..............
.
.. ... ... ... ... ... .. .......................... .... ..... .. .... .... ....
...
.................. ..
. .. .. .. .. .. ............ ... .. ... . .. . .. ...
............................................................................................................................. .. .... .................. .................................................................................... ... .
..
..
... ... ... ... ... ... .
... .............. ..
..
....
. ... ...
.. .... .... .... ..
... ... .......
..... ..... ..... ..... ..... . .. . ..... .. ....................
... ... ............... ... ..... .. .... .... ....
........... ... ... .. .. .. .................
................................................................................................................................ .... ... .. .................... ...........................................................
.... ... ... ... ... ... ... ... ................
.. ... ... ... ... ... ... .............
... . . . . . ..........
......................................................................................................................

2
14) Luis wrote down 1 digit for each number from 1 to 9, 2 digits for each number from 10
to 99, and three digits for the number 100. Thus he wrote down 9(1) + 90(2) + 1(3) =
192 digits in total.

15) Let b and h be the original base and height, respectively, and B and H the new
ones. Then B = b + 0.1b = 1.1b and H = h − 0.1h = 0.9h. The new area is
1 1 0.99 1
2 BH = 2 (1.1b)(0.9h) = 2 bh. Since the original area is 2 bh, the new area is 0.99
or 99% of the original one.

16) The first bulb will flash after 2, 4, 6, 8, 10, 12, 14, . . ., minutes. The first time on
this list that is a multiple of 3 12 minutes is 14 minutes. It follows that the bulbs flash
together every 14 minutes. This means that they flash together at 12:14 am, 12:28 am,
12:42 am, 12:56 am, and 1:10 am. The first time after 1 am that they flash together is
1:10 am.

17) Referring to the diagram below, a + 2a + a + 2a = 18. Thus 6a = 18 and hence a = 3.


It follows that the perimeter of the original square is 24
a a
........................................................................................
.. .. ...
... .
.. .....
.. ... ....
... . ...
.. ...
.. ... ...
... .
.. ...
...
2a ...
..
..
.
...
...
...
2a
... ...
.. . .....
.. ...
... ... ...
.. . ...
.. ...
.. ...
.......................................................................................

a a

18) Let A, B, C, and D be the number of patrons at the four shows that day. Then
C = 2(A + B) and D = 2C = 4(A + B). The money the theatre received was

3.50A + 3.50B + 7C + 7D = 3.50(A + B) + 14(A + B) + 28(A + B) = 45.50(A + B)

Thus 45.5(A + B) = 1183 and hence A + B = 26. There were 4(26) = 104 patrons at
the fourth show.

19) Each side of the original triangle is 25 cm long. After removing the smaller trian-
gles each longer side has length 15 cm. Since each shorter side has length 5 cm, the
perimeter of the resulting region is 60 cm.

. .5. . .
....................................
............................
... ...
.. ...
... .........
...............................................................
.. ...
.
. ...
. ...
.
....
..
15 ....................................... 15
. ...
...
..
..............................................................
..... ...
...
...
...............................................................................................................
..
. ...
.
. ...
..
. .
.....
.............................................................................................
...
... ....
.
...

5 ................................................. 5
... .
.
.
.....................................................................
15

3
20) There are two possibilities to consider: three offices contain one plant each or one
office contains two plants while another has one. In the first case, one plant may be
placed in any of 5 offices, another placed in any of the 4 remaining offices, and the
third in any of the last three offices. Thus there are 5 × 4 × 3 = 60 ways of placing the
plants this way. In the second case, there are three ways of pairing two plants to be
placed in the same office: the cactus and the azalea, the cactus and the orchid, and the
azalea and the orchid. Then the pair may be placed in any of 5 offices and the single
plant placed in any of the remaining 4 offices. Then there are 3 × 5 × 4 = 60 ways
of placing the plants in this case. Combining the two cases, there are 60 + 60 = 120
ways all together of placing the plants.

4
First Round Solutions
Level II (Grades 7–12)
2
1) There are 3 × 87 = 58 white keys on a piano.

2) Since Conchita had $200 left after paying off $600 she owed, she had $800 on hand.
Since this is twice her original savings, she started off with $400.

3) Since Antonio is the uncle of Rosa and the father of Pedro, Rosa and Pedro are cousins.

4) If a single one of the 5 numbers is even then their product is even. Since their product
is odd, all 5 numbers are odd.

5) Since 3 + A + B = 18 and A + B + C = 18, it follows that C = 3. Since C + D + 8 = 18


it follows that D = 7. Since D + 8 + E = 18, we have E = 3. Since 8 + E + F = 18
we have F = 7.
6) First, 5 × 4 × 3 × 2 = 120. Its divisors are 1, 2, 3, 4, 5, 6, 8, 10, 12, 15, 20, 24, 30,
40, 60, and 120. It has 16 divisors. For an alternate solution, note that the prime
factorization of 120 is 23 × 3 × 5. A factor of 120 may contain 0, 1, 2, or 3 twos, 0 or
1 three, and 0 or 1 five. Then there are 4 × 2 × 2 = 16 factors of 120.

7) Since a is opposite an angle of 40◦ , it is 40◦ . Since s is the supplement for a 100◦
angle, it is 80◦ . Since a + s + y = 180◦ , y = 60◦ . Since x and y are supplements,
x = 120◦ .
.........................................................................................................................................................................................................................
... . ...
... ◦..............
...
...
40 ...
....
.
... .....
.....
... .....
.
...
. ...
......
. . .
x .... ... ......
.... ............
.
.
.. . ...................
.... ...
...
...... y ...
... ...
..
....... ... ◦
.. .
........
.
. 100
..
.............
.. .... ......
..
.. .
..... a s .
.
..
.........................................................................................................................................................................................................................

8) To subtract 2007 from 102007 it is easiest to subtract 1 first and then subtract 2006
more. Since the numeral for 102007 is 1 followed by 2007 zeros, subtracting 1 gives the
number with 2007 digits which are all 9. Subtracting 2006 from this number gives the
number with 2007 digits with 9 repeated 2003 times followed by 7993. Then the sum
of the digits is 9(2003) + 7 + 9 + 9 + 3 = 18, 055.

9) Each cycle through the loop consists of 41 channels. So she will be back to Channel 15
after advancing through 41, 82, 123, 164, . . ., 492 channels. This leaves 26 channels
to go, and Maria will end up watching Channel 41.

10) If P is the original price then P − 0.15P = 106.25. Thus 0.85P = 106.25 and so
P = $125.

5
11) If we note only the last digits of the numbers Simon wrote, we obtain 3, 9, 1, 1, 1, 1,
. . .. Then after 2009 times, the last digit is 1.

12) There are 2 choices for the first letter: A or I. There are 3 choices for the second one:
L, P, or Z. Then there are 2 choices for the third letter, 1 choice for the fourth one,
and 1 choice for the last one. Thus there are 2 × 3 × 2 × 1 × 1 = 12 possible orderings.

13) If Maria is wrong then both Luis and Jorge would be right. Since they contradict
each other, this is impossible. Then Maria is right, Jorge is right, and Luis is wrong.
Since Maria is right, there are 3 marbles in the bag.

14) Since the triangles shown are similar,


 2  2
DC 3 DC 3 3
= ; = ; (DC)2 = .
AC 4 1 4 4

3
Then DC = 2 m.

15) If c is the number of cars and m the number of motorcycles in the street then c+m = 44
and 4c + 2m = 144. By eliminating one of the variables, one determines that c = 28
and m = 16. In particular, there were 16 motorcycles in the street.

16) if 3x and 4x are the distances (in centimeters) as shown, then the sum of the diagonals
is 6x + 8x = 14x = 56. Then x = 4. If s is the length of a side as shown, then
s2 = 122 + 162 = 400 and hence s = 20 cm. Then the perimeter of the rhombus
is 80 cm.
.............. ..............
...... .. ...... ...... .. ......
...... .... ........... ...... .... ...........
.
.....
..
..
..... ...... s .
.....
..
..
..... ......
...... 3x ...... ...... 12 ......
...... .. ...... ...... .. ......
....... ... ......
....... ... ......
...................................................................................................
. . . . ...................................................................................................
. . . .
...... .
... ..... ...... .
... .....
...... 4x
...... ....
......
.
.....
. ...... 16
...... ....
.
.....
......
.
...... .. ...... ...... .. ......
..... . .
..
. ..... . .
..
.
...... ... ...... ...... ... ......
...... .. ...... ...... .. ......
............ ............

17) In the first row, Maurice placed 2 + 3 + 4 + · · · + 9 = 44 checkers. In the second


one, he placed 3 + 4 + 5 + · · · + 10 = 52 checkers. In the third row, he placed
4 + 5 + 6 + · · · + 11 = 60 checkers. Continuing in this way, he placed 68, 76, 84, 92,
and 100 checkers in rows 4, 5, 6, 7, and 8, respectively. Then the total number of
checkers he placed is 44 + 52 + 60 + · · · + 100 = 576

18) In order to have a number divisible by 44 it must be divisible by both 4 and 11. Also,
a number is divisible by 4 if and only if the number formed by its last two digits is
divisible by 4. Thus the only numbers we can form which are divisible by 4 are 3764,
7364, 4736, 7436, 3476, and 4376. Among these numbers, the only ones divisible by 11
are 7436 and 3476. These two numbers are the only ones divisible by 44.

6
19) Let H be the point shown in the diagram below. Since AB is 1 m, AH = 1/2 m and
.
C
......
........
.
.... .. ....
. . ..
... . .....
... ... ...
...
. ..
G ...........................................
. F
......... .
. ........
.. ..
. .
. ... ....
.. . ... ...
.... .... .
. ... ....
... .
. .
. ... ...
... .
. ... ...
... .
. .
. ..
.
......................................................................................
A D H E B


HC = 3/2 m. Also, if s denotes the side length of the square then AD = (1 − s)/2 and
DG = s. Since the triangles ADG and AHC are similar,
1 s 1
2 − 2 2 1−s 1
= √ ; = √
s 3 2s 3
2

By cross multiplying, we obtain



√ √ √ √ 3
3− 3s = 2s; 3 = (2 + 3)s; s= √
2+ 3

20) We have
       
x 1 x 2 x 3 x 100
+ + + + + + ···+ + = 100
1 1 2 2 3 3 100 100
       
x x x x
+1 + +1 + + 1 + ···+ + 1 = 100
1 2 3 100
After canceling 100 from both sides,
 
x x x 1 1 1
x+ + + ···+ = 0; x 1+ + + ···+ =0
2 3 100 2 3 100
It follows that x = 0.

21) Let r be the radius of the circle. The larger square has side length 2r and hence has
area (2r)2 = 4r2 . If x is the side length of the smaller square then, by the Pythagorean
theorem, x2 + x2 = (2r)2. Thus 2x2 = 4r2 and hence x2 = 2r2 . Then the area of the
2
square is x2 = 2r2 . The ratio of the larger area to the smaller one is 4r 2r2 = 2.
............................................................................................................................
. ........
.. ........ ...... ...
... ...... x ..... .....
.. ..... ..
.. ................................................................................................. ..
... ... ..
.. .... .. ..
.... .. ... ....
.. . .
.
... ... .... ...
... ... ...
.... .. r....
.
.....
.
. ... ... ...
... ......
..... .. ..... ......
.....
..
...
.. ........
.
...
... ....
..
... . ..
...
x ..
. ..
.............................................................
.
...... .... ... .... .
.... .. .
....
.
. r .
.....
.
.......
.
......
.. ... ....
.
.
. . .
........
.. ... .. .... .... ..... ....
... .... .. .... .
... ... ...
.
.. ..... ... ..
.. .......................................................................................... ....
... ..... ..
.. ...
...... ...
.. ....... ...... ...
.. ... ........ .
.............................................................................................................................

7
22) Suppose first that Andrea tells the truth. Then according to her statement, Barbara
always tells the truth and, according to Barbara’s statement, Carlos tells the truth.
Finally, according to Carlos’s statement, Andrea lies. Since this is a contradiction,
Andrea does not tell the truth. This means that Barbara lies but Carlos tells the
truth. The correct answer is that Andrea and Barbara lie and Carlos tells the truth.

23) Suppose we try to use no even numbers at all. Then we would have to choose 1, 3,
5, . . . , 99, but their sum is only 2500. If we try to use one even and 49 odd numbers
the resulting sum will be odd. Thus we must use at least two even numbers. If we
try to use two even numbers and 48 odd numbers the largest possible sum would be
obtained by replacing 1 and 3 with 98 and 100. In this case the new sum will be
2500 − 4 + 198 = 2694. So we need to replace at least two more odd numbers with
evens. Doing this, we will obtain the largest possible sum by replacing 5 and 7 with
94 and 96. In this case, the new sum will be 2694 − 12 + 190 = 2872. So we still
need at least two more even numbers. We can remove 9 and 11 and replace them with
any even numbers whose sum is 148. For example, we could use 92 and 56. Thus
the smallest number of even numbers required is 6, and this can be achieved by using
{56, 92, 94, 96, 98, 100} ∪ {13, 15, 17, . . . , 99}.

24) Among the 20 consecutive numbers 97, 98, 99, . . ., 116, we find the following six
primes: 97, 101, 103, 107, 109, and 113. We show that this is the maximum number
of primes that can be found. Note first that any 20 consecutive numbers must contain
10 even numbers. These are multiples of two and hence composite. Among the 10
remaining odd numbers, at least three of them must be multiples of 3. These must
also be composite. Also, among the 10 odd numbers, there must be two multiples of 5;
one of them, but not both, can be a multiple of 3 which has already been counted.
Then there must be:
10 numbers which are multiples of 2;
3 numbers which are multiples of 3 but not multiples of two;
1 number which is a multiple of 5 but not a multiple of 2 or 3.
This means that any 20 consecutive numbers must contain at least 14 distinct numbers
which are composite. Therefore, at most 6 can be prime.

25) Suppose his average after n exams was 23 and after n + 1 exams was 22.25. If tn was
his total after n exams then tnn = 23 and hence tn = 23n. If ` denotes his score on
the last exam, then

tn + `
= 22.25; tn + ` = 22.25n + 22.25
n+1

Since tn = 23n we obtain 23n + ` = 22.25n + 22.25 and hence ` = −0.75n + 22.25.
Multiplying throughout by 4 we obtain 4` = −3n + 89. Thus to summarize the
situation, we know that ` is an integer, 18 ≤ ` ≤ 30, and 4` = −3n + 89 for some

8
positive integer n. By trial and error we see that n = 3 is the only possible choice
for n, and in this case ` = 20. Then the student scored 20 on his last exam.

26) Since the equilateral triangle has perimeter 2b+2h, each of its sides has length 23 (b+h).

Also, its area is one-half the product of its base and height. Since its height is 23 of
a side length, its area is
 √   √
1 2 3 2 3
(b + h) (b + h) = (b + h)2
2 3 2 3 9

Since this area is 3 times that of the rectangle,

3 √ 1
(b + h)2 = 3bh; (b + h)2 = bh
9 9
Then (b + h)2 = 9bh and it follows that b2 − 7bh + h2 = 0. Dividing throughout by h2 ,
 2  
b b
−7 +1=0
h h
√ √
Using the quadratic formula, one obtains hb = 7±32 5 . The choice b
h = 7−3 5
2 implies

that hb < 1 and hence b < h. Since this is impossible, hb = 7+32 5 .

27) Let the radius of the sphere and the side length of the cube (both of which are equal)
be denoted by a. Since the center of the sphere is at a vertex of the cube, exactly 18
of the volume of the sphere is inside the cube. Then the volume of the intersection is
 
1 4 3 π
πa = a3
8 3 6

Since the volume of the cube is a3 , the proportion of its volume which is in the
intersection is π6 .

28) Let r1 , r2 and r3 be the roots of the polynomial, with r1 + r2 = 0. Then, on one hand,

P (x) = (x − r1 )(x − r2 )(x − r3 )


= x3 − (r1 + r2 + r3 )x2 + (r1 r2 + r1 r3 + r2 r3 )x − r1 r2 r3

Since r1 + r2 = 0, the coefficient for x2 is −r3 . Also, the coefficient for x is

r1 r2 + (r1 + r2 )r3 = r1 r2 .

Thus P (x) = x3 − r3 x2 + r1 r2 x − r1 r2 r3 . On the other hand, since P (x) = x3 + ax2 +


bx + c we have a = −r3 , b = r1 r2 , and c = −r1 r2 r3 . Among the stated relations, the
one that is certain is a × b = c.

9
29) There are 4 locations for the point Q. Two are such that the angle Q is the same
as B, and two such that the angle Q is the same as A or C. The four locations are
shown below.

B ............... B ...............
. .
... ... ... ...
... .... ... ....
.... ... ...
.
...
.
. ... .
. ...
.
.... ...
... .
.... ...
...
..
. ... ... ...
.
.. ... ..
. ...
... ... ... ...
.
. ... .. ...
Q P .
..
.
.
.
.....................................................................................................................
...
...
Q P
.................................................... ..
.
.
.. ...
...
...... . ...... ... ... . ...... ...
...... .
.
. ...... ... ... ..
. ...... ...
...... .
. ...... ... .
.. ...... ...
......
...... .
.... ......
.......
... ...
... .... ......
.
.
...
...... .. ... ... . ...... ...
. . ...... . .
. ...... ...
......
...... .
.
.
. . ...... .... ...
... .
.
.
.
... .. . ...
...... .
.
.
....... ... ..
. .. .
.... ...
......
...... .
.
.
. ..........
.......
...
... .
.
..
.... ........
Q ...
...
...... ... .......... ... ...
........ ...
......
...... .... ... ....... ... ...
... ... ..........
... ...
...... ...
...... ...
. ... .......
.. ..... Q ... ... ........
................................................................................................
...
.....................................................................................................................

A C A C

30) Of the 30 cards outside of Albert’s hand, 10 are spades and 20 are not. The number
of hands without spades that Barbara could hold is
 
20 20!
=
10 10!10!

The total number of hands Barbara could hold is


 
30 30!
=
10 10!20!

The probability that Barbara holds no spades in her hand is


20!

10!10! 20!20!
30!
 =
10!20!
10!30!

10
Puerto Rican Mathematical Olympiad

Round II

2007/2008

Luis F. Cáceres, University of Puerto Rico, Mayaguez Campus

Jonathan Ho Fung, University of Puerto Rico, Mayaguez Campus

Arturo Portnoy, University of Puerto Rico, Mayaguez Campus

Translated into English by

Raymond McEachin, University of the West Indies, Mona Campus


Second Round Questions
Level I (Grades 4–6)

1) Rosita has a large box with 4 medium-sized boxes inside; in each of the medium-sized
boxes there are two small boxes, and in each small box there are three tiny boxes.
How many boxes does Rosita have?

2) Which number is missing in the following progression: 1, 3, 7, 15, , 63, 127?

3) How many triangles are in the following figure?


................................................................................................
.... ....... ..... .. ...
.....
.. ...... ..... .. .....
.. ..... ........ ... ....
.
... ..... ..... . ..
.
.. ... ...
.. ........ ... .....
.. ..... ......... .....
... ......... ..... ... .........
.. ..... ..... .. ....
................................................................

4) Antonio, Beatrice, Carlos, and Diane were seated in a line with four chairs numbered
from 1 to 4. Emilio came by and said,
• Beatrice is beside Carlos;
• Antonio is between Beatrice and Carlos.
If both statements are false and Beatrice is in Chair 3, who sits in Chair 2?

5) An animal trainer has three tigers and two lions. He would like to line them up in
such a way that no two lions and no two tigers are together. If each lion and tiger is
distinct from the others, in how many ways can he line up his animals?

6) Which number multiplied by 3 corresponds to three-fourths of 120?

7) Each white square below has a side length of 6 cm, and the perimeter of each shaded
rectangle is double that of a white square. What is the perimeter of the figure below?
...............................................
.............................................................................................
..
..
..
.............................................................................
...
..
...
..
. . . . . .. .. .. .. .. .. .. .. .. .. .. . . . . .
.................................................................................................................
...
.. ..............................................................
...
..
...
..
..
.. ..............................................................
...
..
..........................................................................................
...
..

8) On a certain planet, the number of days in a week are the same as the number of
weeks in a month, and this is the same as the number of months in a year. If there
are 1000 days in a year, how many days are in one week?

9) A cake is cut and 13 of it is removed. Then it is cut again and 13 of the remaining
portion is removed; then it is cut one more time and 13 of the remaining part is
removed. What proportion of the original cake remains after the third cut?
10) On Wednesday, everyone at a canteen receives a 10% discount but then has to pay
10% tax on the balance. What is the final price as a percentage of the original price?

11) If you multiply all the odd numbers between 1 and 2008, what is the last digit of the
product?

12) A certain movie theatre has 26 rows with 24 seats in each row. If they are numbered
from left to right, starting from the first row and proceeding toward the rear, in which
row is seat number 375?

13) Maria had the number 4921508 and asked Juan to erase some of the digits in such a
way that the resulting number had three digits and was as small as possible. What
number did Juan obtain?

14) What is the last digit of 22008 − 2?

15) In the diagram below, the horizontal lines are parallel. What is the measure of 6 x?

.........................................................................................................................................................................................................................
... . ..
... ◦ .............
...
...
40 ..
......
.
..
... .....
... .....
.....
. ..
...
. . . ..
......
x ... .. ....
.... ............
.... .
........ .....
.
.... ...
.
........ ...
.... ...
......
. ... ◦
...
......
.....
100
...........
... .....
.
...... . .
.
....................................................................................................................................................................................................................
Second Round Questions
Level II (Grades 7–12)

1) Maria gets channels 2 through 42 on her television. When scrolling through the
channels they cycle so that after channel 42 comes channel 2 again. If she is watching
Channel 15 and advances through 518 channels, which one will she be watching?

2) Last Sunday, Marta received several visitors. When Pedro arrived, Raul was already
there. Jesus and Rita arrived together. Luisa opened the door for Arthur and Arthur
opened the door for Rita. Raul arrived after Rita. Who was the last one to arrive?

3) Some pigeons and piglets are in a cage. If they have a total of 14 eyes and 22 feet,
how many pigeons and how many piglets are in the cage?

4) On a piece of paper there is a three-digit number ending with a 2. If that 2 is removed


and placed at the beginning of the number, the result is a three-digit number which
is 36 less than the original. What is the sum of the digits of the original number?

5) In the figure below, AD = CD, AB = AC, angle ABC is 75◦ , and angle ADC is 50◦ .
What is the measure of angle BAD?
A...........
.... ....................
... .... ............
............
.... ...
. .............
..........
. ...
.
.
.
.
.
. ...
... ......
.
...... 50o D
.
. ....
... ... . ..
....
.
... ... ......
... .. ......
... ... .....
. .......
75o
. .
.. .
...
... . .......
................................................
B C

6) Julio wrote a secret number on a piece of paper and observed that his number has
four digits which sum to 9 and none of them are 0; also, the number is a multiple
of 5 and it is more than 1995. What is the third digit (from right to left) of his secret
number?

7) In the rectangle below, M and N are the midpoints of AD and BC, respectively, and
AC intersects M B and DN at P and Q, respectively. If AD is 5 cm and AB is 3 cm,
what is the area of the quadrilateral M P QD in square centimetres?

M
A ........................................................................................................................................ D
....... . . .
.... ...... .... .... ..
...... .... .... ..
.. ...... ....... ..... ....
..
.. .........
. . ..
.....
. ...
... .... ............ .... ...
.... ....... .... ...
..
... P..
..... ....... Q .....
.
. ...
... ..
... ...... .....
.......... ...
.. .
...
. . .. ...
.. ....... ... .........
.
. ...
.. ......
... ...... . .
..
.
. ....... ..
.. .... ..
. .
. .. ..
.................................................................................................................................
B C
N
8) A large bag contains marbles of 20 different colours. Some are removed at random.
What is the least number of marbles that must be removed in order to guarantee that
you will have drawn at least 100 marbles of the same colour?

9) Juan eliminated one number from a list of 10 consecutive numbers. If the sum of the
remaining numbers was 2008, which number was eliminated?

10) If you write down all of the numbers from 1 to 1000, how many times will you write
the digit 5?

11) A certain cube with sides of length 4 inches is made up of cubes with sides of length
1 inch. How many 1-inch cubes are in contact with exactly 4 other such cubes? [We
say that two cubes are in contact when they share a common face, as shown below.]
....................................................................................................
....... . .
....... ....... ....... ....
...................................................................................................... ..
... ... ...
....
.. ..... ..... ....
... .... .... ..
.. ... ... ...
.. ... ... ...
... .. .. ..
.. ..
.
.. .... .... .............
...............................................................................................

12) A number is called “stuttering” if all of its digits are 1. How many whole numbers less
than 100,000 are there which can be multiplied by 33 to give a stuttering number?

13) In how many ways can you choose one white and one black square on a chess board
so that they are neither in the same row nor in the same column?

14) In the equation 1 + (n2 + n)(n2 + 5n + 6) = 1812 , where n is a whole number, what
is the value of n(n + 3)?

15) Suppose two circles have the same centre and the area between them (which is inside
the larger circle and outside the smaller circle) is 25π 2
2 cm . What is the length of a
chord across the larger circle if it is tangent to the smaller circle?
Second Round Solutions
Level I (Grades 4–6)

1) Rosita has 1 large box, 4 medium-sized boxes, 8 small boxes, and 24 tiny boxes. The
total number of boxes is 1 + 4 + 8 + 24 = 37.

2) After the first term, we may add 2, 4, and 8, respectively, to generate the next three
terms. If we then add 16, 32, and 64 we would obtain 31, 63, and 127. Since these
last two numbers are correct, the missing number is evidently 31.

3) There are five basic triangles which are not made up of smaller ones. There are four
more triangles made up of two smaller ones, and one more triangle made up of three
smaller ones. There are a total of 10 triangles in the diagram.

4) We know that Beatrice is in Chair 3. Also, since Beatrice and Carlos are not beside
each other, Carlos is in Chair 1. Since Antonio is not between Beatrice and Carlos,
he is in Chair 4. This means that Diane in Chair 2.

5) If T denotes a tiger and L denotes a lion, the animals must be lined up in the order
T , L, T , L, T . There are 3 choices for the first position, 2 for the second, 2 for the
third, 1 for the fourth, and 1 for the fifth. The number of possible arrangements is
3 × 2 × 2 × 1 × 1 = 12.

3
6) If n is the number then 3n = 4 × 120 = 90. Then n = 30.

7) Each white square is 6 × 6; suppose that each shaded rectangle is 6 × a as shown.


Then each square has perimeter 24 and each shaded rectangle has perimeter 2a + 12.
Since the rectangles have double the perimeter of the squares, 2a + 12 = 2(24) = 48.
Solving for a gives a = 18. By applying this information to the original diagram, we
see that perimeter of the original figure is 84.
...........................................
. 30 ............................................

6 a .. ............................................................
................................................................................................ ........
.. ...
.......
... ...
.......................
... ... ..................................................................................................................... ..
.. ............................................................
...
... ....
... .
.. . . . . . .
6 ...................................................................................... 6 . . . . ......................................... 12 .
6
..
..
..
..
6 . . . . . . . . . .. . .
...................................................................................................................
..
.........................
..
...............................................................
.. ...
.. ............................................................ .....
...
...
...
...

6 a
...
.
..
. ............................................. ........
..
..
....... ............................................................................................
..
..

8) Suppose there are n days in a week. Then there are n × n days in a month and
n × n × n days in a year. It follows that n3 = 1000 and hence n = 10. There are 10
days in a week.

9) After each cut, the amount of cake remaining is 23 of what it was. After 3 cuts, the
amount remaining is 23 × 23 × 23 = 27
8
of the original cake.

1
10) Suppose p is the original price. After applying the 10% discount, the price is p−0.10p =
0.90p. After applying the tax, the price becomes

0.90p + (0.10)(0.90)p = 0.90p + 0.09p = 0.99p.

The final price is 0.99 or 99% of the original price.

11) If we multiply 1 × 3 × 5 × 7 × 9 × · · · we obtain, in order, 1, 3, 15, 105, 945, . . . In


fact, since 5 is a factor in the overall product its last digit must be 0 or 5. Since the
answer cannot be even, the last digit is 5.

12) The last seat in Row 1 is number 24, the last one in Row 2 is number 48, the last
one in Row 3 is number 72, and so on. It follows that the last seat in Row 15 is
number 360, and so the 15th seat in Row 16 is number 375. The answer is Row 16.

13) Since it is not possible for Juan to have a 0 for the hundreds digit, the smallest possible
hundreds digit is 1. Starting with this, the smallest possible number Juan can obtain
is 108.

14) The powers of 2 are 2, 4, 8, 16, 32, 64, 128, 256, 512, etc. The last digits of these
powers are 2, 4, 8, 6, 2, 4, 8, 6, 2, etc. The 2008th power of 2 will end in 6. After
subtracting 2, the last digit of 22008 − 2 will be 4.

15) Since a is opposite an angle of 40◦ , it is 40◦ . Since s is the supplement for a 100◦
angle, it is 80◦ . Since a + s + y = 180◦ , y = 60◦ . Since x and y are supplements,
x = 120◦ .
.........................................................................................................................................................................................................................
... . ...
... ◦..............
...
...
40 ...
...
.
.....
... .....
... .....
... ..
......
.
..
..... .. .......
x ..... .............
......... ...
. . . ..
..... ..........
.....
.
....
.
....... y ...
...
..... ... ◦
.....
. .... 100
.......
. .
......... ........
.
..
... ..
...
. ...
..
. a s .
.
..
. ... ..
............................................................................................................................................................................................................

2
Second Round Solutions
Level II (Grades 7–12)

1) Each cycle through the loop consists of 41 channels. So she will be back to Channel 15
after advancing through 41, 82, 123, 164, . . ., 492 channels. This leaves 26 channels
to go, and Maria will end up watching Channel 41.

2) If we use the symbol A → B to mean that A arrives before B, then the information
given is that Raul → Pedro, Luisa → Arthur and Arthur → Rita and Jesus. Finally,
we know that Rita → Raul. It follows that the guests arrived in the order Luisa,
Arthur, Rita and Jesus, Raul, and Pedro. The last one to arrive was Pedro.

3) Let B be the number of pigeons and P the number of piglets. Since each animal has
two eyes, 2B + 2P = 14 and hence B + P = 7. Since pigeons have two feet and piglets
have four, 2B + 4P = 22 and hence B + 2P = 11. Using the usual methods of solving
two equations in two unknowns, one obtains B = 3 and P = 4. Then there are 3
pigeons and 4 piglets in the cage.

4) Let the three-digit number be XY 2, where X and Y are whole numbers from 0 to 9.
After moving the last 2, the resulting number is 2XY . From the information given,
we have
XY 2
2 XY
................................................

3 6

It follows that Y = 6 and X = 2. Then the original number is 262 and the sum of its
digits is 10.

5) Since AB = AC, the angle ACB is 75◦ . It follows that the angle BAC is 30◦ . Since
AD = CD, angles CAD and ACD are equal. It follows that angle CAD is 65◦ . Then
angle BAD is 30◦ + 65◦ = 95◦ .
A...........
..... ....................
... .. ............
............
.... ....
. ............
. ... ...........
.
.
.
.
.
. ...
... 50o .
.....
......
D
.
.. ....
..
.
. .
.. ..
...
.
... ... ......
... .....
...
.. ......
.
...
.......
75o
. . .
....
.
... ...
.....
..............................................
..
B C

6) Let Julio’s number be ABCD. Since it is a multiple of 5 and D 6= 0, we have D = 5.


Since his number is more than 1995 it must be at least 2000; thus A is at least 2.
Since the sum of the digits is 9, if A is 3 or more then either B or C would have to
be 0. Since this is not so, A = 2. Furthermore B and C are both 1. Then the third
digit from the right is B = 1.

3
7) Note that the area of ABCD is 15 cm2 . Also, the area of triangle M DN is half that of
rectangle M DCN , and the area of M BN is half that of M ABN . It follows that the
parallelogram M BN D has area 7.5 cm2 . Since the area of the quadrilateral M P QD
is exactly half of that, its area is 3.75 cm2 .

M
A .......................................................................................................................................... D
.... .......... ... . .
.... ...
.. ....... .... ..... ....
.. ...... ....... .... ....
.. ......... . ..
.. ...
... ........... .... ...
.... ................ .... ...
... .... ...... ....
..
.. ....
.P
..
.
.
.
.
....... ....
... ............. .
Q .
.. ...
...
... .
.
.
..
.
. .........
.
...
.. ..
. .. ...... ...
... ....... ... ........ ......
. ..
... .... . .
. ...... .
........ ....
......... . .... .
.................................................................................................................
B C
N

8) To draw as many marbles as possible without having 100 of a single colour you might
draw exactly 99 of each colour. But then, if you draw 1 more marble you would have
to have 100 of a single colour. Then the least number which guarantees that you will
have 100 of a single colour is 19(99) + 1 = 1981.

9) Let the original ten numbers be n, n + 1, n + 2, . . ., n + 9. Their sum is 10n + 45.


Let the eliminated number be n + k for some k ∈ {0, 1, 2, . . . , 9}. Then the sum of the
original ten numbers is also n + k + 2009. Thus
10n + 45 = n + k + 2009; 9n = 1964 + k.
The only 9-multiple in the progression 1964, 1965, 1966, . . ., 1973 is 1971. It follows
that k = 7 and n = 219. Thus the original ten numbers are 219, 220, 221, . . ., 228
and the deleted number is 226.

10) The numbers from 1 to 99 which contain a 5 are 5, 15, 25, 35, 45, 50, 51, 52, 53, 54,
55, 56, 57, 58, 59, 65, 75, 85, and 95. In total, the digit 5 occurs 20 times in these
numbers. Similarly, the digit 5 occurs 20 times in the numbers between 100 and 199,
20 times in the numbers between 200 and 299, and so on. (This will carry us up to
999, and the number 1000 does not contain a 5.) In fact, between every 100 numbers
the digit 5 occurs 20 times, except for the numbers between 500 and 599. In this
group there is exactly one extra 5 in each number, and the digit 5 occurs a total of
120 times. Then the total number of times that you write a 5 is 9(20) + 120 = 300.

11) A 1-inch cube has contact with exactly 4 other cubes precisely when it has two faces
not in contact with any other cube. This means that it has two faces exposed to the
air (which are shaded in the diagram below). There are two such 1-inch cubes on each
edge of the 4-inch cube, and the 4-inch cube has a total of 12 edges. Thus there are
24 cubes having contact with exactly 4 other cubes.
............................
.........................................................
................................................................................................
.......
.......
.....................................................
....... ....
...
..
... ...................................
...
... ...
..
.. ..........................................
...
....
....
..
...
.. ...................................
... ...
..
..
.. ......................
..
...
................
.
...
....
..
............................................ ..

4
12) If we multiply a number between 1 and 100,000 by 33 then we obtain a number
between 33 and 3,300,000. So if the product is to be stuttering it must be one of the
numbers
111, 1, 111, 11, 111, 111, 111, 1, 111, 111
Among these numbers, only 111 and 111,111 are divisible by 3, and of these two only
111,111 is also divisible by 11. Then only 111,111 is a 33-multiple. Thus the only
whole number less than 100,000 which can be multiplied by 33 to give a stuttering
number is 3,367.

13) A chess board has 32 black squares and 32 white ones. Suppose you choose a black
square first. There are 32 ways of doing this. For each way, you may choose any of
the white squares which are not in the same row or column. Since 8 white squares will
be in the same row or column as the black one, there will be 24 white ones to choose
from. Then there are 32 × 24 = 768 ways of choosing two squares as prescribed.

14) We have (n2 + n)(n2 + 5n + 6) = 1812 − 12 and hence

n(n + 1)(n + 2)(n + 3) = (181 − 1)(181 + 1) = 180(182)

The left side is clearly the product of four consecutive integers and the right side
factors as (12 × 15)(13 × 14) = 12 × 13 × 14 × 15. Then n(n + 3) = 12 × 15 = 180.

15) In the diagram shown, let ` be half the length of a chord across the larger circle which
is tangent to the smaller circle. Let also R and r be the radii of the larger and smaller
circles, respectively. Since the area between the circles is the area of the larger circle
minus that of the smaller one, πR2 − πr 2 = 25π 2
and hence R2 − r2 = 25 2
. Since
r + ` = R by the Pythagorean theorem, we also have R − r = ` . Thus `2 = 25
2 2 2 2 2 2
√ 2
and hence ` = √52 . The length of the chord is 2` = 5 2.
............................
............ .......
....... ......
.... ...... .....
.. .. .... ..
..
. .. .............. ................. ...........
.....
..
... ..... .
. ...... ........ ... ......
. .... ....... .... ....
.
...
.
..
...
..
. R ...
.
..... ..... .. .....
.....
.. ... .... ......
` ...
... ... ..... .... ...
... ..... .
....
... .... ..
.. .... ..
........................................ . .
...
... ... ...
...
... ... r .. . ..
. ....
.
... ... .
... ... ... .. ...
... ... ... .. ...
...
...
.....
..... .
.. .... .... ....
.... ....... ..... ... ......
..... ...................................
..... ......
......
....... . .
........
.......... .......
..................................

5
The University of the West Indies, Mona Campus
2014 Junior Mathematics Olympiad
Test for Grades 4, 5, and 6

Student Information

Name:

Grade:

School Information

School:

Principal:

Olympiad Coordinator:

Examination Questions

1) What is the value of 0 + 1 + 2 + 3 + 4 − 3 − 2 − 1 − 0?


(a) 0 (b) 2 (c) 4 (d) 10 (e) 16

2) Shanique bought some cookies. Each one cost $30. She gave the salesperson $100 and
received $10 in change. How many cookies did Shanique buy?
(a) 2 (b) 3 (c) 4 (d) 5 (e) 6

3) In the figure below, the letter U has been drawn on grid paper. How many squares does
the letter U cover?
... ... ... ... ...
.. .. .. .. ..
......................................................................................................
. .. . .. . .. .. . .
....................... .. .......................
.........................................................................................................
........................ ... ........................
............... . .. . .
............................................................................................
.
........................ ... ........................
. .. . . . .. . .
.........................................................................................................
........................ .. ........................
. ................ . ..................
.
....................................................................................................
........................................................................................................
.............................................................................................................................
... ... ... ... ..
.. .. .. .. ..

(a) 10 (b) 8 (c) 11 (d) 13 (e) 12


4) A certain elevator cannot carry more than 150 kg. Four friends weigh 60 kg, 80 kg, 80 kg,
and 80 kg, respectively. What is the least number of trips necessary to carry the four
friends to the highest floor?
(a) 3 (b) 7 (c) 1 (d) 4 (e) 2

5) Which of these numbers has the smallest value?


(a) 2+0+1+4
(b) 201 − 4
(c) 2×0×1×4
(d) 20 − 14
(e) 4+1+0−2

6) Four candy bars and three sweeties cost $450. One candy bar costs $90. How much does
one sweetie cost?
(a) $20 (b) $30 (c) $40 (d) $50 (e) $60

7) From the five numbers below, Liana chose one number. The number is even and none
of its digits are the same. The hundreds digit is double the ones digit. The tens digit is
greater than the thousands digit. Which number did Liana choose?
(a) 1246 (b) 3874 (c) 4683 (d) 4874 (e) 8462

8) Several triangles and squares may be found in the figure below. How many more triangles
than squares may be found?
......
... ......
.
......................
.
... ... .....
...................................
.... .... ... .......
.........................................
... .. ... .........
. .. . .. ...
.....................................................

(a) 4 more (b) 2 more (c) 1 more (d) 5 more (e) 3 more

9) Some students walked to the museum in rows of three. Al, Bob, and Carl noticed that
they were the 7th row from the front and the fifth row from the back. How many students
went to the museum?
(a) 12 (b) 24 (c) 30 (d) 33 (e) 36

10) What is the value of 7001 × 100 + 7001?


(a) 7001007001
(b) 70017001
(c) 7008001
(d) 707101
(e) 77011
11) In the figure below, A, B, C, and D are points on a line. The distance between A and C
is 10 cm, between B and D is 15 cm, and between A and D is 22 cm. What is the distance
between B and C?
• • •
.................................................................................................................................................................................................................. •
A B C D

(a) 3 cm (b) 2 cm (c) 5 cm (d) 4 cm (e) 1 cm

12) Al, Ben, Carl, and Dan each participate in a different sport: karate, soccer, volleyball,
and judo. Alex does not like sports played with a ball. Ben practices judo and often
attends soccer games to watch his friend play. Which of the following statements has to
be true?
(a) Al plays volleyball.
(b) Carl plays volleyball.
(c) Al does karate.
(d) Ben plays soccer.
(e) Dan does karate.

13) Some children went to a picnic. Each child had at least one brother and one sister there.
What is the smallest number of children that could have been at the picnic?
(a) 1 (b) 2 (c) 3 (d) 4 (e) 8

14) During the time that Kaysia eats two bowls of ice cream, Daysia eats three bowls of ice
cream. The two girls ate ten bowls of ice cream in one hour. How many bowls did Kaysia
eat?
(a) 3 (b) 4 (c) 5 (d) 6 (e) 7

15) The grid in the figure below is made of 1 cm × 1 cm squares. What is the area, in cm2 , of
the shaded region?
..................................................................
... .................................. ... ...
........................................................................
... .................................................... ...
. .... ... . . . . . . . ... ... .
.................................................................................................................................
.... ........................................................................ ...
..........................................................................................
.. ........................... .. ...
.. .. .............. .. ..
.............................................

(a) 9 (b) 16 (c) 7.5 (d) 8 (e) 12

16) How many two-digit numbers ab are there with a and b odd numbers which are not equal
to each other?
(a) 15 (b) 20 (c) 25 (d) 30 (e) 50
17) Peter wrote out consecutive whole numbers starting with 3 until he had written 35 digits.
What was the greatest whole number Peter wrote?
(a) 12 (b) 22 (c) 23 (d) 28 (e) 35

18) There were 60 birds sitting in three trees. Suddenly, 6 birds flew from the first tree, 8
birds flew from the second tree, and 4 birds flew from the third tree. Then there were
the same number of birds in each tree. How many birds were in the second tree at the
beginning?
(a) 22 (b) 24 (c) 26 (d) 20 (e) 21

19) The figure below shows a rectangular garden with dimensions of 16 m by 20 m. The
gardener has planted six identical flowerbeds (shaded grey in the diagram). What is the
perimeter of each of the flowerbeds?
.................................................................................................................... ........
...
.
..... .......................................................................................
.......
...
............................................................................................................................................................................... ...
................................................................................................................ ..
................................................................................................................................................................... ...
.................................................................... ... ..
...........................................................................
... ........................................... ...
..
.
.........................................................................................
.. . . . . ... . . . . . . . . . . . . .
16. m
................................................................................................................................................................ ...
...
................................................................................................................................................................................ ..
...................................................................................................................... ...
.................... ... ...
........................................................................................................ . ..
............................................................................................................. ........
.................................. .
.. 20 m ..................................

(a) 24 m (b) 22 m (c) 26 m (d) 20 m (e) 28 m

20) Akeem chose a certain number, subtracted 203 from it, and then added 2003 to his answer.
The final result was 20003. What number did Akeem choose at the beginning?
(a) 23 (b) 17797 (c) 18203 (d) 21803 (e) 22209

21) Suppose a, b, c, d, and e are different digits. If a + a + a = c, b + b + b = d, and c + d = e,


what digit is represented by e?
(a) 0 (b) 2 (c) 6 (d) 8 (e) 9

22) Pedro bought three kinds of cookies: small, medium, and large. The large cookies cost
4 pesos each, the medium ones cost 2 pesos each, and the small ones cost 1 peso each.
Pedro bought 10 cookies and spent 16 pesos. How many large cookies did he buy?
(a) 1 (b) 2 (c) 3 (d) 4 (e) 5

23) A merchant has 6 bottles. Their volumes are 16 oz, 18 oz, 22 oz, 24 oz, 32 oz, and 34 oz.
Some are filled with orange juice, some are filled with cherry juice, and one is empty.
There is twice as much orange juice as cherry juice. What is the volume of the empty
bottle?
(a) 18 oz (b) 34 oz (c) 24 oz (d) 32 oz (e) 22 oz
24) The figure below shows five equal circles. One is in the middle. The other four touch it
and their centres are the vertices of a square. What is the ratio of the area of the shaded
regions of the circles to the area of the unshaded regions of the circles?
....................... .......................
..... ... ..... ...
... ... ... ...
..... .
................................................................ .
. .
... ................................. ............... ...
... .................................................................................................................. .....
...... .................................................................... .....
.............................................................................
..... ........................................................................... ....
. . .. . .. . .. . ..
. .............................. .
........... .......................................... ...........
... ...... ......................................................................................................................................... ..........
.
... ..................................................................................................... ...
..
... ..................................................................................
... ... ...
. .. ..
...
..... ... . .. . ....
. ...
....
.................. ....................

(a) 1 : 3 (b) 1 : 4 (c) 2 : 5 (d) 5 : 4 (e) 2 : 3

25) Mary, Dorothy, Sylvia, Ella, and Kathy are sitting on a bench in the park. Mary is not
sitting farthest to the right. Dorothy is not sitting farthest to the left. Sylvia is not sitting
farthest to the right or to the left. Kathy is not sitting next to Sylvia, and Sylvia is not
sitting next to Dorothy. Ella is sitting to the right of Dorothy, but not necessarily next
to her. Which girl is sitting farthest to the right?
(a) Ella (b) Sylvia (c) Dorothy (d) Kathy (e) Can’t tell

End of Questions

You may mail this completed question paper to:

Junior Olympiad
P.O. Box 94
Mona Post Office
Kingston 7

You may also deliver your entry by hand or by courier directly to the Department of Mathe-
matics at the UWI, Mona. In all cases, an entry must be received by February 17, 2014 to be
considered.

For more information, extra copies of this question paper, and the latest updates, please visit
the following website:
http://myspot.mona.uwi.edu/mathematics/
(see the link to the Junior Olympiad Resource Centre).
The University of the West Indies, Mona Campus
2014 Junior Mathematics Olympiad
Solutions for Grades 4, 5, and 6

1. We will proceed from left to right. We have 0 + 1 = 1, 1 + 2 = 3, 3 + 3 = 6, 6 + 4 = 10, 10 − 3 = 7,


7 − 2 = 5, 5 − 1 = 4, and 4 − 0 = 4. The final answer is 4.

2. When Shanique bought her cookies she gave the salesperson $100 and received $10 in change. So, her
cookies cost $90 in all. Since the cookies cost $30 apiece, she bought 3 cookies in all.

3. We can redraw the grid on top of the letter U. It is now clear that the U covered up 12 squares on the
grid.
... ... ... ... ...
... ... ... ... ...
...................................................................................................................
.................... .. ....................
.......................... ... ..........................
.............................................................................................................
................................. .... .................................
....................................................................................................................................
.................... .. ....................
.......................... ... .............
...........................................................................................................................
................................. ... .................................
.
. .......... . .
............................................................................................................................................
.
.......................................................................
............................................................................................
............................................................................................
..... ..... ..... ..... .....
.. .. .. .. ..

4. No friend who weighs 80 kg can go in the elevator with another friend who weighs 80 kg. Since there are
three such friends, they must go up in three separate trips. However, these are all the trips that are needed.
Indeed, two friends who weigh 80 kg and 60 kg each can make one trip. Then the other two friends can make
two separate trips. They will need three trips in all.

5. We consider each expression in turn. First, 2+0+1+4 = 7. Second, 201−4 = 197. Third, 2×0×1×4 = 0.
Fourth, 20 − 14 = 6. Finally, 4 + 1 + 0 − 2 = 3. The answers produced are 7, 197, 0, 6, and 3. The expression
with the smallest value is 2 × 0 × 1 × 4 = 0.

6. Since one candy bar costs $90, four of them cost $360. But four candy bars plus three sweeties cost $450.
So, the three sweeties cost $450 − $360 = $90. This means that each sweetie cost $30.

7. Among the five numbers 1246, 3874, 4683, 4874, and 8462, Liana chose an even number. So, she did
not choose 4683. She also did not choose a number with repeated digits. So, she did not choose 4874. The
hundreds digits in her number was double the ones digit. So, she did not choose 1246. The two remaining
numbers are 3874 and 8462. But the tens digit in Liana’s number was greater than the thousands digit. So,
she chose 3874.

8. We count the triangles first. There are 4 small (one-by-one) triangles in the diagram, as shown in
Figure 8(a). There are 3 two-by-two triangles as shown in Figure 8(b). There are 2 three-by-three triangles
................... ...................... .................... ....... ........ ........
........................................... .................................... ..................................... ......................... ... .... ... .....
.....................................................
. . .....................
.. .
............... ........................................................ ................................................................... ..................................................... ......................................... ... .. ....
...................................
... .
.. .. ........ ........ ..................... ........................................................ .................................................................... .....................
........................................................................................
... .. ... .. ......
.. . ... .
...............
..
........................................................................................................
.
.. ... ... ....
.............................. ... ............................................................................................
... ...
.... ..
...
... .
..
...
.. ...
....
...
...... . ... ... .... .. ... .
...................................................
.. .
.. . .................... ... . .. . .... . .. .
.....................................................
....
..
... ....
...
...
...
........................................
..
... ...
..
...
...
.........................
...
... ..
...
..... ... .. ... .. ... ...
... .....
...
..
...
..
...
... ............. ...
...
...
.................
...
....
..
...
.......
..... .... ..
... ..
.....
...
...
... ..
................................................................................................. ...................
...
...
...
..
...
.. .. ...
........................................................

Fig. 8(a) Fig. 8(b) Fig. 8(c) Fig. 8(d) Fig. 8(e) Fig. 8(f)
as shown in Figure 8(c). And there is 1 four-by-four triangle as shown in Figure 8(d). There are 4+3+2+1 =
10 triangles in all. We now count the squares. There are 6 small (one-by-one) squares in the diagram, as
shown in Figure 8(e). And there is 1 two-by-two square as shown in Figure 8(f). There are 6 + 1 = 7 squares
in all. Since 10 − 7 = 3, there are 3 more triangles than squares in the figure.

9. There were 6 rows in front of Al, Bob, and Carl, one row consisting of Al, Bob, and Carl, and four rows
behind them. There were 6 + 1 + 4 = 11 rows of students in all. This means that 33 students went to the
museum.

10. First, 7001 × 100 = 701 100. Then 7001 × 100 + 7001 = 700 100 + 7 001 = 707 101.

11. Let BC represent the distance from B to C. The distances from A to C and from B to D do not add
up to 22, the distance from A to D. This is because they overlap between B and C. Since BC is counted
twice, once in each interval, the distance from A to D is actually 10 + 15 − BC. This is 25 − BC, and it is
equal to 22. So, BC = 3.
.......................................................... 15 .........................................................
.
.
.................................... 10 .....................................
.. .
• • • •
......................................................................................................................................................................................................................

A B C D
.. .
........................................................................................
22 .........................................................................................

12. Al does not play volleyball or soccer because he does not like sports with a ball. He does not practice
judo because Ben does. So, the statement “Al does karate” has to be true. The other four statements do not
have to be true. Indeed, the statement “Al plays volleyball” is false (he does karate); the statement “Ben
plays soccer” is false (he does judo); the statement “Dan does karate” is false (Al does karate). the other
statement, “Carl plays volleyball” does not have to be true. Possibly, Al practices karate, Ben practices
judo, Carl plays soccer, and Dan plays volleyball.

13. There had to be at least one boy and one girl at the picnic. However, the boy also had at least one
brother there. So, there had to be at least two boys at the picnic. Also, the girl had at least one sister there.
So, there had to be at least two girls at the picnic. There could have been two boys and two girls there, all
brothers and sisters. Four is the smallest number of children that could have been at the picnic.

14. During one time period, Kaysia ate 2 bowls of ice cream and Daysia ate 3 bowls of ice cream. They ate
5 bowls in all. During the next time period, Kaysia ate 2 more bowls of ice cream and Daysia ate 3 more
bowls of ice cream. They ate 5 bowls of ice crean during that time and 10 bowls in all. So, two time periods
is one hour and each one is one half of an hour. During the first hour, Kaysia ate 4 bowls of ice cream (and
Daysia ate 6).

...............................................................
15. The shaded region consists of 4 small squares and 8 small triangles. Each small ...
.. ................ ..
.. ................. ..
...
. ...........
................................................................................................... ..............
square has an area of 1 cm2 . Each small triangle is exactly one half of a square. So, its ... ................................................................... ...
.................................................................................................................
..................
  ...................................................................
.. .................................. . ................................
1 2 2 1 .............................................................................................................
... .............................. ...
..................
area is cm . The total shaded area, in cm , is (4 × 1) + 8 × = 4 + 4 = 8. ...
.......................................................................
... ..................
2 2
16. There are 5 choices for the tens digit a: 1, 3, 5, 7, or 9. For each one, there are 4 choices for the ones
digit b: it may be any odd digit except a. There are 5 × 4 = 20 such numbers in all. They are:

13 31 51 71 91
15 35 53 73 93
17 37 57 75 95
19 39 59 79 97

17. The first numbers Peter wrote were 3, 4, 5, 6, 7, 8, and 9. These there are 7 of these numbers in all, and
they were the first 7 digits he wrote. The next numbers he wrote were 10, 11, 12, . . ., 19. There are 10 of
these numbers in all, and they consisted of the next 20 digits Peter wrote. At this stage Peter had written
7 + 20 = 27 digits in all. The next four numbers he wrote were 20, 21, 22, and 23. These four numbers have
a total of 8 digits. At this stage, Peter had written 27 + 8 = 35 digits in all. He stopped there, and the last
whole number he wrote was 23.

18. Let x be teh number of birds ineach tree after some of them flew away. Before they flew away, there
were x + 6 birds in the first tree, x + 8 birds in the second tree, and x + 4 birds in the third tree. So, there
were
(x + 6) + (x + 8) + (x + 4) = (x + x + x) + (6 + 8 + 4) = 3x + 18
birds in the trees. This was equal to 60 birds. So, 3x + 18 = 60. Subtracting 18 from both sides gives
3x = 42. Dividing both sides by 3 gives x = 14. After the birds flew away, there were 14 birds in each tree.
Before they flew away, there were 14 + 8 = 22 birds in the second tree.

19. Let x be the width of a rectangle and let y be its height. From the middle two flowerbeds (which are
up-and-down in the figure) y + y = 16 m. So, y = 8 m. From the width of the rectangular garden, we see
that y + x + y = 20 m. This means that 8 m + x + 8 m = 20 m. So, 16 m + x = 20 m. Subtracting 16 m from
both sides gives x = 4 m. So, each flowerbed has width 4 m and height 8 m. The perineter of a flowerbed is
4 m + 8 m + 4 m + 8 m = 24 m.
.................................................................................................................
....... ..........
.... .................................................................... ...
... ............................................................... ..
...
.............................................................................................................................................................. ...
............................................................................................................... ... .......................
...................................................................................................................................................... ... .... ...
...................................................................................... ... .. ... ...
.................................................................................. ... ... ...
... ................................................... ... ... ... y
... ............................................................................................... 16 m ... ...
...
............................................................................................................... ... ... ...
............................................................................................................................................... ... ...
.............................................................................................................. ...
... ........................
............................................................................................................................ ...
................................................................................................................. ...
... ... x
... ................. .
.......................................................................................................................... ..........
.. .
...................................... 20 m ......................................
. .

20. Let x be the number Akeem chose. When he subtracted 203 from it, the result was x − 203. After he
added 2003 to it the result was x − 203 + 2003 = x + 1800. This was equal to 20,003. So, x + 1800 = 20003.
Subtracting 1800 from both sides gives x = 18, 203.

21. It is not possible that a = 0. Otherwise, we would have a + a + a = 0 + 0 + 0 = 0. So, we would have
c = 0. But a and c cannot be the same digit. Also, a cannot be 4 or more. Otherwise, a + a + a would be 12
or more and so c would be 12 or more. But c is a digit and cannot be more than 9. So, the only possibilities
are a = 1, 2, or 3. In the same way, b = 1, 2, or 3. Thus c = 3, 6, or 9 and d = 3, 6, or 9. Since c and d
must be different digits, c + d could be any of these expressions:

3 + 6; 3 + 9; 6 + 3; 6 + 9; 9 + 3; 9+6
These sums are equal to e. They are 9, 12, 9, 15, 12, and 15, respectively. Since e is a single digit, the only
possibility is e = 9. This happens when c = 3 and d = 6 or c = 6 and d = 3. In turn, this happens when
a = 1 and b = 2 or a = 2 and b = 1.

22. Pedro bought at least one small, one medium, and one large cookie, and bought 10 cookies in all. He
coud not have bought 4 or more large cookies. If he did, he would have paid at least 16 pesos for 4 large
cookies, at least 2 pesos for 1 medium cookie, and at least 1 peso for the other 5 cookies. He would have
spent at least 16 + 2 + 5 = 23 pesos. Pedro coud not have bought 3 large cookies either. If he did, he would
have paid 12 pesos for the large cookies, at least 2 pesos for a medium one, and at least 1 peso for the other
6 cookies. He would have spent at least 12 + 2 + 6 = 20 pesos. Pedro could not have bought 2 large cookies
either. If he did, he would have paid 8 pesos for the large cookies, at least 2 pesos for a medium one, and at
least 1 peso for the other 7 cookies. He would have spent at least 8 + 2 + 7 = 17 pesos. So, Pedro bought
1 large cookie. He also bought 3 medium cookies and 6 small cookies. This would be 10 cookies in all and
their cost in pesos would be (1 × 4) + (3 × 2) + (6 × 1) = 4 + 6 + 6 = 16.

23. The bottles with capacity 18 oz., 24 oz., and 34 oz. are filled with orange juice and the bottles with
capacity 16 oz. and 22 oz. are filled with cherry juice. In this case, there is a total of 18 oz+24 oz+34 oz = 76 oz
of orange juice and 16 oz + 22 oz = 38 oz of cherry juice. So, there is twice as much orange juice as cherry
juice. The bottle with capacity 32oz. is empty.

24. Divide each circle into four equal parts as shown in the figure below. The shaded regions consist of 8
equal quarter-circles. The unshaded regions consist of 12 equal quarter-circles. The ratio of the area of the
shaded regions to the area of the unshaded regions is 8 : 12. This is equal to the ratio 2 : 3.

................... .................
...... ... ........ ..... .... ........
... ... ... ... ... ...
... . . ... . ..
.......................................................................................................................
... ............................... .. ...............................
. ...
... ............................................................................
.
. ...
..... ..................................................................... ....
................................................................................................
... ............................................................................. ...
... ...................................................................................... ...
. . . . . . . .. . . . . .. .
...........................................................................................................................
...... ................................................................................................................. .....
.... ............................................................. ...
.
....................................................................................................................................................
... .... .. ... .... ..
... .
. . .... ... .
. .....
..... .. .... ..... .. ....
...................... ......................

25. Let the seats be numbered 1, 2, 3, 4, and 5. Since Mary does not sit furthest to the right, she is in seat
1, 2, 3, or 4. Since Dorothy is not sitting furthest to the right, she is in seat 2, 3, 4, or 5. However, Ella is
....................................................................... M .....................................................................
.
.

.........................................
.. D, S ........................................
.

.......................... .......................... .......................... .......................... ..........................

1 2 3 4 5

sitting to the right of Dorothy. So Dorothy cannot be in seat 5; she is in seat 2, 3, or 4. Since Sylvia is not
sitting furthest to the right or left, she is also in seat 2, 3, or 4. Finally, Sylvia is not sitting next to Dorothy.
So, either Sylvia is in seat 2 and Dorothy is in seat 4, or vice-versa. Suppose Sylvia is in seat 2 and Dorothy

S D E
.......................... .......................... .......................... .......................... ..........................

1 2 3 4 5

is in seat 4. We know that Ella is sitting to the right of Dorothy. So, Ella is in seat 5. But then Kathy
would be sitting next to Sylvia, which she is not. So, Dorothy is in seat 2 and Sylvia is in seat 4. As before,
........................................................................ M .....................................................................
.

K D S
.......................... .......................... .......................... .......................... ..........................

1 2 3 4 5

Mary is in one of the first four seats. Since Kathy is not stiing next to Sylvia, she is in seat 1. Then Mary
is in seat 3, and Ella must be in seat 5. That is, Ella is sitting furthest to the right.

You might also like